Skip to main content

Full text of "The teacher's hand-book of algebra; containing methods, solutions and exercises illustrating the latest and best treatment of the elements of algebra"

See other formats


m 


K 


BRA 


EACHER'S  HAND-BOOK 


LIBRARY 

OF  THE 

University  of  California. 


Gl  FT    OF 


^ 


..k.. 


I'ri-^^A-^.ivvl^^W't^^ 

Class   ^ 


j. 


-   $^.    IStf.  ~- 


M.  3    Cage's  ^atljematital  Scries. 

THE    .TEACHER'S 

Hand-Book  of  Algebra  ; 


CONTAINING 

/ 

METHODS,  SOLUTIONS  AND  EXERCISES 


ILLUSTRATING 

THE   liATEST   AND   BEST   TREATMENT   OF   THE   ELEMENTS 

OF  ALGEBRA. 


BY 


J.  A.  McLELLAN,  M.A.,  LL.l)., 

M 
HIGH    SCHOOL   INSPECTOE    FOR    ONTABIO. 


The  object  of  pirre  Mafliemati'r<>.ivhicli  is  mwther  vanipfor  Alfiehra,istheunfoldivo 
of  tlie  laws  of  the  human  intelligence.'' — SyiiViiHTEB. 


FIFTH  EDITION-REV ^SEO  AND  ENLARGED. 


W.    -T.    GAGE    &   COMPANY, 

TOKONTO  AND  WINNIPEG. 


Entered  according  to  Act  of  Pt^rliament  of  Canada  in  the  year 
1880  by  W.  J.  Gage  &  Coitpany.  in  the  office  or  tne  iviinisier 
of  Agriculture. 


PREFACE. 


This  boot — embodyincr  the  substance  of  Lectures  at  Teachers' 
A.ssociations — has  been  prepared  at  the  almost  unanimous  request 
o^  the  teachers  of  Ontario,  who  have  long  felt  the  need  of  a  work 
to  supplement  the  elementary  text-books  in  common  use.  The 
following  are  some  of  its  special  features  : 

It  gives  a  large  number  of  solutions  in  illustration  of  the  best 
methods  of  algebraic  resolution  and  reduction,  some  of  which  are 
not  found  in  any  text-book. 

It  gives,  classified  under  proper  heads  and  preceded  by  type- 
solutions,  a  exeat  number  of  exercises,  many  of  them  illustrating 
methods  and  principles  which  ai'e  unaccountably  ignored  in 
elementary  Algebras. 

It  presents  these  solutions  and  exercises  in  such  a  way  that 
the  student  not  only  sees  how  Algebraic  transformations  are 
effected,  but  also  perceives  how  to  form  for  himself  as  many 
additional  examples  as  he  may  desire. 

It  shows  the  student  how  simple  principles  with  which  he  is 
quite  familiar,  may  be  applied  to  the  solution  of  questions  which 
he  has  thought  beyond  their  reach. 

It  gives  complete  explanations  and  illustrations  of  important 
topics  which  are  strangely  omitted  or  barely  touched  upon  in  the 
ordinary  books,  such  as  the  Principle  of  Symmetry,  Theory  of 
Divisors,  Factoring,  Applications  of  Horner's  Division,  &c. 

A  few  of  the  exercises  are  chiefly  supplementary  to  those  pro- 
posed in  the  text-books,  but  the  intelligent  student  will  find  that 
even  these  examples  have  not  been  selected  in  the  usual  appar- 
ently aimless  fashion;  he. will  recognise  that  they  are  really 
expressions  of  certain  laws  ;  they  are  in  fact  proposed  with  a  view 

l(w220 


IV  PREFACE. 

to  lead  liim  fr,  investigate  these  laws  for  himself  as  soon  as  he 
has  sufficiently  advanced  in  his  course.  Nos.  8,  9,  10  and  11 
afford  instances  of  such  exercises. 

Others  of  the  questions  proposed  are  preparatory  or  interpreta- 
tion exercises.  These  might  well  have  been  omitted,  were  it  not 
that  they  are  generally  omitted  from  the  text-books  and  too  often 
neglected  by  teachers.  Practice  in  the  interpretation  of  a  new 
notation  and  in  expression  bv  means  of  it,  should  always  precede 
Nits  use  as  a  symbolism  itself  subject  to  operations.  Nos.  23  to 
36  of  Ex.  iii.,  and  nearly  the  whole  of  Ex.  xv.  may  serve  for 
instances. 

By  far  the  greater  number  of  the  exercises,  are  intended  for 
practice  in  the  methods  exhibited  in  the  solved  examples.  As 
many,  as  possible  of  these  have  been  selected  for  their  intrinsic 
value.  They  have  been  gathered  from  the  works  of  the  great 
masters  of  analysis,  and  the  student  who  proceeds  to  the  higher 
branches  of  mathematics  will  meet  again  with  these  examples 
and  exercises,  and  he  will  find  his  progress  aided  by  his  familiar- 
ity with  them,  and  will  not  have  to  inten-upt  his  advanced 
studies  to  learn  processes  properly  belonging  to  elementary 
Algebra.  In  making  this  selection,  it  has  been  found  that  the 
most  widely  useful  transformations  are,  at  the  same  time,  those 
that  best  exhibit  the  methods  of  reduction  here  explained,  so  that 
they  have  thus  a  double  advantage.  A  great  part  of  the  exercises 
have,  of  necessity,  been  prepared  specially  for  this  work. 

Articles  and  exercises  have  been  prepared  on  the  theory  of 
substitutions,  on  Elimination,  &c.,  but  it  has  finally  been  decided 
to  hold  these  over  for  Pt.  ii,,  which  will  probably  appear  if  the 
prSsent  work  be  favorably  received. 


CONTENTS. 


Chapter  I. — Sobstitution,  Horner's  Division.  &c. 

PAGE. 

Sect.  1. — Nnmeiical  and  Literal   Substitution 1 

Sect    2.— Fundamental  Formulas  and  their  Applications 10 

Sect.  3. — Horner's  Methods  of  Multiplication  and  Division,  and  their 

Applications    21 

Chapter  II. — Principle  of  Symmetry,  &c. 

Sect.  1.— The  Principle  of  Symmetry  and  its  Applications 33 

Sect.  2. — The  Theory  of  Divisors  and  its  AppUeations..- 39 

Chapter  in. — Factoring. 

Sect.  1. — Direct  Application  of  the  Fundamental  Formulas   62 

Sect.  2. — Extended  Application  of  the  Formulas 71 

Sect.  3.— Factoring  by  Parts 79 

Sect.  4. — Application  of  the  Theory  of  Divisors 83 

Sbct.  0. — Factoring  a  Polynome  by  Trial  Divisors 90 

Chapter  IV. — Measures  and  Multiples,  &c. 

Sect.  1. — Division,  Measures  and  Multiples  101 

Sect.  2. — Fractions IO9 

Sect.  d. — Ratios 122 

Sect.  4. — Complete  Squares,  &o 130 

Chapter  V. 
Simple  Equations  op  One  Unknown  Quantity 138 

Preliminary  Equations.  Resolution  by  Factors.  Fractional  Equa- 
tions. Application  of  Ratios.  Equations  involving  Suids, 
Higher  Equations,  Sec. 

Chapter  VI. 
Simultaneous  Equations I70 

Equations  of  Two  Unknown  Quantities.  Systems  of  Equations. 
Application  of  Symmetry.  Equations  of  Three  Unknowns. 
Systems  of  Equations. 

Chapter  VQ. 
Examination  Papers 207 


:ir.R^R>t^ 


CHAPTER  I. 
Bection  I. — Substitution. 


Exercise  i. 
1.  If  a  =  1,   6  =  2,  c  =  3,  rf  =  4,  a;  =  9,  ?/  =  8,   find  the 
value  of  the  following  expressions  : — 


l_a_(l-l-a;)}. 
a-{x-y)-{b-c){d-a)-{y-b){x  +  c). 
x - !j^y  -  {y  —  a)\d  +  c(h-- c)v']■ 
{x+d){y+b+c)+{x-d){a—b-(^)+(y+d](a-x  -dy 
{d-x)^  +  {c  +  v^* 

la-b){c^  -b-'x)  -{c-d){b^  -a'-'x)  +  {d-b-c){d^  -f'*'^ 
d  —  ad  +  c        (\d  +  b 
d  +  a        d—G  d  —  b 

2.  If  a  =  3,  6  =  -  4,  c  =  -  9,  and  2s'  =  a  +  b  +  c,  find  the 
value  of  the  following  expressions : — 

s{s  —  a){s  —  b){s-c). 
««H-(5-a)2+(s-5)^+(s-c)2. 
««  -  (s-a){s  —  h)  -  {s  —  b){s  -c)  — (s  — c)(s— a). 
2{s  —  a)[s  —  b){s  —  c)-j-a{s-b){s-c)  -\-b{s-c){s~a)+c{s-  a){s-b). 

8.  If  ffi  =  2,  6  =  -  3,  c  =  1,  a;  =  4^,  find  the  value  of  the 
following  expressions  : — 

a2_62      „2+/,2     (a-^,)2      (a-h)^  * 

c^^Tb^'    ~a^b^'    (a  +  by'    (a  +  by' 

a^^ab  +  b^     a^-b^      a;  (2x-3         3«-l  ).x-l 


a'^-ab^b^      a^-b^      2 
{a  +  b)\{a  +  by-c^\^ 

Ab^c^-{a^-b^-c^)^' 
a'{b  -  g)  Vb''{c-a)^",^{a-b)^ 
-        Ja-b)(b-G){c-a) 


2  SUBSTITUTION. 


\ 


4.  If  a  =  G.  ^  =  5,  e  ==  -  4,  rf  =  —  3,  find  tlie  value  of  tin 
following  expressions : — 

y{b-'  +ac)+  y(c2— 2ac),  yib^+ac+  ^(c^— 2«c)}. 

5.  If  a;  =  3,  i/  =  4,  2;  =  0,  find  the  value  of: — 

{3a;-v/(a;3+?/2)}2{2a;+v/fa:2+2/2+2)}. 

G.  Calculate  the  values  of  {x+y+z)^-^{x^+y^  +  z^,  ^^^^ 

xyz 

(a)  9^=1,  y  =  2,  z  =  H. 

(b)  x  =  %y  =  3,z  =  4:. 

(c)  a;  =  3,  2/ =  4,  2  =  5. 
(J)  a:  =10,  .v  =  ll,  2=12. 

7.  Givcu  x=S,  y  =  4:,  z=  —5,  calculate  the  values  of 

{x+y+x}^ -S{x+y+z)  (xy+yz+zx). 
x^{y+z)-{-y^{z+x)+z^{x+y)  +  2xyz. 
x'^[y-z)-{:ij^z-x}+z^x-y). 
(5«-4z)2+9(4a;-2)2-(18a;-5«)2. 

{Sx  +  Ay  +  5z)^  +  {Ax  +  8y  +  12z)Z-(5x+5y  +  lSz)*, 

8.  If  s  =  a+ 5 +c,  find  the  value  of 

(2s_a)2  +  (2s-ft)2-(2s+c)2,  given 
(1)  a  =  S,  6  =  4,  c  =  o,  (2)  a  =  21,  6  =  20,  c  =  29, 

(3)  a=119,  6  =  120,  c  =  169,     (4)  a  =  S,  b=  -4,  c  =  5, 
(5)  ^=5,  6  =  12,  c=-13. 

9.  If  a  =  l,  6  =  3,  c  =  5,  f?  =  7,  f  =  9,,/"=ll,  prove  that 


<i  +  6  +  c  +  (i  +  e+/= 


2 


1+1+1+1+1=1(1-1 

ab     be      cd      cle      ef       2   \  a      / 

A+  V  +  l_+±=l/l-l 

abc      bed      cd^      def      4   i  fl6      et 

J_+J^+_l.=l(l-  M. 

o6';i      6cde      cdef       6  \a6p      t/(// 


SUBSTITUTION. 

a2^h2.\.c2_ab  —  bc  —  ca  =  h^+c^+fl^  —  bc  —  ed  —  db  = 
t.2 4. ,f2 _|_ ^2  _ cd - de -ec  =  d^+e^ -\-P  -de- ef-fd. 
10.  lia=l,h  =  %  6  =  3,  d  =  A,  e=o,f=&,(j  =  l,  prove  that 
a+b  +  c  =  U-d,  a-{-b  +  c-\-d  =  ^de, 
a^b+c+cl+e=  W,  a  +  b+c+d-\-e+f=  ^fg, 

aa^fc3+c.3=  ^4l+^),  a2_{.b3  4.^2+^3=: 

de{d±^)^  «3+^,3 +,3+^3 +,3  =  ^0 
«3  +i2  +,2  +,/3  ^.,2  +  /^  =    M+'/J. 

a3_|_i3  4.^3  4-^3  =  («  +  J4.(.+  rf)8, 

a^-\.b^4.rS  +  d^+e^-{-p  =  {a+b+c  +  d-\-t+f)» 
A  ,  lA  .    A.      cd(c+d)(c^d—l) 
6c(o+c) 


a4  4-64  4.^4 4_J4  = 


6ie((i+e)(crfe— 1) 


^44.,44.,4+J4  +  ,4  =  €Ai-^i^), 

,4  +  ,4+,44.d4+,44./4=  ^(Z+^)  (^^^tI). 
c2+t^2=e3^  C3+J3  4.,3=y3 

11.  Assume  any  numerical  values  for  x,  y,  and  z,  and  calculate 
the  values  of  the  following  expressions  : — 

(a;5  -  10:c3  4. 5a;j 2  +  (5a;4  -  10a;2  + 1)3  -  (a;3  + 1) 5 . 

(a,4.i)3_2(x+5)3-(a;+0)34-2(a;+ll)3  +  (x+12)3-(a;+16)3. 

(^2  _^3;3  4.(2x^)2  _  ^^2  4.^2)2 
(a;3_3x^3)34.(^);c32/-^3)3_(a;3  4.^2)3. 

(3x3  +4a;i/+2/2)3  +  (4a;3  +2a;(/)2  -  (5a;S+4a;.'/+i/2)2 
(a;_2/)34-(v-5)34-(5-a;)3-3ia;-2/)(2/-5;(«-aj). 
Art.  I.     If  X  =  any  number,  aS,    for  example,  3,    then  x- 
(which  =  x.a;)  -  3*,  x^  (which  =  x.x'^)  =  ?..(;2,  a;*  (which  =  x.x^)  = 
Bx-3,  &c.     Oi-  3  =  x,  3j;  =  a;2,  3a;3  =0;^,  '6x'^  =  x^,  &c.     Hence  prob- 


4  SUBSTITUTION. 

lems  like  the  follomng  may  be  solved  like  ordinary  arithmetical 
problems  in  "  Keduction  Descending." 

Examples. 

1.  Find  the  value  oi  x^  —  '2.x  —  9  when  x^H. 

x^-2x-9 
5 

5x 

-2x 

3» 
5 

15  Explanation, 

-9  x^^6x, 

.-.  a?2-2a;  =  8a;=15,  and 

6  .-.  a;2-2a;-9=:15-9  =  6. 

2.  Find  the  value  of  a;*  —  *^  —  ^x"^  —dx—b  when  x~'^ 

x'^—x^  —  4.x'^—Sx  —  5 
3 

p^ 3x3 

Tj  2a;3 

3 

Pa  ^*^ 

-4a;3 

r„  2ar2 

3  .-.  x4-a;3-4j;«-3a:-6  =  4 

—  if  ic  =  3. 

7>s  ^-^ 

-3a; 

Tj  3a! 

3 

P4  9 

-5 

%  *• 


SUBSTITUTION, 

Explanation. 

:.   x^—x'^  =  2x^  =  6x^ , 
,•.    x^-x^  —  ix'^  —2x^  —  6x, 
:.   x'^-x'^-ix^-Sx  =  3x^-9, 
:.    cc^  — x^  —  4a;2  —  3a;  — 5  =  4. 
8.  Find  the  value  oi  2x^  +  V2x^  4-6a;3  -12x+ 10, 
Using  coefficients  only,  we  have 

2+12+6-12+10 
-5 


Pi  •• 

.  -10 

+  12 

r, 

+  2 

-  5 

Va     ... 

-10 

^2    ■•• 

+  6 

Vm.     ... 

-   4 

'9   ••' 

-   5 

1t>,     ... 

20 

r3 

-12 

fg  ... 

8 

-6 

Pa      ... 

-40 

r4 

+10 

r.  ... 

-30 

the  quantity  =  —30  if  ar=  -5. 
Art.  II.  If  the  coefficients,  and  aiso  the  values  of  x  are  small 
numbers,  much  of  the  above  may  be  done  mentally,  and  the  work 
will  then  be  very  compact.  Thus,  performing  mentally  the  mul- 
tiplications and  additions  (or  subtractions)  of  the  coefficients, 
and  merely  recording  the  partial  reductions  r^,  r^,  r^,  and  the, 
result  r^,  the  last  example  would  appear  as  follows  : — 


6 


SUBSTITUTION. 


-5)2     4-12     +6     -12     +10 
2 
-4 
8 

Art.  III.  In  the  above  examples,  the  coefficients  are  "brought 
iown"  and  written  below  the  wodncts  p^,  p^,  p^,  p^,  and  are 
added  or  subtracted,  as  the  case  may  require,  to  get  the  partial 
reductions  r^,  r^,  r^,  and  the  result  r^.  Instead  of  thus  "  bring- 
ing down  "  the  coefficients,  we  may  "  carry  up  "  the  products  j9^, 
P2'  Ps'  Pv  writing  tliem  beneath  their  corresponding  coefficients, 
and  thus  get  r^,  r^,  r^,  r^  in  a  third  (horizontal)  line.  Arranged 
in  this  way  Ex.  2  will  appear 

1      -1      _4     _3     _5 
+  3     +6     +6     +9 


1  +2     +2     +3;       4; 
and  Ex.  3  will  appear 

2  +12       +6     -12     +10 
-10     -10     +20     -40 


-5 


2       +2       -4       +8;   -30 
Comparing  these  arrangements  with  those  first  given  (Ex.  2 
and  3),  it  will  be  seen  that  they  are  figure  for  figure  the  same, 
except  that  the  multiplier  is  not  repeated. 

Art.  IV.  When  there  are  several  figures  in  the  value  of  a;, 
they  may  be  arranged  in  a  column,  and  each  figure  used  sepa- 
rately, as  in  common  multiplication.  Where  only  approximate 
values  are  required,  *•  contracted  multipHcation  "  may  be  used. 

4.  Find  the  value  of  3a;5-lG0a;4  +  344a;3_^700a;3-1910a;+ 
1200,  given  a;=  51. 

3     -160     +844     +700     -1910     +1200 

1  3-7-18  37         -23 

50  150      -350     -650         1850     -1150 


•7       -13       +37 
result  is  27. 


-23;       +27 


SUBSTITUTION. 


5.  G'ven  »;=  1-1S3,  find  the  value  of  CAx*-lUx+4:5  correct  to 
three  decimal  places. 

64         0  0  -144  +45 

64  75-712  89-5673  -38-0419 

6-4  7-5712  8-9567  -3-8042 

5-12  6-0570  7-1654  -3.0434 

•192          -2271  -2687  --1141 


1 
1 

8 
8 


64,      75-712,     89-5673,        -88-0419, 
.'.   result  is  —-004. 


•0036. 


1. 

2. 

3. 

4. 
5. 


Exercise  ii. 
Find  the  value  of 

xi-Ux^-Ux^-lSx+n,  fora;=12. 
xi  +  r)0x^-lGx^-16x-61,  for  ic=  -17.' 
2x4  +  249a;3-125a;2-|.l00,  fora:=  -125. 
2.^3  _478a;3_  234a;- 711,  for  a;  =  200. 
x'^—iix'^-8,  for  a;  =  4. 

6.  a;6-515a;S-3127a;4+525a;3-2090a;2+315Ga;- 15792,  for  a: 
=  521. 

7.  2a;5+401a;4-199a;3  +  390a;3_602a:+211,  for  «= -201. 

8.  1000x4  -  81a;,  for  a;  =  •!. 

9.  99a;4  +  117a;3-257a;2-325a;-50,- fora;=lf. 

10.  5a;^+497a;4  +  200a;3  +  19Sa;3- 218a;- 2000,  fora;=-99. 

11.  5a;5-620a;4-1030a;3  +  1045a;2-4120a:+9000,  fora;  =  205. 
Calculate,  correct  to  three  places  of  decimals, — 

12.  a;3  4-3a;2-18a;-38  for  a;  =  3-58443,  for  a;  =- 3-77931,  and 
for  a;  = -2-80512. 

13.  7/4- 142/2 +1/+ 38  for  t/  =  3-13131,  for  y=  -1-84813,  and 
for  ?/= -3-28319. 

Exercise  iii. 
What  do  the  following  expressions  become  (1)  when  x  =  a,  (2) 

when  x=  -a? 

1.  a;4 -4ra3  +  6fl2a;3_4fl3a;-l-rt,4, 

2.  y\x^-ax+a^).  3.   y(x^  +  2ax+a^). 
4.   (a;2-|-aa;  +  rt3)3-(a;2-ax-  +  r/2)3. 

If  a;  =  1/  =  3  =  «,^nd  the  value  of  the  following  expressions  r 


8  SUBSTITUTION. 

5.  {x-y)  (y-z)  (z-x). 

6.  (x-i-y)^  {y  +  z-a)  (x+z-a). 

7.  x{y  +  z)  (y2  +z2  -x^)  +^ ^^,  +  x]  {z^  +x^  -y^-)+z{x  ^y)  (x^  -^ 

8.  -^   +   JL  +  ^_. 
y+z        x+z       x+y 

Find  the  value  of 

n     ■^     ,     X     1  abc 

9.  —  +  — .wtienjc= 

a  b  a-{-b 

10. +  — +  — -,  when  x=  —  (a-b+c), 

a[b  —  x)         b[c  —  x)         a{x  —  c)  a 

11.  ^+  -^-,  wheua.=  ^(Ll«)_. 

a  b  —  a  b[b+a) 

12.  (a  +  x)  {b+x)-n{b  +  c)+x^,  when  a;  =  —. 

b 

13.  bx-\-cy-\-az,  when  x  =  b-\-c  —  a,  y  =  c  +  a  —  b,  z  =  'i-^-b  -c. 

14.  <l±^l±b^  -  __1^  _,  when  a:=  -a. 

a(l+6)  —bx        a-2bx 

15.  — —      -  —! —,  when  a;  =*(/;- ^0- 

\x+bj  x  —  a  —  2,b 

16.  (p  —  q)  {x+2r)  +  {r  —  x)  (p+q),  when  a;  =  ^  ^  ^^  ~  ^■'■. 

17.  ffl3(6-c)-f62(c-a)  +  c2(a-6),  when  a- 6  =  0. 

18.  (a-\-b  +  c)  (6c+m+rt6)  -  (rt+6)  (6-f-c)  (e+a),  when  rt=  — 6. 

19.  (a+6  +  c)3-(a3+63_^c3),  whena+6  =  0. 

20.  {x+y +z)'^  -  (x+y)'^  - {y+z^  -  {z+x}'^ -\-x'^  +y*  +z*,  when 
x-i-y-r-z  =  0. 

21.  a3(c-62)  +  53(a_c2)_}.c3(j_a2)+a6c(a&c-l),  when6-a» 
=  0. 

22.  aW«_!+!iV  +  6^/^^l±lT.  when  a^ +6^=0. 

23.  Express  in  words  the  fact  that 

(a-&)2=a2_2rt6+62. 

24.  Express  algebraically  the  fact  "  that  the  snra  of  two  quan- 
tities multiplied  by  their  difference  is  equal  to  tjie  difference  of 
the  squares  of  the  numbers." 


SUBSTITUTIOSI.  9 

•  25.  The  firea  of  the  walls  of  a  room  is  equal  to  the  height  mul- 
tiplied by  twice  the  sum  of  the  length  and  breadth :  what  are  the 
areas  of  the  walls  in  the  following  cases  : 

(1)  leuglh  I,  height  h,  breadth  h. 

(2)  height  x,  length  b  feet  more  than  the  height,  and  breadth 
6  feet  less  than  the  height. 

26.  Express  in  words  the  statement  that 

{x-^-a)  {x+h)=x^-^{a+b)z+ab. 

27.  Express  in  symbols  the  statement  that  "  the  square  of  the 
sum  of  two  quantities  exceeds  the  sum  of  their  squares  by  twice 
their  product." 

28.  Express  in  words  the  algebraic  statement, 

(x+y)^  =x^  +y^  +  Sxij{x-\-y). 

29.  Express  algebraically  the  fact  that  "the  cube  of  the  differ- 
ence of  two  quantities  is  equal  to  the  difference  of  the  cubes  of 
the  quantities  diminished  by  three  times  the  product  of  the 
quantities  multiplied  by  their  difference," 

30.  If  the  sum  of  the  cubes  of  two  quantities  be  divided  by 
the  sum  of  the  quantities,  the  quotient  is  equal  to  tl:  ^  -square  of 
their  difference  increased  by  their  product ;  express  this  algebrai- 
cally. 

81.  Express  in  words  the  following  algebraic  statement: 

""Lzyl^ix+yy-xy. 
x-y 

32.  The  square  on  the  diagonal  of  a  cube  is  equal  to  three 
times  the  square  on  the  edge ;  express  this  in  symbols,  using 
I  for  length  of  the  edge,  and  d  for  length  of  the  diagonal. 

83.  Express  in  symbols  that  "  the  length  of  the  edge  of  the 
greatest  cube  that  can  be  cut  from  a  sphere  is  equal  to  the  square 
root  of  one-third  the  square  of  the  diameter." 

34.  Express  in  symbols  that  any  "rectangle  is  half  the  rectan- 
gle eoutained  by  the  diagonals  of  the  squares  upon  two  adjacent 
sides."  [The  square  on  the  diagonal  of  a  square  is  double  the 
square  on  a  side.] 

85.  The  area  of  a  ckcle  is  equal  to  x  multiplied  into  the  square 


10  SUBSTITUTION. 

of  the  radius ;  express  this  in  symbols.     Also  express  in  symbols 
the  area  of  the  ring  between  two  concentric  circles. 

36.  The  volume  of  a  cylinder  is  equal  to  product  of  its  height 
into  the  area  of  the  base,  that  of  a  cone  is  one-third  of  this,  and 
that  of  a  sphere  is  two-thiids  of  the  volume  of  the  cii-cumscribing 
cylinder  ;  express  these  facts  in  symbols,  using  h  for  the  height 
of  the  cylinder,  and  r  for  the  radius  of  its  base. 

Exercise  iv. 
Perform  the  additions  in  the  following  cases  : 

1.  {b-a)x+{c-b)y,  and  {a+b)z+{h+c)y. 

2.  ax-lnj,  {a  —  b)x-{a+b)y,  and  {a  +  b)x-{b—a)y, 

3.  (y~z)a^+iz-x)ab  +  {x-y)b^,  and  {x-y)a^-{z  -y)ab-{x 

4.  ax+by  +  rz,  bx+cy+nz,  an."c.>t4-.''.'y  +  &z. 

5.  (a+b)x^+{b+c)y^+{a+c)z^,  {b  +  c^x^ +{a+c)y^  +  {a  +  b)z9, 
{a+c)x^  +  {a  +  b)y^  +  {b+c)^^,  and-(a+/;+c)  {x^+y^+z^). 

6.  x(a-b)2 +y{b-c)2-\.z{c-a)*,  y(^a-b)^ +z{b-c)^+z{c- 
^)^,  and  z{a  -  b)^+x{b  -  c)3  +y(c-a)^, 

7.  {a-b)x^+{b-c)y^+{c-a)z^,{b-c)x^  +  {c-a)y^+{a-b)z^, 
and  [c-a)x^  +  in-b)y^+(b-c)z^. 

8.  {a-^b)x  +  {b+c)y-{c  +  a)z,  {b  +  r,)z  +  {G  +  a)x-{n  +  b)y,  and 
(a+c)y+(a  +  b)z-{b  +  c)x. 

9.  rt3-3rt6-^^/;2,  263-363+C3.  ab-^,b^+b^,  and  2a&-^?;». 

10.  ax^-nbx'',  -Qaaf+lbaf,  and  -  8bx" +  I0ax'' . 

11.  What  will  {ax-by  +  cz)-\-{bx  +  cy-((z) -{cx  +  mj+bz)  be- 
come when  X  -  y  -  z  =  l  ? 


Section  II. — Funovmental  Formulas  and  theib  Appcication. 

4.  By  Multiph  cation  we  get 

{x  +  r)  (x  +  s)=x^  +  {r  +  s)  x  +  rs A. 

(x-hr){x-ts){x  +  t)  =  x''  +  {r  +  s  +  t)x''  +  {rs-tst  +  tr)x  +  rst B, 

From  A  we  immediately  get 
(a; -}-j/)  2=^2  +2x^+2/2 [1] 


FUNDAMENTAL    FORMULAS.  11 

{x+7j  +  z)^=x^+2xy  +  2xz  +  y^  +  2yz  t  z^  [2] 

(2«)^  =  2rt2  +  2  nab [3] 

{x+y){x  —  y)^x'-^—y^ [4] 

From  B  we  derive 

{x±y)^  ^x^±8x^y  +  Sxy^±ir  [6] 

=  x^±/j^±3xij  {x±y) [6] 

{x  +  y4  »)^=a;2  +y'-^+z^  +'Sx-{y+z)  +  8y^{z  i- x)  i-Sz^{x  +  y) 

+  6xyz • [7J 

=  x^+y^  +  z^  +  3  {x  +  y)  (y^-z)  (z  +  x)    [8] 

=  x^  +y^+z^  +8  {x+y  +  z)  {xy  -\-  yz  +  zx)  —  Sxyz...  [9] 

(2a)3^2a3  +  8^a^b -\-  Q^ahc  [10] 

[The  symbol  £  means  the  sum  of  all  such  terms  as] 
Formula  [1] . — Examples. 

1.  We  have  at  once  {x  -\-  y)'^  +  (^  — y)'^  =  2(^2  _j_  ^2^^  a^^j 
{x  +  yY  —{x  —  yY=4.xy. 

2.  (a  +  6  +  c  +  d)  '^  +{a  —  h  —  c  4-  d)  '^  may  be  written 

{{n  +  d)  +  (6  +  c)}-^   -h   {{a  +  d)  —  (6  +  c)}2,  which  (Ex.  1;  == 
.    2{('<  +  J)2i-(i  +  f)^}  ;  similarly 

l^a  —  h  -f  c— (/)2+  [a  +  b—c  —  d)'^  =  {{a  —  d)  —  {b-c)}-^-^ 

.-.  (a+  6  +  C+  tf)2  +  [a  —  h  —  G  -ir-dY  -f  {o.  —  h  +  c  —  d^Jr 
(a  +  6-c-(/)3  =  2{(a  + J)-^  +  (6+c)s+(a-c/)2  +  (/,  _c)a}  ^ 
(again  by  Ex.  1)  4(«3+63+,.2_i.,i2). 

3.  Simplify  («  +  ?j-fc)s  -  2{a+b+c)c  +  c^  ; 

This  is  the  square  of  a  binomial  of  which  the  first  term  is 
^aJrb-^c)  and  the  second -c;  the  given  quantity    .•.  = 
{(«+6+c)-c[-  =  (a 


12  FUNDAMENTAL    FORMULAS. 

4.  Simplify  {a+b)* -^a'-i  +  b^)  [a-i-b)^ +  2{a*  +  h^). 

By  Ex.  1.  2[aA  +  b't)=:{ct2^b^)^  +  {a» -b^)^  ;    :.    given  quan- 

tity  =  («  +  6)4-   2[U^   +  //2)  («  +  6)2  4-  („2    +02)2   +  („2  _  ^>2^a  = 
{{a  +  6)2  _^a3+62j}2  +  ^,i2_63)2=rt4+2«2y3^  /;i  =  (a2   +62j2. 

Exercise  v. 

1.  (a;+3//2)2  + (a; -32/3)3,    |i,,,3  +  3J2)2  _  ^,,2  _  37,2)2. 

2.  Shew  that  {mx-{-n;j)2  +  {nx-my)^  =  {m^-^n^)  [x^  +  i/^). 
B.  "  "  {mx  —  ny)'^—{Hx  —  iny)^  =  \^m''^—n^){x^-i/^). 
4.  Simplify  ;./  + 3ij2_^2(«H-36)  ((i-^jH-(a-&)3)  {a-b\^. 

6.         "         (a;+ 3)3^  (x -1-4)2 -(a;+ 6)3,  and  (4^3-2^2^2 - 

(i(/2+2a;3)2. 

6.  Simplify  (a  +  6+c)2  +  (6  +  (;)»-2(6+c)  {a  +  b+c) 

7.  Shew  that  ['ix  +  by)^  +  {cx-\-dyY-^{ay  -  bxY -\- [cy  - dx)^  = 
(rt2  +  i2  +  c3+(Z3j  (^3.34.^3^. 

8."  Simplify  (a;-3y2)3^.(3^2  _^)3  _  2(3a;3-^)  (a:-3v/2). 

9.  "         (a;3-i-a;^-^2)3_(^3_a.^„^3^3^and(l  +  2a;+4a;3)2 
-|-(l-2a;+4x2)«. 

10.  If  «  +  />=  -  Jc,  shew  that  (2(t-6)2  +  (26 -c)3  +  (2c-a)2  + 
2(2«-6)  (26-c)  +  2(2Z>-c)  (2c-«)-|-2(2c-a)  (2«-?>)  = /^jc2. 

11.  Simplify- 2 (ff- 6) 2 -(a -26) 2;   (a^+^ah-^b^)^ -{a^-\-b^y. 

12.  "  ((/  +  M2-(6-H6-)3  +  (c  +  t/)2-(^+«)2. 

13.  "         (^a;-2/)3+ai/-«)2+a^-a;)2+2(ia:-2/)  (Az-cr^ 

+  2(A2/-^)(i-^-^)  +  2Q^-2/)(i2/-=)- 

14.  Prove  that  [x- yf  +  {y -zf +{z-xY  =  '2{x-y)  {z- y)  + 

^{y-x)  {z-x)  +  ''l{z-y)  {z-x). 

15.  Simplify  (l  +  o;)* -2(1 +a;2)  (1+^)3  +  2(1 +x4). 

16.  "        (a;-l-.(/+,^)2-(.c+?/-z)3-(^+2-a;)2-(z+a;-y)2. 

17.  "        (a;-2y+3z)3  +  (3z-27/)3+2(a;-2?/  +  3z)i2?/-32). 

18.  "        (r/2  4.62-,,. 3)3  _f.(c2_  62)2 +2(/;2_^.2)(rt2_(.62_c2), 

19.  "         {x+yy  +  {x-y)^-\x-y}-[x  +  y)». 


FUNDAMENTAL    FORMTJIiAS.  18 

20.  "         {5a+3b)^  +  16{da+by^-{lSa  +  5b)9. 

21.  Shew  that  (3a-ft)2+(3fe-6)34-(3c-a)3-2(i-3fl)(36-c) 
+  2{'db-c){Sc-a)-2{a-Si.){3u-b)-i{a  +  b  +  c)2=0. 

22.  If  z2=  2x7/,  prove  that  (2x3 -2/2)2 +  (22 -2^ 3)2 +(a;3-2z2)3 

-2(2x3-2/2)(22-2^2)^_2(x2-2z2)(23_2j/3)_ 
2(x3  -  222)  (2x2 -1/2)  =  (x+2/)*. 

23.  Simplify    (l+x+x3+x3)2  +  (l-x-x2+x3)2  + 

(1-X  +  X3-X3)2  +  (1+X-X2-X3)2. 

24.  Simplify   {ax-{-by)^-2{a^x^  +  b^y'^)  (ax+by)^  + 

2{a^x^  +  bhj^). 

Formulas  [2}  and  [3] . — Examples. 

1.    (l-2x  +  3x3;2  =  l_4x+6x2 

+  4x3-12x3 

+9x* 


=  l-4x+l(ix-- 12x3 +9x4. 

2.  (ah  +  bc  +  ra)^=a^h^-\-2ab^e  +  2a^bc-{-b^c^-{-^abc^  +  c^a^  = 
a2b^+b^c^+c^a^+2abc{a  +  b  +  c). 

3.  \{x  +  y)^+x^  +  y^}^^{x+ij)^  +  2(xi-y)^{x2+l/^)-\-x^  +  2x^ 

y2^^4  =  (a.  +  j^)4  +  (^|.y)3|(a;-H^)2   ^   (x-y)^}    t  X-^  +  2x^7/3  +  z/4 
=  2,X  +  ^)4+(x2  -  7/'3, 3 +^4+2x2^/3  +yi  =  ^{ix+y)^  +  X*  +  7/4}. 

4.  (x3+X?/  +  /y3j3=x*+i^.X-'^   +  2x2^/3  +x3^2   -f  2x7/3    +  ^/^  = 
(x+7/)2x2+a;'//3+7/2(x  +  7/)3. 

5.  In  Ex.  3,  substitute  5  -c  for  x,  f  -a  forj/,  and  consequently 
b  —  a  for  x+7/,  then  since  (b  —  a)^  =  {a  —  b)^,  Ex.  3  gives 

|(rt_/,)2  +  (6_c)2+(c-a)2}2  =  2{(a-Z>}4  +  (5-c)4  +  (c-fl)4}. 

6.  Making  the  same  substitutions  in  Ex.  4,  we  have 
(a2+63^c2  -ah-bc  -caY  =  {a  —b)^{b  -  c)2  +  (6-c)2(c-  a)2  + 
(c  — a)2(a  — fi)2,  or,  multiplying  both  sides  by  4, 

{(a_6)3^(5_c)3   +  (c-a)3}2=4(rt_6)2(J_c)2  +  4(fe_c)2  X 

(c-a)2+4(c-rt)2(a-6)2.  and  .-.  from  Ex.  5,  (a-i)4  +  (^6-o)4  + 
(c-a)4  =  2(«-6)2(i>-c)2+2(fe-c)8(c-a)2  +  2(c-a)3>(a-6)3. 


14  FUNDAMENTAL    PORMULAS. 

Exercise  vi. 

1.  (l-2.'c+3a;3 -4x3)3,     {l-x-tx^-x^)». 

2.  {l~2x+2x^-3x^-x^)^,     (l  +  3a;+3a;3+a;3)2. 

3.  (2a-6-c2_l)2,      (l_;^  +  y  +  2)3,      (la;-  1?/  + 03)  2. 

4.  (:c3-x2^+a;^3_^3)3,     („a;+te2+cx3  +  (/a;4)3. 

o.  Shew  that  (a^  ^b'^  +c^)  (.c^  i- y2  ^z2 ^  _ (^^x  +  bij  +  •z)^  = 
{a/j  -  6a;) 3  +  (ex  -  flz) 3  +  (65  -  cy ) 3 . 

6.  Prove  that  {a  +  b)x  +  (6  +  c)y  +  (c  +  ^/)3  multiplied  by  {a  —  b)x 
-^[b  —  c)y  +  (c  —  a)z,  is  equal  to  the  difference  of  the  squares  of 
two  trinomials. . 

7.  Shew    that    (a-b)  (a-c)  +  {b-c)  (b-a)  +  {c-a)  (c-b) - 
i-{(a-6)2  +  (6-c)34-(c-a)3}=0. 

8.  Simplify  {a-(6-c)}3 +{6-(c- a)}»  +  {c-(a-6)}». 

9.  Shew  that  {a^+b^ -x^- )^ +{ai  +  bl-x^)^  +  2{aa^ +bb,y 

^(,,2  ^  ,t2_a;3)2+(t2_l_i,2_^2)2+2(a6  +  ai6i)3. 

10.  ^rovethsii{{a-b){b-c)  +  {b-c){c-a)-]-{c-a){a-b)}^  = 
(a-bY(b-GY  +  {b-cY  (c-a)3-h(c-rt)3  (a-6)2. 

1 1 .  Square  2(«  —  \bx  —^cx-\-2dx. 

12.  If  a;  +  ?/  +  2  =  0,  shew  that  x^  +  //*  +  2*  =  (^2 -7/2)3 + 

(^3_22)3+(22_^2)2. 

13.  Provethata3(6  +  c)2-}.//2(c  +  rt)3-fc3(rt  +  6)3  +  2a6c(a  +  6  +  c) 
=  2(rt6  +  6c+crt)2. 

Art.  V.  To  apply  formula  [4]  to  obtain  the  product  of  two 
factors  which  differ  only  in  the  signs  of  some  of  their  terms  : — 
group  together  all  the  terms  whose  signs  are  the  same  in  one  fac- 
tor as  they  are  in  the  other,  and  then  form  into  a  second  group 
all  the  other  terms. 

Examples. 

1.  Multiply  a  +  6  — c-f  t^  by  a-6  — c  — d;  here  the  first  group  is 
a  —  c,  the  second  b-\-Ll\    :.  we  have 

{(a -c)  +  (6  +  <0}   {{a-c)-{b+d)}={a-cy-{b-\-d)^. 


PUNDAMENTAIi    FORMULAS.  15 

2.  (1  J-  Sx-^Sx^  +  x^)  (1  -  nx  +  Bx^  -  x^.)  =  {(1.+  ?>x"}  + 
{Sx-^x-)}  {(1+3x3)  -  ('3a;+«3)}={l+3a;3)2  -  (3a;+a-)3  =  1- 
3x^  +8x4 -a;«. 

3.  Find  the  continued  product  of  a +/'+c.  h+c  —  a,  c+n  —b  and 
n  +  h  —  c. 

The  first  pair  of  factors  gives  {{b  +  c)-\-a}  {{h-\-c)  —  <-i\  ={6+c)^ 
-o8  =  62+26c+c2-«2. 

The  second  pair  gives  {a  —  {h  —  c)]  {a  +  {h  —  <:)]  =a^  —  h^-\-1hc 
—  c2  ;  the  only  term  whose  sign  is  the  same  in  both  these  results 
is  26c  ;  hence,  grouping  the  other  terms,  we  have 

{26c  +  (/y2+22_a2)}  |2k-(63+c3   -  a^)]  = 
(263)2  -(63+c2-a2)3  =  2a262  +  2^2,•3+2c3»2  _„4_ft4_c4. 

4.  Prove  (^i.^-^ah^h)^  -aU^  =  {a'^+ahY  +  {ah  +  b^)^ . 

The  expression  ^{>t^-\-h^)  {n^  +  2ah+h^)  =  {a'^  +b^)  (^+6)2  = 
a2(a+6)2+63(«  +  6)2  =  (a2_f.a5)2_}-(a6+63)2. 

Exercise  vii. 

1.  (a2+2a6  +  62)  („2_2a6+62). 

2.  {:L;c^-xy+y^){hx^-\-y^+xy). 

8.  (r<2_n6+262)  (o3^.«?,+262);  (.'c4+4.T2/)  (a:4_4a;y). 

4.  {(x  +  y)x-y{x-y)]  {{x-y)  x-y{y-x)). 

5.  Simplify:  (.c+3)  (x-8)  +  (a;  +  4)  (a;- 4)-(a;  +  5)  (a- 5). 

6.  "  (l+a-)'i+(i-a;)4-2(l-a;2)2. 

7.  (a;2+j/^)2-(2^?/)3-(a;3_y2)3. 

8.  (2a2-362+4c2)  (2«3+3J2_4c2). 

9.  (2a+6-3c)  (6  +  3c-2a);  (2a— 6-3c)(/^-3o— 2a). 

10.  (x4+2/4)  (a;2+i/2)  (x+?/)  (x-7/). 

11.  (a;2+a;2/+?/2)  (aj3  _a;?/+?/3)  (x* -a:3?/2+y4). 

12.  (a+A-aZ;-l)  (a+6+a6+l). 

13.  Prove  (a2 +62 +6-2)(6'-i+(-'-  _  as^^^s+as  _fe2)  ^^2  _j./,2_c2j 
=  46*0*  when  a4  =  64  _j_c4. 

14.  (a!2+t/2-|a;i/)  (a;2  +  //2+|xy). 

15.  (a-*— 2x3^3x2— 2a; +1)  (a;* +2x3+3x2 +2a;+l). 


16  FUNDAMENTAL    FORMULAS. 

16.  Multiply  (2:k— 2/)a2  -  {x-\-2j)ax  +x^  by  (2x-y)a''  + 
{x+y)ax-x^. 

Prove  the  following : 

17.  {a^A-h^+c^+ab  +  bc+m)^-(ah  +  bc  +  ca)-  =  {a  +  h  +c)3 
x(a3+62-|-r.2). 

18.  (a3-|-/>2  +c3  +a6-j.5c+ca)3  -  {a^  +ah  +  ca-hcf  = 
{{a+h)[b^c)]^  +  {(b+c)  (c  +  «)}2. 

19.  4.{ab+cdY-{a^+h^-c^-d^)^  = 
<a->rb-\-c-d)  {a  +  b-c  +  d)  {c  +  d->ra~h)  {c-\.d  —  n-{-h). 

20.  Find  tlje  product  of  a;2+//2 +2'- 2x//  +  2xz-2?/0  and  a;=  + 
.,3+22 ._  2xy  -  2x3+2^/2. 

21.  (a;2  +;/2  +.r//|/2)  [x^  -xy^2+y^)  (a;*  -//"). 

22.  (l-6a+9ri2)(i  +  2a+3a2). 

23.  {(m+»)  +  (/?+r^)}  (m-9+p-nj. 

24.  Obtain  the  product  of  1+x+x^,  x^  +x  —  l,  x"  —  x-f-l,  and 
H-a;-a:3. 

25.  («-ft2)2  (a+i8)3  (rtS4-64)3  (a4^.ft8)3. 

26    Shew  that  {xP-  +  a;y  +  ^/^^s  (^,2  _  xij  -f  ;y2)2  _  {x^y^y  = 

Formula  A. — Examples. 

1.  Multiply  x'  —x+5  by  x^  —x  —  1 :  here  tKe  common  term  is 
a;2  _3.^  ^}ie  other  terms  +5,  and  — 7,  hence  the  product  =  (x^  —xY 
4_(_7+5)  (a;?-a;)+(-7x5)  =  (ic^-a;)2-2(a;2  -  x)-  %o=x^- 
2x^-x^+2x-S5. 

2.  {x—a)  (x—3a)  (a;+4a)  {x+6a):  taking  the  first  and  third 
factors  together,  and  the  .second  and  fourth,  we  have  the  product 
=  {x''+Sax  -  4a2)(a;2  +  3aa;-18a2)=n(a;2+3aa;;2  _  (ia^  +  lSa^) 

x(a;2+3aa;)-72a*  =  &c. 

Exercise  viii. 

1.  (a;2+2a;+3)  (a;2  +  2a;-4);  (x-y  +  Zz)  (x-y-^^z). 

2.  (x+1)  (a;+5)  (x-l-2)(a;+4);    {x^  Jra-b)  {x^+^b-aX 

3.  (a2-3)(a2_l)(a2  +  5)  (a2  +  7);    {x^  +  x' +l){x*+x'' -2). 

4.  {(a;+2/)^-4a;i/)}  {(x+y)^ +5a;//}. 


FUNDAMENTAL    FORMULAS,  17 

6.  (?«a;+?/+3)  (nic+y+7). 

7.  (x+a-y)  (x+a+dy). 

8.  (x-"  +a;"  -a)  (x^"  +a;''  -*). 

9.  (ia;4-2/-'  +  2)(|aj4-2/2-4). 

10.  (-+--;5      -+-  +  2i)  . 
\x  '  y     2j   \x  '  y  '     'I 

11.  Multiply  together  a- 2 -f  1/2,  a;- 2 +  1/3,  a;-2--i/2,  and 
K-2-  v/8. 

12.  (x+a  +  b)  (x+h-c)  {x~a  +  b)  [x  +  b  +  c). 

13.  (a  +  b+c)  {a  +  b-{-l)-\-{a+c-\-d)  {b-\-c+d)  -  {a+b+c+cl)K 

14.  Prove  that 

(2aH-26-c)(2//+2c-a)-H(2c  +  2a-6)(2a4-2i-o)+(2//  +  2c-a) 
(2c  +  2a  -b)  =  9{ab+bc+cu,). 

Formulas  [5]  and  [6] . — Examples. 

1.  We  get  at  once 

(a;+^)3-(a;-2/)3  =  22/(3a;^+2/2). 

2.  Simplify  (rt+6+tf)3-3(«  +  i+6')2c4.3(fl+fc  +  c)c2-r3. 
This  plainly  comes  under  formula  [5] ,  the  first  term  being  a+5 

+c,  the  second  —c;  hence  the  expression  is  {(a  +  i+c)  —  c}' = 
{a+b)K 

3.  Shew  thsit  {x-"  +xii+y-)^  +  {xij -x-  -y^)^ - 
6xy{x*  +a;2y3  +?/*)  =  8x^y^. 

This  comes  under  formula  [G] ,  the  first  term  being 
{x^+X7/+y'^),  and  the  second-  (a;-  —xy+y^) ;  we  have  therefore 
{{x^+xy+y-)-  {x^-xy+y^)}^  =  {2xyy  =  8x-y». 

Exercise  ix. 

Simplify 

1.  (l-a;')3  4.(i+a;2)3^    (x^ +xy^)^ -(x^ -xy^)^. 

2.  (a  +  2fc)»-(rt-6)3,     (3a_6)3_(3«,_2i)3. 


18  FUNDAMENTAL    FORMULAS. 

4.  (a-fc)3+(a  +  &)3+6«(a2_62). 

5.  (x-i/)^+{x+!/)^  +  d{x-yy-  {x+y)^3{7j-x){x+y)\ 

6.  (l+a;+x^)s-(l-a;+a;2)3_6a;(l+a;-+a;4). 

8.  (3a;-4?/-r02)3-(52  -  4?/)3  +  3(52  -  4^)2(3a:-4^  +  5z}~ 
■S(3x-4.y  +  5zy{5z-iy). 

9.  (l+^+x-)3  +  3(l-x3)(2+a;-)  +  (l-:c)3. 

10.  Shewtbat  «(a-26)3-^6-2«)3  =  (rt-^/y)(a+c/)3. 

11.  Shewthata3(rt3_263)3+i,3(2a3-i3)3  =  (rt3_i3)(«3^i3)3 

12.  (a;3+u:?/+i/'-*)3  +  6(a;-+i/-)  {x^+xy+y^)-\-{x^  -xy+y^)^. 

13.  Shew  that  aS^^s  +  2^3)3  +  b3{2a-  +  63)3  4-  (8a-62)3  =r 

14.  Simplify  {aa!;-\-l>y)^+0'^y^  +b^x^  ~Sabxy{ax-i-by). 

15.  What  will  a^+b^+c^  —3abc  become  when  a+b+c  =  0  ? 

16.  Find  the  value  of  x^  -y'^+z'^  +  'dx-y^z'^  when  x-  -y-  -fz^ 
=  0. 

Formulas  [7] ,  [8]  and  [9] . — Examples, 

1.  Simplify  (2x-3?/)3+(4^_  5a;)^4-(3-c-y)3- 
.■i(2x-3^)(4^-5a;)(3a;-^). 

By  [8]  this  is  seen  to  be  {(2a;-  3jr)+(4^  —  5a;)  +  (3a;  -  ^) }  3  = 
(0)3  =  0. 

2.  Prove  that  («-/;)3  +  (6-t)3  +  (c-a)3  =  3(a-6)  ib-c){c-a). 

In   [8]    substitute  a  —  6  for  x,  b  —  c  for  y,  and  c  — aforz;  for 
these  values  x+y+z=-0,  and  the  identity  appears  at  once. 

3.  Prove  (a  +  6  +  c)3 -(6  +  6--a)3_(a  +  c-i)3-(a-h6-c)3  = 
24Labc. 

In  [7]  let  a;  =  6+c  — a,  y  =  c+a  —  b,  z  =  a-\-b—c,  and  therefore  at 
+  7/  =  2c,  y-{-z  =  2a,  z-\-x  =  ''Ib,  and  this  identity  at  once  appears. 


-  FUNDAMENTAL    FORMULAS.  19 

Exercise  x. 

1.  Cube  the  following:  1 -x-\-x^ .  a-b-c  1  -  2:r+3a;2_4a;3. 

2.  SimpUfy  (a?2  ^  9a:_l)3  +  (2a;-l)(a:^  +  2a;- 2)  — 
(a;3  +  3a;2-l)3. 

3.  *Prove that. {x+y){y+z)(z+x)  +xf/z  =  {z  +  y-\-z){.r]/ -\-i/z+zx). 

4.  Prove  that    {ax  —  b?/)^  +  a^i/'^  —  b^x^  •^'dahxy{ax  —  bij)  = 

5.  Simplify  (a;-22/)3  +  (?/-2z)3  +  (z-2a:)3  +  3(2'-?/-22)  x 
(y-z  -2a;)  (2-a:-2?/)+(a;+?/+z)3. 

6.  Simplify  (2a;-'  -  Sy^ +422)3^(2^2  _  g^s  +  43^2)34- 
(222-3x2+4|/-)3. 

7.  Simplify  (2fl;B-fe?/)34-(2?>7/-c5)3  +  (2rz— ffa;)3  + 
8f2fla;+6i/  -  rz)  {^by+cz  —  o^a;)  (2r2+rta;  -  by). 

8.  Prove  (x^  +  Sx'^y-y^y  +  \'^xy{x +jj)]^  =  \(x-  y)^+^x^y] 
x{a;'^+a^+|/%'-3. 

9.  '  Prove  9{x^  +y^+z^)  -  (x-l-y  +  z)^  =  (4a;  -f  4y-!-  z)  {x  -  tj)^  + 
{4y  +  -iz+x)  (y-z)-+{iz+4:x+y)  (z-x)^. 

10.  U  x+y-\-z  =  0,  shew  that  a;3  4-7/3  _|_23  :=3a^2_ 

11.  If  a;=  27/+3z  shew  that  a;^  -8y3  _27z3  -  18a;^2  =  0. 

12.  Shew  that  (x^+xy  +  y^)^ +  {x^ -xy+y'')^  +  8^^- 
^z''  (a;4  +a;22/2  +?/4)  =  0,  if  x^  +?/2  +2^  =  0. 

13.  Prove  that  8{a  +  b  +  r)3  -  (a  +  6)3  -  (6  +  c)3  -  (c+  a)3  = 
3(2a+6  +  c)  (a  +  2i4-c)  (a  +  6+2c). 

Prove  the  following : 
14     (ax  —  by)^  +0^;/^  =a^i;^  -\-?>abxy{by  —  ax). 


. .  . ^ 

*Note  that  the  right-hand  member  is  formed  from  the  left-band  one  by  changing 
additions  into  multiplications,  and  miMipHcations  into  additions ;  hence  in  (x+y+ 
s).(x.y-i-y.z+z.xi  the  signs  +  and  .  maybe  interchanged  throughout  without  alter- 
ing the  value  of  the  expression. 


20  FUNDAMENTAL    FORMULAS. 

15.  a^+b^+c^-3nbc=i{{a-b)*  +  {b-c)^  +  (c-a)^}  X 
{a+b  +  c). 

16.  {a  +  b  +  c)  {{a  +  b-r)  {b +c-a}  +  {b  +  c-a)  {c+a-b)  + 
{c+-a-b)  {a  +  b  —  c)}  =  {a  +  b  —  c)  {b  +  c  —  a)  (c+a  —  b)-+8abc. 

17.  a^  +  b^+c^+2iabc  =  {a  +  b  +  c)^-S{a{b-cy+b{c-a)^-\- 
c{a-b)^}. 

18.  (fl.  +  />+7c)(a-i)3  +  (5  +  c+7a)(6-c)«  +  (c+a+76)(c-fl)a 
=  2(a  +  fe  +  f)3 -54rt/>c. 

19.  (a  +  b  +  c)  {{2a-b)  {2b-c)  +  {2b-c)  {2c-a)  +  {2c-a)x 
{2a-b)}  =  i2a-b)  {2b-c)  {2c-a)  +  {2a  +  b -c)  {'Ib  +  c-a)x 
{2c  +  a-b). 

20.  li  x^{y-'rz)  =  a^,  y^{z+x)  =  b^,  z^(x  +  y)  =  c^,  and  xyz  =  abc, 
shew  that  a^ -{-b^  +  c^ +-2abc  =  {x+y)  {y-\-z)  (z+x) 

Expansion  of  Binomials.  • 

We  have  from  formula  [5] 

{a+b)^  —n^  +Sa^b  +  3ab^  +b^  ;  multiplying  by  a  +  b  we  get 
la+b)'^=a^  +  4rt.36  +  Sa^b'^^ +iab^+-b^  ;  multiplying  this  by 
o+i  we  get 

[a-\-b)^=a^  +  5a^b+10a3b^  +  10a^b^  +  5ab^  +  b*. 

From  tiiese  examples  we  derive  the  following  law  for  the  form- 
ation of  the  terms  in  the  expansion  of  a+6  to  any  requu-ed 
power  : — 

(1).  The  index  of  a,  in  the  ^rsf  term,  is  that  of  the  given  power, 
and  decreases  by  unity  in  each  succeeding  term ;  the  index  of  b 
begins  with  unity  in  the  second  term  and  increases  by  unity  in 
each  succeeding  term. 

(2).  The  coefficient  of  the  first  term  is  unity,  and  the  coefficient 
of  any  other  term  is  found  by  multiplying  the  coefficient  of  the 
immediately  preceding  term  by  the  index  of  a  in  that  term,  and 
dividing  the  product  by  the  number  of  that  preceding  term.  It 
will  be  observed  that  the  coefficients  equally  distant  from  the 
extremes  of  the  expansion,  are  equal. 


MULTIPLICATION    AND    DIVISION,  21 

Exercise  xi. 

1.  Expand  (ic+i/)«,    (a;  +  2/)^    («+?/)%    {x+ijY*. 

2.  What  will  be  the  law  of  signs  if  —  y  be  -written  for  y  in  (\)  ^ 

3.  Expand  (a-Z>)^    (a-26)4,    (26-a)4. 

4.  Expand  (l  +  m)6,    (w^-fl)^    (2m+l)e. 

5.  What  is  the  coefficient  of  the  4th  term  in  {a—by°  ? 

6.  Expand  (a;2- 2/) 4,    (a-2''.^)^    (a3-2/>3)G. 

7.  In  the  expansion  of  [a  —  bY^,  the  thii-d  term  is  %Qa'^*^lt^,  iind 
the  5th  and  6th  terms, 

8.  Shew  that  {x-{-yY -x^ -y^ —^xyix+y)  {x^ +x>i-{-y'^). 

9.  From  (8)  shew  that  2 {(a  -  by  -^r  {h  -  cY  +  [^c  -  ay]  =-. 
5(a-6)  (6-c)  {c-a)  {{a-hy +  {b-cY +  {c- aY). 


Section  III. — Hoknek's  Methods  of  Multiplication  and 

Division. 


Examples. 

1.  Find  the  product  of  kx^-\-ix'^ -'rvix-\-n  and  ax'^-\-hx-^r. 
Write  the  multiplier  in  a  column  to  the  left  ot  the  muitipiicaud, 
placing  each  term  in  the  same  horizontal  line  with  the  partial 
product  it  gives  • 

kx^  .       +ix'^  +mx         -fw  Q 


ax^ 


•^bx 
+  c 


akx^        +'.'?j;*         +a)iix^      -\-a}ix^  Pj 

-\-hhx'^        -\-'ilx^         -\-bmx~-\-bnx     ■■■'P^ 

-\-rkx-^        ~\-(:tx^    ■\-cinx-'rcn p^ 


akx^  -f  (rt/  +  6/.-)a;*  +  ( '.nn  +  bl  +  ck)x^  -f- {an + bm  +  cl)x'  + 
(b7i+cm)x-{-cn P. 

Art,  VI.  The  above  example  has  been  given  in  full,  the  pow- 
ers of  X  being  inserted  ;  in  the  following  example  detached  coeffi- 
cients are»used.  It  is  evident  that  if  the  coefficient  of  the  first 
term  of  the  multiplier  be  unity,  the  coefficients  of  the  multiplicand 
will  be  the  same  as  .those  of  the  first  partial  product,  and  may  be 
used  for  them,  thus  saving  the  repetition  of  a  line. 


22 


MULTIPLICATION    AND    DIVISION. 


2.  Multiply  3^4 -2.7-3- 2a: +  3  bya;3+3a;-2. 


1 
+3 

-2 


3     -2     +0      -2      +3 

+9      -G     +0     -G     +9 

-6     +4     -0     +4     -6 


3x^  +  7x^-12x^+2x^-Sx^  +  rdx-Q. 
3.  Find  the  product,  of  (^-3)   >-  +  4)   (a: -2)  (a; -5). 


+^ 

1 

-3 

+4      -12 

+  21 

-2 

1 

+  1      -12 

-2-2 

-6 

1 

-1      -14 
-5      +5 

+  24 

+  70 

-120 

a:4 

-6x3_9a;3 

+  94a; 

-liO. 

4.  Multiply  a;3  -  4a;3  +  2x  -  3  by  2a;3  -  3 


1 

-4      +2 

-3 

2 

0 

0 
-3 

2 

-8     +4 

0         0 

0 

-6 

0         0 

0         0 

-8    +12 

0 
-6   +   9 

2.'c« 

-8a;-^  +  4.r4. 

-.9a;3+12a;2 

-63;  +9 

[x^  X  Z^  =X*] 


In  this  example  the  missing  terms  of  the  multiplier  are  suppliecl 
by  zeros  ;  but  instead  of  writing  the  zeros  as  in  the  example,  we 
may,  as  in  ordinary  arithmetical  multiplication,  "  skip  a  line  " 
for  every  missing  term. 

5.  Multiply  a;4  -  2a;3  + 1  by  a;4  _  a;2  +  3. 


1 

-1 
3 


1    +0    -2   +0   +1  [x*xa'*'-xn 

_1    _o   +2   -0    -1  L«    xa-   -X  J 

+  3    +0    -6    +0   +3 


x° 


-3aj« 


+  6a?* 


•7a,-2        +3 


MULTIPLICATION    AND    MYISION, 


ss 


G.  Find  the  value  of  {x+  2)  (a;+3)(a;+4)  {x  +  5)-  9(x  +  2)(a;-i-  3} 
X^^^-4)  +  3(x•+2)(.^■^-3)  +  77(a;  +  2)-85. 

1      +5 
-0 


+  4 


+  3 


+  2 


1      -4 
+4 

-IG 
+   3 

-39 

+77 

1      +0 
+3 

-13 
+   0 

1     +3 

+  2 

-13 

+  6 

+38 
-26 

+  79 
-85 

+  5.^3-    7.f3  +  12a;   -   9 

7.  Find  the  coeflficieut  of  a;*  in  the  product  of  x  —ax^+bx^~ 
cx  +  d  and  x^+px-\-q. 

1      —a     +6      —c 
—  ap 


+  ? 


-{-d 


+  {b-ap-{-q) 

Exercise  xii. 
Find  the  product  of 

1.  (l+x+x3+a;3+a;'ij(l  ~x+x^  -x^  +x^  -x'^^+x^^). 

2.  {l'+x^){l  -x^+x'^){l+x+x-^+X'i+x^). 

3.  (x-5)  (x+Q)  {x-7){x+8);  (2^--a;3  +  i)  (^4_^+2). 

4.  (aj3  +  5a;3  -  16a;  - 1)  (a;3- 5^-3- l(3.c+l). 

6.   {6x^-x'^-j--2x^-2x^  +  2x^  +  Wx+6)  (3a;2+4a;+l). 
Obtain  the  coefficients  of  x'^  and  lower  powers  in 

6.  {l  +  ix-ix^-\-^\x^-^§^xi)  (1  -  .^x-ix^-^^^x-^-^^^x*). 

7.  Multiply  2x''  -x^+Sx  -4:hj  3z^ -2x^ -x-l. 


24  MULTIPLICATION    AND    ERISION. 

Simplify  the  following : 

8.  (x+l)  {x-\-2)  (a;+3)  +  3(a;4-l)  (a;+2)  -  10(a;+l)+9. 

9.  x{x+l)  (£c  +  2)  {x-h'd)--dx{x+l)  [x+2)-2x{x-hl)  +  2x. 

10.  x{x-l){x-2){x-3}  +  dx{x-l){x-2)-2x{x-l)-2z, 

11.  {x-1)  (x+l)  {x+3)  {x  +  5)-U{x-l)  [x-tl)-rl. 

VI.  Given  that  the  sum  of  the  four  following  factors  is  —  1,  find 
(1)  the  product  of  the  first  pair ;  (2)  the  product  of  the  second 
pair  ;  and  (3)  the  product  of  the  sum  of  the  first  pair-  by  tiie  sum 
of  the  second  pair. 


(1) 

X 

+^^ 

+xi3 

+  X^* 

(2) 

x^ 

+  X8 

+  ^9 

4. .-IS 

(3) 

x^ 

+^-» 

+a;i3 

+x^^ 

(4) 

x^ 

+  a;' 

H-r'o 

+a;ii 

13.  Given  that  the  sum  of  the  three  following  factors  is  equal 
to  —  1,  find  their  product. 

(1)  X     4a;'    4a;8      4a;»8 

(2)  ic2    -ha;3    +xio    4a;* ' 

(3)  x^    +x*^    +x^      +x^. 

Art.  VII.  Were  it  required  to  divide  the  product  P  in  the 
first  of  the  above  examples  by  ax'^  +bx+c,  it  is  evident  that  could 
we  find  and  subtract  from  P  the  partial  products  ^g,  ^3,  (or  what 
would  give  the  same  result,  could  we  add  them  with  the  sign  of 
each  term  changed),  there  would  remam  the  partial  product  ;>, , 
which,  divided  by  the  monomial  ax^,  would  give  the  quotient  Q, 
This  is  what  Horner's  method  does,  the  change  of  sign  being 
secured  by  changing  the  signs  of  b  and  c,  which  are  factors  m 
eacli  term  of  _pg,  p^,  respectively. 


BITJLTIP LIGATION    AND    DIVISION. 


25 


+ 

H  «  3 


+  ' 


+ 

^^    ^ 

•^  5  "1^ 


+ 


+ 


+  I 
+ 

"5 


* 

-      O 

a 
.2 

*-i3 

r-l 

03 

Co 

a-i 

CO 

o 
O 

<D 

CD 

a 

*  1— H 

o 

f-H 

■  ^ 

•  ^H 

p— < 

O 

r-4 

> 

CD 

"^ 

-<-i» 

S 

a 

> 

P^ 

> 

c3 

o 

r---l 
-4-3 

a 

fcD 

J? 

'Hi 

•8 

a 

•  <— 1 

r-i 

o 

O 

n3 

•"3 

IN 

1— ~i 

•1-H 

«J-I 
O 

CO 

-1-3 

a 

a 

•  »-4 

CQ 

P* 

CO 

o 

r^ 

"« 

}-\ 

.2 

a" 

<s 

Pi 

•  * 

Sh 

'Ph 

S» 

'^ 

Q) 

'o 

o 

o 
S3 

aj 

Ph 

_a 

•  1— ( 

c3 

a 

o 

'S 

■4J 

CD 

'to 

O 

CO 

a 

•  1— t 

g 

CD 
O 
O 

a 

'3 

-»3 
>—< 

« 

&. 

fl 

r^ 

g 

CD 

■—1 

-4^ 

■ 

2 

OQ 

■  1— 1 
1 — I 

*  1-^ 

CO 
■4^ 

o 

-1-3 

1 

c 

a 

CD 

^-4 

-4J3 

^ 

3 

O) 

o 

"*^ 

O 

TS 

1— H 
-4-3 

C^"* 

-f 

-             f 

a 

o 

CD 

a 

1— ■ 

o 

3 
o 

i 

o 

-k3 

o 

o 

P^ 

a 

o 

(—1 

a 

O 

o 
CO 
00 

•  1— t 

-1-3 

to 

■73 

a 

1 

et-c 
O 

g 

■73 

a 

rH 
CD 

•  f-H 
-4-3 
O 

a 

<D 

O 

CO 

•rH 

-4J 

o 

a 

■-3 

CD 
CD 

a 

CO 

1 

-1-3 

c3 

O 

-< 
so 

en 

-4-3 

a 

o 

g 

_a 

> 

P 

■73 

a 
o 

CO 

cr" 

CD 

-1-3 

0) 

P-. 

<D 

f^ 

r— * 
-4^ 

■4-4 
O 

CD 

a 

•  r-( 
1 1 

o 

^ 

4 

'             1 

-    4 

"73 

■73 

a 

c3 

CD 

■  I— 1 

-4J 

(0 

'q3 
1  -^ 

-1-3 
00 

'Hi 
•■— ( 

cu 

3 

g 

o 
o 

g 

r-4 
-1-3 

CO 

o 

+3 

■^ 
OS 

c5 

Jj 

o 

o 

<D 

tp 

-*-3 

-4J 

ca 

0) 

^ 

?H 

*^ 

fcB 

.^ 

^ 

o 

o 

CO 
•i-H 

> 
•r-( 

a-t 

CO 

3 

> 

00 

a 
•p— 1 

Pi 

-4-3 

-1-3 
■M-l 

O 

i       e; 

> 

n3 

1 

o 

o 

a 
o 

■73 

CO 

o 

-4-3 

a 

CD 
CO 
O 

4 

-      4 

CO 
-1-3 

O 

r-» 

-a 

-t.3 

O 

-1^ 

Ol 

p—H 
-1-3 

O 

CO 

-1-3 

60 

_a 

-4-> 

o3 

(D 

"3 

o 
o 

-4J 

a) 

« 

-1-3 

CD 
O 
>4 

o 

g 

CD 

a 

-4-3 
-t-3 

1 

'cS 
CO 

g 

a 

Pi 

CD 

Ph 

■73 

g 

eg 

CO 

• 

i-H 

-4-3 

^ 

^ 

Ol 

tfl 

so 

CD 

rid 

'o 

CO 

g 

•           • 

r— 1 

a 

-1-3 

O 

-a 

c* 

<x> 

02 

o 

^ 

c3 

■73   n3 

o 

o 

•  f-l 
> 

g" 

g 

2 

a 
o 

o 

CQ 

CO 
0) 

-^ 

ri 
■  1— ( 

a    a 

ce     eg 

o    o 

m 

•f-H 

CO 

o 

26 


MULTIPLICATION    AND    DIVISION. 


2.  Divide  Sx"  +1x^  -  rix*-i-2x^  - 3a;3  +  18x-- 6  by  z'+Zx-2. 
3     +7     -12  +2   -3   +13   -6 


-3 

+  2 


_9     +6-0+6-9 

+   6-4+0-4+6 


{x''^X'^X'>'} 


8a;4_2a;3+   0    -2x  +  3 


Compare  this  example  with  the  secoEd  example  of  Homer's 
Multiplication,  performing  a  step  in  multiplicatio.u,  then  the  cor- 
responding step  in  division  ;  then  another  step  iu  multiplication 
and  the  second,  (corresponding)  step  in  division,  and  so  on. 

3.  Divide  x''  -  Sx*^  +  ia;-^  +  18x-^  -  7x.+  12  bv  x^  -  Sx^  +  'dx  - 1. 


-3 
+  1 


-3      +0      -4  +18  +0      -7  +12 

+  3      +0      -y  -36  -27 

-3      -U  +9  +36+  27  [x"  - x^  =.'f4] , 

+  1  +0  -3      -12  -9 


>  x-i   +U      -3a;3-12x-   9;       (lr-'+   8x  +3 

The  quotient  is  therefore  x'^  —ox'^  -  IJx  — 9,  and  the  remainder 
6x2+8a;+3. 

4:  Divide  x^  -  Zx'  -ox''  +2a;4  +  5x3  +  Ix-  +  1  by  xS  4.  2a;-l. 
The  zero  coefficient  in  the  divisor  may  be  inserted,  or  it  may  be 
omitted  and  allowance  made  for  it  in  the  2x  — iiue.  See  examples 
4  and.  5  in  multinlication. 


-2 
■+1 


1  _3  +u  _5  +'2  +.J  +4  +0  +1 
-2  +6  +4  -4  -f;  +2 

1-3-2  +2  +3  -1 


I  1  _3  -2  +2  +3   1;  0+5  +0 

\_x^  ~-  x^  =x^] .    The  quotient  is  therefore  x^  —  3x^  —  'Ix^  4-  2a;' 
+  3a;  — 1,  and  the  remainder  5x. 

5.  Divide  lOx^ -lla;»-3a;4+20x3  +  iOa;3  +  2  by  bx'^-3x^-i- 


OF 


MUI.TIPIiTCTI  TON    ANT)    r)I\aSlON. 


27 


Arranging  as  in  the  ordinary  metliod,  we  have 


+3 

-2 
+2 


10   -11    -3  +20 
6-3-6 

-4   +  2 
+   4 


+  10   +0  +2 

+  12 

+   4-8 

-2-4+8 


2      _i    _2  +  4 
Quoiieat  =  2x^-x^  -  2x+4  + 


24-12+10 

24a;2....i2a;+10 


5x^-Sx^+2x—2 

We  first  draw  a  vertical  h'ne  with  as  many  vertical  cohimns  to 
the  right  as  are  less  bj'  unity  than  tlie  number  of  terms  in  the 
divisor.  This  will  mark  the  point  at  v,'hich  the  remainder  begins 
to  be  formed.  We  then  divide  5  into  10,  and  thus  obtain  the 
first  coefficient  of  the  dividend.  We  next  multiply  the  remaining 
terms  of  the  divisor  by  the  2  thus  obtained.  Adding  the  second 
vertical  column  and  dividing  by  5,  we  obtain  —  1  ;  we  multiply 
by  the  —  1,  add  tlie  next  column  and  divide  the  sum  by  5,  and  so 
on  for  the  others. 

This  method  is  not,  however,  always  convenient.  If  the  first 
term  of  the  dividend  be  not  divisible  by  the  first  term  of  the  divi- 
sor, the  work  would  be  embarrassed  with  fractions.  We  may 
then  proceed  as  in  the  following  exaTQples  : 

6.  Dividea;5-3a;4+a;3+3a;2-a:  +  3  hy  2x^+x^ -Bx+l. 

Let  2x  =  y,  or  x  =  — . 
^  2 

Substitute  -^  for  a;  in  the  dividend  and  divisor,  and  wo  have 
2 


y' 

3,y4         y3         Sy^ 

V 

2^ 

24     -r  23     '      22      " 

2 

+  3  - 


%^ 


+ 


y 


_  y^ 


23         '     23 

-2  X  82/^1  +22^/3  +  23  X  3^3  - 24?/+2s  x  3 


37/ 

—  +  1 
2    ^ 


25 

y3+y3-2x3y  +  23 

—  p     '■ 

2/ 6  -  &ij^^  +4y3 +  242/3 -16y +96 


^2/3+,/2_6iy  +  4. 


.A. 


28 


MULTIPLICATION    AND    mviSION. 


Dividing  ?/»  -6^/*  +4»/3  +247/2  -  IQy+dQ  by  y^+y^  -  6?/+4  by 
the  ordinary  method,  and  the  quotient  by  2'  we  have 


y2-7y  +  17 
23 


1_     39y2-  1147/  -28 
23*    ^3_f.,/2_6^_|_4  • 


Substituting  for  ?/  its  value  2x,  and  simplifying  we  get 
a;2         7iB    .     17         1       39a;2-57a;-7 


-T  + 


8 


8"    2a;3+:c3-3a;+l 


B. 


By  comparing  the  dividend  of  A  with  the  original  question,  we 
find  that  we  have  multiplied  the  successive  coefficients  of  the  divi- 
dend by  2°,  2^^,  22,  &c.,  and.  omitting  the  first  term,  we  have 
multiplied  the  successive  coefficients  of  the  divisor  by  tlie  same 
numbers.  Dividing  then  by  Horner's  division  we  get  tlie  coeffi- 
cients 1,  —7,  17,  and  for  coefficients  of  remainder,  -39,  114, 
and  28.  The  first  of  these  divided  by  2,  22,  2^  are  the  coeffi- 
cients of  a;2  &c. ;  and,  -39,  &c.,  are  divided  by  1,  2,  22.  Hence 
the  work  will  stand  as  follows  : — 


a.5  _.;:)^4 +a;3  +   8.c2-     x+   3-2a;3-fa;2-3«+l 
1248  16     82  124 


1-6     +4 

+  24 

-   16  +  96 

-1 

-1      -f? 

-17 

+'•> 

+  6 

-42 

+  102 

4 

-    4 

+  28-68 

.1-7  +17 

-39 

+  114-f28 

1    -6  +4 


"Quotient  = 


x' 


X' 


2 


7x         17 
4    "'"    8 

1 
§■ 

7x         17 
4           8 

1 

8 

S9x^ 


lUx 

2^ 


28 
4 


2x3  +3:3  -'Sx  +  1 
39x2  _  57a;  -  7 


2x^+x2-Sx+V 


*lt  will,  in  general,  be  as  convenient  to  multiply  the  dividend  by  such  a,  num- 
ber aa  will  make  its  first  ti'vin  exactly  divisible  by  the  ftrst  torm  of  the  divisor,  and 
afterwards  divide  the  quotient  by  this  multiplier. 


MTJLTTPTJrjATION    AND    DIVISION. 

7.  Divide  5x^+2  by  Sz^-2x+B. 


29 


5x-' 
1 


0         0         0  0        +2  -f-  Sx^-2x+S 

3         9       27  81         243  13 


5 

0         0 

0 

+  2 

10   +20 

-50 

-9 

-45 

-90 

5     -|.io   -25-140 


0  +  486 
—280 
+225   +12G0 


-2   +9 


-   55  +1746. 
10        25         140 


Coeffs.  of  Quotient  = ^.   -    - 

3         32        o^ 


3* 


Quotient  = 


5x^ 


+ 


lOiB^ 


25^ 


140 


J^    55 -H^ 
34"  3_2  +  3- 

55a;- 582 


9  27  81 

Exercise  xiii. 


81    3a;2-2a;+3' 


1.  Divide  6ar-^  +  5^4  -  17x3  -  6x^  +  10.r-  2  by  2x-  +?.x  - 1. 

2.  (5a;«+6a;5+l)-f  (a;-^+2a-  +  l). 

4.  (x5-4a;-//3  _ 8x273. „  i7a;,y4_  127/5) ^(cc3  _2^v/_3^2). 

Divide 

6.  4a;'i  +  8x3-3a;+lby  a;3-2a-+3. 

7.  10x5+ox-4-00a;3  _4i^.2+i0a;+l  by  a;2-9. 

8.  x^ —x^y-{-x^ii^—x'^ll^^xy'^-y-'  hyx^—y^. 

9.  Multiply  a;4-4a;3a  +  Ca;3«2  -4a;«3  4-  ^4  ^y  a;2  4.  2x(<  +  a", 
ftnd  divide  the  product  by  x*  —  2ic3a+2a;a3  —a*. 

Divide 

10.  x^  -  ax^  +  I'X^  -  bx^  +r,x  - 1  by  a;  - 1. 

11.  6a;^+7.T-i+7a;3+6x3+0a;+5  by  2a;2+^  +  l. 

12.  m{x^-\-y^)-\-S)lxy{x'^-y'-^)  by  12a;3  -  13a;//  +  5.y*. 


•jO  multiplication  and  division. 

13.   6ae-481a;'^  +  79a;4  +  81a;3_81a:2-f86x-481  l.y«-80. 

15.  a(«  +  2//)3_/;(2rt4-6)3  by  («- A)^. 

16.  (x+uy^-{-3{xi-ijyz-{--d{x  +  y)z"-+z^hy{z+y)^  ^^ 

—  17.  10a;"J  +  ina;C  +  10a:3-200  by  a;' +x3 -x  +  l. 

18.  bjnx^ +  {bji-}-cin)x^ +ciix~ +abx  +  ac  hy  bx+c. 

19.  Multiply  1+  2^a;-18a;3  by  l-L3a;3  +  3a;3   aud  divide  the 
product  by  l+Va;  — 8x2. 

Find  the  remainders  in  the  following  cases  : 
-r   20.   {xJ-\-nx^  +  -i.x  +  5)-{x~2). 

^   21.  (x^~Sx^-]-x-S)~{x-l). 

22.  (a:^+4.«3  +  Ga;  +  8)-(a;-i-2) 

23.  (27a:i-?y*)^(8a;-2y). 

24.  {3x''  +5x^  -3x^-\-7x^  -r)x  +  8)-^{x"  -2x). 

25.  (5a;4-f90a;3  4.80a;2-100a;  +  500)-^(a;+17). 


Art.  viii.     The  following  are  examjiles  of  au  important  use  of 
Horner's  Division  : 

1.  Arrange  x^  —6x^-\-lx—  5  in  powers  of  a;  — 2. 

11  -6  7—5 

2  2  _8         -2 


2 


1  . 


-4 
2 


-1 
-4 


-7 


—  2 
-1-2 


-5 


I  1;  0 

Hence,  x^ -Gx^ +lx-5  =  {x—2)^ -5{x-2)-7,  or  as  it  is  gen- 
erally  expressed,  x^  -  Gx^  -\-7x—  5  =  y^  —  5y  —  7  if  y  =  X'-2. 


MULTIPLICATION    AND    DIVISION. 


81 


a.  Express  x^+l2x^+'i7x^+G()x+28  in  powers  of  x+6. 


-3 

12 
-3 

47 

-27 

60 
-GO 

2S 
-18 

-3 

9 
-3 

20 
-18 

6; 
-6 

10 

-3 

6 
-3 

2- 
-9 

0 

-3 

3; 
-3 

-7 

-*-    ) 

0. 

Hence  a;4 +12x3+47x2 +66a;+28  =  //'i -7y/- +10  if  y=a5+3. 

After  a  few  solutions  have  been  written  out  iu  full,  as  in  the 
above  examples,  the  writing  may  be  lessened  by  omitting  the 
lines  opposite  the  mcrements  (—2  in  Ex.  1,  and  3  in  Ex.  2),  the 
multiplication  and  addition  being  performed  mentally.  The  last 
example  written  iu  this  way  would  appear  as  follows  : 

1  12  47         66         28 

1  9  20           6       (10) 

1  6  2          (0) 

1  3  (-7) 

1  (0) 

Exercise  xiv. 

1  Express  x^  —  5x^  +  3a;  -  8  in  powers  of  x  —  1. 
x''-^Sx-+Gx+0  "  x+1. 

;c4  _  8x3  +  2  Ix'-'  -  32x + 97  in  powers  of  x  -  2.  < 


-3 


2. 
3. 

4. 

5. 

6. 

7. 

8 

9. 


x-'  +  12x3  +  5x3-7 
3x'-x3  +  4x-+5x-8 
x^  -7x3  +  11x3 -7x  +  10 
x3-2x--4x+9 

x3-9x3//  +  (JX^2_8^3 

X*  —bx^ij-k-^xy^--y^ 


(< 

x+2. 

<4 

x-2. 

(< 

x-n 

(( 

x-h 

({ 

x-2.y 

<f 

x-y. 

32  SYMMETRY. 

10.  "  8a;S+12x2y  +  10«.'/2+8?/3  ••  2z+y. 

11.  «'  ,r3-|a;2+§a;-f^  "  ia^-rV 

12.  *'  a;'^  +  8a;3-15x-10  ♦*  x+2. 


CHAPTEB  II 


Section  I. — The  Principle  ov  Sym^metky. 


Art.  ix.  An  expression  is  said  to  be  symmetrical  with  respect 
to  two  of  its  letters  when  these  can  be  interchanged  without 
altering  the  expression  : 

Thus  if  in  a^+a^x  +  ax^  +  .v^,  we  write  x  for  a,  and  a  for  x,  we 
get  x^+x-a+xa^+a^,  which  is  identical  with  the  given-  expres- 
sion. So,  in  a;2+i3^_f.^^_|_rt23.  if  ^e  interchange  a  and  6,  there 
results  a;- +«-iB+ai'+62a;  which  is  identical  with  the  given  ex- 
pression ;  but  it  will  be  seen  that  the  expression  is  not  symmetrical 
with  respect  to  x  and  b,  or  x  and  a. 

An  expression  is  symmetrical  with  respect  to  three  of  its  letters 
a,  b,  c,  when  a  can  be  changed  into  b,  b  into  c,  and  c  into  a,  without 
altering  the  expression. 

Thus  a3  -^i,s^sS—Sabo  remains  unaltered  by  changing  a  into  b, 
h  into  c,  and  c  into  a,  and  is  therefore  symmetrical  with  respefct 
to  these  letters.  So,  aH-\-b^a+a^c+c^a-irb^c-\-hc^ ,  and  {a-h)'^ 
_|.  (h  —  c)^  +  (c  —  «)',  are  each  symmetrical  with  respect  to  a,  b,  c. 

Again  (x-a)  {a-by  +  {z-b)  (b-c)^  +  (x-c)  {c-ay  is  sym- 
metrical with  respect  to  a,  b,  c,  but  not  with  respect  to  x  and  any 
of  the  other  letters. 

Generally,  an  expression  is  symmetrical  with  respect  to  any 
number  of  its  letters  a,  b,  c,    .   .    .    h,  k,  when  a  can  be  changed 

into  h,  b  into  c,  c  into  d h  into  Jc,  and  k  into  a,  without 

altering  the  expression. 


BYMMETRY.  83 

A  symmetric  function  of  several  letters  is  frequently  represented 
by  writiDg  each  type-term  once,  preceded  by  the  letter  2  ;  thus  for 
a-\-b-\-c-\-  ...  .  +/.  we  write  2a,  and  for  rt^+rtc-f  atZ+  .  .  .  . 
+  6c  +  k/+  .  .  .  [i.  e.  the  «um  of  the  products  of  every  pair  of 
the  letters  considered)  we  write  "Zab. 

Exercise  xv. 
Write  the  following  in  full : 

1.  2a^h,  2(a-i)3,  y_a{b—c),  ^ab{x-c),  ^aH^c,  ^{a-\-h) 
X{c-a){c—b),  2  {(a+c)--63},  and  va(i-fc)3,  each  for  a,  h,  c. 

2.  y.abe,  y.aH,  ^a-bc,  2  (a- i),  and  2a 3 (a _jj^  each  with 
respect  to  a,  b,  c,  d. 

She"w  that  the  following  are  symmetrical : 

3.  {x-\-a)  (a +6)  {b->rx)-'rabx,  with  respect  to  a  and  b. 

4.  (a+6)^+(a.  — 6)3  with  respect  to  a  and  b,  and  also  with 
respect  to  a  and  —  b. 

5.  {ah -xyY^ -{a -^-h-x-y)  {ab{x-{-y)—xy{a+b)}  Mniih.  respect 
to  a  and  b,  and  also  with  respect  to  x  and  y. 

6.  a'^ {b  ~c)  —  b^{a  —  c)  —  c^ {b  -  a)  with  respect  to  a,  b,  c. 

7.  {ac+bd)-+{bc-arl)-  with  respect  to  a'  and  i-,  and  also 
with  respect  to  c-  and  (P . 

8.  x^  +y^ +?>x>/{x^  +xy + y^)  with  respect  to  x  and  y. 

9.  •{«^-?/3+8x^(2a;-f2/)}3  +  {2/3-a;3+3a;?/(2|/  +  a?)}3  with  res- 
pect to  x  and  y. 

10.  a(a+26)3+6(6+2a)3  with  respect' to  a  and  6,  and  also 
with  respect  to  a  and  —  b. 

11.  ab[{{a  +  c)  (b+c)  -f  2<7(a  +  ft)}2  _  (a_e)2  (j_c)«]  with 
respect  to  a,  b,  c. 

12.  a'^b-  -{■b'^c'^  -\^c^a-  -^'2>abc{a-\-b-\-c)  with  respect  to  afc,  6c,  ca 
With  respect  to  what  letters  are  the  following  symmetrical  ? 

13.  xyz-\'5xy-\-'l{x--\-y'^). 

14.  2(a3a;-'  +  h-h/)  -  2ab{xy  +  //z -\-zx). 

15.  {P  - /t--^)-2  +  4//2(  z'+Zi r-'  4- ( 2/7^  - 2.7-)». 


34  SYMMETRY. 

16.  (x+y)  (x—z)  {7j  —  z)  —  xyz. 

17.  «,252^Z»2c3+.c2a2-2fl6c(a  +  ^-c). 

18.  x^  -y^  +z^  -^x^  -y-^){y^  -z^)  (z^  +  a;'). 

19.  (a+ft)2+(a  +  c)2  +  (//-(-)*- 

20.  (a  +  i)4  +  (,i_^)4^(/,+c)4  +  («  +  f)4. 

Select  the  type-terms  in  : 

22.  ,,2a.2ai-f/*2+2k+?^+2ca 

23.  a(b^  -t"~)+h[c^  -a^)+r{(r-  -b"-)-\-{a  +  h)  (h-\-c)  {c-k-a). 

24.  «(6  +  '0'+^'(''-l-«)^  +  ^'(«  +  ^)^-l'^«/;c. 
Write  down  the  type-terms  in  : 

25.  (a;+y)5,  (a;-?/)^  (x  +  y)^ -a^s  -  ^/S. 

26.  (x+yy  +  ix-yy,  {x+!/y-{x-yy. 

27.  (a;+7/-f2)4,    (a;-y-^)4. 

28.  (a+b  +  c  +  dy,   {a-+h^-+c^+d^y. 

29.  (a  +  &)3+(z,  +  c)3  +  (c4.«,)3. 

Art.  X.  In  reducing  an  algebraic  expression  from  one  form 
to  another,  advantage  may  be  taken  of  the  principle  of  symmetry  : 
for,  it  will  be  necessary  to  calculate  only  the  type-terms,  and  the 
others  may  be  written  down  from  these. 

Examples. 

1.  Find  the  expansion  of  {a  +  b-\-c  +  d-^e-{-&c.y 

TJiis  expression  is  symmetrical  with  respect  to  a,  b,  c,  &c. ; 
hence  the  expansion  also  must  be  symmetrical,  and  as  it  is  a  pro- 
duct oi'  iM;o  factors,  it  can  contain  only  the  squares  a^,  b^,  c^,  &c., 
and  the  products  in  pairs,  ab,  ac,  ad  .  ,  ,  ,  be,  bd,  &c.  ;  so  that 
rt^  and  ab  are  type-terms. 

Now  {a-^by  —a"^  -{-2ab-^b^  ;  and  the  addition  of  terms  involv- 
ing a,  b,  c,  SiC,  will  not  alter  the  terms  a^  -f  2fli>,  but  will  merely 
give  additional  terms  of  the' same  type.  Hence  from  symmetry 
we  get 


SYMMETRY.  35 

(a  + !)  +  (■-{-  I +  e  +  &Q.y-'  ^  a'-'  +2ab-^2ac  +  2ad  +  2ae+ 

+  b-    +2bc+2bd+2be  + 

+  6-2    -{-2cd+2ce-\- 

+  fZ2   +2de  + 

This  may  be  compactly  written 

(2a)3=Xa2  +  2Safe. 

2.  Expand  (a  +  h)'^. 

This  has  been  found  by  actual  multiplication — see  formula  [5] 
—but  we  may  also  proceed  as  follows  : 

(1)  The  expression  is  of  three  dimensions,  and  is  symmetrical 
with  respect  to  a  and  b. 

(2)  The  type-terms  are  a^,  a-b. 

■  Hence  [a-^-b)^  =  a^  +h^  -\-niu'^b-}-b-a),  where  n  is  numerical. 

To  find  the  value  of  n,  puta  =  ft  =  l,  and  we  have  (1  +  1)3  = 
1  +  1  +  »(1  +  1);   .-.  n  =  S. 

3.  Expand  (a;+^ +2)3. 

This  is  of  three   dimensions,  and  is  symmetrical  with  respect 
to  X,  7j,  z.     We  have 

{x-^U+z)^  =  {{x+y)+z]^  =  (x+^)3  +&c. 

=  x^  +  ^x'^y-\-kQ.,  which   are  type-terms,  the  only  other  possible 
type-term  beiug  xyz. 

Now,  since  the  expression  contains  ^x-y,  it  must  also  contaia 
^x'-z,  that  is,  it  must  contain  Qx-{y-\-z).     Hence 

(x+y+z)^  =  x^-\-dx''{y-{-z) 
+2/^+3y-(z+a;) 
+  z^  +  -iz^{y^x) 

+     n{xyz),  where  n  is  numerical,   and 
may  be  found  by  puttinjf  x-y  =  z  =  l  in  the  last  result,  giving 
(l  +  ]+l)3  =  i  +  i^.i+3(i4.iA4.3(i_^l)_j.3(l_^l)_^,,. 
.•.     n  =  6, 


36  SYMMETRY. 

4.  Similarly  we  may  shew  that 

{a  +  b-\-c  +  U)^=  ai-\-nn-{b+c  +  d)  +  (ibcd 
+  hs+3b2{c  +  d+a)  +  Gcda 
+  c^  +  3c^d-+a+b)-\-Gdab 
-f  d^  +'Bd^(<i  +  b-i-c)-\-6abc. 

5.  Expand  (a+6+c+&c.)3. 

Tho  type- terms  are  a^,  a^b,  abc. 

Expanding  (a  +  b-\-c)^,  we  get  a^  +  Ba^h-\-6abc-{-&0, 

Hence  by  symmetry  we  have 

6.  Simplify  (a  +  &  -  2c)  2  +  ( fc + c -  2(y) f  +  (c + «  -  2b)  2 . 

This  expression  is  symmetrical,  involving  terms  of  the  types 
a^  and  ab.  Now  a^  occurs  with  1  as  a  coefficient  in  the  first 
square,  with  4  as  a  coefficient  in  the  second  square,  and  with  1  as 
a  coefficient  in  the  third  square,  and  hence  6a^  is  one  type-term 
of  the  result :  ab  occurs  with  2  as  a  coefficient  in  the  first  square, 
with  —4  as  a  coefficient  in  the  second  square,  and  with  —4  as  a 
coefficient  in  the  third  square,  and  hence  —  6ah  is  the  second 
type-term  in  the  result:  hence  the  total  result  is  6  {a^+b^+c^ 
•—ab  —  bc—ca). 

7.  Simplify  {x-\-y+z)^-\-{x-y-z)^+{ij-~z-xy -\- (z  —  x-y)^. 
This  is  symmetrical  with  respect  to  x,  y,  z;  and  the  type-terms 

are  x^,  Sx^y,  (ixyz  : 

(1)  ic*  occurs  in  each  of  the  first  two  cubes,  and  —  os^  in  each 
of  the  second  two  cubes,  .".  there  are  no  terms  of  the  type  x^  in 
the  result. 

(2)  3x^y  occurs  in  the  first  and  third  cubes,  and  —  Bx^y  in  the 
second  and  fourth,  .•.  there  are  no  terms  of  this  type  in  tlie 
result. 

(3)  6xyz  occurs  in  each  of  the  four  cubes,  .'.  24a;^z  is  the  total 
result. 

8.  Prove  (a^  +  b^+c^+d^)  (w^ +x^ -^y- +z'')- 

{aw  -r  bx-\'Cy  -f-  dz)  -  =  (ax  —  bw)  ^  +{ay  —  \cw)  ^  +  (az  —  dw)  2  -f 
(by-cx)^  +  {l)z-dxY  +  (cz-dy)^. 


SYMMETRY.  87 

The  left  hand  member  (considered  as  given)  is  symmetrical 
with  respect  to  the  pairs  of  letters,  a  and  iv,  b  and  x,  c  and  y, 
d  and  t,  that  is,  any  two  pairs  may  be  interchanged  without 
affecting  the  expression.  As  the  expression  is  only  of  the  second 
degree  in  these  pairs,  no  term  can  involve  three  pairs  as  factors  ; 
hence  the  type-terms  may  be  obtained  by  considering  all  the 
terms  involving  a,  b,  w,  x',  these  are  a^ic'^,  a-x^,  b^w^,  b^x^, 
—  a^w^,  —b^x^,  —  2«fettic,  and  are  the  terms  of  [ax  — bw)'^ -which  is 
consequently  a  type-term.  From  (ax—bw)^  we  derive  the  five 
other  terms  of  the  second  member  by  merely  changing  the 
letters. 

9.  Prove  that 

(x^-yz)^-j-{y^-zxy  -\-{z^ -X!jy-B{x^ -yz)  (y^-zx)  {z^-xy)k 
a  complete  square. 

The  expression  will  remain  symmetrical  if  (x^—yz)  (y^—zx)- 
(z^—icy), instead  of  being  multiplied  by  —3,  be   subtracted  £>-om 
each  of  the  preceding  terms,  thus  giving 

(Kg  -  yz)  { (a;3  -  yz)  ^-(y^-xz)  {z^  —xy) } 
-f  (?/2  -zx)  {(y^  -zx)^  —  {z^  -xy)  {x^  -  yz^} 
-\-[^^'-xy)   {{z2-xyr--(x'--:.z)  (y^-zx)} 
=  (x^  —yz)z[X^-^y^-\-z^  —  Bxyz) 
+&c. 
+&c. 

=  iX'^-j-y^+z^  —  Bxyz)  {x^-^y^-\-z^  -  Sxyz). 
Exercise  xvi. 
Simplify  the  following : 

1.  {a  +  b  +  G)^-\-{a-\-b-c)2  +  {h+c-a)^-\-{c-\-a~-b)^ 

2.  (a-6-c)2-f(i-a-c)3  +  (c-a-6)2. 

3.  (a-}-6-|-c-d)2  +  (6+c+cZ-a)3-|-(c-f£/-{-«_i)2_|_ 
(d+a+b-c)^. 

4.  {a-\-b-\-c)^-a(b-^c-a)-b{a+c-b)-c{a+b-c). 

■     5.  (x-\-y+z+ny^+{x-y-z+7i)^+{x-y-{-z-ny-\- 
(x+y-z-n)^. 

6.   (a+6+c)3+(a+6-c)3-|-(6+c-a)34-(c+a-Z>)3. 


38  SYMMETRY. 

7.  (x-2>/-Bz)^-{-{y-^z-dx)2-{-{z-9x-Sy)^. 

8.  (ma-\-u/j-\-rc)^  —  (ma-\-nb—rc)^  —  (nh-j-rr  —  tim)^  — 
'  (rc-\-ma  —  nb)^. 

9.  aib-\-r)ib^-i-c.2-a2)-{-'j(c+a)(c2-{-a^  -b^)  + 
c(a+&)(a2+62_c2). 

10.  (ai  +  6c-+m)3  -  2rt6c(a+6+c). 
Prove  t)ie  following  : 

11.  (ax-\-b;/+rz)''-{-{hx-\-cy-{-azy  +{cx+a?j+hz)^  + 
{nx-\-cy-]-bz)''^-\-{cx-\-l>i/-^az)^-\-{hx-{-ay-\-cz)^ 

=  '2{ci'+b^-{-c--){x^--{-y2+z-^)-^i{ab-{-bc+ca){xy^yz-{-zx). 

12.  {a-^b+cy-^{b+c-a)^+{c+a-b)^-\-yu-i-b-o)^ 

13.  {a-\-b-\-cy  =  2a^  +  iZa^b+Q-LaH^  +  12.Za^bc. 

14.  (£rt)4  =  2rt*  +  42rt3/,  +  62a3i2  +  122a3ic  +  24SrtZ-(,-t^. 

15.  (^a^ -\.b^  +c2)^  +  2{'ib+bc  +  ca)^  -3{a^ -i-b^  +c^)X 
f^ab  +  he  +  ca)  3  =  («  3  +  0  ^  _|_ c  3  _  ^abc)  3 . 

16.  («-/>)2(6-c)3  +  (^>-c)2(c-a)2  +  (c-a)3(rt-/;)2  =  ' 
^^2  +63  +c2  -ab-  ac  -  bc)^. 

17.  (2a-ft-c)-'(26-c-a)2  +  (26— c-a)2(2c-a-6)2-f. 
(2c-a-&)2(2(*-6-c)2=9(«s  +  />2+6'2-a6_6c-ci/)2 

18.  (rtr2+26?-s-|-cs-)((ifx2  +  26a;y  +  c^2)_ 
{  arx  -{-b{ry  +  sx) + csy  }2  =  {ac  —  b^)  (ry  —  sx)  2 . 

19.  {a^+ab+b^){c^  +  cd+d^)  =  {aG  +  ad  +  bdy  + 
{ac  +  ad+bd)  (be  —  ad)  +  {be  —  ad)-. 

20.  Sbew  that  there  are  two  ways  in  which  the  given  product 
in  the  last  example  can  be  expressed  in  the  form  p-  +iyq+q^,  and 
two  ways  in  which  it  can  be  expressed  in  the  form  jJ^  —pg+q^- 
21.  6(ti'2  +a;2  +?/2  +z2)2  =  (w  +  x)^  +  (rt'-a;)^  +  (»,-+?/)  *  + 

{iv-'y)^  +  {w  +  z)^  +  {w-z)^  +  {x+i/)^-\-{x-y)^  +  {x-\-z)'^  + 

{x-z)^  +  {y+z)^My-^y' 

22.     |{(a+i+c)-'  +  (a-fe-c)6  +  (6_c-a)«+(c-a-i)5}=s    . 
i{(a+i  +  c)3-fi>-^.-e)3  +  (i_c-a)3  +  (c_a-i)3|x 
^{{a+b  +  c)^  +  {a-b-c)^  +  {b-c~a)^-\-{c-a-b)2}. 


THEORY   OF   BfVISORS.  39 

Section  II. — Theory  op  Divisors. 


Any  expression  which  can  be  reduced  to  the  form  ax"-]-hx''~'^  + 
cx^~^+    .    .    .    .   -\-    .    .    .    .    +Jix  +  k,  in  which  n  is  a  positive 
integer  and  a,  h,  c,    .    .    .    .   h,  k  are   independent  of  x,  is  called 
a  PoLYNOME  in  X  of  degree  n. 

The  ■expressions ./'(ic)",  F{xy,  <p(x)"',  are  used  as  general  symbol.-; 

for  polynomes  ;  the  index  n.  m,  indicates  the  degree  of  the  polj- 
nome. 

Theorem  I.  If  the  polynome/(a;)"  be  divided  by  a?— a,  the 
remainder  will  be /(«)". 

Cor.  1.    /(x)" —/(a)"  is  always  exactly  divisible  by  a;  —  a. 
(Particular  case:  ic"  — a"is  alv/ays  exactly  divisible  hy  x—a). 
Cor.  2.     If /(a)"  =  0,  /(a;)"  is  exactly  divisible  by  x—a,  i.e.,  /{xf 
is  an  algebraic  multiple  of  x  —  a. 

Cor.  3.  If  the  polynome  /(as)"  on  division  by  the  polynome 
^(a;)'"  leave  a  remainder  independent  of  x,  such  remainder  will  be 
the  value  of /(a;)"  when  <f){x)"'  =  0. 

Examples. — Theorem  1. 

1.  Find  the  remainder  when  x^  —7x^-\-lSx^  —  IGx^  +  dx— 12  is 
divided  by  x  —  5. 

The  remainder  will  be  the  value  of  the  given  polynome  when  5 
is  substituted  for  x.     (See  Art.  III.). 

1     -7     +13         IG     +9     -12 
5-10         15-5         20 


1-2  3       -1         4;         8 

Hence  the  remainder  is  8. 

2.  Find   the   remainder   when  (x—a)^ -{■  (x  —  b)^ -\-(a^h)^    is 
divided  by  x-\-a. 

For  a;  substitute -a,  then  ( -  2a)3  +  ( -  «  -  h)^  +  (a.+b)^  =  _  8^3. ' 

8.  Find  the  remainder  vfhen  x^^a^-\-h^-\-{x+a){x+b)[a-\-L) 
is  divided  by  a;  +  a+6. 


40  THEORY    OF    DIVISORSi 

For  X  substitute  —  (« +  6)  and  we  get 
~{a+l,)^-\.a^-\.l^J^ah{ci^h)  =  -1ah{a-^h).      See  Formnla  [6]. 

4.  Find'the  remainder  when  {x^^'-lax  —  '±a^Y{x^-'lax-%i-) 
+  32(a;-a)4(j;4-a)4  is  divided  by  x'^  -  2rt.3. 

x^  —  2«2  may  be  struck  out  wherever  it  appears. 

This  reduces  the  dividend  to 

{'hnxf{-1ax)->r^1{x-aY[xA-aY=  -  16a4.^4  ^32(a;3 -a*)*. 

In  this  substitute  2a3  for  x'^  and  it  becomes 
-64a«^-32a8=-32a^ 
which  is  the  required  remainder. 

Exercise  xvii. 

1.  Find  the  remainder  when  3a;4+60a;3  +  54a;3  — 60.'c4-58  ig 
divided  by  a; +19. 

2.  Fmd  the  remainder  when  ^^x^  —  Sr/x-'-f-Sj-rc  —  s  is  divided  by 

3.  Wliat  number  added  to  ^x^  A- 34a;4  +  58.6-3+21x3  -  123x-  41 
will  give  a  sum  exactly  divisible  by  2x+13  ? 

4.  What   number   taken    from    10a;' °  ~  20.'c8 -lOaj^  - -SOa;*  — 
8*9a;3+.20will  leave  a  remainder  exactly  divisible  by  10a;'-  — 11  ? 

Find  the  remainders  from  the  following  divisions  : 

5.  {x-\-\Y-x'>  ~x  +  \,  and  (a;+a+3)3  -  (a;  +  ,/  +  l)3  -^a:  +  2. 
■  6.  a;"+y"  -^  x-y;  a;2"+?/2»  -r-  a;+// ;  a;-"+i+y-:'i+i  -^  ^j-f^. 

7.  (a;  +  l)3+a;3  +  (a;-l)3-=-a;-2. 

8.  (a;-a)3(x+^f)3  +  (a;3-2i2)3  -i-a;3+Z)2. 

-^a;2+-2^Y2. 

10.  (9rt24-6a^+i/;2)(9«3_6a6+462)(81a4— 36a2i3  +16i4)-r 
(3a-2i)2. 

11.  a2(a;-a)3+fe2(a;-i)3-i-a;-a-&. 

12.  (rta;  +  %)3-j-«3y3_|_j3jg3_3(^i;jj^^(-(2;^jy^  -^  (a+&)(a;  +  y)' 

13.  a;3  +  a3 +.  63 _  3Q^jj._i_^_^  _j,^.    also -i- a;  +  a— 6  also-*- 


THf  ORY    OF    blVISORS. 


41 


H.  Anr  polynoine  divided  by  a; -1  gives  for  lemaindev  tlie 
aam  of  tlie  coefficients  of  the  terms. 

Examples. — Cor.  1. 

1.  ar-^-f-!/'  is  exactly  divisible  by  x-\-y. 

In  "a;^  -  n^  is  exactly  divisible  by  ic— «,"  siibstitnte  -y  for  «., 

2.  m.r'^  -  px'^'  +qx-{-v}  -\-p +q  is  exactly  divisible  by  x-^1. 
This  may  be  written 

{v.x--  -jjx^ -{-ox}  -  {iii{-l)^ -  p{~l)-  +q{-l)}  is  exactly  divi 
sible  by  jc-  (-1). 

3.  (a;3+6a,vy-I-%3)5_|_(a;2_t-2a;?/+4?/2)5   is  exactly  divisible  by 

{r+2y)^.     ¥oi- (x^-hQxy-\-iy^y-{-x"-2xy-iy^)''  is  exactly 
divisible  by  (x^-\-()xy+'iy^)-  (  -x-2  _  2a;//  -4?/3),  which  is 
2(a:3-4-4a;.v+4?/3 )  =  2(a;4-27/)2. 

Exercise  xviii. 
Prove  that  the  following  are  cases  of  exact  division  : 

I.  a;^"+i+y2«+i  ^x  +  y;  a;^"  -t/^"  -f-  a,-+y. 

also  -^  X-  +?/-. 

3.  (rt.a;+/^^)'  +  {l>'^+('yy  ^  («  +  '')(a;  +  ?/)• 

5.  (22/-a;)"-(2a;-y)"-^3(2/-a;). 

6.  {2y-xf''+^-{-{'2x-y)^'"+^^y+x. 

7.  {my  —  nxY  —  {vix  —  ny)^  -=-  (jh  +  w)  (y  —  x). 

8.  (.r-{-7/)«  +  (a;-j/)«-^2(a;2+7/2). 

9.  (x2+a;//+7/2)3  +  (a;2-a;2/  +  2/2)3--2(a;34-//2). 

10.  (7  +  ?')^-(«-^)^ -^2i(Sr/3  +  Z;2). 

II.  (-.«-'  +  5ix-+/>2)'+(a;2-Z>u;  +  ^2)T^2(a;+i)». 

12.  (a+6)'i»+--^+(«-6)4"+2--  2(^(3 _|_ia). 

13.  {a;3  +  3a^?/(a;-?/)-y3i3  +  {a;3-9a://(a;-v/)-2/3}3^2(x-v/)3. 

14.  3j;a-5x2+4a;-2-r-a;-l. 


12  THEORY    OP   Dn'ISORS. 

15.  Any  polynome  in  x  is  divisible  by  a;— 1  when  the  sum  oi 
the  coefficients  of  the  terms  is  zero. 

16.  Any  polynome  in  x  is  divisible  by  ic+1,  when  the  sum 
of  the  coefficients  of  the  even  powers  of  x  is  equal  to  the  sum  of 
the  coefficients  of  the  odd  powers.  (Tiie  constant  term  is  in- 
cluded among  the  coefficients  of  the  even  powers). 

Examples. — Cor.  2. 

1.  Show  that  a(a+2Z>)3  —h{^a-\-hY  is  exactly  divisible  bv  (i-f-fc. 
By  Cor.  2,  the  substitution  of  —6  for  a  must  cause  the  polynome 
to  vanish. 

Substituting;  o(a-2a)3+a(2.'?-fl)3=  -a^^a^  =  0. 

2.  Show  that  {ah  —  xy^^^  —  {ci-\-h  —  x  —  y)\ah{x-\-y)  —  X]j{a-^}))\  it. 
xa,ctly  divisible  by  {x  —  a)(T/  —  a),  also  by  [x  —  h){y-b). 

For  X  substitute  a  and  the  expression  becomes 

{ah-ayY-{b-y){ah{a-\-y)-ay{a-!rb)}^ 
aHb-y)^  -{b-y){a^h-y)]  =0. 

The  expression  is,  therefore,  exactly  divisible  by  a;— a.  But  it 
is  symmetrical  with  respect  to  x  and  y,  hence  it  is  divisible  by 
y—a,  and  as  aj  — a  and  j^  — «  are  independent  factors,  the  expree- 
sion  is  exactly  divisible  by  {x  —  a){y  —  a).  Again,  the  given 
expression  is  symmetrical  with  respect  to  a  and  b,  hence,  making 
the  interchange  of  a  and  b,  the  expression  is  seen  to  be  divisible 
hj{x-b){y-h). 

3.  Show  that  %{a^+b^+c^)-b{a^+b^-^c^){a»  ^-b^  +  c*)  is 
exactly  divisible  by  a+fe+c. 

For  a  substitute  —  (6+c)  and  the  result  which  would  .be  the 
remainder  were  the  division  actually  performed,  must  vanish. 

(3|_(64-c)^+65+c5}_5{-(&+c)3+63+c3||(J^c)?^J3+c9} 

=  6|_(ft+c)s  +  6^+c^}+306c(6+c)(62+6c  +  c2).  See [1] and [6] . 

The  expansion  being  of  the  6tli  degree,  and  symmetrical  in  h 
and  c,  it  will  be  sufficient  to  show  that  the  coefficients  oih^,  h^c^ 
b^c*  vanish,  the  coefficients  of  b^c^,  be*,  c*  being  the  coefficients 


THK(mY    Of    DIVISORS.  43 

of  tbe  foi-mer  terras  in  reverse  order.     Calculating  the  coefficients 
of  tiiese  type-terms  we  get 

6{_5/;4c-1063c2.-...}+30(64f  +  263c3-l-...), 

which  evidently  vanishes.     Hence  the  truth  of  the  proposition. 

In  the  last  example  it  lias  been  proved  that  the  dilference  of  the 
(quantities  here  declared  to  be  equal,  is  a  multiple  of  a  +  6+c,  i.e., 
in  this  case,  a  multiple  of  zero.  Hence  under  the  given  condition 
they  are  equal. 

Exercise  xix. 

Prove  that  the  following  are  cases  of  exact  division  : 

1.  (ax-b7jy  +  {bx-ay)^—{a^+b^){x^-y^)^a,b,x,  y,  a-\-\ 
x-y. 

2.  ax^  —  (a2  -f  6)a;3  +52  ^  ox—b.     (Substitute  ax  for  b.) 

o     \  (ax+by)^  -  {a-  b){x+z){ax+b7j)  +  {a-b)^xz  -^  x+y. 
^'  \  {ax-by)^  -[a  +  b){x-{-z)(ax-by)  +  {a  +  b)^xz^  x  +  y. 

4.  da^x^—iax^  —  l'^^axy-~oa'^xy  +  2x^y-{'Oy^  -i-2ax-y, 

5.  l-2a*x-16-22a^x^  +^-8a^x^ -{--dax^  - x^  -^  •Gax^2x^. 

6.  x^+x^y^+x^y+y^  ■^x'^^y. 

7.  {o-d)a^  +  6{bc-bd)a+9{b^c-b2d)  -^  a  +  3b. 

8.  itix-^^yy+yi^^r-^-yy  -'^x-y. 

9.  a(a+26)3-6(6  +  2a)3  -^a-h,  ;dso  -=-  a+b. 

10.  a5-2rt4^)+a363+a3a;3_2rtaa;3+63a;3  -^  (a-h)(x-ha). 

11.  a(6-cj3+6(c-a)3-f  c(a-6)3  --  (a-b),  {b-c),  {c-a). 

12.  a^{b-c)  +  b^{c-a)  +  c^{a-b)  -r  (a-b),  {b-c),  (c-a). 

13.  a*(6-c)  +  6*(c-a}  +  c4(a-6)  -^  (a-b),  (b-c),  (c-a). 

14.  (a-6)2(c-d)3  +  (6-c)3(cZ-a)2-(rf-6)3(a-.)2  ~  (a-b), 
{b-c),  {c-d),  {d-a). 

15.  {{a-b)^+{b-cy'  +  {«-a)-'}{{a-b)^rJ  +  (b-c)^a^  + 
(c-a)363}-{(a-6)3c-i-(6-c)2a  +  (c-a)2i}3    ^   (a_i),   (i-c), 

(e—a). 

16.  (:s+t/)(t/+s)(2+a;)+a;2/z-ra:+y+a. 


44 


THEOKY    OF    DIVISOKS. 


18.  (cib  -  bc-ca)l-a^b^-h2c2-c^u^-.a-\-b-c. 

19.  («  +  26)3  +  (26-.3c)3-(3c-a)3+a34.s^3„27cS  + 
+2^»-3r. 

20.  a^b^-hb^c3^cSa3  -Sa^li^c^~<ib^bc-Jrca. 

Examples. — Cors.  3  and  2. 

1.  Find  the  value  of  Ax'"  -^-^x^  -  ox^  +  23a;+6  when  1x^  =  3ar  -  4. 

Since  2a;2  —  3a; -{-4  =  0,  we  have  simply  to  find  the  remainder  on 
division  by  'Ix^-'Sx+A,  and  if  it  is  iudependent  of  «;,  it  as  th( 
value  sought.  Cor,  3. 


4 

0 

9 

—  5 

23 

6 

3 

6 

9 

15 

-3 

-4 

-8 

-12 

-20 

4 

2         3  5      -    1;       0       10 

Hence  the  required  value  is  10. 

2.  What  value  of  c  will  make  x^  —  ox^  +7a;  —  c  exactly  divisible 
by  a;— 2. 

If  2  be  substituted  for  x,  the  remainder  must  vanish.  Cor  ^. 

1-0  7-6- 

2-6         2 


1;  2-c 


1     -3 

Hence  2  — c=:0,  or  c  =  2. 

3.  What  value  of  c  will  make  6x^  —  5x*+cx^—20x''^19x-5 
vanish  when  2a;2=3a;_i  ? 

By  Cor.  8,  the  remainder  must  vanish  when  the  given  poly- 
nome  is  divide  by  2x^—  dx+1.  We  may  divide  at  once  and  find, 
if  possible,  a  value  of  c  that  will  make  both  terms  of  the  remainder 
vanish,  or  we  may  fii'st  express  cx^  in  lower  terms  in  z,  and 
then  divide  and  find  the  required  value  of  c  from  the  remainder. 

1st.  Method,  (see  page  28), 

6-10  Ac       -160  304  -160 

3  18         24     12C+36      ,S6c-420 


-2 


-16       -8c -24 


8  4c+i2  12c -140;    28c- 140 


24c -{-280 
•24C+12U 


THEORY    OF    DIVISOKS.  45 

Hence  28c  =  140  and  24c  =  120.     Both  of  these  are  satisfied  by 

2nd  Method,     x^  =  ix{Sx  -1)=  ^x^  -  ix  =  f  (3.t-  1)  -^x  = 
2^a;-f  —  ia;=l|a;-i  ;  .-.  cx^  =  IJca;— ^c. 

Substituting  for  c.v^  in  the  given  polynome  it  becomes 

6x^-5x*-20x^  +  [lic  +  l9)x-ic-5. 
Divide  and  apply  Cor,  3. 

6  -10  0  -160  28c +  304    -24c- 160 

3  18  24  36  -420 

_2  -12  -16  -   24  280 

6  8  12  =ri"40;  28c -140  "^^24^+120 

We  thus  obtain  the  same  remainder  as  by  the  former  method, 
and  consequently  the  same  result.  A  comparison  of  the  two 
methods  shews  that  they  are  but  slightly  different  in  form,  but 
the  second  method  shows  rather  more  cleaiiy  tliat  c  need  not  be 
introduced  into  the  dividend  at  all,  but  the  proper  multiples  of  it 
found  by  the  preliminary  reduction  can  be  added  to  or  taken 
from  the  numerical  remainder,  and  the  "true  remainder"  be 
thus  found,  and  c  determined  from  it. 

Exercise  xx. 

Find  the  value  of 

1.  x* -3a;3  +  4a;2  —  3a;+4,  given  x2  =a;_i. 

2.  x^-2x^-4x^  +  13x^-nx-10,  given  (a;-l)3  =  2. 

3.  2x'' -lx'^-{-l2x^-llx"  +  '2x~5  given  (x-l)2+2==0. 

4.  3a;«+lla:5+10.'c3+7a;2+2a;  +  8  given  a;?  + Sa:^ -2a;T5  =  0. 

'    5.  6a;'+9a;*5 -16a;* -0x3- 12a;3_ 6a; +  60  given  3a;4+a;— 4  =  0. 
What  values  of  c  will  make  the  following  polynomes  vanish 
under  the  given  conditions. 

6.  a;*  +  13a;3  +  26a;2+52a;  +  8c,  given  a;+ll=0. 

7.  a;4-2a;3— 9a;2+2ra;-14,  given  3a;+7  =  0. 

8.  a;*  -  4a;3  -  a;3  + 16a;  +  6c,  given  a;3=  a; +6. 

9.  2a;*-10x2+4ea;+6,  given  a;2  +  3  =  3x. 

10.  2a;4+a;3-7cx2  +  lla;+10,  given  2a;  =  o. 


46  THBORy    OF    DIVISORS. 

11.  4.1-4 +  ra;2-f- 110a;- 105,  given  2.t3  -   5a;-}- 15  =  0. 

12.  Sx^-lijx^-+cx--i-5x^-lUx+200,  giveu  x^  =  3x-4. 

13.  What  values  of /_>  aud  q  \f ill  make  x'^  +  ix^ -lOx^ -j'x+q 
vanish,  given  x^  =  3(a;  —  1)  ? 

14.  What  vahies  of  p  and  ^y  will  make  a^^  -5a^"  +  lOrt »  -  15rt  '• 
+29a4  -/^rt2  +,^  vanish,  given  (a^  -2)2  .=a2  _3  '? 

Theorem  II.     If  the  i^olynome /(a;)"  vanish  on  substituting 
for  a;  each  of  the  71  (different)  values  a-^,  a^,  a^     .     .     .     .     a„ 

f{xY  =  A{x  —  a^){x-a^){x~a^)     ....     (a;  — flj 
in  which  A  is  independent  of  x  aud  consequently  '6  the  coefScient 
of  a;"  in /(a;)". 

Cor.  If  /(a;)"  aud  9(3;)"*  both  vanish  for  the  same  m  different 
values  of  x,  f{xY  is  algebraically  divisible  by  <p{x)'^. 

Examples. 

1.  x^+ax^+bx+c  will  vanish  if  2,  or  3,  or  —4  be  substituted 
for  X,  determine  a,  h,  c. 

The  coefficient  of  the  highest  power  of  a;  is  1  ; 

.-.    a;-'^+(/a;^+6a;+c=(a;-2)(a;-3)(a;  +  4)=.a;3  -x-3  -  14a;+24. 

.-.    a=  ~  i:  b=  -14:  <;  =  24. 

2.  x^+bx^+cx-\-d  will  vanish  if  —3  or  2,  or  5  be  substituted 
for  X,  determine  its  value  if  3  be  substituted  for  z. 

The  given  polynome  =(x+3)(a;  — 2)(a;  — 5) ; 

.-.  the  required  value  is  (3  +  3)(3-2)(3- 5)  =  -12. 

3.  (ta;3+3/;a;2  +  3ca;4-rf  will  vanish  if  for  a;  be  substituted  —3, 
or  I;,  or  1^,  but  it  becomes  45  if  for  x  there  be  substituted  3  ; 
determine  the  values  of  a,  b,  c,  d. 

The  coefficient  of  the  highest  power  of  a;  is  « ; 

.-.    aa;3  +  36.r2  +  3ra;  +  ^Z  =  rt(a;+3)(a;-i)(a;-l^) 
.-.    a(3  +  3)(3-i)(3-U)  =  45;      .-.    a  =  2. 
.'.    2a;3  +  3/'x2  +  3ca;  +  <.'  =  2(a;  +  3)(a;-i)(a;-  li) 
...  b  =  ^,  c=  -3i,  d=  4i 


THEORY    OF    DIVISORS.  47 

4.  Ir'  x^  -^j'X-  -\-(]x-{-r  vanish  for  x^a  or  b,  or  c,  determine  p,  q, 
and  r  in  terms  of  a,  b,  c. 

x"  -i-px-  +qx-\-r={x  —  a){x  -  b){x  -  c) 

=  x^  —  (a+b  +  c)x'^  -\-{ab  +  br  +  <-Ci)x  —  abc 
:.  p=  — (rt-f /)  +  c)       or  —2". 
q=       ab  +  bc-\-ca  or        ^nh 
r  =  —  abc  or  —  2  "^c. 

5.  U  x^  +px^  -{-qx+r  vanish  for  x=n,  or  b,  or  c,  determine  the 
jiolyaome  that  will  vanish  for  x  =  b  +  c,  or  c  +  a,  or  a  +  b. 

Since  x^-{-2)x^  +qx+r  vanishes  for  x  =  n  or  I)  or  c, 

a;3  —px'^-{-qx  —  r  will  vanish  for  x=  —a  or  —6  or  —  c, 

and  —  /'  -—  a-\-b+c; 
But  the  required  poljmome  will  vanish  for 

x=  —p—a,  or  —p  —  h,  or  —  p— c; 
that  is,  for  x-\-p=   -a,  or  —ft.  or  —  c. 
Hence  it  is         (x+p)^  ~p{x+p)^+q{x+p)—r  = 
x^  +'lpx^  -\-{j}^  +  q)x-{-pq  —r. 

The  following  is  the  aalculation  in  the  last  reduction.     (See 
page  31). 


-p 

9 

—  r 

p 

0 

? ; 

2^q-r 

p 

p; 

p^  +  q 

p 

1     • 

2p 

6.  In  any  triangle,  the  square  of  the  area  expressed  in  terms  of 
the  lengths  of  the  sides,  is  a  polyuome  of  four  dimensions ;  and 
the  area  of  the  triangle,  the  lengths  of  whose  sides  are  3,  4,  and 
5,  respectively,  is  6.  Find  the  polynome  expressing  the  square 
of  the  area. 

Let  a,  b,  and  c  be  the  lengths  of  the  sides,  and  A  the  required 
polynome. 

1st.  The  area  vanishes  if  any  two  of  the  sides  become  together 
equal  to  the  third  side,  hence  ii  a  +  b  =  c,  A  =  0,  and  consequently 
A  IS  divisible  hy  a -\-b  —  c.  Similarly  it  is  divisible  by  b-\-c-a 
and  bv  c+a  —  b. 


48  THEOKY    OF    DIVISORS. 

2ncl.  Tha  area  vanishes  if  the  three  sides  vanish  together, 
hence  if  a-{-b-}-i-  =  0,  .-1  =  0,  and  consequently  A  is  divisible  by 
a-i-h  +  c. 

We  have  .thus  found  four  linear  factors,  but  A  is  of  only  four 
dimensions. 

.-.  A  =  vi(a  +  b  +  c){b4-c-a){c+a-b){a+b-c), 
in  which  w  is  a  numerical  constant. 
But  63  or  3G  =  (»(3+4+'5)(4-f  5-3)(5  +  3-4)(3+4-5) 
=  576;;/ ;     .•.  m  =  ^^. 

(The  above  includes  all  the  ways  in  which  the  area  of  a  triangle 
can  vanish,  for  the  vanishing  of  only  one  side  involves  the  equal- 
ity of  the  other  two,  or  if  a  =  0,  b  =  c,  and  .*.  a-\-h  —  c,  which  is 
included  in  1st.  ;  if  two  sides  vanish  simultaneously,  the  three 
must  vanish). 

Examples  on  the  Corollaky. 

7.  Prove  that  (x+l)'^ —j?' ^ —2«- 1  is  divisible  by 

2x3  +  dx-+x. 
Factoring  the  latter  expression  we  find  it  vanishes  for  x  —  0,  or 
—  1  or  —  ^.     Substituting  these  values  in  the  former  polynome, 
it  also  vanishes.     But  these  are  different  values  of  x,  hence  the 
truth  of  the  proposition. 

8.  (x-f  ?/+ 2)"'  -x^  —  y^—z^  is  divisible  by 

{x-\-}j+z)^  —  x^  —  y^  —z^. 

The  latter  expression  vanishes  if  a;=  —y,  so  also  does  the  former. 

By  symmetry  they  both  vanish  if  ?/=  —  2  and  ii  z=—x.  Hence 
they  are  both  divisible  by  (x-\-y){y+z){z+x).  But  this  expres- 
sion is  of  three  dimensions,  as  also  is  the  latter  of  the  given  poly- 
nomes,  hence  it  is  a  divisor  of  the  former. 

9.  Prove  that  { 0/4-6)^ -f(c  +  ri) 5}  (rt- 6) (c-./) + 

{{b+cy'  +  {a  +  dy}{b~c)[a-d)  +  {{b  +  dy+{c-{-ay}{b-d)(c-a) 
is  algebraically  divisible  by  {a--b)(c  —  d)[b  —  c)[a  ~d){b  —  d){c  —  a) 
X  («  +  6  +  f  +  '0>  ^^^^  ^^^  ^^1^  quotient. 

Let  a  =  b  and  tlie  former  polynome  reduces  to 
[{a+cy+{'i-^dy'}{a-c){a-d)  +  {(a+d)-''-[.{c-\-n)^\{a-d){c-a) 


THEORY    OF    DIVISORS. 


49 


which  vanishes,  the  second  complex  term  differing  from  the  first 
only  in  the  sign  of  one  factor,  having  (c  —  a)  instead  of  {a  -  c). 

Hence  the  former  polynome  is  divisible  by  a  —  b,  and  by  sym- 
metry it  is  also  divisible  by  a  —  c,hya  —  d,hjb  —  c,hyb  —  d,  by  c  —  d. 

Again,  {a  +  b)^ +{c-{-d)^  is  divisible  by  (a +  h)  +  {r -r  d) ;  for,  on 
puttinga+6= -(c-t-f/),  it  becomes  {—{<■+<! )]■'■ +{c  +  d)^  which 
=  0. 

Similarly  the  other  terras  of  the  former  of  the  given  polnomes 
are  each  divisible  by  a-^b-\-G+d,  and  consequently  the  whole  is 
so  divisible. 

Now  all  these  factors  are  different  from  each  other,  hence  the 
former  of  the  given  polyuomes  is  divisible  by  the  product  of  these 
factors,  i.e.,  by  the  latter  of  the  given  polynomes. 

Both  of  these  polynomes  are  of  seven  dimensions,  hence  their 
quotient  must  be  a  number,  the  same  for  all  values  of  a,  b,  c,  d. 

Put.<  =  2,  6=1,  c  =  0,  d=-l,  and  divide.  The  quotient  will 
be  found  to  be  —5. 

...    i^(a  +  hy'-{.{c  +  d)^}{a-b){<,--d)-\-{{h  +  r)^  +  {a  +  d)^}  X 
{h_.c){a-d)  +  {{h-^d)-''+{c  +  a)^}{l>-d){c-o)=  -o{a-b]{c-d) 
X{b-c){a-d)il'-d){c~a}{a -{-!>-{-,,• -h'l). 

N.B. — It  is  not  always  necessary  to  find  the  factors 
of  the  divisor,  as  the  following  examples  show. 

10.  Prove  that  x^  +x+l  is  a  factor  of  3;^  4+j-''  +  1. 

.f  2  4-x+l  will  be  a  factor  o?  z^'^-\-x''  +1  provided 

a;i  4  _}.a;7  _f.  1  =  0  if  x2 +X  + 1  =  0. 

Ifa;2+a;+l        =0 

.-.  x^+x-^x        =0 

.-.  x^+x--{-x+l  =  l 

.'.    «3  =1 

..    .*  =  landa;i2  =  l 

x^  =x  and  x'^'^  =x'^ 
..  ^i4+a;'+l       =.x^+x  +  l  =  ^ 
,■ ,  x'^ 4-a;-f  1  is  a  factor  of  x^  *  +j;'  -{-1 . 


50  THEORY    Od"    divisors. 

Art.  XII.     Two  other  methods  of  proving  this  proposition 
are  worthy  of  notice, 

1st.  x^+x  +  1  will  be  a  factor  of  x^^-\-x''  +1  provided  it  is  a 
factor  of  {{x'^'^+x''  +1)  ±  a  multiple  of  {x^+x  +  l)\. 

x''-  "^  -\-x'  -{-I  differs  by  a  multiple  of  x'^  +x-4-l  from 
x^^+x^'{x^-+x  +  l)+x^{x''+x-hl)-\-x'+x'^{x'-^+x  +  l)  + 
x[x'^+x  +  l)  +  l 

=  x^-'[x^  +x-rl)+x^{x^-\-x-\-  i)-\-x'-{x-^-\-x-{-l)^  x^[x-'  +x-rl)  -{- 

(x^+x+l) 
=  {x^^+x^+x''+x^-\-l){x-^-tx-^l). 

Hence  x^+x  +  1  is  a  factor  of  «*'*+.<;• -fl. 
2nd. —   =    . — 

(x^i-l){[xi^-l)-x{x^^-'l)} 

(a;^^-l)(a;'^'-l)    _    x{x^^ -l){x^i -I) 
{x^-l}{x^^  {x^-l}{x'^'^^; 

Bitt  we  see  at  once  that  on  reduction  both  of  these  fractions 
give  an  integral  quotient,  hence  {x^'*^-\-x''  -\-l)  ~x^ -{-x  +  l  gives 
an  integral  quotient. 

11.  x^-\-x+l  is  a  factorof  (x  +  1)''  -x^  -1. 

If  a;-+.«  +  l  =  0,  {x  +  iy  -x''  —  1  will  vanish  also,  for  in  such 
case  aj+l  =  —x^. 

.-.    {x-\-iy  -X'  -1  =  {-X^)'  -X'  -1=  -x-14_^7  _l^ 

which  by  the  last  example  vanishes  if  x''-^-\-x+  1=1); 
.-.  a;-+.i  +  l  is  a  factor  of  [x-[-iy  —x''      1. 

For  X  substitute   —  and   multiply   by  i/^   and  ?/^   respective! v 

y 

a.nd  this  example  becomes 

^^-\-X!J+i/^  is  a  factor  of  {x+y)''  —x''  —i/'. 


THEORY    OF    DIVISORS.  61 

Exercise  xxi. 
Determine  tlie  values  of  n,  b,  c,  d,  e,  iu  the  lollowing  cases : — 

1.  a;3  +  36.u2  4-3(M—|-// vanishes  for  x  =  1,  or  3,  or  4. 

2.  x*^  -it-cx^  +dx  +  e  "         "    a;=l^or  —3  or  4^. 
^.  x^+hx-+cx  +  'i^  "         "     r/;  =  2or-3. 

4.  .^6-">-f/;a;2+rx+90.        "         "    a;  =  3or-5or2. 

5.  ax^+>-jyi  -?>Qx  +  e.        "         "    x=  1|- or  -4,  or  2^. 
Q.   Qlx'^  +  iScx'+Ux  +  e     "         "    a;=l|  or  -81- or  1^. 

I.  iix'^+1>x^-\-cx'^-'dl       "         "    .^•  =  for|or3. 

8.  ax'^+cx^  +  ilx+e  ♦*         "    x  =  2  or  1^  or    -1   and  be- 
comes 14  for  x=l. 

9.  ax^-\-cx-\-d  vanishes  for  x=l\.,  or  2|,  and  becomes  49  for 
aj  =  3,  determine  its  value  for  x=  —Q. 

Given  that  x^  -  px^  +qx  —  r  vanishes  for  x  =  a,  or  b,  or  c,  deter- 
termine  the  polj'nome  that  vanishes  for 

10.  x  =  tt-[-l,  ovb  +  1,  or  t'  +  l. 

II.  .f  =  a  — 1,  or  i— 1,  or  c— 1. 

Ill 

12.  x=l  -  — ,  or  1  — -T",  or  1  — — . 

13.  x  =  ((b,  or  be,  or  ca. 

lA.  x  =  a^,  or  b^,  or  c^. 

(  r  ) 

15.  x  =  a{b  +  c),  or  b(e  +  a),  or  c(a+b).  Ui{b-\-c)  =  q 1. 

(  "  ) 


a4-b        b4-c        c-\-a  («+^ 

16.  a;-     —  or    — -  or  -f— .  '    ^ 

c  a  b 


Prove  that  the  following  are  cases  of  exact  division : 

17.  (a;-l)i3_a;G4-(a;2-x-i-l)2  -x^ -2:k^ +2x- 1. 

18.  (x-l)i«--a;«  +  (:^2  -a;+l)«^a;3-2a;2  +  2.c-l. 

19.  (a;-2)i"(2a;-5)'0-a;i"  +  2iO(x2-4a;  +  5)5^ 

a;3_6«2  +  13x-10. 

20.  (x2  +  4.«  +  8)i«-ic'^-3;--5x-3-a;3  +  Ga;2+8.f-  +  3. 

21.  (9x-4)2  i  (a;-  1)2  '-•a;2  i  -  {^x"-  -  14a;+4)2  ^  ^(;c_  i)  x 
(9x-4)(9a;'^-Ma;  +  4). 

22.  {a(.'K-l)f '3_(.2a;3  +  3a;-4)'3  +  (2uj2_3;c  +  2)i3^ 
(2a;2 +3x  -  4)(2a;2  -  3a;  +  2)(a;- 1). 


52  THEORY    OF    DIVISORS, 

23.  {2(x+l)(x-2)}'^''-\-{x^-Sx+dy^-(3x^-ox-iy-^ 
{X'^lrx-2){x^  -Sx+3){dx^  -5x^1). 

24.  {6(a;-l)]i6-(2a;3+3x-4)i«-(2x2— 3.c+2)i«  + 
2{2x^+Sx-A)^2x^  -'3x+2)^-^{x-l)(2x^+dx-A)[2x'^  -Sx+2) 

25.  {2(a;+l)(a;-  2)[ 20  _  ,,.3  _Sx+B)^'' -('Sx^  -  5x-  1^  + 
2(x^'  -3x  +  d)\3x^-5x-iy^'^{x+l){x-2){x^  -3x-^'3)  x 
(3a;2-5a;-l). 

26.  l+x'^+x^  -^  l-f.r4-a;8. 

27.  a;io +0:5^5  +  //!  0  -^  a;2+a;?/  +  ?/2. 

28.  l  +  a;3+a;«+a;9+x'2  -^  l  +  x+.t:2+«3*+a:4. 

29.  l4-x4+ic«+.fi24-a;ie  -^  i+x+x^+x^+x"^. 
80.  a;i^+s;iO//-5+a;5iyio^7is  ^  ^3_|.;^2_,^_j.a.^2_}_y3; 

31.  x^'  +x'>'+x^-\-x+l  ^  x^+x^+x^+x  +  1. 

32.  l+x+x'+x^+x^'+x^'+x^^  ^ 

l+x+x^-\-x'^+.c'^+x'-+x^. 

Find  tbe  quotient  of  the  following  divisions  in  which  D  denotes 
the  product 

{b-c){c-a)ia-b)(a-ri){b-d)ic-d)  ; 

33.  (62c2  +  «V2)(6-c)(a-(/)  +  (c3a2  4.62(Z3)(c_a)(6-rf)4- 
{a^b^  +c^d'-'){a-b){c-d)  -f-  D. 

34.  (fcc+ad)(62  _c2)(«2_^3)+(ca  +  6cZ)(c2-a2){62— d2)  + 
(ab+cd){a^-b^){c^-d^)  H-  D. 

35.  (i  +  c)(a+'Z)(i2_c2)(fl2_t?2)^the  two  similar  terms  -=-  D. 
86.   (/>2+c2)(a2+r/2)(i-c)(a_</)+  ".  '*  -i-  D. 

37.  {bc{b  +  c)^+ad{a  +  d)^}{b-c){a-d)+  "          -^  D. 

38.  {bc{b  +  c)+ad{a  +  d)}{b^-c^){a^-d^]+  "           -7- i*. 

39.  {bc(b^+c^)+ad{a^-{-d^)}{b-c){a-d)-{-  "          -^7). 

40.  (Z;+c-a-rf)^(&-c)(«-(/)+  "          -^  D. 

41.  The  sum  of  the  fractions  |,  |,  i, \,  increased  by  the 

sum  of  their  products  two  by  two,  increased  by  the  sum  of  their 

products  three  by  three, increased  by  their  product  is 

equal  to  n. 


^3 


THEORT    OF    DIVISORS.  O 

42.  Ill  any  trapezium  the  square  of  the  area  expressed  in  terras 
of  the  lengths  of  the  parallel  sides  and  the  diagonals,  is  a  poly- 
nome  of  foiu*  dimensions,  determine  that  polynome. 

43.  In  any  quadrilateral  inscribed  in  a  circle,  the  square  of  the 
area  expressed  in  terms  of  the  lengths  of  the  sides,  is  a  polynome 
of  four  dimensions,  find  that  polynome. 

Theorem  III.  If  the  polynome /(a:)"  vanish  for  more  than 
n  different  values  of  x,  it  vanishes  identically,  the  coefficient  of 
every  term  being  zero. 

Cor.  If  a  rational  integral  expression  of  n  dimensions  be  divi- 
sible by  more  than  n  linear  factors,  the  expression  is  identically 
zero. 

Examples. 

{x-a){x-h)  (x-b){x-c)         {z-c){:v-u) 

1-  ^c_a)(c-/>)  +  {a-b){a-c)  "^  (b-c){b-a)  "r^-"'  ^^  «' 
b,  and  c  are  unequal ;  for  this  is  a  j^olynome  of  two  dimensions  in 
X,  but  it  vanishes  for  x  =  a,  aud,  therefore,  by  symmetry  for  x=b, 
and  for  a;  =  c,  that  is,  for  three  different  values  of  x,  hence  it 
vanishes  identically. 

2.   \{a  +  h)^-h{'-\-<l)^](^-b){c-d)  +  {(c-hh)2  +  {h  +  d)^ 
{b-c){a-d)  +  {{c-{-'()-+(''  +  dy}{c-a){b-d)  =  0. 

Substitute  b  for  a  p.nd  the  expression  becomes 

{{b+cy^Mb+dr^}{b-~c){b-d)-h{{c+by'  +  {b+dy\[c-b){b-d) 

which  vanishes,  hence  the  given  expression  is  divisible  by  a— 6, 
and  consequently  by  symmetry  it  is  divisible  by  («—?;),  (b-c), 
(^c  —  d),  {a-c),  (b  —  d),  and  (a  —  d),  But  the  given  expression  isof 
only  four  ditaensions,  while  it  appears  to  have  six  linear  factors, 
hence  it  vanishes  identically. 

Exercise  xxii. 

Verify  the  following : 


54 


THEORY    OF    nwiSOBS, 


1 


{x+a)[x-\-h){x^-c)' 

Q     o+x a-\-y a-]-z ^     a^ 

x{x-y)(x—z)   "^   y{v-x) {y  - z)         z{z -  x)  {z  -  y)    ~   xyz 


a'{h-c)+b-[c-a]+c-{a-h) 
8.   {iiclf+h.-f-\-he(l-acc)^-\-{hce-\-aed-^acf—hilj)-   =3 

{a-b){i,-c){c~<i)  ~ 

10.  (-a:+?/+z)(x  — 7/+2)(a:+y— ■•  -^xix  —  y+z^ix-Vy-z)-^ 
il{x+y-z){-x  +  y+z)+z{-x  +  y\-::)\x-y-\-z)  =  'ixyz. 

(a3-/;S)3-|-(/;3-c2)3_|_^C^_^2)3 

•  (a-+T)(6+c)(c+a)       ""     ;    - 

(a-i)3  +  (/y-c)3  +  (c-a)3,. 

12.  x-{y  +  zY-{-y-[z^xy'-^z^{x+yY+2xyz{x+y-\-z)  = 
2{xy+yz-\-zx)-. 

Theorem  IV,  If  the  polynomes  /(a;)",  f  (a;)'"  (n  not  less  than 
m)  are  equal  for  move  than  n  different  vAliics  of  x,  they  are  equal 
for  a//  values,  and  the  coefficients  of  equal  powers  of  x  in  each 
are  equal  to  one  another. 


THEORY    OF    DIVISORS.  55 

(This  is  called  the  Principle  of  Indeterminate  Coefficients,  The 
full  use  of  it  cannot  be  exhibited  till  the  student  is  able  to  work 
simultaneous  equations.) 

Examples. 


+    71— :^7T— a71— ,T>     + 


{a-h){a-c){a-d)    ^   {b-a){b- c){b- d) 

{G-a){G-b){c-d)    "*"    (7i"(iy(7Z-6)(rf^j    ^    ^' 
Assume 


{x-a){x  —  b)(x-c){x—d) 

A                B                 CD  ,   > 
j^   . I 1. (a) 

x  —  a  x-b  x—c  x—d 

in  •which  A,  i>,  C,  D  are  independent  of  x. 

Mnltiplv  by  {x-a){;x-b){x-c){x-d). 

:.   x^  =  [A 4- B-{-C-\- L')a;2 +terms  m  lower  powers  of  x. 

Now  this  equality  holds  for  more  than  three  values  of  x,  hold- 
ing in  fact  for  all  finite  values  of  x. 

Again  multiply  both  sifes  of  (or)  by  a;—;6 

x^  ,,    I    B  C  D    \  ,        . 


(x  —  b){x  —  c){x  —  d)  \x  —  b  x  —  c  x  —  d 

Put  x  =  « 


(a  —  b){a  —  c){a  -d) 


By  symmetry      — ^  =   b,  &c. 

{u—,a)[b  —  c)[b  —  d) 


Adding 


«2  62  c2 

+     7Y. :Y7, x-71 T^     + 


{a-b)\^a-c){a-d)    ^    {b-a){b- c)[b -d)    ^    {c  —  a){c-b){c-d) 

+  id^:aji^b)ld-c)    =    A^B+C+D  =  Ohyi&). 

2     a^(a  +  b)ia+c)  b^{h+c)ib+a)         c^{c+a){c  +  b) 

{a-b){a-c)      "^     (b-c){b-a)     "^     (c-aXc-6) 
;=(a-f  6  +  c)-, 


58 


THEOHY    OF    DIVI90ES. 


17     a(^  +  h){a+c){a  +  d)  ■     -,      . 

*■ '  •    7 7w w T\  +  three  similar  terms* 

a-(a+b)(a+c){a^d)  ^^  ^^ 

•'°'      {a-b){a-c)(a-d)     "*" 

aS(a+b)(a+c){a  +  d)  ^^ 

^^-      {a-b)[a-c){a-d)     "^ 

bc{b-\-c) 

20.  , Y^,    -     ,  +two  similar  terms. 

[For  numerator  use  x^-\-2px--\-ip^-^q^x-\-('jiq  -  r).] 

(2a+6:(2a  +  c)  .       ' 

21.  -7 fw r-  +  two  similar  terms. 

(a— f>)(a— c; 

[For  numerator  use  x^  —'lpx^^^^(J)^■\^q)x  —  (yq  —  r^^ 


+  two  similar  terms. 


"  {(.t  —  b){a—c) 

[For  numerator  use  a;(a;  +  ;j).] 
h-\-c.-\-d  ■ 

23.    7 Tw \7 j\  +  three  similar  terms. 

{a  —  o){a  —  c){a —a) 

a^(hc-\-cd  +  db) 

"^-    (a-6)(a-c)(a-rf)  ^ 

hc-i^cd-\-db 

"^^^    (fl--6)(a-c)(a-J)  ^ 

Extract  the  square-root  of  (to  4  terms) :  » 

26.  l+x.       .1     27.  1-a;.  I     28.  l  +  2.^+3a;*+4z3 -t  &c. 

29.  l-4a;+10x2-20a;3  +  35a;4-56ic5+84a;«. 

30.  Extract  the  cube-root  oil+x.     (To  4  terms). 

Art.  XI.  1.  Find  the  condition  that  px^'  +  ^qx+r  andi  p'x^ 
+  2q'x+r'  shall  have  a  common  factor. 

Multiply  the  polynomes  hj  p'  and  p  respectively,  and  take  the 
difference  of  the  products,  also  by  r'  and  r  respectively,  and 
divide  the  difference  of  the  products  by  x. 


p  'px^  -{-  2p  'qx 4-/J  'r 
pp'x^  4-2pq'x+pr' 


2(  pq'—p'q)x+{pr'  —p'r) 


p7-'x^  +2qr'x-\-7r' 
p'rx^  +  2q'rx-{-r'r 


(J)r'-p'r)x  +  2{qr'  -r'q). 


Multiply  the  former  of  these  remainders  by  (pr'—p'r)  and  th$ 
latter  by  ^{pq'—p'q),  ^'^'^  the  difference  of  the  products  is 
^^r'-p'r)^-^pq'-p'q){(p--r'q). 


THEORY    OF    DIVISORS.  C9 

But  if  the  given  polynomes  have  a  linear  factor  this  remainder 
must  vanish,  or 

ipr'  —p'r)^  =4:(pq' —p'g){qr'  —  r'q). 
If  the  given  polynomes  have  a  quadratic  factor,  the  linear  re- 
mainders must  vanish  identically,  or  (Th.  III.) 

pq'—p'q  —  O,  pr'  —p'r  =  Q,  and  qr< — r'q  =  0, 

par 
ox,  ±-   =    J-   =    — 

p'  q'  r' 

2.  Find  the  condition  that  px^  -\-3qx^  +Srx-^s  shall  have  a 
square  factor. 

Assume  the  square  factor  to  be  (x  —  a)^.  On  division,  the 
remainder  must  be  zero  for  every  finite  value  of  x,-  and  conse- 
quently (Th.  III.)  the  co-efficient  of  each  term  of  the  remainder 
must  be  zero.     Divide  by  (x  — a)^,  neglecting  the  first  remamder. 

P  Sq  3r  $ 

a  *  pa  pa^ -\-dqa 

p         pa  +  3q  pa^-{-Sqa-\-3r  ;     R 

a  pa.  ^pa"^ +dqa 

\~~p       ^a-\-3q ;     3{pa^ -\-2qa+r) 

.•.  pa^-\-2qa-\-r  =  0; 

:.  joaj^+^aj+z- is  divisible  by  a;  — a  (Th.  I.  Cor.  2), 

or,  px^  +3qx'^  -\-3rx-\-s  and  jDa;2+25a;+r  have  a  common  divi- 
sor. Multiply  the  latter  polynome  by  x  and  subtract  the  product 
from  the  former,  and  the  proposition  reduces  to 

lipx'^-\-3qx^ -\-orx-{-s  have  a  square  factor,  ^;a;--f-2g'a;-|-r  and 
qx--\-^lrx-\-s  will  have  the  square-root  of  that  factor  for  a  com- 
mon divisor. 

3.  If  joa;3  +  3ga;^4-3ra;-|-s  vanish  for  a;  =  a,  or  h,  or  c,  find  in 
terms  of  x,  p,  q,  r  the  value  of 


X  —  a        x  —  b        X  —  c 
Eeduce  to  a  common  denominator  and  add  the  numerators 

•      3x^  -1{a  +  h  +  r)x  +  {ab  -irhc+ca) 

[x  —  a}{x  —  h){x  —  c) 


60  THEORY    OF    DIVISOES. 

Multiply  both  numerator  and  denomiuator  by  p  and  reduce  by 
Tii.  II.,  and  Ex.  4  of  Th.  11. 

px^ -\-''dqz'^ -\-'6rx-\-s 

a.m+1  y,m+\         rj,m+l    ^    'd{px'"-+^  +  2qx''^+'^-\-rx:^+'^) 

'  '   X  —  a         x  —  b        X  —  (■  px^ -\-'iqx^ -\-or.c-\-s 

4.  li  2}x^  -VQqx^  -\-^rx-\-s  vanish  foi-  x  =  a.,  or  h,  or  c,  express  in 
terms  of/>,  q,  r,  s,  the  following,  a +  6-}- e,  a~-\-h~  +c^,  a^  +  h^  -\-c^ 
,     a"* +  6'" +<;'«. 

Divide  a;"'+^  bv  x  —  a. 

1 


a 


a         ft'  a^  ft™  a"^+i 


1  a         a2  ^s ^m  .        a"^+^ 

Similarly  divide  x''"'+'^  by  x—b  and  also  by  x—e. 
add  together  the  quotients 

/^m+l  r^m+1         ajW+l  • 

1-  ■ 7  +  =3x"'  +  (a  +  b  +  r)x'"-'  +  [a-  +  b^  +  e^hf-^ 

X—  a         .V—  h        X  —  G  ^  '  ^  ' 

+(rt.34-63+c3)a;'"--rf-  ,1'c. 

Hence,  by  tlie  last  example,  the  required  expressions  are  the 
coefficients  taicen  in  order,  beginning  with  the  second,  of  the 
terms  in  the  quotient  of  3(/>a;'"'+3  +29'a;""''+^+rx"'+^)  -=-  {iix^ -^-^qx^ 
+  3/-a;4-«).     These  may  now  be  found  by  Horner's  Division. 

5.   Writing   s^    for   a+/'+c,  s^  for  ii~-\-h"-^c-,   &c.,    express 
(a— 6)4+(6— c)4  +  (c  — fl')4  in  terms  of  s^,  .Sg,  Sg,  s^. 
By  actual  expansion 

{x-a:)^-\-{x  -  6)4  +  (a;-r)4  = 

3x4-4(a+/.  +  c)a;S+G(«2+i2_j.c3)a;2_4(„s_|.^3^c3)a-4- 
<j4  ^  ^4  _i_  (:4  =  ga.4  _  4s^a;3  -f  Gsgic^  -  4.v3rc  +  s^. 

Puta;  =  «,    =  i,    =c  in  succession. 

(a_6)4  _|_(c-fl)4    =3^?4  _4s^a^+6s2fl2_.4.<,^,,^,,_^ 

(ft_c)4    _[_((._rt)4     =3r4   -4SiC3+6s2c2  -4s3r+S^ 

...  2{f«-&)4  +  {i-r')4  +(f:_a)4}=3.S4-4.s,.^3  +  6.s:--4s3S,+3.v^   . 
in  which  s^  is  written  for  3  or  1  +  1  +  1,  i.e.,  a^  +  Z^o+'j". 


THEORY    OF    DIVISOBS. 


61 


Exercise  xxiii.    (a). 

1.  Determine  the  condition  necessary  in  order  that  x^  frV^-\-9. 
and  x^-{-p'x+q  may  have  a  common  divisor. 

2.  The  expression  x^-\-Za^x^+Zbx*-{-cx^-\-Mx--\-^e^x-\-P 
will  be  a  complete  cube  if 

e  d         c  —  a^ 

•^  ~    a     ~     b     ~      6a2       ~      ~ 

3.  Prove  that  ax^+bx  +  c  and  a+6a;*-f  ca;*  will  have  a  common 
quadratic  faekir  if 

Z)2c2  =  (c2  -  a2 +&2)(c2  -  a^+ah). 

4.  Prove  that  ax^+bx^+c  and  a+bx^+cx^  will  have  a  com- 
mon quadratic  factor  if 

«362  =  (rt2  _  cS)(«2  _  c3 +5c). 

6.  Prove  that  ax^+bx^-^cx+d  and  a+bx+cx^-^dx^  wiilhwe 
a  common  quadratic  factor  if 

{a-\-d)     3  =  {b-c){bd-ac). 

6.  x^  +px'  +qx+r  will  be  divisible  by  x'^+a^'  +  b  if 

a^ -2pa^-\-{p^+q)a-}-r—pq  =  0,  and  b^  —qb^  -t  rpb  —  r^  =  0. 

7.  a;*4-pa;-4-5'  will  be  divisible  by  x^  4-ax-\-b  if 

rt6  -  4?a3  =^3  and  {b^- -¥  q){b^  -  qY  =JJ-bK  ^ 

8.  Determine  the  condition  necessary  in  order  that  x* +4pa;3 
-{•Qqx^+Arx+t  may  have  a  square  factor. 

JI  x^+Apx^  +  Qqx^ -jr^Li-x+t  vanish  for  a;  =  a,   or  b,  or  c,  ovd, 
find  in  terms  of  x,  p,  q,  r,  t,  the  value  of 

a;"  x"  a;"  a;" 

x  —  a        x  —  o        x—c        x  —  d 

10.  2  a,    2a^    ^a^,    S«*,    2  «^    2  a". 

11.  2(a-&)S    S(a-fc)*. 

12.  Determine  the  values  of  the  expressions  in  Ex.  9,  10, 11,  for 
the  poiynome  a;*  —  14a;^  +a;  —  38. 


62  rAOTORING, 


CHAPTER  in. 


Section  I. — Dieect  Application  of  thk  Fundamental  Formulas 


Formulas  [1]  and  [21.  {x±yy  =x^±1xy-\-y^,  &c. 
Art.  XII.  From  this  it  appears  that  a  trinomial  of  which  the 
extremes  are  squares,  is  itself  a  square  if  four  times  the  product 
of  the  extremes  is  equal  to  the  square  of  the  mean,  and  that  to 
factor  such  a  trinomial,  we  have  simply  to  connect  the  square 
root  of  each  of  the  squares  by  the  sign  of  the  other  term,  and 
write  the  result  twice  as  a  factor.  • 

Examples. 

1.  4a;4-80a;3//2  +  400?/4  =  (2a;2-20y2)(2a;3_!20?/3) 

2.  l-12cc2?/24-36a;42/4^(l_6a;22/3)(i_6ic2y2). 

3.  (a_6)2_|.(5_^)24.2(a_6)(&-c).     This  equals  (a- i  +  ^-c) 
X  (a  — 6+fe  — c)  =  («— c)(a— c). 

4.  x^ ^xj^  -li-z^ -\-'ixy -1xz--%jz. 

Here  the  three  squares  and  the  three  double   products  suggest 
that  the  expression  is  the  square  of  a  linear  tnnowial  in  x,  y,  z. 

An  inspection  of  the  signs  of  the  double  products  enables  us 
to  determine  the  signs  which  are  to  connect  x,  y,  z:  we  see  that 

ist.     The  signs  of  x  and  y  must  be  alike, 

2nd.     The  signs  of  x  and  z  must  be  different. 

3rd.     The  signs  of  y  and  z  must  be  different.     Hence  we  have 
^^y—z,  or  —x—y-\-z=  -  i^x-k-y-z),  and  the  factors  are 
{x+y-z){x  +  y-ri). 

Exercise  xxiv. 

1.  9m 2  + 12m +  4;  c2"'-2c"'  +  l. 

2.  ?/6_2?/323+z«;   16a;2v/2  +  -iGa:i/3  +  47/4. 

i).  9^262 4.i2«^c-i-4c^  ;  'di5x'y^-'lixy^'\-iii'^ 


FACTORING.  63 

0.  (a4  6)2+c3_2e(a4-6)  ;  S)^'^  -  fx4y2  4-^j^y4. 

/  a  \  -'"       /  6  \  ^^ 
G.  ^2^.^^_,^)3_2^(x-y);    jy)      +  (-]      -2- 

8.   (rc^— x'?/)2— 2(cc2  —xi/)(x)/  —  y^)  +  {x}j  —  y')^. 

0.  (a  +  ft+c)2-2c(rt+/)-fc)-trc2  ;    ff./?'^-2/>3(^2^.y.^4. 

10.  (8.c-4/y)3  +  (2a;-3//)-  -2(3a;-4)/)(2ic-37/). 

1.1.    (a;3  -.,;y-i-y2)3  +  (a;3  +^^  +  i/2)2  +2(s;* +.-^2^3+7/4). 

12.  (o.r2  +  2a;?/+:.73)34.(4x-24-6//2)8  _2(4a;'^+6?/3)x 
■5x^J^2xy-h7y--^). 

13-  (t)  +(t)  -Mt)    • 

14.  a?  +  63_j.c2_2fl6-26c+2rtc. 

15.  a4  +  i4+c4-2a-53_2«3,;3+262c2. 

16.  {a- i)2  +  (t  -  <')^  +  (f;  ~  '0^  -f-  ^(«  --  &)(6 -c))  - 2(a-5)(c-a) 
+  2(?>-c)(«-c). 

17.  4^,4  _  12^26+  :)/;3  +.16rt3c  +  16o3  -24&C. 

ForjiutjA    [4].     a;3  —  ?/3  =  (cc-}-2/)(a;  — ?/), 

Art.  XIII.  In  this  case  we  liave  merely  to  take  the  square - 
foot  of  each  ox  the  squares,  and  couueet  the  results  with  the  sign 
4-  for  one  of  the  factors,  and  with  the  sign  —  for  the  other. 

Examples. 

1.  {a-\-b)''-{c->rd)^. 

This={(a+6)  +  (^+(0}{(«+6)-(c+fO} 
=  (,',+6  +  'j  +  d)(a  +  6-c— d). 

2.  Factor  {x-  +^rij-\-i/^)^  -{x^  —xy-^y^)^. 
Here  we  hav^e 

{{x^-  +  oxy+y^)  +  {x^-x!i+y^-)]{{x"-+5xi,-^y^)-{''^-xy+y^-)\ 

This  =  a2  -  (i>  -  cf^  =  (rt  +  6  -c)(«— fc+c). 


64 


FACTORING. 


4.    EeSolvG  (a2  4-53)2_(<j8_52)2_(„2+ftS_c3)2. 

This  =  4rt2/,2_(a2  +  /,2_,.2')2 

The  former  of  these  factoi-s  =  (a  +  i)"-^  —  c^  =  (^a  +  h-\-<:)(a-}'h  —  t') , 
and  the  latter  =  c2  —  (a  —  b)^)  =  (c+a  —  b){c  —  c(,-{-h). 
.•.  the  giveu  expression 

=  {a+b+c){a+h  —  c){c-\-a—b){c--a+b). 

Exercise  xxv. 


1. 

49^8  _452^ 

9. 

81a4-l. 

2. 

9a2_ife3. 

10. 

ai-16b*. 

3. 

81a4_i664. 

11. 

a»e-6i6. 

4. 

100a;2-36y2. 

12. 

a2_52^26c-c*. 

5. 

5rt.2&-20&a;22/a. 

13. 

(rt+26)2-(3a;-4^)-=. 

6. 

9x<5- 162/4.      » 

14. 

(0,2+^3)2  _  43,2^2.' 

7. 

9  ^2        1 

y^C       —  i. 

15. 

{x  +  yf-iz^. 

6. 

4]/4-|a;2z3. 

16. 

{Sx+5y-{6;e-i'd)^  - 

17.  4a;22/2-(a;2+y2-z2)2. 

18.  (^x-2+xy-y-'y  -{x^-  -ary-7r-)\ 

19.  (»2_,y2+22)2_4.,.223. 

20.  {a+b-\-c+d)^  -{a-b+c-dy. 

21.  (2+ya;+4a;3)3_(2-3a:+4:c-')». 

22.  («2+62+4a6)2-(a2+i3)2. 

23.  {a^-b^+c^~d2)^-{2ac—2hd)^. 

24.  (a;3_y2_  23)3  _  4-^223. 

25.  (a6-a3t3^iC)3_(^C_5fi3/;3_j_i8)2. 

•20.  ai2_/>i2  +  Ga9/>3_6i9a3  +  SZ?»fi3-8r/S6». 
27.  (a;24-?/2+23  -xy  —  ijz.—zx)'^  —  {xy-^yz+zx)*. 
!i8.  (ic^  +7/2  +22  -  2x,y  +  2x2  -  2?/^)-(;?/+s)2. 
29.  2a2i2  4.2&2^.2+26-2a3_^4_i4_,;4. 

SO.    .S4  +^4  _i.24  _  2::2^2  _  2^223  -  2»3a;2. 


PACTOEING.  66 , 

FoxiMULA  A.     (x->rr)(x+s)=X'  +  {r  +  .i)x+r$, 
Examples 

2.  {x^ij)^+x-y-110  =  {x-7j-^h){x-y-10). 

^(aS  _  ab  +  b2)-^{2a  +  Sb)}  {(fl3  _rt?;_{-62)  _  (2«-  3&)}. 

4.  (a;2  -  5a;) 2  - 6(x3  - 6x)  - 40  =  (a;3  - 5a;+4)(a;2  -ox- 10). 

5 .  («a; + i?/ + c) 2  —  (77?,  —  w) («.r ■+by-\-c)—mn 
=  [ax-i-  by  -\-c  —ni){ax+by  +  c-{-n). 

Art.  XIV.  It  will  be  seen  that  the  first  (or  common)  term  ot 
khe  required  factors,  is  obtained  by  extracting  the  square  root  of 
the  first  term  of  the  given  expression,  and  tlfat  tlie  other  terms 
are  determined  by  observing  two  conditions  : 

(1)  Their  product  must  equal  the  third  term  of  tlie  given  ex- 
pression. 

(2)  Their  sum  (algebraic)  mnltiplied  into  the  common  term 
already  found,  must  equal  the  middle  term  of  the  given  expres- 
sion. Hence,  to  make  a  systematic  search  for  integral  factors  of 
an  expression  of  the  iormx^±bx  +  c,  we  may  proceed  as  follows  : 

Ist.    Write  down  every  pair  of  factors  whose  product  is  c. 

2nd.  If  the  sign  before  <;  is  +,  select  the  pair  of  factors  whose 
sitm  is  b,  and  write  both  factors  x+,ii  the  sign  before  6  is  +  ;  a;  — , 
if  the  sign  before  6  is  — . 

3rd.  But  if  the  sigh  before  c  is  — ,  select  the  pair  of  factors 
whose  difference  is  b,  and  write  before  the  larger  factor  x+  or  a;  —  , 
and  before  the  other  factor  a;-  or  a;+,  according  as  the  sign  be- 
fore 6  is  +  or  — . 

Examples. 
1.  a;3  +  g.x-l-20.     The  factors  of  20  in  pairs  are  1  and  20,  2  and 
10,  4  and  0.    'The  sign  before  20  is  +,  hence  select  the  lactors 
whose  sum  is  9..    These  are  4  and  6.     The  sign  before  9  is  +, 
hence  the  required  factors  are  (a;+4)(a;-f5). 


66  FACTORING. 

2.  a!2_8a;^.i2.     Pairs  of  factors  of  12  ore  1  and  12,  2  and  6, 

3  and  4.  Sign  before  12  is  +,  therefore  take  pair  whose  sum  is- 
8.  These  are  2  and  6.  Sign  before  8  is  — ,  heiice  the  factore 
are  (x  — 2)(x— 6). 

3.  a;2-21a;-100.  Pairs  of  factors  of  100  are  1  and  100, 
•2  and  50,  4  and  25,  6  and  20,  10  and  10.  Sign  before  100  is  - . 
therefore  take  the  pair  whose  difference  is  21.  These  are  4  and 
25.  The  sign  before  21  is  — ,  therefore  x—  goes  before  25,  the 
larger  factor,  and  the  factors  are  (a;+4)(a;  — 25.) 

4.  a;2+12a;-108.  Pairs  of  factors  of  108  are  1  and  108, 
2  and  64,  3  and  36,  4  and  27,  6  and  18,  9  and  12.  Sign  before 
108  is  -  ,  therefore  take  the  pair  whose  difference  is  12.  These 
are  6  and  18.  Sign  before  12  is  +,  therefore  x+  goes  before  18, 
the  larger  factor,  and  x  —  before  6,  the  other  factor ;  hence  the 
factors  are  {x  —  Q)){x-\-lQ). 

Note. — It  will  be  found  convenient  to  write  the  factors  in  two 
columns,  separated  by  a  short  space.     Taking  Ex.  2  above,  pro 
ceed  thus  :     Since  the  sign  of  the  third  term  is  + ,  write  the  sign 
of  the  second  term  (in  this  case  — )  above  both  columns. 

1  12 

(a!-2)  (x-Q) 

Ex.  3  above.  Since  the  sign  of  the  third  term  is  — ,  write  tbe 
sign  of  the  2nd  term  (in  this  case  — )  above  the  column  of  larger 
factors,  and  the  other  sign  of  the  pair  +,  above  the  other  column. 

+ 

1  100 

2  50 
(a;+4)          (a; -25) 

6.  a;'»-84a;4-64. 

Here  we  have  the  factors 

1,         64 

x-%  a;-32 
4,         16 

and  since  the  last  term  has  the  sign  -+•,  and  the  middle  term  has 

the  sign  — ,  we  write  —  over  both  columns. 


FACTORINOc  57 

6.  a;3+12a;-64. 

+ 

1,  64 

2,  32 

X-4:,   x  +  lQ. 

Here,  since  the  last  term  has  the  sign  - ,  we  write  the  sign 
(  +  )  of  the  middle  term,  over  the  column  ol  larger  factors,  and 
the  sign  —  over  the  other  column. 

7.  jc*- 10x3 -144. 

Here  we  have  the  pairs  of  factors : 
+  - 

1,  144 

2,  72 
4,         36 

x  +  S,  u;-18. 
And  since  the  sign  of  the  third  term  is  — ,  we  write  the  sign  ol 
the  second  term   (in  this  ease  -)  above  the   column  of  larger 
tactors,    and   the   other   sign  (of  the  pair  +)  atove  the  otiier 
column. 

Exercise  xxvi. 

1.  x-^-^5x-U;  x^-Qx+U;  a;2+7;B+12. 

2.  a;2  -8x4-15  ;  x^  -  19x4-84  ;  x^  -7x-60. 

3.  4x3-2x-20;  9x3-150x4-600. 

4.  ix3 4-4*x- 36  :  25x'^  4-10x4-15;  9x«-27x34-20. 
y5.  ^^x^  +  Hx+12;  16x4-4x3-20. 

^-^6.  ^4_u,34-63)x3+.y2/,2;  4(x4-2/)2-4(x4-^)-09. 

7.    (a;2+.v-^)3-(a2_fe3)(a;2+y2)_a2i2. 

'^7'(:;rHp4ij[#+2/3)(x+2^ 

10.  {a+bf-iab{a  +  b)-{a^-b^f. 

11.  (x3  4-x//4-!/2)2+x3-?y3_5xi/-2^3-2x». 

12.  a-^-<ia{b-c)-^{b-c)\ 


68  FACTOKINa. 

y^ 3:3.  (x^+y^)^-h^a^{x^-\-y^)  +  a^~b^. 

14.  (a;2-10aj)3-4(a;3-10a;)— 96. 

15.  (a;2_l4a;  +  40)2-25(a;2-14a;+40)-150. 

16.  {x^  -x>j  +  :i^)^+2xy{x-^  -xij-\-y^)  ~3x^y^. 

17.  z4-3z2  +  2;  x^ -2x^-3:  9x^+9x^y^ -lOy^. 

18.  c^'"  +  c™  -  2  ;  a;«  -  a;3  —  2  ;  a;^'"  -  2a;"'2/»  -  8^/'". 

19.  a;^"—  (a  —  b)x"Y  -  « V- 

Art.  XV.  Trinomials  of  the  form  ax^  +  bx+c  (a  not  a  squai  e) 
may  sometimes  be  easily  factored  from  the  following  couaiJera- 
tions  : — 

The  product  of  two  binomials  consists  of 

1st.  The  product  of  the  Jirst  terms. 

2nd.         "  "         second     '« 

3rd.  The  sum  (algebraic)  of  the  products  of  the  terms  taken  dia- 
'^oxially. 

Tnese  three  conditions  guide  us  in  the  converse  process  ol 
resolving  a  trinomial  into  its  binomial  factors. 

EXAIIPLES. 

1.  Resolve  6ic2-13ar7/+ 6^/2. 

Here  the  factors  of  the  first  term  are  x  and  6x,  or  1x  and  3a; ; 
those  of  the  third  term  are  y  and  Qy,  or  2y  and  oy.  These 
pairs  of  factors  may  be  arranged 

(i)  (2)  (3)  (4) 

»  2»  y  2.i/ 

6a;  3x  6?/  dy 

Now,  we  may  take  (1)  with  (3)  or  (4),  or  (2)  with  (3)  or  (4) ; 
but  none  of  these  combinations  will  satisfy  the  third  condition. 
If,  however,  in  (4)  we  interchange  the  coefficients  2  and  3,  then 
(2)  and  (4)  give 

2a;         'dy,     and 

3x        'ly,     where  we  can  combine  the  "  diagonal" 
products  to  make  13,  and  the  factors  are 


FACTORING. 


69 


2a;  —  Sy,  and 
Sx  -  2y. 
The  coefficients  of  (2),  instead  of  those  of  (4),  might  have  been 
iuterchauged,  giving  the  same  result. 

2.  ea;2-15a;?/+6y2. 

Here,  comparing  (2)  and  (3),  Ex.  1,  we  see  that  their  diagonal 
products  may  be  corabined  to  give  15,  and  the  factors  are 
Ix—y,  and  dx—Qy. 

3.  Qx--'iOxy+%y^. 

Here,  again  referring  to  Ex.  1,  we  see  at  once  that  it  is  useless 
to  try  both  (2)  and  (4),  since  the  diagonal  products  cannot  be 
combined  in  any  vv'ay  to  give  a  higher  result  than  IQxy.  But  com- 
paring (1)  and  (4),  we  obtaiB  by  interchanging  the  coefficients 
in  (4)  x—oy,  and 

6a;— 2^,  which  satisfy  the  third  condition. 
Or,  v/e  might  interchange  the  coefficients  of  (3),  and  take  the 
resulting  terms  with  (2),  getting  2x—Gy,  and 

8x-  y. 

4.  iJx^ -i-Sbxy  —  6y^ . 

Here  the  large  coefficient  of  the  middle  term  snows  ai  once 
that  we  must  take  (1)  and  (3)  together.  Interclmnging  the  co- 
efficients of  (1)  we  have 

6x—  y,    and 
ai  +  6?/.    The  same  result  will  be  obtained  by  inter- 
changing the  coefficients  of  (3). 

Exercise  xxvii. 


1.  6a;2_37a;?/  +  6?/2. 

2.  6x2-f9x(/-6;/-. 

3.  56a;2-76a;?/+20?/2. 
•i.  56a;2-36x2/-20y2. 

5.  56x2-1121a:*/+207/2, 

6.  56a;2-68£c?/ -1-201/2. 

7.  56a;2-558.-c(/-20?/2. 

8.  5Qx^  +  'dQxy-2Qy-. 

9.  56a;2  -67x^+207/2. 
10.  5e:i;2+3.c^-20i/2. 


11.  Qx^  -IQanj  —  Qy'^. 

12.  6a;3+5a;//-67/2. 

13.  56a;-^-H562a;//+ 207/3. 

14.  56a;--122ic/y  +  20?/^ 
15:   56a;-'-102a;i/-20?/2. 

16.  56x2 -229a;?/ +20^/2.' 

17.  5 Sa;3 -94x^+207/3. 

18.  56x2- 276a;)/ -20//3. 

19.  3Gx- — 33x^  — 15?/2. 

20.  72x3- 19x1/ -4U(/2. 


70  FACTOBINO. 

Art.  XVI.  More  fjeneralbj,  trinomials  of  the  form  ax^-\-hx-rO 
[a  not  a  square)  iaay  be  resolved  by  Formula  A,  thus 

Multiplying  by  a  we  get  a^x^  +bax+ac.  Writing  z  for  ax  this 
becomes  2=^+ '!/z+'i';.  Factor  thid  trinoaiial,  restore  the  value  of 
z  and  divide  the  result  by  a. 

Examples. 

1.  6.t3 -\-5x-4:.  Multiplying  by  6,  we  get  (6a-)2  +5(Ga;)  - 24  or 
2^  +  02 -2i.  Factoring,  we  get  (-2;-3){c  +  3),  hence  tlio  required 
factors  are  ^(6x-'d)(Qx+'S)  =  {2x—l){\ix+-±}. 

2.  6a;2  -  13x1/  +  6»/3 .     Factoring  z^  -  Idzy  +  36?/3  we  get  {z  -  4t/) 
{z  —  9y),  hence  the  required  factors  are  i«(Da;  — -i^j(6a;  — %)  = 
{3x-2y){2x-3y). 

3.  33-14a;-40a;3.     Factoring  1320- Ue-g^  we  get 

(^30  — 5;)(444-z),  hence  the  required  lacfcors  are  ^^^(30- 40a;)  x 
(44+40^)  =  (3- ^^(ll +  10.'-c). 

NoxB. — The  factors  may  conveniently  be  arranged  in  two  col- 
umns, each  with  its  appropriate  sign  above  it. 

+ 
Ex.  1,  above  1  24 

2  12 

^(6a;'-3)(6a:+8)  =  (2a:-l)(Sx-f4). 

Ex."  2,  above  1  3G 

2  18 

3  12 

^  (6a;  -  4)  (6x  -  9)  =  (3j;  -  '2 )  (2»  -  3). 

[Another  method  of  factoring  trinomials  of  the  form  ax'^~{-lx-\'e 
is  as  follows : 

Multiply  by  4a,  thus  obtaining  4:a^x^  +4ff&^-+4ac.  Add  6*  -  6*, 
vhich  will  not  change  the  value,  Aa^x^  +  4:abx+b'  —b^  +  iac  ;  by 
[1]  this  may  be  written  {2ax+by~-—{h^  —  'l:ac).  Factor  this  by 
[4]  and  divide  the  result  by  4a. 


FACTORIXO. 


71 


Ex.     Factor  56x»  +  137a; -27885.      Multiply    by    4x56    or 
2x112,  1122ic3 +  2.137.1120;— 6216240.     Add   1873-1372,  then 
il2-'.«3  + 2.137.112.^+1373- (1373 +P.246240)  =  (112^-  +  137)3 - 
6265009  =  {{112a;  +  137)+2503}{(112.r+ 137) -2503}  = 
(112a;+2640)(112.c-23GG). 

Y/e  multiplied  by  4  x  56,  we  must,  therefore,  now  divide  by  that 
number.     Doing  so,  wc  obtain  as  factors  (7a;  +  165)(8.£— lCij).j 

Exercise  xxviii. 


1.  10a;3+a5-21 

2.  10.c3  -  29.e  -  21. 

3.  10.«2  +  29a;-21. 
;.  6a;3-37a;+55. 

5.  12«3_5«_2. 

6.  12a;2-37a;+21. 
12*2  + 37a; +  21. 
ISaS  +  lSaS^S-OOS* 


7. 


9.  12.r^-a;-l 

10.  'dxhj^ -'dxy^ -e,y<^, 

11.  4x3+8.v^+3//-. 

12.  662x2 -7ia;3-3.r*. 

13.  &x^-x^y^ 

14.  2a;4+a;3_45. 


•35//*. 


■%*. 


15.  Ax^~'61x-y'^- 

10.  4(2; +  2)*  -87a;2(:/j+ 2)2-1- 9a;*. 

17.  6(2a;  +  37/)3  +  5(Ga;3  +  5.r//-6//-)-6(3a;-2?/)3. 

18.  6(2a;+3?/)*  +  5(6.j;3  +  5is,//-6//-)2  _G(3a;-2,v)*, 

19.  6(a;3+a;//+y2)3+i3(a.4+.^2^^2+y4)_385(a;^^-a;'/+?/2)3. 

20.  21(a;2  +  2a:^+2j/2)3-6(ic-^-2x^+2?/2)3-5(a;4+-i^-i'). 


Section  il. — Extended  Application  of  the  Formulas. 

Art.  XVII.  The  methods  of  factoring  just  explained  may  be 
a-pplied  to  tind  the  rational  factors,  where  such  exist,  of  quadratic 
multinomials. 

Examples. 

1.  Eesolve  12u;2_a;//-20?/2+8a;+41y-20. 
In  the  first  place  we  find  the  factors  of  the  first  three  terms, 
!7hich  are 

Ax+5y,  and 
Bx  —  Ay. 

Now,  to  find  the  re^naining  terms  of  the  required  factors,  we 
must  observe  the  following  conditions : 


72  fACTORINd. 

1st.     Their  product  must  =  —  20. 

2ad.  The  sum  (ah/ebraic)  of  the  products  obtained  "by  xuulti- 
plyiug  them  diagoually  iuto  the  y's,  must  ==41?/. 

3rd.  The  sum  of  the  products  obtained  by  multiplyiug  them 
di.agonally  into  the  x's,  must  =8a;. 

We  see  at  once  that  —4  with  the  first  pair  ah'eady  found,  and 
+  5  with  the  second  pair,  satisfy  the  required  conditions,  and  .". 
tlie  factors  are 

4:X+5y  -4,  and 

8a;  — 4?/ 4- 5. 

Here  the  factors  oi p^+  V']  —^q^,  are 

p+2^,  and 
p  —  q. 
Now  find  two  factors  which  will  give  -  3r-,  and  which  multi- 
plied diagonally  iuto  the  p%   and  qs,  respectively,  will  give  2pr. 
and  Iqr  ;  these  are  found  to  be  —  r  taken  with  the  first  pair,  and 
+  3r  taken  with  the  second  pair.     Hence  the  required  factors  are 

j9  +  2g— r,  and 

■    ■  .  / 

Art.  XVIII.     But  the  following  examples  illustrate  a  surer 

method. 

3.  ic2+a;?/-2?/3+2a:3  +  7?/*-32». 

Reject  1st  the  terms  involvings, 
2nd.         "  "  2/, 

3rd.         "  "  jc,       • 

and  factor  the  expression  that  remains  in  each  case. 

1st.  x^-{-xu-2y^  =  {x~y){x+2y). 
2nd.  x^+2xz-Sz^  =  {x+Sz)[x-z). 
8rd.    -2//'^+7//2-3z3    =  (-y+Sz){2y-z). 

Arrange  these  three  pair  of  factors  in  two  sets  of  three  factors 
each,  by  so  selecting  one  factor  froin  each  pair  that  two  of  each 
act  of  three  may  have  the  same  coefficient  of  x,  two  may  have  the 


?ACTORIN(J.  73 

game  coefficieut  of  y,  and  two  the  same  coefficiant  of  z  {mefficimi 
including  su/n).     In  this  example  there  are 

X—   y,    x  +  3z,    —   y-\-?iZ, 
and  x-\-1y,   x—  z,        ^y—z. 
From  the  first  set  select  the  common  terms  (inchuling  signs^ 
and  form  therewith  a  trinomial,  x—y  +  Bz. 

Eepeat  with  the  second  set,  and  we  get  x-\-2y—z. 

:.x^+xy-2y^  -\-2xz-\-7yz~Sz^  =  {x-y-{-3:){x-\-2y-z). 
4.  3a;2-acy-8//2-i-30a;+27. 

1st.         Sx^-8xy-3y^    ={Bx  +  i/)(x--By). 

2nd.       8a;2-L30x  +  27      =  (3a;+3)(a;+9). 

8rd.   -%2  ^_27      =(j,^-3)(_3y  +  9). 

.'.  the  factors  are  {3x+y+'d){x  —  3y-\-2). 

6.  6a2-7a6  +  2rtc-2062  +  646c-48c3. 

1st.  6«3_  7«6-20Z)3    ={2a~5b)(3a-i-'ib). 

2nd.         6rt3+  2rfc-48c3    =(2r«+6c)(3a-8c). 
8rd.    -2062+ 646c -48c3    =  (_.56  +  Ge){46-Sc>, 

/.  the  factors  are  (2a  -  u6+Gc)(3a  -j-  46  —  6c). 

Exercise  xxix. 

1.  7x^~xy-6y9-6x-20y-16. 

2.  20x2-15a;?/-5t/S_68a;-427/-88. 

3.  3«4+a;27/3_4y4  +  i0a;2_i7^2_i3, 

4.  20a;2- 20^3  _^9.i-?/  + 28^4-35?/. 

5.  72a;2-8?/3  +  55a;?/-}-12!/— 169a;  +  20. 

6.  x^  —xy—  12y^  —  5x-~15y. 

'  7.  8a;3  +  18a;(/+9^3_^2a;2— s*. 

8.  6x^+6y^-13xy-8z^-2yz-{-Sxz. 

9.  6a;*- 10?/^ +  lla;2;/2 -2532 -[-107/2 +25!/2z2-15:K2^10a;2z3 

10.  16.c4  -16^4  - 22a;3j/2  j.  1524 _[.  14^222  ^. 50a;222, 

11.  4rt3_i552_4aj_21c2  — 366o— 8ac. 

12.  «*  +  &4+c*-2rt»62-263c2-2c2a». 


OFTHc 
lift.ii<<___ . 


74  FACTORING. 

Art.  XIX,  Trinomials  of  the  form  ax^  +  bx^  +  c  can  always  be 
broken  up  into  real  factors. 

If  a  and  c  have  different  signs,  the  expression  may  be  factored 
hj  Art.  XVI. 

If  a  and  c  are  of  the  same  sign,  three  cases  have  to  be  consid- 
ered :  i.  6  =  2v/(ac),  ii.  6>2v/(ac),  iii.  6<2v/(rtc) 

Case  I,  b  =  2y^{ac).  This  case  falls  under  Art  XII.,  formula 
[1] .  where  examples  will  be  found. 

Case  II.  i>2y(ac).  This  case  falls  under  Art  XVI.,  where 
examples  will  be  found.  The  following  additional  examples  are 
resolved  by  the  second  method  of  that  article. 

Examples. 

Here  we  see  that  (^y^)^  will  make,  with  the  first  two  terms, 
a  perfect  square,  and  we  therefore  add  to  the  given  expression 
(fy^)'~(f!/^)'-     T^^®  expression  then  becomes 

=  (2x3+12/2  +  12/ =^)(2a:»+|-y»-f?/2) 
=  (2a;=  +  22/3)(2a;'  +  ij/^')  =  {x^  +  r/^){4x^  +y"). 
2.  3a;* +  6x2 +2. 

Here  multiplying  by  4x3,  and  completing  the  square  as  in 
Ex.  1,  we  have 

86x* +72^2 +  62  + 24 -62  =  (6x2  +  6)2 -12    . 

=  (6x3  4-6-i/12)(Gx=+6  +  i/12),  which  divided  by  4x3  give 
the  required  factors. 

8.  ax^+hx^+c. 

Proceeding  as  in  Ex.  2  we  have,  by  multiplying  by  4a, 

ax^^bx-  +c  =  {ia^x^-^iabx"  +63  - b^  +4ac}  -•  4a 
=:{2ax^+b  +  y/{b^-Uc)}{2ax^+b--i/{bi-4.ac)}^U. 


FACTORINO.  75 

Exercise,   xxx. 

1.  ic^+TxS+l;  4a;4  +  14a;3+l. 

8.  40:4+10x3+3;  S{xA-y)^  +  oz^{x+yy +2'^. 

4.  a;4  +  7cc32/2+3J?/4;  x^ +7x2^/3+8^7/4. 

5.  4x4+9xV+tI^*;  4(rt+6)4+10c2(a+i)2 +-3c*. 

6.  3x^+8x2i/=  +  4-T\jr*;  36x^+96x3+55. 

7.  5x*+20x3+2;  4a*  +  12rt3  +  l. 

8.  4(x+?/)*  +  12(x+?/)^z3+24;  5.r4+20x3j/2+2T/*. 

9.  9x4  +  14x2+4;  2x*  +  12x3(f/+2)2  +  15(t/+z)4. 
10.  2x4+12x3  +  15;  7x4+40x3+45. 

n.  8x4  +  36x3?/2+29?/4:  7x4+20x-^2 -20?/*. 

12.  7(a-i)4  +  16(a-5)36-2+5c4;  ^a^  +  SaH^+b*. 

13.  8x4  +  6x^vM-2//4;  S(a+by+Q(a^-b^y  +2(a-b)K 
U.  49a4_84a2i'^+2264  ;  25TO4-{-60w2n3  4.27«4. 

15.  49(m+7i)4-84(»i2  -»43)3+22(m-M)4. 

Case  III.     &<2;/(ac).      This  case  may  be  brought  under 
Art.  XIII.     The  following  examples  illustrate  the  process  oi  re 
duction  and  resolution. 

ExAJfPLSS. 

1.  x4 -7x3+1. 

"We  have  to  throw  this  into  the  form  a*  —  6'  : 
a;4_7a;3  +  l  =  (^2  +  1)3  -9x3  =  (x3+H-3x)(x-  +1  -Sa;). 

2.  9x4  +  3x3?/2 +42/4  =  (3x3  +  22/3)2 -9x3y3 
=  (3x= +2^/2 -8x>/)(3x3 +  2^/2 +3xy/). 

3.  x4 +2/4  =  (a;S +2/3)2  _2x3|/S 

=  (X3  +J/2+XV  ,/2)(x3+2/3_x2/-/2). 

4.  x4 -ixs^/^* +1/4  =  (x^ +2/3)3 -la;  V     • 
=  (x-^+y3  -^%xy){x^  +.v2-i^?/)- 

5.  «x4+ix3+c  =  (,/«.  a;3  +  |/c)3-{2v/(ac)-6}a;» 
=  {ya.  x3+|/6--|/(2;/^-i)x}x 

il/a.  x2+i/c+v'(2v^^— ^)a;}. 


76  FACTORING. 

Art.  XX.  It  is  seen  from  these  examples  that  we  have  merely 
to  add  to  the  given  expression  what  will  make  with  the  first  and 
last  terms  (arranged  as  in  Ex.  5)  a  perfect  square,  and  to  subtract 
the  same  quantity.  In  Ex.  2,  e.  g.,  the  square  root  of  9z'^  =  3x^, 
the  square  root  of  4t/*  =  2v/ 3,  /.  3a3^+2?/2  is  the  binomial  whose 
square  is  requii'ed  ;  we  need  .".  12x^1/'  ;  but  the  expression  con- 
tains Sx^y'-^ ;    .".    we  have  to  add  and  subtract  12x^y'^  -  ox^'y^  = 

Hence  we  derive  a  practical  rule  for  factoring  such  expressions. 

(1).  Take  the  square  roots  of  the  two  extreme  terms  and  con- 
nect them  by  the  proper  sign  ;  xhis  gives  the  first  two  terms  of 
the  required  factors. 

(2)  Subtract  the  middle  term  of  the  given  expression  from 
twice  the  product  of  these  two  roots,  and  the  square  roots  of  the 
difference  will  be  the  third  terms  of  the  required  factors. 

6.  x*+^\x-y^-\-y'^.  Here  ^/a;*  =a;^,  l/y*  =  i/2,  and  the  first 
two  terms  of  the  required  factors  are  x^-\-y^  ;  twice  the  product 
of  these  is  -\-2x^y^,  from  which  subtracting  the  middle  term, 
Y^x^i/^,  we  get  xe^^i/^  j  *^^  square  roots  of  this  are  +izy. 
Hence  the  factors  are  x--\-y^±:-lxy. 

Note  that  since  s/y^=  -^u'^,  ov  -y^,  it  may  sometimes  hap- 
pen that  while  the  former  sign  will  give  irrational  factors,  the 
latter  will  give  rational  factors,  and  conversely. 

7.  x'^  —  lix^y^+y*.     Here,  taking  -{-y^,  we  have 

x^+tj^+xy  /13,  and  a-»  +y-^  -xy  v/13. 
But  taking  —y^,  we  have 

a;3  ^yZj^Zxy,  and  x"^  -y^  —  3xy, 
Sometimes  both  signs  wiU  give  rational  factors. 

8.  lQx'*'  —  l'lx^y^-\-y^.     Here  we  have 

(^ix^Jf-y^J^'dxy){4.x'^+y^  -'dxy,  and  also 
{^x"^  ~y^-Yoxy){Ax" -y^ -oxy). 


FACTORING.  77 

Exercise  xxxi. 
4.  a;4— 7a:-+l,  x-4-f9,  Ja;4+^4  _  ^3a-.'^2. 

6.  4a;4+y4_8i^.3^,/3^  ^4 +^4_^7^^3_,/-2^  4;t;44  1. 

7.  a;^"'  +  64i/*'»,  a;*"' +  %*'",  ia;*+y\v/4-5fx37/2. 

9.  m^x^  +  n^i/^~{2tiin-\-ij)x''^ij2,  .t^"' +  2^"'-y". 

10.  16a;'i-2o.62  +  9,  4:;;4  _  le^s  4.4^  ISx'^y^ -dx^  -  ii/-'. 

11.  4a;'t-12^|x-2//3-l-()?/4,  x4+6.x2+25. 

12.  a^~\-h^^(a  +  b)^,  l+«4+(l_|_a)4. 
Vl3.  (;«4-Z/)'^-722(^-+t/)2+24. 

■\    14.    (rt-f  i)4+7(.2(«_|_ft)3+c4, 

15.  16</4-j_4(i_c-)4_9«2(i_,.)3. 

16.  4(a  +  Z))4  +  9(a-i)4-21(./3_/,3)3, 

17.  (a;2+y2_^^)4_7(^34.^3)24.(^.+^)4. 

18.  (rt2_|_rti  +  /;3j4^7(^,3_63)3_|_(a-i)*. 

19.  16rt4  +  4rt2  4-l,  ;f;4-41a;2+16. 

20.  x4+8l7/8-63a;2i/4,  14-24+2528^ 

21.  (a24-l)4  +  4(rt24.i)2^2_|.iea4^  (.«+l)*  +  2(a;2  _  1)24. 
{»(a;-l)4. 

Art.  XXI.     We  can  apply  [4] ,  Art.  XIII.,  to  factor  expres- 
sions of  the  form  ax'*'-\-bx^+rbx—r^a.     This  may  be  written 
a{x*-r'^)+bx{x''-\-r)={a{x^-r)-\-bx}{x^+r). 

Examples. 

1.   6x-4  +  4a;3  +  i2x-54.     This 

=  6(a;4  -  9)  +  4a;(x3-f-3)  =  {x^-+S){6{x^  -  3)  + 4a} 
=  (a;2  4-3)(6a;2+4a;-18). 


rs 


FACTORING. 


2.  11^4  + 10a;' -40a; -176.     This 

=  n{x^-lQ)  +  10x(x^-4:)  =  {x''-i){ll{x^+.i)+10x} 
=  (x^-A){nx^+10x+U). 

3.  40a;4 +  30x3+ 60a; -160.     This 

=  10(4a;4  -  16)  +  1.5a;(2a;2+4)  =  (2a;2  +4){10(2.t;3  -4)4-15a:} 
=  (2a;2 +4)(20a;^  +  lya;- 40). 

Note. — To  determine  r,  take   the  ratio  of  the  coefficient  of  x^ 
to  the  coefficient  of  x. 

Exercise  xxxii. 
Resolve  into  factors 

1.  a;4+2a;3  +  6a;'-9. 

2.  2.r*  +  2a;3-f6a;-18. 
.3.  a;4  +  3a;-^+12a;-16. 

4.  3a;^+a;3_4a;-48. 

5.  5a;4  +  4a;3-12a;-45. 
>lf>.  10a;4  +  5a;3+30a;-360. 


3  _    S  ^_    4 


7.  ix^-\-20x^+4x-j^^. 

8.  2;"^a;4-40a;3+8a;-l. 

9.  37ia;'i-30.^;3+48x-96. 
>il0.  63a;4- 39x3  + 52a; -112. 

11.  810x4  +  Va;3+|a;-2i. 

12.  242a;4-33a;2-3a;-2. 


13.  ix^  +  ^y^x^-^2^x 

14.  80a;''  -  32a;37/+64a;?/-  320;/*. 

15.  24a;4  -  12.x-3y+30a;/y3  -  ISO*/*. 

16.  2x^  +  ^z^i/-8xy^-512ij*. 

17.  lla;*  +  10a;3-12a;-15fi 

18.  40a;*  +  30a;3  +  60a;-160. 

19.  13a;4-12a;3y+72a;?/3-468?,'4. 

20.  3a;*  +  3a;3^+12a;//3-48?/4. 

21.  oa;4+4a;3//-12a;</3_45(/4. 

22.  4x4  -  14x3/7+28x^3  -16i/*. 

23.  x4+80x3?/+16x//3-^i3.y*- 

24.  2x4 -x3?/+6x^3_  72^/4. 


Art.  XXII.     Formulas  [1]  and    [4]    may  sometimes  be  ap- 
plied to  factor  expressions  of  the  form 

ax^-\-bx^-\-cx^-\-7-bx-{-r^a. 

This  may  be  put  under  the  form 

afx*  +r2)  +  6x(x2  +/-)+cx8  =  a{x^  +r)3  +  bx{x^+r)  + 
(e  — 2ar)a;3,  which  can  sometimes  be  factored. 

Examples. 

1.  x*+6x3+27x2+162x+729. 

We  "have  x4+729  +  6x(a;2+27)+27x2. 

=  (x2+27)3  4-6x(x2+27)  +  9x3-36x2 

=  {x3+27+3x}  3  -  3Gx3,  which  gives  the  factors 


X 


2-3X+27,  and  x2+9x+27. 


FACTORING.  79 

2.  x^  +  ix^+ix' +20x^-25.     This 

=  (a;2 +5)2  +  4a.-(a;3 +5)  -  Ga;3 
=  (a;3  +  5)3 +4a;(a;2 +5)  +  4a;3  -  10a;3 
=  {a;2  +  5  +  2x-xVlO}{a;3+5+2u;+aVlO>. 

Exercise  xxxiiu 
Eesolve  into  factors  : 

1.  a;4_6cc3  +  27a;3- 162a; +729. 

2.  a;H-2a;3+3a;2+8x+16. 

3.  a;4+^3  +  a;3+a;-j_i. 

4.  a:4-4x3+a;3-4a;  +  l. 

6.  4a;4_i2a;3_6a;2-12a;+4. 

6.  a;4  +  14a;3-25a;2-70a;+2?. 

7.  16.T*-24a;3-16x3  +  12a;+4. 
V8.  a;*  +  5:«3-16x2  +  2Ux+16. 

9.  a;4  +  6a;3  _  11x3 -12x+4. 

10.  a;*+4a;3?/+ic22/2+12a;y3_|.9^^4. 

11.  x^+6x^-9z^-6x-\-l. 

12.  a;4+-4^•32/-19a;2J/2_}_4,^.y3^,^4. 

13.  4a;4 +4a;3|/- 65x2^2  _io.ri!/34_25,2/* 

14.  x*^  +6x^y-9x^y^  -Gxij^-^y*. 

15.  a;*-+6.c3//  +  10a;22/2+12ic^3^4^4. 

16.  ^x^-\-18x^y - 52x^1/- -12xy^  +  i7j*. 

17.  lla;4+10x37/+39/^V-22/3+2Ua;?/3  +  442,'«./ 


Section  III. — Factoking  by  Parts. 


Art.  XXIII.  To  factor  an  expression  which  can  be  reduced 
to  tlie  form  a.F{x)+b.f{x). 

When  the  expression  is  thus  arranged,  any  factor  common  to 
a  and  b,  or  to  F{x)  and  f{x),  will  be  a  factor  of  the  whole  ex- 
pression. The  method  about  to  be  illustrated  will  be  found  use- 
ful in  cases  where  only  07ie  power  of  some  letter  is  found. 


80  rACTORiNa. 

'  Examples. 

1-  Factor  acx"  —nbz  —  hc^x  +  b^c. 

Here  we  see  that  only  oae  power  of  a  occurs,  and  we  therefore 
group  together  the  terms  involving  tliis  letter,  and  those  not  in 
volving  it,  getting 

a{cx^-hx)-bc^x+h^c 
=  ax{cx  —  b)  —  bc(cx  —  b)  --  (ax  —  bc)(ex  —  b). 

2.  Factor  m^x^ -mna^x  —  tnnx  +  n^a^. 

Here  we  observe  that  a  occurs  in  only  one  power  («^). 
Therefore  we  have 

—   a^  [^)imx~  71^ )  + m^  x^ —  7nnx 

=  —na^[mx—n)-\-nix{mx  —  n) 

—  {mx  —  n)(^wx  —  na^). 

3.  2x^+4:ax  +  Sbx+Gab. 

Here  we  observe  that  the  expression  contains  only  one  power 
of  both  a  and  b.  W"  may,  therefore,  collect  the  coeflficieuts  in 
either  of  the  following  ways  : 

a(4x-{-(jb)+.{2x^+3bx), 
or,  b{3x  +  Ga)-\-{2x^+Aax). 

Now  the  expressions  in  the  brackets  ought  to  have  a  common 
factor,  and  we  see  that  this  is  the  case.     Hence, 

a{ix  +  6b)-\-{2x^+Sbx) 
=  2a{'2.x  +  Sb)+x{2x  +  Sb)    =  (2a; +36)  (a;  +  2a). 

4.  abxi/  +  b^ij^-\-acx  —  c^ 
=    a(bx!/-[-cx)-rb^y^ —c^ 

=  «x(%+5)  +  (%+c)(%-c)    =(by  +  c){ax+bi/-c). 

5.  yS  -  {2u  +  b)ij^  +{2ab-\-a^)y  -  a^b 

=  _f,(y2  ^2ay-}-a^)  +  y^  -2mj^+a^y 
u:  -  b{y^-2ay+t(2)  +  y(y2  _  2((//  +  a3) 

=  {y-b)iy-a)^' 

6.  2x^y-\-2bx^-bx^y  +  4:abx2y  -x^y^+iaxy-  -  2abxy^-2ay^. 
=  b{2x^-x^y+4:ax^y-2axy^)  +  2xhj-x^y^  +  4:axy^-2ay^ 
=zbx{2x^-x'-^y  +  iaxy-2ay'^)  +  y{2x^-x-y+iaxy-2ay-^) 

=  (y  +  bx){2x^  -z^y+iaxy  -  2ay^). 


FACTORINQ, 


81 


And  2a?«  —  a;2 }/ 4- 4aa;y  —  2a?/» 
=  a{4x!j  -  2?/2 )  +2x3  —  x^?/ 
=  2ay{2x  -  ly)  +x2(2x  -  y)    =  {2aij+x^){2x -  y). 

7,  x^  +  {2a-b)x^  -{2ab-a^)x-a^b 
^h{-x^~2ax-o^)+x^  +  2ax-+a^x 

=  -b{x  +  a)^+x{x  +  a)^    ={x-b){x+a)^. 

8.  px^-(;:p  —  q)x^-\-{p-q)x+q 
=  q{x^  —x-^l)+px^  —px"^  -\-}>x 

=  q{x^-X+l)+px{x^-X  +  l)     ={pX  +  q){x^-X-il). 

Exercise  xxxiv. 

6.  x^ —b^x-  ~a^x  +  a^b^. 


7.  x' 


-a^x- 


•62a;3  +  a362 


8.  8a;3  +  12aa;+  lOhx  4- 15^(6. 

9.  a^^{ac-h^)x^  ■\-bcx^. 
10.  a^+{ac-b'^)x^-hcx^. 


1.  a;2y— iK^z—  "y^ -\-yz. 

2.  abxi/-\-b'^y^ -{-acx  —  c^. 

8.    ic322^.flja;2_^322_a3.      , 

4.  2a;2— «a;  — 46a;+2a6. 
6.  a;2+26a;  +  8a^+6rt&. 

12.  ;?a;3_(p+y);j.3_j_(^^^)aj_j, 

13.  a2-\-ab  +  2ac-2b^+7bc-Bc^. 
\14:.  x^+{a  +  l)x^  +  {a  +  l)x+a. 

15.  ynpx^  +  {mq  —  n]))x^  —  {))i;r+7iq)x-\-nr. 

16.  x^  —  {a-\-b-\-c)x'^ -\-{cLb-\-bc  +  ac)x-abc. 

17.  x^  +  (a  —  b  —  c)x^  —  [ab — bG-\-ca)x+abc. 

18.  x^  +  {a-\-b  —  c)x^  —(be — ca  —ab)x  —  abc. 

"VlD.  a^x^ —  a^x^y  —  a^Xy  +  a^y^  —ax^yz-{-x^z  —  xyz-\-ciy^z, 

20.   a^bx^-{-nb^xy  +  acdxy -f  hcdy 2  —  at'/icz  —  befyz. 
^1.  a2a;3-a(6-e)a;2-5-c(a-6U-  +  c2. 

22.  7Ha;3  —nx^y-\-rx^z  —  mxy^-\-ny^—ry^z. 

23.  amx^  -{-{mby — nay-\-incz)x  —  nby^ —ncyz. 

24.  (aw.  —  bcm)x-  -{-{mn  —  bcn)x-^an-\-nax. 

25.  a262c3-6=c2a;</-a2c2?/2  +  6-2a;^32_a2i22a;  +  ft2^.3^2^rt222^ 

26.  a;5  -m^x^  -{n-n')x^  +  {m''-n-m'^n^)x^  -a{x--^n^  -n). 

27.  l-(a-l)j;-(a-/;  +  l).«2_^(a+6-c)a;^-(6  +  c)a;4+ric5. 

28.  a3a;3  -a^{b-  c+d)x^y  -  {abc-abd+acd)xy^  +bcdy^. 

29.  m^npx^  —  {n-p  —  m"}i^  -  m^j.q)x^  —  (rr'  +  n]  q  —  m^nq)x  —  n^q. 


82 


FACTOEING. 


—  [n^q-  -\-n'^q^z)y'^. 

Art.  XX IV.  Sometimes  an  expression  which  does  not  come 
directly  under  the  preceding  form,  may  be  resolved  by  first  find- 
ing the  factors  of  its  parts. 

Examples. 

1.  ahx^ -{-ahy^ —a-xy  —  h^xy.  ' 

Here,  taking  ax  out  of  the  first  and  third  terms,  and  by  out  of 
the  second  and  fourth  terms,  we  hav£ 

ax{bx  —  ay)  —  b>j(bx  —  ay),  and  hence 
{ax  —  by){bx  —  ay) . 

2.  x^-{a+b)x^  +  {a^b+ab2)x-a^b^. 

Here,  taking  the  first  and  last  terms  together,  and  the  two 
middle  terms  together,  we  have 

{x^+ab){x^-ab)-{a+b)x^+ab{'i-{-b)x 
=  (x^  i-ab)[X' —  ab)- (a  +  b)x{x^     ab\ 
=  [z-  - ab) [x-  +ab -  {a-\-b)K}  =  {x*  —ab){x-a)(x  - h)» 

3.  a:3»*-4a;"»+3.     This  eq  lals 

3^m  _  a;m  _,  s(x^i  -i)  -  x"'{x""'  -  1)  -  o(a;"'  —  1) 
_  x^(x"''  + 1 )  (a;'"-  - 1 )  -  3  (a''"  - 1 ) 
=  (a;"^-l){a;"^(a;^'^+l)  — 3}-. 

Exercise  xxxv. 


1.  a^  —  ab-^ax  —  bx. 

2.  abx^  +  b^zy  --a-xy  - aby^ . 

3.  x^  +ax^  —a^z-  «"*. 

L  a^x  +  'la^x^^^ax^+x"-. 
5.  acx^  +  {ad  —  bc)x  —  Ixl. 
0.  'I'ux'*'  -5x^+x   -1. 

7.  a~  —  A-  +ax  —  ac  —  bx  +  br. 

8.  a^  +  {l+a)ab  +  b^. 

9.  z^  +  2xy{x^-y-)  -y^. 
'.0.  x^-y^+x-'+xy+y^. 
A.  2b+{h^-4:)x-2bx^. 
12.  a;3-4-3x3-4. 


13. 


P^ 


.p2q 


2pq^  +  2q^. 


U.  a3  4-a2_2. 

15.  '3an^-2ab^  -1. 

16.  //3-3f/+2. 

17.  2a^-a''-b    -aV--\-2b^. 

18.  &:^'»  +  i2,7i_2. 

1 0  7/3n  _  2_,y2»2v  _  2i/"2':^"  H-2"-\ 

20.  a3_4rt52_j_3i3. 

21.  a'^"i-3ft'"f"+2c^". 

22.  ax^-{a^+b)z^+b^. 

23.  35.-«^»-6a2a;»_9a4. 

24.  ^^3^2_|.2rt/,r3-rt3<---^)-'.a 

25.  aia^ —iib^ -\-b^m  — )ii^. 
2ti.  i  -  tta'  -4-27::'' 


FACTORING.  83 

27.  (x-y)^  +  {l-x+y){z-y)z-t^. 

28.  2Am9 -28in^n+Gmn^  -7n^» 

29 .  a;"*+" + a;"?/" + a;"'|/"' + ?/'" +" . 

80.  x^  +  2x^y-a''x^+xhj^  -2axy^  -y*. 

Section  IV. — Application  of  the  Theory  of  DiVvSORS. 

Art.   XXV.     By  Theorem  I.  we.prove  that 

a;"  —  «"  is  divisible  by  x—  a  ahcays 

jcR  —  a"  *'         "         "  X  +  a -whenn  is  even 

af  +  a''"         "         "  x-\-a  yfhen  n  is  odd. 

By  actual  division  we  find,  in  the  above  cases ; — 

=    a;"-^+a:"-^«+     .     .     .     Jca^-'+a"-' (1). 

x  —a 


af  —  a" 
X  +a 


=   a;"-'-a;"--<^+     .     .      +«rt"-^-a"-^ (2). 


'^^'^    =  a;'-^-a^'-V(+     .   •.      -xa^-^+a^' (3). 

Examples. 

1.  Eesolve  into  factors  x^  —  y^  ;  here  a;—?/  is  one  factor  and  by 
(1)  the  other  is  a;- +a;?/ 4-2/^ • 

2.  Eesolve  n'^-\-{h  —  c)^  ;  here  a  +  (b—c)  is  one  factor;  and  by 
(3)  the  other  is  a2_^a(6-c) +  (i-r) 2. 

3.  Eesolvo  a; '5  -f-1024?/^o.     This  =(x''y  +  {{2y)^}^,  one  factoi 
of  which  is  a:^  +  (2_!/)2,  and  by  (3)  the  otlier  factor  is 

(a;?)4-(x3i^(4?/2)  +  (a;3)2(4.v2)3_a;3(4?/2)3  +  (47/3)4. 
=  a:i2_4a;9y2^1Ga;'^2/4-64a;3v'5  +  256i/8. 

4.  Eesolve  (x—2y)^-\-{2x  —  y)^  into  factors. 
Here  by  (3)  we  lia-.e 

:.   the  factors  are 

'dix-y)(7x'^-ldxy+7y^). 


S4  FACTORING. 

0.  Piesoh-e  x^+x^y+x^ij^  +x^  11^  +x}/^  + If  ^  : 

x^  —  y^        (x" -{-i/^){x^ —y^) 

-By  (!)  we  see  that  this  =  ^ —  = 

■^  ^  ^  x   —y  x—y 

=^{x->ry){x^  -xy  +  y'^)(x^ ■\-xy+y^). 
6.  Eesolve  x^'^ -x^^'a+x^a^ —  x^a^+x'' a*' -  x^a^+x^a* 

-x^fl''  +x^a^  -x^a^-\-xa'''  -a^K     This  =  r— 

_  {x^+a^){x^-a^)  __   {x^  +a^){x^  -  a^){x^  -\-d^) 
~  x-{-a  ~  x-j-a 

Exercise  xxxvi. 

Factor  the  following  : — 

1.  x«-.vS  x^-1,  x^+S,  8a^-27x'^,  S-{-a^x^. 

2.  j;5-(i>o,  27a3_G4,  ai2_i8^  3.io_32_y6, 

3.  find  a  factor  which,  multiplied  into 

a*+a3/,_|_rt362^„/,3_f./,4^  will  give  a^-b^. 

4.  By  wiiat  factor  must  x^  —  ix'^ y-{-16xy^  —  My^  be  multiplied 
to  give  a;*  -  250//*  ? 

5.  Fa.Gtor  x^  +x'''y+x-'''y^  +x*yS-\-x^y^-{-x-y^  i-xy~^  fy'. 
Find  the  factors  of  the  follow! iv 

6.  (32/2 -2a;2)3_(3a;3 -22/2)3,  a«~16i*. 

7.  x^ —y^ —x{x^—y^)+y{x  — y). 

10.  a;6-t/6+2x2/(a;4+x22/^+2/''). 

11.  (a2_ftp)3  +  8&3c3,  a;^/»_a-t'». 

12.  x^-dax-^+Ba^x-a^  +  bs. 

13.  a;3  4-8y3+4a;?/(a;2-2xv  +  4?/2j. 

14.  8x^-Gxy{2x+Sy)  +  21y3. 

15.  l-2a;4-4x2-8.c3. 

IG.    t/5+u4it-|-f/3i2c2_Ua253c3^«i4c*+6''r». 


FACTORING.  85 

Art.  XXVI.  The  principles  iilnstrated  *in  Section  II.,  chap. 
II.,  may  be  applied  to  factor  various  algebraic  expressions,  ap  in 
the  following  cases  :  ^ 

Examples. 

1,  Find  the  factors  of 

{a-{-b-\-c){ab+bc-i-ca)~~{a  +  h){b+e){c+a). 

1st.  Observe  that  the  expression  is  symmetrical  with  respect 
to  a,  b,  c. 

2nd.  If  there  be  any  monomial  factor  a  must  be  one.  Put- 
ting a  =  0,  the  expression  vanishes.  .".  a  is  a  factor, 
and,  by  symmetry,  b  and  e  are  also  factors.  .".  abc 
is  a  factor. 

•8rd.  There  can  be  no  other  literal  factor,  because  tlie  given 
expression  is  of  only  three  dimensions,  and  ahc  is  of 
three  dimensions. 

4th.  But  there  may  be  a  numerical  factor,  vi  suppose,  so  that 
we  have 
{a-\-b+c){ab-{-bc-^ca)—  {a  +  b){b-\-c){c  +  a)  =:mabc. 

To  find  m,  put  a  =  h  =  c  =  l  in  this  equation,  and  w  =  l, 
.".    the  expression  =  aic. 

2.  Eesolve  a^{h-c)+b^{c-a)+c^{a-b). 

1st.  For  rt  =  0  this  does  not  vanish.  .*.  a  is  not  a  factor, 
and  by  symmetry  neither  is  b  nor  c. 

2ad.  Try  a  binomial  factor ;  this  will  likely  be  of  the  foi  ni 
b  —  c  ;  put  0  — 0  =  0,  i.e.,  h  =  c  in  the  given  expression, 
and  there  results 
a.2[c-c)+c^{c—a)+c^{a-c),  which  =  0, 

.'.  b~c  is  a,  factor,  and  by  symmetry  c  —  a  and  a  —  b  are  fac- 
tors. Since  the  given  expression  is  only  of  three 
dimensions,  there  can  be  no  other  literal  factor ;  but 
there  may  be  a  numerical  factor,  m  (say),  sq  that 

a^ib-c)-^r'{c-a,)-{-c^{a-b)  =  m{a-b)(b-c){c-a). 
To  find  the  value  of  m,  give  a,  b,  c,  in  this  equation,  any  values 
which  will  not  reduce  either  side  to  zero;  leta=l,  b  =  2,  c  =  0, 


8G  FACTORING. 

and   we  iiave  2  =  m(  —  2),  or  to=  —  1 :  so  that  the  given  expres- 
sioii=  —  (a  —  b)(J)  —  c){c  —  a),  or  {a-b)[b  —  c){a  —  c). 

3.  Eesolve  aS{h-[-c^)+b\{c  +  a^)+c^{a+b^)+abc{abc+l). 
Here  we  see  at  once  that  there  is  no  vwnomial  factor : 

put  &+6'2  =  0,  i.e.,  h—  —c^,  and  the  expression  becomes 
«3(-r3+c2)-c6(c  +  rt2)+c3(a+c4)-f3«(_c3a+l)  which  =  0; 
.*.  i  +  c2  is  a  factor,  and  by  symmetry  c  +  a^  and  a+b^  ars  also 
factors  ;  and  proceeding  as  in  former 'examples  we  find  m  =  l ;    /. 
the  expression  =  (6 +  c2)(c-f-«^)(«  +  i^). 

4.  Eesolve  into  factors  the  expression 

(a-i)3+(6-c)3  +  (c-a)3. 

As  before,  we  find  that  there  are  no  monomial  factors. 

Let  a  —  b  =  0,  or  a  =  b,  and  substituting  b  for  a  the  expression 

becomes  zero  ;  hence 

a  —  b  is  a  factor. 

By  symmetry,   b—  c         " 

and         c—a         " 

Hence  the  factors  arc 

vi(a  —  b)(b  —  c){c~a). 

To  find  TO  let  a— 0,  6  =  1,  r,  =  2,  and  we  hav« 

6  =  2m,  or  «i=3. 
The  factors  are,  therefore, 

S{a-b){b-c){c-a). 

5.  Eesolve  into  factors 

a^{b  -  c) -f-63(c--a)+c3(a  -  b). 
A-s  before,  we  find  that  there  are  no  monomial  factors. 
Let  a~b  =  0,  or  a  =  b  ',  substituting  b  for  a,  the  expression  be^ 
comes  zero  ; 

therefore  a—b  is  a  factor. 

By  symmetry     b  —  e         " 
and     c  —  a         " 

Now  the  product  of  these  three  factors  is  of  tJiree  dimensions, 
while  the  expression  itself  is  of  four  dimensions.  There  must, 
therefore,  be  another  factor  of  one  dimension.     It  cannot  be  a 


FACTORING.  87 

monomial  factor,  for  the  expression  has  no  such  factors.  It  can- 
not be  a  binomial  factor,  such  as  a+b,  for  then,  by  symmetry, 
6+c  and  c+a  wonlii  also  be  factors,  which  would  give  an 
expression  of  six  dimensions.  It  cannot  be  a  trinomial  factor, 
unless  a,  b,  and  c  are  similarly  involved.  For  instance,  if  a  —  b+c 
were  a  factor,  then,  by  symmetry,  6  — c-f  a  and  c  —  a-{-h  would  also 
be  factors,  and  the  dimensions  would  be  six  instead  oifour.  The 
other  factor  must,  therefore,  be  a-\-h-{-c.     Hence, 

a^{h-c)+b^{c-a)-^c^{a  —  b)=m{a—b){h  —  c){c-a){a-\-o  +  r). 

To  find  m,  put  a  =  0,  6  =  1,  and  c  =  2,  and  we  have 
—  6  =  6v//, ; 
.*.    m  =   —  1. 

Hence  the  factors  are 

—  (a  —  b){b  —  c){c  —  a)(a  -\-h+c), 
or,         {a  —  b){a  —  c)(b  —  c){(i-^b  +  c). 

6.  Prove  that  «• 

a3+b3  +  c^-]-S(a-hh){b  +  c){c  +  a) 
is  exactly  divisible  by  a-j-6  +  c,  and  find  all  the  factors. 

Let  a4-ft4-c  =  0,  or  a=  —{b-{-c);  substituting  this  value  for  n, 

we  have 

-  (b+c)^  +  b^  -\-c^-^Sbc{b-^c),  or 
-(6  +  c)3  +  (i  +  (;)3  which  =  0,  and 

therefore  a-\-b-^c  is  a  factor. 

As  before,  we  find  that  there  are  no  monomial  factors.  Since 
a-{-b-{-c,  the  factor  already  obtained,  is  of  one  dimension,  the 
other  factor  must  be  of  two  dimensions,  and  cannot,  therefore,  be 
a  binomial ;  for  if  a-\-b  were  a  factor,  by  symmetry  i  +  c,  and  c  +  a 
must  also  be  factors.  The  factors  in  that  case  vv'ould  give  a 
qi;antity  of  four  dimensions,  while  the  expression  itself  is  only 
oi  three  dimensions.  Nor  can  a^-\-b^-\-c^  be  a  factor.  For 
if  so,  the  other  factor  must  involve  a  numerical  multiple  of  the 
first  power  of  a,  and,  therefore,  on  taking  the  first  power  of  a  out 
of  terms  involving  first  and  third  powers,  we  should  have  left 
some  numerical  multiple  of  a^-\-b^+c^,  instead  of  wiiich  we  get 


58  FACTOBINa. 

a2_|-3(6-]_e)».  Nor  can  af-i-{h+cy  be  a  factor,  for  symmetry 
would  require  two  other  factors,  viz. :  b- -{-{c-i-a)-,  awl  c-  +(«  +h)^, 
tiius  giving  a  quantity  of  seven  dimensions. 

The  only  factor  admissible  is,  therefore,  {a-^h+c)^. 
Hence 

a^-i-h'-^  -i  9^--hB{a-*-b){b+c){c  +  a)  =  ?n{a-\-h  +  i:\a-Jrb  +  :)' 

=  m{a+b-![-c)'^. 
To  find  m,  let  a  =  l,  6  =  0,  and  c  =  0,  and  we  nave  1  =  »«. 
Hence  the  factors  are 

(^a  +  h  +  '^)(a  +  h  +  c){a+b+c). 

7.  Simplify 

a{b+c)^  +b{a  +  cy  -^c{a  +  b)^  —  {a-'rb)(a-c){b-c) 
-{a-b){a-c){b  +  c)  +  {a  —  b){b  —  c){a  +  c). 

Let  a  =  0,  and  the  expression  becomes 
bc"-^cb^-{-bc{b—c)  —  bc(b-^c)  -bc{b-  c),  vriiich  equals  zf;ro  ;  there- 
fore a  is  a  factor  ;  by  symmetry  b  and  c  are  also  factors. 

The  expression  is  of  three  dimensions,  and  abc  is  of  three 
dimensions,  there  cannot  therefore  be  any  other  hteral  ff,ctoi\ 

Hence  the  expression  =viabc. 

To  find  m,  let  a  =  b  =  c=l,  and  v/e  have 

m  =  12. 
.-.  the  expression    =12abc. 
In  the  preceding  examples  the  factors  have  been  linear,  but  the 
principle  applies  equally  well  to  those  of  higher  dimensions.    (See 
Th.  ii.  Cor.) 

8.  Examine  whether  «"  +  !  is  a  factor  of  a;3"  +  2a;-"+3.f"  +  2. 
Let  a;" +  1  =  0,   or  x-"=— 1,  and   substituting,    the   eiipvessiou 

vanishes,  therefore,  x"-\-l  is  a  factor. 

9.  Examine  whether  a^  +  b^  is  a  factor  of 

2a4+a36+2a2t/2+a^3, 

Let  a^  +  b-  =0,  or  a^  —  —b",  substituting,  we  have 
2^-i_a63_2i4+a63  which  =  0,  and 
therefore     <^^-\-  6^  is  a  factor. 


FACTORING.  89 

10.  Prove  that  a^+h^  is  a  factor  of 

a^-^a^^b+a^h^  +  a-b-'^ab^  +  b'^. 
Lei  a3_}./,3_o,  or  a^  =  -b^;    substituting,  we  have 
-a?b^  -ab^-b^+a^b^-^ab^-{-b^,  wliich  =  0,  and 
therefore  a^-{-b^  is  a  factor. 

Exercise  xxxvii. 
Eesolve  into  factors 

1.  {x-i-i/+z)^  —  {x^+y^+z'^)' 

2.  bc(b  —  c)—ca{a~(;)  —  db{b~ti). 

8.  (a3-63)3  +  (62-f=^)3+(c2-aa)3. 

4.  x{y+z)^+7j{z+xy+z{x  +  ijy-4.xijz. 

5.  (^a+b)^  ^{b-\-c)^  +  {c-  a)^. 

6.  a(6-c)3+6(c-«)3  +  c-(«-6)^ 

7.  (a-i-b-\-c){ab-{-bc-{-ca)—abc. 

8.  a^{c-b^)  +  b^{a-c'^)+c^{b~-a^)+abc{abc-l). 

9.  «3(6  +  (;)4.i2(f  +  a)  +  c2(a  +  /^)4-2(/ic. 

10.  (a  -b){c-h){c-k)-{-{b-c){a-h){a-k)  +  {c-  a){b-  'i){b-Ic). 

11.  a;4^3+,c2^4+a;4s3_^a;2z4+^423^y3244.2a;2^223. 

12.  (a-^)5  +  (fe-6-)5+(c-a)s 

13.  afc(a+6)+M^+c)+«a(f+rt)+(a3  +  6»+c-3). 

14.  a4(c-63)+64(rt_c3)+c'i(6-a3)4-a66-(a262c3-l). 

15.  a;4(y3_23)+^4(23_^2)4.24(a;2__,/3). 
IG.    a;*  +  ^4  ^24  _2aj22/3_  2^223  _  222^3. 

17.  (6-c)(a;-fc)(x-c)+(c-a)(a;-c)(a;-a)4-(«-^)(^-«)(^-6). 

18.  {a+by^  +  {h  +  c)^  +  {c  +  a)s  + 

B(rt+26  +  6-)(6+2c+rt)(6-+2a+6). 

19.  Shew  that  a^  +(t^b^-ab^  -  b^  has  a^  -b  for  a  factor. 

20.  Shew  that  {x  +  yY  -x'  -y'  =lxy{x+y){.v^  +.'-V/  +  ^/-)^ 

21.  Examine  whether  x^  —  5x-\-iJ  is  a  factor  of 

^3_9^2_^26a;-24, 


90  fACTOEINO, 

22.  Shew  that  a  —  b-\-c  is  a  factor  of 

23.  Shew  that  a"  +Sh  is  a  factor  of 

and  find  the  other  factor  . 
24.  Find  the  factors  of  W^ib  -  c)-i-b^{c  ~  a)-{-c^(a—b'). 


Section  V.. — Factoring  a  Poltnome  by  Teial  Divisors. 


Art.  X,XVII.  To  find,  if  possible,  a  rational  linear  factor  of 
the  poiynome. 

a;»^6a;"-i+ca;"-'+ ^^nx+k. 

Substitute  successively  for  x  every  measure  (both  positive  and 
negative)  of  the  term  k,  till  one  is  found,  say  m,  that  maT\es  the 
polynome  vanish,  then  x  -  m  will  be  a  factor  of  the  polynome. 

Examples. 

1.  Factor  a;3+9a;2+16a;+4. 

The  measures  of  4  are  +1,  +2  and  +4.  Since  every  coefS- 
cient  of  the  given  polynome  is  positive,  the  positive  measures  of 
4  need  not  be  tried.  Using  the  others <•  it  will  be  found  that  —  2 
makos  the  polynome  vanish ;  thus 

1        9        16        4 

-2     -14     -4- 


-2 


17  2;       0 

Hence  the  factors  are  (a3  +  2)(x3-|-7a;-f2). 

The  labour  of  .substitution  may  often  be  lessened  by  arrang- 
ing the  polynome  in  ascending  powers  of  x,  and  using  1  -h 
(measure  of  k)  instead  of  the  measures  of  k.  (This  is  really 
substituting  1  -r  measure  of  k,  for  l^x).  Should  a  fraction 
occur  during  the  course  of  the  work,  further  trial  of  that  measure 
of  k  will  be  needless. 


FACTORING. 


91 


Examples. 

2.  Factor  x^  -  lOz^  -  63a;+60. 

The  measures  of  60  are  +1,  +2,   ±3,  ±4,   +5,  &c.     Neither 
4-1  nor  -  1  will  make  the  polynome  vanish.     Try  2 ;  thus 

60         -63         -10         1 


1^ 
2 


63 
30 


30         -16^ 

A  fraction  occurring  we  need  go  no  further.     —  5  will  also  give 
a  fraction,  as  may  easily  be  seen.     Next  try  3  ;  thus 

60         -63         -10         1 


•63 

20 


20 


-14i 


A  fraction  again  occuring,  we  may  stop. 
fwaotion.     Next  try  4  ;  thus 


—  3  will  also  give  a 


1 

T 


60 


-63 
15 


^10 

-12 


Next  try  —4. 


-1^ 

T 


15 


60 


-12 


-63 

-15 


5* 


-10 


15         -19i 
Next  ti:yiu^  5  we  find  it  fails,  then  try  —  5,  thus 
60 


-1 


-63 
-12 


-10 

15 


1 
-1 


12         -15  1;       0 

The  remainder  vanishes  as  required  ;  the  factors  are,  therefore, 
(a;+5)(a;3-15a;+12). 

Art,  XXVIII.  When  k  has  a  large  number  of  factors,  the 
number  that  need  actually  be  tried  can  often  be  considerably 
lessened  by  the  following  means. 

Add  together  all  the  coefficients  o£  x  (including  the  constant 
term  k) ;  let  the  sum  be  called  k^. 


92  FACTORING. 

From  the  sum  of  the  coefficients  of  the  even  powere  of  x 
(including  k)  take  the  sum  of  the  coefficients  of  the  odd  powers  of 
x;  let  the  remainder  be  called  Jc^.  (In  the  coefficients  are  in- 
cluded the  signs  of  the  terms). 

1st.  If  k^  vanish,  x  —  1  will  be  a  factor  of  the  polynome. 

2nd.  If  /Cg  vanish,  x+1  will  be  a  factor  of  the  polynome. 

3rd.  If  both  k^  and  Ag  vanish,  x^ —1  will  be  a  factor  of  the 
polynome. 

4th.  If  neither  ky  nor  k^  vanish,  (writing  p  for  "  a  positive 
measure  of  ^  greater  than  1 ") ; 

(a)  We  need  not  try  the  substitution  of  p  for  x  unless  ^  —  1  be 
a  measure  of  ky,  and  ^-fl  a  measure  of  k^. 

{0)  Nor  need  we  try  the  substitution  of  —p  for  x  unless  p-^-l 
be  a  measure  oi  k^,  and  p  —  1   a  measure  of  kc^. 

(In  trying  for  measures,  the  signs  of  k^  ky,  and  k^  may  be 
neglected. 

Examples. 

1.  Find  the  factors  of  x^  -  lOx^  -  63a;+G0.     (^ee  Ex.  2  above). 

Here  ^  =  60  ;  ^1=     1 -10-63  +  60= -12, 
A;2  = -1-10+63+60  =  112. 

Tabulating  the  trial-measures  we  get 


12 

1,' 

2, 

3, 

4, 

60 

2, 

3, 

4, 

5,     6, 

10,     12, 

112 

4, 

7, 

12 

3, 

4, 

6, 

60 

2, 

3, 

5,     6, 

10, 

112 

1, 

2, 

4, 

(It  is  evident  that  12  is  the  highest  measure  of  60  we  need  try 
in  the  upper  table,  for  the  next  measure,  15,  would  give  14  as  a 
trial-measure  of  12,  and  higher  measures  of  60  would  give  higher 
trial-measures.  Similarly,  10  is  the  highest  measure  that  need 
be  tried  in  the  lower  table.) 


FACTORING. 


93 


In  the  upper  table,  8  is  the  only  measure  of  60  that  gives  a 
full  column ;  hence  of  the  positive  measm-es  of  60  we  need  try 
only  the  substitution  of  3  for  x. 

In  the  lower  table,  2,  3,  and  5  give  full  columns,  hence  we 
must  try  the  substitutions  —2,  —3,  —5  for  x. 

On  trying  the  four  substitutions  to  which  we  are  thus  restricted 
we  find  —  5  is  the  only  one  for  which  the  polynome  vanishes. 
(See  Ex.  2  above). 

-      2.  Find  the  factors  of  x^+12x^  -4:0x^  +Q7x-120. 
&=-120;  /.-i=H-12--40+67-120=  -80; 
A,  =  1  - 12 -40  -  67  - 120  =  -  238. 


80 

120 
238 

1, 

2, 

2,  4,  5, 

3,  4,  5,  6, 

7, 

8, 

10. 

12, 

15,  &c. 

80 
120 
238 

2, 
1. 

4,  5, 

3,  4,  6,  6, 
2, 

8. 
7, 

10, 

i 

16, 

15,  20, 
14,  21, 

24, 

&c 

The  upper  table  gives  us  6  as  a  trial-measure,  and  the  lower 
gives  us  —3  and  —15. 


Trying  these 

)  we  get 

-120 

67 

-40 

12 

1 

1 

-20 

6 

-  20 

U 

-120 

67 

-40 

12 

1 

-1 

40  - 

3 

-40 

36f 

-120 

67 

-40 

12 

1 

-1 

8 

—  5 

3 

-1 

16 

-  8 

6 

-  3 

1: 

0 

94 


FACTOEINO. 


Hence  a;+ 15  and  x^  —  Sx^  +  5x  -  8  aie  the  factors.     The  latter 
cannot  be  resolved,  for  our  tables  above  tell  us  we  need  try  only 
x—6,  x+3,  and  a;+15.     The  first  two  have  been  found  not  to  be 
factors,  and  15  will  not  measure  8. 
4.  Factor  a;"  _  27a;2  +  Ux+ 120. 

k=120]  ^'i  =1-27+14+120  =  108 
/.•2  =  1-27-14  +  120=    80. 


108 

1, 

2, 

3, 

4, 

9 

120 

2, 

3, 

4, 

5, 

6, 

8, 

10, 

12, 

15,  &c 

80 

4, 

5, 

16, 

108 

3, 

4, 

6, 

9 

120 

2, 

3, 

4, 

5, 

6, 

8, 

10, 

12, 

15,  &c 

80 

1, 

2, 

4, 

5. 

The  upper  table  gives  us  3  and  4,  the  lower  table  gives  us  -  2, 
—3,  and  —5.     Using  these  in  order  we  get 


Hence  x  —  3  is  a  factor. 

Hence  a;  — 4  is  a  factor. 

Hence  ar+2  is  a  factor, 
and  there  remains  x-\-o,  a  factor. 

Hence  the  factors  are  {x-B){x—4:){x-\-2,){x  +  5). 
5.  ¥&ctor  x*-px^-\-{q-l)x^+px    q. 

k=-q;  k^  =  l-p  +  {q-l)+p-q  =  0; 
k„  =  l  +p+{q-l)  -p-q  =  0. 
Since  both  ki  and  /c,  vanish,  the  polynome  is  divisible  by  both 
x—1  and  x-\-l. 


1 

120 

14 

40 

-27   0   1 
18  -3  -1 

3 
1 

40 

18 
10 

-3-1;  0. 

7   1 

4 
-1 

10 

7 
-5 

1;  0 
-1 

2 

5 

1; 

0 

1 

1 

-V 

1 

q-1 
-p  +  1 

P 

q-p 

1 

1 

1 

-;,+l 
-1 

q-p 
+P 

9; 
-9 

0 

1 

-P 

9; 

0 

FACTORINa. 


95 


Hence  the  othei-  factor  is  x^  —px-^q. 

6.  Factor  x^+2ax^+{a^  +  a)x-'-{-2a^x+a^. 

k^  =  1  -2a+(a2  +a)  - 2a^  +a^  ^a^-a^ -a-1. 

The  positive  measures  of  k  are  1,  a,  a^,  a^.  Of  these  1  may 
be  rejected  at  once,  since  neither  k^  nor  k^  vanish,  and  a^  and  a^ 
may  also  he  rejected  since  k^  or  (a +  1)3  is  not  divisible  by  eithei 
a^  +  l  or  a'  +  l.  But  k^  is  divisible  by  a-\-l,  and  k^  is  divisible 
bya— 1;  thus  we  need  only  try  the  substitution  of  -aforx. 
(See  4  j5,  page    92) 


1 

2a 

a2+a 

2a3 

flS 

—  a 

—  a 

-a2 

-a3 

-a? 

1 

a 

a 

a3; 

0 

—a 

—  a 

0 

-a2 

1 

0 

a; 

0 

Hence  the  factors  are  {x-{-ay^{x^-\-a). 

7.  Factor  a;3_(rt+c)a;2 +(&+ac)a; -6c. 
^:    =  —  6c  ; 

4j  =      l  —  {a+c)-^{b+ac)  —  bc  =  l—a+b—c  +  ac  —  bc 
fe,  =  -  1  -  (a+c)  —  (6+ac)  -  5c  =  —  (1  +a+i+c  +  ac+6c). 

The  factors  oi  k^,  other  than  1,  are  b  and  c.  k^is  not  divisible 
by  either  6+1  nor  by  c+-l.  However,  Zr^  is  divisible  byc-1, 
and  k^  is  at  the  same  time  divisible  by  c+1,  /.  we  need  only  try 
the  substitution  of  c  for  x.     (See  4  «,  page  86). 


(a+c)  (6  +  ac)      —6c 

c  —  ac  ic 


1  —a  6; 

Hence  the  factors  are  {x—  c){x^  —  ax+b). 


96 


FACTORING. 


Exercise  xxxviii. 


1.  a3_9„2_^l(5a_4. 

2.  a;3-9a;2+26a;-24. 

3.  a;3-7a;2+lGx-12. 

4.  x^-Ux+16. 

5.  x^  +  Sx^  +  5x-\-B. 

7.  x^-dx+2. 

8.  a;4  +  2a;2+9.  22 

9.  m^-Sm^-n+^mn^-27i^.         23 

10.  x^-\-2x^-\-2.  24 

11.  jftS  _  5m2n-\-8t7in^  —  4n^. 

12.  63+62c+76c2  +  39c3. 

13.  m^  —  4mn^  +  3n'^. 

14.  a4_7a36  +  28rti3_i664. 

29.  x^-18x^  +  UBx^-288x+252. 

30.  a;4-9a;3i/4-29x22/2_3ga;i^3T-i8y4, 


15.  a;3_llx2+39a;-45. 

16.  x^-\-5x2+lx-\-2. 

17.  «3_3o,2_i93rt_|.i95. 

18.  p^-Sp^-6p-8. 

19.  a4+3,t3_3rt2_7fl_|_a; 

20.  a'''-6a*"+lla"'-6. 

21.  aA-Ua^b~  +  lGb^. 

rt4_rt3/,2_2a63H-264. 


^,3_4^;2^67;-'4. 

a;'"+4a;""  -  5. 
25.  y4-5?/M'8i/2-8. 
26    a4-2a3  +  3a3-2«+l. 

27.  a3+a253^a62_363. 

28.  2a''' -a'" -a" 4-2, 


Art.  XXIX.     To  find,  if  possible,  a  rational  linear  factor  of 
the  polynome 

aaf+bx''-^-\-car-^+ +hpB  +  k. 

First  Method.     Multiply  the  polynome  by  a"~^ 

(axy+b{ax)'^^+ac{azy-^'  + -^a^-'hiax)  +  a'^-'k ; 

or  writing  y  for  aXy 

ynj^lyn-i  ^  acy"-^  + +«"-'*%  +  a^-^k. 

Factor  this  polynome  by  the  method  of  the  last  article,  replace 
y  by  ax,  and  divide  the  result  by  a"~\ 

Example. 

Factor  3j;4-|-5a;3  -33a;2+43a;-20. 
Multiply  by  33  and  express  in  terms  of  Sx. 

(3a;)4  +  5(3a;)3-99(8a;)2+387(3a5)-540  ; 
or,    2/4+52/3-99^/3 +  387// -540. 


FACTORING. 


97 


Here /t=: -540;  i5:i  =  1  +  5-99+387 -540==  -246: 

5  -  99  -  387  -^540  =  - 1030. 


k,  =  l 


246 

640 

1030 

246 

640 

1030 


1,  2,     3,     6,     41,     82,     123,     246. 

2.  3,     4, 

5, 


3, 
2, 
1, 


6,     41,     &c.  (Trying  by  factors  of  246 

5,  instead  of  by  factors  of  640, 

for  convenience). 


The  only  factors  of  540  in  full  columns  are  4  in  the  upper 
table  and  2  in  the  lower  one ;  hence  we  need  try  only  the  substi- 
tutions 4  and  —2. 


1 

-540 

387 
-135 

-99 
63 

5 

-9 

1 

-1 

4 

-135 

63 

-   9 

-1; 

0 

Hence  ?/- 4  is  a  factor.     The  substitution   —2  need  not  now 
be  tried,  since  we  see  that  135  is  not  a  multiple  of  2.     The  other 
factor  is  therefore  y^-\-2y^ —Q^y-^1^6. 
Replacing  y  by  3a;  and  dividing  by  27  ; 

^V(3a;  -  4)(27a;3+-81aj8-189a;-}-135) 
=  (3a;  -  4)(a;3 +3a;2 -7a;+5), 
wnich  are  the  factors. 

Art.  XXX.  Second  Method.  Writing  m  for  "a  measure  of 
a,"  and  p  for  a  "  measure  of  k,  positive  or  negative  ;" 

For  aj  substitute  every  value  of  p-wtill  one,  say;?'-r-m'  bb 
found  which  makes  the  polynome  vanish  ;  then  m'x—p'  will  be 
a  factor.  Should  a  fraction  be  met  with  in  the  course  of  substi- 
tution, further  trial  of  that  value  oi  p-rvi  will  be  useless. 

Should  k  have  more  factors  than  a,  it  will  generally  be  better 
to  arrange  the  polynome  in  ascending  powers  of  x  and  use  values 
of  m^p  instead  of  p-i-m,  making  2^  positive  and  ?«  positive  or 
negative. 


98  FACTORn<[G. 

To  reduce  the  number  of  trial-measures,  calculate  k^  and  k^,  as 
directed  on  page  92,  tken  1,  2,  3  hold  as  on  that  page,  but  in  4 
read^  — m  for^  — 1  andp-j-m  for^j+l.  , 

1^^  Examples. 

1.  Factor  36a;3  + 171x2 -22X+480. 

;t  =  480,  yti=     36+171-22-1-480  =  665 
;;;2= -364-171+22+480  =  637. 
m  may  have  any  of  the  values  +1,  ±2,  +3,  +4,  ±6,  +9, 

+  12,  +18,  +36. 

In  forming  the  table  write  out  the  measures  of  ^j ;  take  each 
measure  in  succession  and  add  to  it  each  value  of  m  separately, 
should  the  sum  measure  480,  i.e.,  k,  add  to  it  the  same  value  of 
m,  and  should  the  new  sum  measure  637,  i.e.,  k^,  keep  the  mea- 
sure of  480,  writing  above  it  the  value  of  m  used.  Should  the 
sum  in  either  case  iiot  be  a  measure,  another  value  of  ni  must  be 
tried  ;  "^hen  all  the  values  of  m  have  been  tried,  another  measure 
of  665,  i.e.,  k^  must  be  tried  till  all  have  been  tested.  (Measures 
of  yfc,  or  665  have  been  used  in  this  instance  because  they  are 
much  fewer  than  those  of  480  ;  measures  of  k^  or  637  would  have 

done  equally  well). 

• 

m=+3,  +1,  +3  -2  -8  -9  -3 

665        1,  5,  7  5  7  19  19 

480      4,  6,  10  3  4  10  16 

637       7,  7,  13  1  1  1  18 

Hence  the  only  substitutions  that  need  be  tried  are 

8J_8      z^     zl     Zl     Z^     iox   L 
T'    6'    To'       3'       4'      10\    16'  x' 

Arrangement  in  ascending  powers  of  x. 

By  actual  trial,  as  below,  we  find  ^1  is  the  only  one  of  these 
giving  a  zero  remainder. 


FACTORING. 


99 


4 
1 

3 


10 

-2 

8 
-8 

4 
-9 


480 


-   22 
360 


171 


86 


10 
-3 


120 

84i 
80  - 

80 

144 

48 

12-2 
-320 

228 

-266 

160 

-114 
360 

133; 

-230 

120 

-  95i 
-432 

48 

-45-4 
-  90  . 

21 

-36 

16         30         -     7  12;  0 

(The  coefficients  are  written  only  once,  and  understood  for  the 
other  hnes  of  substitution.) 

Hence  the  factors  are  3a; +  16  and  12a;2  _  735+30, 
■     The  latter  factor  cannot  be  resolved,  for  16  will  not  measure 
30,  and  all  the  other  factors  left  iox  trial  by  the  tables  above, 
have  been  tried  and  have  failed. 

2.  Factor  10a;* -a;3(15|/+4z) -a;3(40y» -6y«)  + 

Here  m=  ±1,  +2,  ±5,  or  ±10.     k=  -2.4t/»z. 

k^  =  l0-{15y+iz)-(4:0y^-eyz)  +  (G0y^-\-16y^z)-2iyH 
=  10- 151/ -  402/2-1-602/3 -2;s(2-3y- 82/2 -M2?/S) 
=  (5-2z)(2-3y-82/24-12.y5). 
A--  =  (5  +  2z)(2+3?/—8y2 —  127/3),  as  may  easily  be  found 
by  making  the  calculation. 

We  get  at  a  glance   2z  a  factor  of  k,  2z  —  5  a  factor  of  Ic^,  and 
2z+5  a  factor  of  k,^  ;  hence  taking  w  =  5,  we  are  directed  to  trj' 

2z 
the  substitution  -_  for  x. 
5 

10     -{15y+iz)      -{iOy^-6yz)      {my^  +  16y^z)      -24^/3- 

4z  —Qyz  .  -16y'-z  24?/ "^2; 

^%2  12^^3';  0 


6 


2     -3y 


100 


FACTORING. 


Hence  5x~2z  is  a  factor,  tlie  other  being 

2x^-3x^y-8xy2-i-12y3. 

•    The  latter  factor  being  homogeneous,  the  method  of  this  article 
may  be  appHed  to  it. 

'^i=+lor±2,    k  =  12,    ki=8,    k^  =  15. 
m  =  l,         2,         1,  -1 

1,         3,  3  The  other  columns 

3,        4,  2  are  not  full. 

5,         5,  .1 

Hence  the  trial- substitutions  (arrangement  m  ascending  powers 
of  x)  are  i,  |,  ^,  :±. 


3 

1, 

12 

2, 

15 

3, 

12 

-8 

-3 

2 

1 

6 

-1 

-2 

2 

6 

-1 

-2; 

0 

2 

4 

2 

3 

2 

1; 

0 

Pinal  factor  is  2>j+x» 

Hence  the  factors  are  {z—2i/){2x-3i/){x+2y),  and  these,  with 
the  factor  5x  —  20  already  found,  give  the  complete  resolution  of 
the  polynome  proposed. 

(The  factor  5x  —  2z,  might  easily  have  been  got  by  the  method  of 
Art.  XXIII.,  page  79,  but  the  present  solution  shows  we  are  inde- 
pendent of  that  article.  It  may  also  be  obtained  by  rearranging 
the  polynome  in  terms  oiy). 

Exercise  xxxix. 

Factor 

l.aa;3_20fl;»+38a;-20;  x^ -Ix^y  +  lQxy^ -I2y^. 

2.  12x^  +  5x^y-\-xy^+dy^  ;  8a;3_i4a;+6. 

3.  Bx^-15ax+a^x-5a^;  2x^-\-9x^y-\-7xy^  -Sy^. 

4.  2^*4_7J3c_462c2  +  6c3-4c4  ;  15a3+47a36  +  13a6»- 1263. 

6.  UOx'^  -725a;3y+98ia;3y2  ^Q20xy^  - 1152^4, 

7.  d6x^-6{9  -7y)x^ - 7{9  +  Uy)x^y+3{4:9-40y)xy^  +  180y3. 

8.  lOx*  -x^l5y  +  iz)  +  x^{4:0y^-\-Gijz)+x{60y^ -Hj^-^z)-24.y^z 


DIVISION.  101 


CHAPTEE  IV. 


Section  I. — Division.      Measures  and  Multiples. 


Art.  XXXI.  When  one  quantity  is  to  be  divided  by  another 
the  quotient  can  often  be  readily  obtained  by  resolving  the  divisor 
or  dividend,  or  both,  into  factors. 

Examples. 

1.  Divide  a'i-2,ab-{-b^ -c^-{-2cd-d^  hy  a— \b+c—d.  Here 
we  see  at  once  that  the  dividend  ={a—b)^-{c  —  d)^,  and  .•.  quo- 
tient =  a-b  —  {c-d)  =  a  —  b  —  c-^d. 

2.  Divide  the  product  of  a^-\-axi-z^  and  a^-\-x^  by  a*'-{-a^x'^ 
•{■z^.  Here  a^-[-x^  =  {a-\-x){a'^  —  ax-\-x^),  and  the  divisor  = 
(a^-^ax+x^){a^  —  ax+x^)  :.  the  quotient  is  a -fx. 

3.*Divide  a^+a^b+a^c—abc-b^c-bc^  by  a^-bc.  The  divi- 
dend is a{a^  —bc)-\-b{a^  -6c)4-c(a*  —be)  /.the  quotient  =a+b-Jr-c, 

4.  (a3+b^-c^-^3abc)^{a-{.b-c). 

Dividend  =a^-\-b^+Sab{a+b)-G^ -3ab{a+b)-\-SabG={a'^b)^ 
—  c^  —  Bab{a -{-b  —  c)  which  is  exactly  divisible  by  a +6— c;  quotient 
=a^-^b^ -\-c^ —ab-\-bc-\-ca. 

5.  Divide  x^ —x^y-{-x^y^ —  x^y^ +xy^—y^  bjx^  —  y^. 

The  dividend  is  (Art.  XXV.)  evidently  (x^  —y^)  -^  (a'+*')j  and 
this  divided  by  x^  —y^  =  (^x^-\-y^)  -h  (x+y)  =  x^  —  xy-\-y^ . 

6.  Divide  b{x^  +a.3)-{-ax{x^  -  a^)+a^{x-{-a)  by  {a-^b){x^-a). 
Striking  the  factor  x  +  a  out  of  dividend  and  divisor  we  have 
b{x^  —  ax-\-a^)-\-ax{x  —  a)-^a^  =  b(x^  —  ax  +  a^y-\-a{x^  —  ax-\'0-) 
=  {a-\-b){x'^  —  ax+a^)  .'.  quotient  =x^  -ax-\-a^. 

7.  Divide  apx^  +x^(aq+bp)-\-x^{ar-\-bq-\-pc)-\-x{qc-\-br)  +cr  by 
ax'-^bx-i-'' 


102  DIVISION. 

Factoring  the  dividend  (Art.  XXIII.)  we  have 

(ax^  +  bx+c){px^  -^qx+r). 
:.   the  quotient  =  the  latter  factoi". 

S.  Divide  Gx^  -  ISax^  +  ISa^x^  -  13a^x~  5«4  by  2a;^  -  Sax- a''. 

This  can  be  done  by  Art.  XVII.  The  divisor  is  2x^  ~a^-  Sax, 
and  we  see  at  once  that  3x'+5a'^  must  be  two  terms  of  the  quo- 
tient. 

Multiplying  diagonally  into  the  first  two  terms  of  the  divisor, 
and  adding  the  products,  we  get  -{-7a^x^  ;  but  -^ISa^x'^  is  re- 
quired. .'.  -\-Qa-x'^  is  still  required,  and  as  this  must  come  from 
the  third  term  multiplied  into  —  3ax,  that  third  term  must  be 
—  lax;    :.  the  quotient  is  3x^ -^-ba^  —2ax. 

jJoTE. — By  multiplying  the  terms  -  lax,  —  3ax,  diagonally  into 
the  ic^'s  and  a^'s  respectively,  we  get  the  remaining  terms  of  the 
dividend  ;  it  is,  of  course,  necessary  to  test  whether  the  division 
is  exact. 

9.  Divide  2«*  -  a^J  - 120252  _  s^js _{.4J4  by  ««  -  ja  -  2a6. 

Here,  as  before,  one  factor  is  a^  —  h'—1ah;  :.  ft<7o  terms  of 
the  other  factor  are  2a2-462.  Multiplying,  as  in  the  last 
example,  we  get  -&a^h^;  but  -lla^^b^  is  required.  .•.  —  Sa^fts 
is  still  needed,  and  -\-3ah  is  the  third  term  of  the  requii-ed  quo- 
tient, which  is  therefore  2a2_463_|.3a/,. 

Prove  that 

10.  (l-fa;-|-a;2+  -      -  -   -^x-'-^){l-x+x^  -   ....   -fa;"-^) 

=  l-fa;2+a;'i+    ....    +x' 
1  -  X"       1  -1-  a;" 


^i»— 2 


Product  = 


1—x        l-\-x 
l  —  a?^ 

z ^    =    1+X^+X*'i      ....    +^-»-». 

1  —X^ 


11.  Divide  (a^ -bc-y -{-Sb^c^  by  a^-\-hc. 

=  (a^  -bc)^  +{2bc)^  hy  {a^  -bc)  +  2bc 
«  (a3  _ic)3  -(«2  _  be)  X  2k-f-(26c)» 
=  a4^.4a3ic-f763ca. 


DIVISON.  103 

12.  Divide  l+23579-i7691a;9  by  l-llx+121a;3 
Dividend  =l  +  (lla;)^  ^ 

=  [1  -(llx)3+(ll.'c)«}{H-(lla:)3} 

Divisor=  {H-(lla;)3}  ^(1+lla;). 
.-.  quotient  =a-(ll£c)3+(llx)«}(l  +  lla:). 

Exercise  xl. 

Find  the  quotients  in  tlie  follo\ving  cases : 

1.  l-x-\-x^-z^-^l-x. 

2.  l-2a;4+a;»H-a;4+2a;3  +  l. 

4.  x4+4x3j/3-32?/4^a;_2?/. 

5.  l-4:X^+12x^-9x'^-^l  +  2x-Sx'^. 

6.  (rt2  -2ax+x^-)(a^+3a^x+3ax^-{-x^)  ^a^  -x». 

7.  x^—y^+z^+dxyz^x  —  y-rZ. 

8.  6a4-a36  +  2,'i262  4-13«63_f.4j4  ^  2rt» -3a6-f  4fc*. 

9.  4«4-a;32/2+6a;;/3-9^4  ^  2a;2+3»/3-arj/. 

10.  rt*+/)4_c4_2«252  -^rt3_&2_c2. 

11.  21a4-16rt3J4-16a363-5a63  +  264  -r- 3«2 -a&^-J*. 
12.-2rt3_7a3_4Grt_21  ^  2a2+7a+3. 

13.  {a^{b~c)+b^{c-a)-\-c^{a-b)}  -r  a-{-i+c. 

14.  x^-Sax-+Sa^x-a^+b^  -^x-a+b, 

15.  a;*  -  ^4+4  +  2a;2z2- 2^3-1 -7- a;3 -1/8+22-1. 

16.  a:*  — (a+c)a;3  +  (6  +  ac)x3— ica;  H- a;-c. 

17.  x^-^x^y  +xy^-{-y^  -i-  a:+?/. 

18.  x'' —x^y+x^y^  —  x'^y^-\-x^y*-x^y^+xy^—y''  -^z^+y*, 

19.  (j44.;,4_c4_2a2i2_2c3_l  -H  a2-62-c2_l. 

20.  a4-rti3_rtc3_2a3&+264  +  26c3  +  3a3c-363c-3c4 
-5-a  +  8c-26. 

21.  a^b-bx~+a^x-x^  H-  (a:  +  6)(a-a;). 

22.  a(6-c)3  +  6(c--a)3+c(a-6)3  ^a^-ai-ac+Sc, 


104  MEASUKES    AND    MULTIPLES. 

23    aH^  +  'lnhc--a^c^ -ir-c^  ^ab+ac-bc. 
2A.  x^+y^  +  -d.v.i/-l  -y  x  +  y-1. 

25.  x^-x^-2  ~  x^-x  +  1. 

26.  a^-2da2-50a-21  -^  a^~5a-l. 

27.  (2x-'y)^a^-{x+y)^a^x^+2{x+'i/)ax'^-x^  -^ 
{2x  —  y)a--t-(x  +  y)ax  —  x^. 

28.  {x^-l)a^-{x^+ao^-2)a^+{4:X^+dx+2)a-d{x+l)  * 
-=^  {x-l)a^~{x-l)a-{-d. 

Art.  XXXII.  The  Highest  Common  t  actor  of  two  algebraic 
quantities  majj  in  general,  be  readily  found  by  factoring.  The 
H.  C.  F.  is  often  discovered  by  taking  the  sum  or  difference  (or 
sum  and  difference)  of  the  given  expressions,  or  of  some  multiples 
of  them. 

Examples. 

1.  Find  the  H.  C.  F.  of  {h-c)x--\-{2ah-2ac)x  +  a"-h-a'^c,  and 
iah  —  ac-\-b'''^  —  bc)x-\-a^ c -\- ah^  —a^b-  abc. 

Taking  out  the  common  factor  b  —  c  we  get  (b  —  c){x^  +2ax-\-ub} 
and  {b  —  c)\(a  —  b)x-a^-i-ab}  ; 
.'.  b  —  c  is  the  H.  C.  F.  of  the  i^iven  expressions. 

2.  Find  the  H.  G.  F.  of 

1  —  x+y  +  z  —  xy  +  >iz  —zx-  xyz,  and 
\  —  z—y—z^xy-\-yz-\-zx—xijz. 

Their  difference  is  %j-\-2z  — 2xy-'lzx  =  2(1  — x){}j-{-z). 
Their  sum  is  2  -  2x-Y2yz  —  2xyz  =  2(1  — a;)(l  +^2). 
.'.  theH.  C.  F.  is  (1-a;). 

3.  Find  the  H.  C.  F.  of  a;"'  +  Sa;^  -8^2  -9a;-3,  and 

jc-^ -2a;4-6a;3+4a;2  +  13a;+6. 

The  annexed  method  of  finding  tlie  H.  C.  F.  depends  on  the 
principle,  that  if  a  quantity  measures  two  other  quantities,  it  will 
measure  any  multijjle  of  thek  sum  or  difference. 


MEASURES    AND    MULTIPLES. 


1 
1 

+  3 
-  2 

0-8-9-3  (a) 
-6  +  4  4-13  +  6  {h) 

0 

+   6  -12  -22-9  (c) 

2 

1 

+  6 
-  2 

0  -16  -18  -  6 
_  6  +  4  +13  +  6 

3 

+  4 

-6-12-5      (f/) 

15 
16 

+  18  -36  -66  -27 
+20  -30  -60  -25 

-2-6-6-2 

1  +  3  +  3  +  1  (f) 

25  +30  -60  -110  -45 
27  +36  -54  -108  -45 

-2  -  6  -  6  -  '2 

(a)x2 
(6) 


(c)x3 
(£i)x6 


(r)x5 
(ti)x9 

1  +  3  +  3  +~1  (gr) 
E.  C.  F.  =  (a;+l)3. 
The  coefficients  are  written  in  two  lines,  (a)  and  (6).  They 
are  then  subtracted  so  as  to  cancel  the  first  terms,  (a)  is  next 
multiplied  by  2,  and  added  to  cancel  the  last  terms.  K  (c)  and 
(d)  had  been  the  same  their  terms  would  hav.e  been  the  coefficients 
of  the  H.  C.  F.  Since  they  are  not,  we  proceed  with  them  as 
with  (a)  and  (6)  till  they  become  the  same.  When  {a)  and  (b) 
do  not  contain  the  same  number  of  terms  it  is  more  convenient 
to  find  only  (r),  aud  then  use  this  with  the  quantity  containing  the 
same  number  of  terms.  The  general  rule  is  to  operate  on  hues 
containing  the  same,  or  nearly  the  same  number  of  terms. 

4.  Find  the  H.  0.  F.  of  Sx^  +  2x^ -Ux  +  8,  and 

6a;3-lla;3  +  13a;-12. 

8    +    2-14   +   8  (a) 

6   -11   +13   -12         (6) 
6~+  4   -28  +16  (a)  x 2 

15   -41   +28         (c)  (6)-(«), 

(5_7)(3-4) 

H.  C.  F.  =  3a; -4.  (d) 

If  (a)  and  (b)  have  a  common  factor  its  first  term  must  measure 
8  and  6,  and  its  last  term  must  measure  8  and  12.     (c)  is  not 


106  ME4SURES    AND    MULTIPLES. 

therefore,  the  H.  C.  F.  Eesolve  (c)  into  factors.  5a;-7isnota 
factor  of  (a)  and  (b).  If,  therefore,  (a)  and  {b)  have  a  common 
factor  it  is  3a; -4.  On  trial  3a; -4  is  found  to  be  a  factor  of  (a) 
and  .-.    it  is  the  H.  C.  F.  of  (a)  and  (6). 

6.  1{  X-  -{-px+q,  and  a;2  +  >'a;+s  have  a  common  factor,  prove 
that  this  factor  is 

x"+ .     If  x—a  be  the  common  factor  then  the  remainders 

p  —  r 

on  dividing;  the  given  expressions  by  x—a,  must  be  zero,  i.  e., 

a^+pa+q  =  0,  and  a--\-ra+s  =  0,  or 

s—q 
i  (^  - r)a  =  *-?,.-.  a  =  ^3^,  and 

s —q  q— s 

x-a  =  x—   =x+   r~~Z' 

p —r  P  —  ^ 

6.  What  value  of  a  will  make  a^x^+{n-{-2)x+l,  and 
a^a;2-fa2— 5,  have  a  common  measure. 

They  cannot  have  a  monomial  factor.  Neither  can  they  have 
one  of  two  dimensions  unless  {a +  2}  vanishes,  i.e.,  unless  a=  —  2, 
in  which  case  the  expressions  become  Ax^  +  l,  and  4a;'^  —1,  which 
have  no  C.  F.  Hence  if  the  given  quantities  have  a  C.  F.,  it 
must  be  of  the  form  a;+»i;  dividing  a3a;2+a2 —  5  by  x-\-m,  we 
have  for  remainder, 

5  — a'  1 

fi2„i2  +  fl,2_5  =  o,  orm3=   — ~^- ;  .-.  w=  —  ^/'(S-a^),  in  which 

l/{5  —  a^)  must  be  possible  and  integral,  .-.  a2=4,  («2  =  1  gives 
values  to  m  which  on  trial  fail)  and  a=±  2,  of  which  the  positive 
^  alue  must  be  taken,  and  .-.  2a;+ 1  is  the  C.  F. 

7,  If  the  H.  C.  F.  of  a  and  b  be  c,  the  L,  C.  M.  of 

(a+6)(a3-63),and(a-6)(a3+fe3)is ^^— • 

Lei  a  =  mc,  b  =  'nc,  and  .-.  a^=Tn^c^.  b^=n^c^.     Thus 
(a  +b  )  =  c  {tn  +n  );  («  -b  )  =  c  {m  —n  ),  and 

,«.  (a  +  6)(a3  _  53)  _  c4^TO+n)(wi3  -n^),.  and 
(a-6Va3+63)  =  c4(w-n)(m3+?i3). 


MEASURES    AND    MULTIPIiBS. 


107 


The  H.   C.  F.  of  the  last  expressions  is  c'^(m*—n^),  .-.  the 
h.  C.  M.  =  c4(m6-»6\^   _v — -^_  — 

8.  If  (x—a)^  measures  x^  +  qx+r,  find  the  relation  between  q 
and  r. 

Let  a;  +  m  be  the  other  factor,  then 
x^+qx  +  r=^-^c)"{x+m)=x'^ +  im  —  2a)x%+(a^  -  2,ou>n)x-\-ma^ 
equating  coefficients, 

m  —  'ia  =  Q,a^—2am  —  q,ma^=r 

.*.»»  =  2a,  and  .'.  a^  — 4a2  =q,  2a3  =  ,-,  and 

q  q'^  r  r' 

a*  =  -  Y'  or  ««  =  -  2^ ;  and  a*  =  -^  or  a^  =  -^ 

rS  ^3  ,.8        qZ 

'''  T=~  27'  °^*    ^"^  27  =^ 
Or  thus : — 
Dividing  x^  +  qx+r  by  (a;— a)'  we  find  the  remainder 
\  {q  +  3a^)x+r-2a^ 

and  as  this  vfiil  be  the  same  for  all  values  of  x,  we  have,  by  equat 
ing  eoefOciente, 

2+3a»=0, 
and         r— 2a3  =  o, 
or  3-3=— 27a« 

and  r3  =       4a® ; 

therefore  ^  +  27  =  Of  as  before. 

Exercise  xli. 

Find  the  H.  C.  F.  of  the  followmg  : 
1.  2x^+dx^  +  ox^+9x-d,  dx^-2x^  +  10x^  -6x^3. 

a.  a;^  +  (a  +  l)x^+{a+l)x-fa,  x^  +  {a-l)x^  --{a-l)x+a. 

3.  px^-{p  +  q)x^+{p-q)x  +  q,  px^ -{p+q)x^ +(p+q)x-^q. 

4.  ax^-{a-b)x^-{b~c)x-G,  2ax^t+{a+2b)x^-\-{b  +  2c)x+o. 

5.  l-3|a;-3ia;3+ia;3_a.4,  i^i^i^a;_3a;3-[.l^a;3+a;4. 


103  MEASt)KE5    AND    MUL,TIPLES. 

7.  a^x^  i-a''  -2abx^  +  b'-'x^  +a5b'*  -2a^b,  and 

8.  {ax-\-hyY  —  {a—  b)[x-\-z){ax->rby)-^{a  -  b)^xz,  and 
(ax  —  hy)^  —  {a-\-  h)  {x-]rz)  {ax  —  by)  +  (a  +,  b)  '^xz. 

9.  «(i2-c3)  +  fe(c2-rt3)+c*(a2-62)  and 
a{b^-G'')  +  h{c^-a^)  +  c\a^-b^). 

10.  a^"'+a-"'-{-a^-Jfl,  and  a^m  „«'■*'»  ^-a^-l. 

11.  If  x^  +ax^  +bx+c,  and  x^+a'x+b',  have  a  common  factoi 
of  one  dimension  in  x,  it  must  be  one  the  factors  of 

{a  —  a')x^  +  {b-b')x+c. 

12    Determine  the  H.  C.  F.  of  (a-fe)^+(6— c)6+(c-a)»,  and 

18.  Find  the  H.  C.  F.  of 

2(7/3 _2?/2-y  +  2)a;3+3(y2_i)a;a_(2?/3_j,2_2y  +  i^,  and. 
3(2/*-4?/3+5y-2)a;3+7(2/2-2y+l):B-(3i/3-5v=5+2/-fl). 

14.  If  a;2  4-i^a;4-9,  and  x2+OTa;+"  have  a  common  hnear  factor, 
shew  that 

{n—q)^-\-  n{m—p)^=m(m—p){n  —  q). 

15.  Find  the  L.  C.  M.  of  x3-3a;2  +  3a;_l.^  a:3-a;8-a!+l, 
a;4_2a;S  +  2a;-l,  and  x^ -2a;3  +  2a;3 -2a;+l. 

16.  Find  the  L.  C.  M.  of 

a;3  +  6a;2  +  lla;+6,  a;3 +  7a;3  +  14a:+8. 

a;3+8a;3  +  19a;+12,  and  x^+dx^  +  2Gx-^24^. 

17.  Find  the  value  of  y  which  will  make 
2(y^-{-i/)x^  +  illy-2)x+4:  and 
2(^/3^-2/•^)x3+(lV-2^/)a:2  +  (2/3^-5y)a;  +  5^y-l,  have  a 

common  measure. 

18.  The  product  of  the  H,  C.  F.  and  L.  C.  M.  of  two  quantities 
is  equal  to  half  the  sum  of  their  squares,  one  of  them  is 
2a;3-lla;3+17a;-6  ;  find  the  other. 

1.9.  If  a;+a  and  x  —  a  are  both  measures  of  x^+pz^  +qx+>\ 
shew  that  pq  =  r. 


FRACTIONS. 


109 


20.  IS  x^+qx+r  and  x^  -j-mx+n   kaye   a   common   measure 
[x—a)^,  show  chat  q^n^=m^r^. 

21.  II'  the  H.  C.  F.  of  x^  +px-^q  and  x^  +mx+n,  he  x-\-a,  their 
L.  C.  M.  is 

x^  +  {m  —  a)x^-\-px^+{a^  +  mp)x-\-a{m  —  a){a^+p). 

22.  If  x--{-qx  +  l,  and  i3^jr,^3_|_^aj^l^  liave  a  common  factor 
of  the  form  x+a,  shew  that  (jb-1)3  —  ^(jo  — 1)  +  1  =  0. 

23.  'iix^-\-px''-^q,  and  x^ +mx+n,  have  x+a  for  theii-  H.  C. 
F.,  shew  that  their  L.  C.  M.  is 

x'^-\-{ni  —  a-\-p)x^-{-p(in  —  a)x'^-\-a^{a—p)x  +  a^{a-'p){m  —  a). 

24.  If  x^-\-p)x  +  l,  2i,udiX^+px"-\-qx-^l,  have  x  —  a  iox  a  com- 
mon factor,  shew  that  a  = 

1-q 

25.  Find  the  H.  C.  F.  of   {a^  ~b^Y  +  {b^- -c^yJr{o^ -a^Y, 
ai,ndia^{b  —  c)-\-b^{c  —  a)-\-c^{a-b). 

26.  If  a.  be  the  H.  C.  F.  of  b  and  c,  &  the  H.  C.  F.  of  c  and  a, 

y  the  H.  C.  F.  of  a  and  b,  and  X  the  H.  G.  F.  of  a,.b.  and  «,  then 

a6c5 
the  L.  C.  M.  of  a,  b,  c,  is   — ^t"* 

27.  If  a;+c  be  the  H.  C.  F.  of  x^  +ax-i-b,  and  x^+a'x+b',  their 
L.  C.  M.  will  be  x^  +  {a+a'-c)x^  +  {aa' -G^)x+{a-cJ{a' -c)c. 

28.  Shew  that  the  L.  C.  M.  of  the  quantities  in  Ex.  2  (solved 
above)  will  be  a  complete  square  if  x  =  y^-^z^  -y^z^. 

29.  Kind  the  H.  C.  F.  of  x^+^x^+dx"^  -2a;3  +  l,  and 

ex^+  x'+nx'--7x^-% 


Section  II. — Feactions. 


Art.  XXXIII,  When  required  to  reduce  a  fraction  to  its 
lowest  terms,  we  can  often  apply  some  of  the  preceding  methods 
of  factoring  to  discover  the  H.  C.  F.  of  the  numerator  and  de- 
nominator. 


110  feaotions. 

Examples. 

^       ac+by+a7j-{.be     _     c{a+b)+y{a+b)    _     c-\-y 
aJ-{-2bx+2ax+bf  "  /(a+5)+2a;(a+6)   "  /+2a;* 

a(«+i)(a-6)2  a 

g     z^+x^y-\-x^y^+z^y^+xy*+y^ 
x^—x^y+x^y^—x^y^-\-xy^-'y^ 

Here  the  numerator  is  eyidently  (x^  —y^)  -^  (x-y),  and  the 

denominator  is    ^.     The  result  is  .'.  --I^-^. 

a+y  x-y 

{x+y)^—x^—y^  5x^y+Wx^y^  -{-lOx^y^  +  5xy* 

{x+yy+x^-^y^    =    ■(cc+^)4-a;V  +  (^^+2/l)f-aJ,V 

_  5a;?/-{a;^+y^  +  2a;j/(a;+y)} 

~    (a;S+2/2+a;'/){(a;+?/)^+a;«/+a;2+?/2-iC2/} 

5a;y(a;+y)(a;^  +xy+y^)    _        5.ry(a;+y) 
2{x^+xy-\-y^y  ^X'+xy+y^) 

V  *2-12a;+35 


a;3_i0a;3+31a;-30 
Here  we  see  at  onee  that  the  numerator=(a;-5)(a;  — 7) ;  and 
it  is  plain  that  x— 7  is  not  a  factor  of  the  denominator;  we  .•.  try 
x  —  5  (Horner's  division),  and  find  the  quotient  to  be  x^  —5a; +6. 
x-7 


.  the  result  = 
6. 


x^-5x+Q 

a.4+2a;2+9 


a;* -4x3  + 8a; -21 

The  factors  of  the  numerator  are  at  once  seen  to  be  a;'+2a;+3, 
and  x^  —  2x+B,  of  which  the  latter  is  one  factor  of  the  denomin- 
ator, the  other  being  (Horner's  division)  ?:"  -  2x— 7  :  .'.  the  result 

x^+2x-\-B 

z^-2x-7 


FRACTIONS.  Ill 

« 

Exercise  xlii 

Reduce  the  following  to  their  lowest  terms  : 
a.2_7a;  +  6  Sm/'' -ldxy-\-Ux 


2. 
8. 


5. 


x^-2x"--8x-96  72/3-172/2^-62/ 

x^+ax^  -a^x  —  a^'        x^ —5x^ -{-Ix-S' 

a;3-3a;+2  x^ +2x^+9      ^ 

^  +  4a;2_5'        ^  _ 4a;3  -I-  4:X^  -  9* 

2+J.r  xl  +  2x^+^ 

26+(62_4)a;  -  26a;2'  a;5+4a: 

5a^+10a4.r+5ff.3a;^  20a;4+a;--l 

a3jc+2^^3  +  2rta;3  +a;4 '        25a;4  +  5a;3  -  a;  -  l' 


a;'  —  iv^jf +x^y^  —x'^y^  +x^y^  —x^;/^  +^y^  — y' 
^'    x''  +x^'y+x^y^  +a;*2/H- jc^z/^ ^a;2,/5  _|_ ^yG~^y7' 


I  a  b 

Sa'^x^  -  2ax^  - 1 


^'+  r  +  ^h»z/+2/* 


8. 


abc{a  — h)(b  —  c){c  -  a) 


^-    aS(6-c)  +  &3(c-a)+c3(a-6) 
10.  From  Ex.  4  (solved  above)  show  that 

.   (a-6)5  +  (6-c)5+(c-a)«    ~         5(a-6)(6-c)(c-a) 

(a;+y)^-a;''-2/^ 
(a;+2/)^-a;^-2/'* 

12.  Shew  that 

\a-hy-^[h-cy.  +  {c-a)''      7  ,,       ^^,     ^,      ^ 


112  FRACTIONS. 

Art.  XXXIV.  In  reducing  complex  fractions  it  is  often 
convenient  to  multiply  both  terms  of  tiie  eompiex  fraction  by  the 
L.  C.  M..  of  all  the  denominators  involved. 

EXAMPIiES, 

1.  Simplify   K^-Mi)-t(l-t:«).. 

Here  the  L.  C.  M.  of  all  the  denominators  involved  is  12 ; 
.-.  multiplying  both  terms  of  the  complex  fraction  by  12,  and 
removing  brackets,  we  have 

6a;-f8-8+6a;    _      12a;      _     ^x 
21~4a;-17      ~   4-4*  '"    1-x 

a  —  b 


II.    a- 


l+ah 

^^aia-b)^    Here  multiplying  both  terms  by  1  ^-ah,  wa  get 
l-\-ab 

a{l+ah)-a+o         &(a2  4-l)  . 


3. 


Here  multiplying  both  terms  of  the  frac- 

tion  which  follows  x-1  by  4-a;,  the  given  fraction  becomes  at 

gjjQg  ,  and  now  multiplying  both  terms  by  4,  we 

4  —  X 
«-l+ 


have 


4 
4  4 


4x— 4+4  — a;  3a;  . 

It  may  be  observed  that  when  the  fraction  is  reduced  to  the 

form  -^  -^   — ,  we  may  strike  out  any  factor  common  to  the  two 

b  d 

denominators,  and  also  any  factor  common  to  the  two  numerators ; 
it  is  sometimes  more  convenient  to  do  this  than  to  multiply 
du-ectly  by  the  L.  C.  M.  of  all  the  denominators. 


FE  ACTIONS.  113 

la+b        a-b\        la^  +  b^        a*  -h^\ 

4.  Simplify   [^^  +  ^  ^  [^^^-ZTbi  -  ^s+li'  ' 

Here  the  numerator  of  the  first  fraction  is  {a  +  by^  +  {a  —  b)^ 
and  the  denominator  is  a^  -b^  ;  tlie  numerator  of  second  fraction 
is  (a2+Z,3)a_(«2_j2)3^  and  the  denominator  is  a-^-b*",  the 
former  denominator  cancels  this  to  a^-i-b'^,  which,  of  course,  be- 
comes a  multiplier  of  the  first  numerator  : 

.-.    Wehave  — ^2_j_^3^2_^^3_^2X2  -         2.(3/yJ 

Occagionally,  we  ai  once  discover  a  common  com^^iex  factor, 
strike  this  out,  and  sin^plify  the  result. 

a    "^     /'     "^     c  M  1  \  8       1 

5.  -:; :  ^T       ^  :    here  the  den.  = V  ~t\     —~^ 

^  "^  iis"  -  "^  +  ^ 

/I         1         IWl         1  1\ 

ss 1-  ■-; — I —  +  -7- ,  and  cancellmg  the  eom- 

\a  b  c  I    \a  b  c  j  ° 

mon  factor  we  have 

1 

1,1  1 ,  and  muItiDlying  by  tibc,  this  = . 

■ r  ~r   —   —  "  bc-^ca  —  ub 

a  o  c  ' 

Exercise  xiiii. 
Simplify  the  following : 


l-^{l-^(l-aj^}  a-b  '^  a+6 


a-\-b  a  —  h 

a~b  g+ft 

^  - ^{l-i(l  -^)T        <»+6  a^) 

a  —  b  ~  a  +  b 
X          jc_          _3_           1 

x-i-y       x—y         1—a       \-\-a 


2.f 


^  -  +  '- 


8.    1  + 


aj*-  y*  1  -a        1  -\-a 

1 
a  x—y 


1+a  x-\-y 


114 


4. 


FHACTIOWS. 

a'+h^         2^>2  1  1 

a+b         a~h  h^ 


c-\-d   ^    c-d 
a+b         a~h' 


+  :r—i         «+&+- 


a 


6. 


a;— 1         ?/.— 1         T*:*?* 


7. 


yz~  zx  —  xu  1  1  1 

—  +  —--{-  — 

—   4.  —         A         g'^  +  fe^-f  c* 

^2     +     ^2     +     ,.3     +     ~7ii3,.3 

a                 /)                 r; 
-r—     -U    -I-    

DC         ac     '     ab 


/«+6        a^+fegy  /„^5         a"--b^\ 

\a-b  "*■  rt2-62  I    ^  \^+^  ~  a^^^j 

1  1 

—  4- 

'■1  11    ^^  26c         [ 


a         b 


■c 


\x-\-i'  I     '      .c-  a  i 

tr^^T"  m ranMiTrf-iTi — n — fwn 

(1— .)+!  WJ+^+t-J 


2(1 -a;)  (l-a;)2 

— >^ 4.  > 4.1 

14-x     ^  (1+a;)-  ^^ 
l-a; 


a; 

+  1  + 

J/_ 

1 

— 

y3 

0 

7; 

a; 

x-^ 

^« 

a; 

7' 

• 

V  ' 

— 

-1  + 



1 

-p 

.7 

X 

/ 

X  • 

FKACTIONS. 


]15 


/a~bY      (a--bY         {ci~h\ 

^[^nl  -[-v-J    -  3(— -7/4-1 


13. 


x^ +yAi/-'rx^y^+x^'^y^-{-xy^+y^     '    \x+y/ 

'   [l^^   +   l~x+xV  "^   [l+x-hx^    ~    1+xy' 

16.  Find  the  value  of 

Zi^^^v  +  ^b^^2^x  ^l^enc.  =  i(a+6). 

17.  Find  the  Talue  of  y'{l  -i/{l  -x)} 

\l  +  b}  \l+bl 

18.  Find  value  of 

l/ia+hx)  +  i/{a-hx)    ^^^^  ^  ^       2ae     ^ 
l/{a'{-bx)  —  i/{a-bx}  6(ln-c2) 

Art.  XXXV.  When  the  sum  of  several  fractions  is  to  be 
found,  it  is  generally  best,  instead  of  reducing  at  once  aU  the 
fractions  to  a  common  denominator,  to  take  two  (or  more)  of 
them  together,  and  combine  the  results. 

Examples. 

1,  Find  the  sum  of 

x+y  y-x        '  x^-y* 


2x-2y         2x+-2y         x^+y^ 
Here  taking  the  firgt  two  together  we  have 

i^+y)'  +  i^-y)'     _    t±yi ;  now  add  this  to  _  -^IH^'. 
2(a;3-?/3)  x"-y'  x^+y" 


and  we  get 


a;*— 2/*  a;*— 2/* 


116 


FRACTIONS. 


2.  Find  the  sum  of 


l-\-x  4x  8x  1—x 

r^  "^   1+x^   +  T+^  ~  l+i' 
Here,  taking  the  first  and  the  last  together,  we  have 
(l+^)^-(l-a;)3    _      Ax 
l-a;2  -   i_a;3' 

taking  this  result  with  ihe  second  fraction,  we  have 

8a; 


W-^-   4-  -^—]    -    - 
\l+x^   ^  1-xV   -    1 


.+x^     '     l-aV         1-x^ 
now  take  tliis  result  with  the  remaining  fraction  and  we  get 

^\i-x^  +  iT^j  =  r::^' 

ar'"  a;-"  11 

8.    ^;rri   -  ^r:^    -   ^^rzi   +  ^jqH'      Taking   in    pair. 

those  whose  denominators  are  alike,  we  have 

ic"^  —  l        a;^"  —  1 
•      ^iTZl  -  ^T^  =x"""+x"  +  l-(a;"-l)=ar"'+2: 

The  work  is  often  m  zde  easier  by  completing  the  divisions  repre- 
sented by  the  fractions. 

2a;+l  4ic+5 

4.    Pmd  the  sum  of  1+  oT^ITTv   —   o^lTo'    By  dividing  num- 
erators into  denominators,  this 

3  1  3  1 

=  1  +  1+  sr^  -2- 


2a;-2   ~"       2a;+2   "    2a;-2  ~   2a;+2 
3a;+3-a;  +  l    _    a;+2 
-        2x3-2         ^  a;2-l' 
x  x—9        x-\-l        x—S 

2  2  2'  2 

1+  ^32  +1-  ^=7  -^-  ^:ri  -1+  ^THS'^^ 

2_         ^_         ^_  2 2(2a;-8)  2(2a;-8) 


fl;-2    ^  a;-6         a;-7         a;-l   ~(a;-2)(x-6)        (a:-l)(a:-7) 

f        _1 1  ) 

=  (4a;-16)  |a.2_8a.^l2   "  a;3  -  8a;+7/ 

=  (80 -20a;)-=-(a;4-16a;3  +  83a;3  -  152.r+84). 

[denommator=(a;2 -8a;)2+19(x3     8x)+84]. 


FRACTIONS.  117 


6,  Find  the  value  of 


TT'  + Kl  when  X  =  — r  \ 

x-2a     '    x—2b  a  +  b 


Bydmsion,l+^32;^-h  1+^32^ 

=  2  +  4|-~|j-  + ^,\;   but  the   quantity  in  the  brackets 

(a-\-b)x~4:ah 
=  ~{^^2a)i^~2h  =  0  «^^^^  {a  +  b)x  =  Aab 

:.  the  value  of  the  given  expression  is  2. 

Exercise  xliv. 

Simplify  the  following  : 

,     ct  —  a         x^+ax+a^  x^  —  a^ 

5  x+a  x^—a^ 

^         a^+bs  a^-Sa^b  +  3ab^  -b^         a(a - h)  -  b(a -  h) 


/     1  _1_  2a     X 

\a  -\- X  a  —  x  a^+x^J 

I     1                J. 2a;     \ 

\a  +  a;  a  —  x  a^  +x^l' 


a  — 

a; 

1 

a  — 

« 

h 

a  — 

b 

2  - 

Bx 

,  a  h  ab  ah 

4.    _ J_ —    -f -. 

a  •\-  h         a  —  b         ab—h^  a'-+ab 

.     3+2a;         2  -  3a;         IQx—x^ 

^'    2-a;  ~    2  +a;    "*"     a;2-4 

1  1,1 


4a3(«+«)  4a3(a-a;)    ^   2a2(a2+a;3) 

.    1   /3^+2y)\     _  1  /3a;-2yl 
'•  2  \3x-2i/)/         2  l3a;+22/i 

a;+l  a;-l  l-3a;      ,  x 


'^'    '-      .1    ~  2.C+1        a;(l-2a;)  "*"  a;(4a;2-l)   "^  a;(l'6a;*-l)  • 


4  9  x-\ 

-I- 


.2x+2  a;+2    ^    2(a;+3)         (a,-+2)(a;+3) 


12. 


H8  FRACTIONS. 

10.    ^(^+y)    _   %-^)    _  4(a;'^-y")    _^   Hx*+1/^) 
x  —  y  x+y  x^-{-y^  «^  — 2/* 

(       1  1       ) 

\x  -\-  a         X  +  b  f 
I  a+x  4:ax  8a^x  a-x  \ 

\  a^x  "*"   a^x^   "*"   'oM^   ~    «+«  J    "*" 

I  ffl2_^2     +    a4  +  a;*    ~   a^+x^\' 

^o     5a;-4         12a;+2  lO.r+17 

13.     -I- —    — • 

9        ^    llic-8  18 

,^     12a:  +  10a     ,     117a+28x         _ 

Jo.  +        fc H —     —     lo. 

3a;+a  9a  +  2a! 

4a; -17         8a; -30         10a; -3  5a;-4 

^^-    ~^1T     "    2a; -7     +    ^^T    ~    ^T 

17.    Find  the  value  of  ^:^:^32Z-  +   ^+b^rd 

nhena-\-h=    --_r^. 

ar'"  ^"a;"*  a;""  «'" 

a;"-?/"         af+r/"         a;"-)/"  ^  a;"H-J^"' 


19. 


{a-bf-  (a  -  by-     _  1  1 


1  1 _  1      _ 

1+a;  1 -^  2       _     2a;3_ 

^-^-    l-a;3    +  i+a;3    ~   l-a;3         a;«  +  l* 


FRACTIONS.  119 

Art.  XXXVI.  The  following  are  additional  examples  in 
which  a  knowledge  of  factoring  and  o.  the  principle  of  symmetry 
is  of  advantaofe. 

Examples. 

Cancelling  the  common  factor  z  —  y-{-z  in  the  two  terms  of  the 
first  fraction,  there  results  ■^- ,    hence   by    symmetry,   the 

denominators  of  the  other  two  fractious  will  be  x-\-y+z,  and  the 
numerators  will  be  y-\-z-x,  z+x-y;  .'.  sum  of  the  three 
numerators = 33 +?/+0,  and  the  result  =1. 

^     „.       ,.,  ab  be  ca 

2.    Simplify  7         .,       ..    +    ,,     .,.     — s   + 


{c-a){c~h)   ^   {a-b){a-c)   ^   {b-c){b-a) 

The  L.  C.  M.  of  denominators  is  evidently  {a—b){b  —  c){c  —  a). 
This  gives  for  numerator  of  first  fraction  —ab{a  —  b),  and  by  sym- 
metry the  other  numerators  are  —bc{b-c),  —ca(c  —  a), 

ab(a-b)-i-bc(b  —  c)-^ca(c-a) 

.  :  we  have  —  -. txtt — w ^^ ' 

(a  —  b)[b  —  c){c  —  a) 

_  {a  —  b){b  —  c){a  —  c) 

{a  —  b)[b  —  c){c  —  a)    ~ 

2.  Eeduce  the  following  to  a  single  fraction  : 

a  h  c 

+  -nr—xn — z^7:7-^^   + 


{a-b){a-c){x-a)    ^    {b-a){b-c){x-b)    "^  {c- a){c-b){x-c)' 

Here  the  L.  C.  M.  is  {a-b){b  —  c){c-a){x—a){x-b){x—c) ;  the 
numerator  of  the  first  fraction  is 

~a{b  —  c){x—b){x  —  c),    and   .*.   by   symmetry  that   of 
second  is  —b{c—a){x  —  c){x  —  a),    and  that  of  third  is 

—  c{a  —  h){x  —  a){x  —  b);  and  their  sum  is 

—  {a[b  —  c){x-b){x  —  c)-^b{G-a){x  —  c){x  —  a)  + 

c{a  —  b){x  —  a){x~h)). 
This  vanishes  if  a  =  6,  hence  «  —  &  is  a  factor,  and  .*.  by  sym- 
metry b  —  c  and  c—a  are  also  factors.     Now*  the  product  of  these 


i'^i)  FRACTIONS. 

is  of  the  third  degree,  while  the  whole  expression  rises  only  to 
the  fourth,  hence  x^  cannot  be  involved.  The  other  factor  must 
therefore  be  of  the  form  nx+n,  in  which  mis  a  number. 

To  determine  n  put  a;  =  0,  and  the  expression  becomes 
abc{a  —  h-¥b—c-\-G  —  a]=Q;    .-.  n  =  0,  or  the  other  factor  is  mx. 

To  determine  m  put  a  =  0,  h  =  l,  c=  -1,  and  m  will  be  found  tc 
be  1.     The  numerator  is  .-.  x{a  -  lj){b  —  c){c  —  a),  and  the  result  is 

X 

lx  —  a){x—b){x—c) 

o     cj-       Tf              a  +  h                         b+c  c  +  a 

8.  Simplify      ^ _j_ + 


{b  —  c){c-a)   ^  {c-a){a~b)  {a  —  b){b-c) 

L.  C.  M.  of  denominators  is  {a  —  b)[b  —  c){c—a)  ; 

.'.  first  numerator  is  a^  —b^,  and  By  symmetry 

second  "  b^—c^,s.ndL 

thkd  "  c2-a2; 

the  sum  of  these  =  0,  which  is  the  required  result. 
4.  Eeduce 

2  2  2  (x-2/)2+(7/-z)2  +  (2-a;)« 


+   :--   +  -   ..  + 


x-y         y-z  ^  z-x  ^  {x -y){y - z)(z-x) 

Here  the  numerator  becomes 

^y-z){z-x)^-^x-y){z-x)  +  '>.{x-y)(:y-z)  + 
{x—yY-¥{y—z)--\-{z-x)-,  which  is  evidently 

{(•■-2/)+(^y-^)+C--^)p=o. 

ffl3  +  263]3  (2a3+£3,3 

Observe  that  the  denominators  become  the  same  by  changing 
the  sign  between  the  fractions,  and  that  the  expression  is  sym- 
metrical with  respect  to  a  and  b.  The  numerator  of  the  first 
fraction  is  a^*  +  6a^i3_^.l2a^6^^-8a369,  and  by   symmetry  that 

of  the  other  is  -b^^ -Qb^a^ -l'2.b^a^ -Qb^a^ .     Their  sum  is  .". 
ai 3  _  &i  2  _j_6a3/;3(rt6  _  ^fi)  _  8a3i3((^e  _  ^6)_ 

—  ^^^li^l)S'^(^a'^-.^J^^i^  and  since  the  denoniinatur  of  the  given 
expression  is  (a'^—b^)'^  :.  the  result  is  a^-\-b'^. 


FRACTIONS. 

Exercise  xlv. 
Simplify  tht!  following  : 


121 


1-    ^U  +  //j      -^^  \x+y 


\  a  —  b  I  \0  —  a  I 


\ 

a-{-b  O-^-r                      __*'  +  * 

^-     (ir-7)(c-a)    +  (7-</)(.i-6)    "^    (^-b)(b-c)' 

_    1 1                     1 

*•    (a-/;)(a-c)    +  {b-a)(b-c)    +    (6--aj(f  -  i)' 

((-6          /'~V-  c-a          (a  —  b){b  —  c){c-u) 


(^a:^'}j)^a  +  c){x  +  ay(a-{-b){b  -  c)(a;  +  b)     {a+c){b  -  c){x-\-c) 


'^'    {x-y){x-z}   +    {>J-x){y-  z)    "^   {z~x){z-y) 

,,3  /,3    C3 

11.  ^. ' ^,  + , 


(b^.~S2^)(^c+a-2b)    ^'   (c  +  a-26)(a-i-6-2c) 

1 

(a  +  b-2c){b-}-c-2a) 

b2-c^  c^a^         a^-l]^ 

1Q  "  I I 

■     {a-b){a-c){x-a)    "^     {b - a){b -  c){x- b)    ^ 
c2 


{c  —  a]{c  -  b){z  —  c) 


122 


RATIOS. 


14.        ^iy+^)         ,         y(z+x)         ,         z{x+y) 


+  iT.'^'z^nrh.   + 


(x-y){z-x)  ^  {)/-z}{x—y)         {z-x){y-z) 

15.    [(^  +  ir-+{(^-o)'+{a+cy^     _  -^    _    ._!_    a.    ---^. 

(a4-6)(6  — c)(a  +  c)  a  +  c         b  —  c     '     a-\-b 

1ft  1  1  1 


x[x  —  a){x  —  b)         a(b  —  a){x  —  a)         b{b-a){x—b) 


Section  III. — Eatios. 


Art.    XXXVII.     If  -^  =  -j      •••  ad  =  hc.     Now, 

b          d 
dividing  ad -he  by  ca  we  have          -            .... 

(1) 

a            b 

"       ad  =  bchvcd       "           —    =    —.... 

i'2) 

,     ,    1                               ^'            c  • 

•^                           da 

(3). 

ma  +  nc                 - 
Also      ,  ,      ,   —   each  of  the  given  fractions      .     .     . 

(4) 

,           mb[  ,  )+«(/(   ,  )     Imb+nd)    , 
_      wa-\- nc          \  b  /  '       \d  /     ^              '    b       o            r 

mb-\-nd          mo    +nd                 imi  +  nd           b            d 

A  very  important  case  of  this  is  m  =  1,  n=  ±1,  hence 

a             c           n -{-(■,          a  —  c 

b     ~     d     ~    b+d    ~    b  —  d 

(5). 

a  —  h          e  —  d. 
Also .     — ; 

(6). 

■^^s"    (,^b          cH-ci 

For  by  (2)  and  (5) 

(f            b           a—b         a+h         a — b         c  —  d 

c     ~     d     ~    c  —  d    ~    c -\-d    "    a-j-b    ~    c-\-d 

^    X,          a-b    _     b              _     d                   c-d 

a+b          «     ,    1            '^       ,            c+d 
d                   d 

RATIOS. 


123 


Generally,  to  prove  that,  if  —    =    — ,  anv  fraction  whose  nu- 

merator  and  denominator  are  homogeneous  functions  of  a  and  b, 
and  are  of  the  same  degree,  will  be  equal  to  a  similar  fraction 
formed  with  c  instead  of  a  and  d  instead  of  h  : — Express  the  first 

fraction  in  terms  of  — ,  and  for  —  substitute  its  equivalent  — 

b  b  W 

and  reduce  the  result. 

By  (2)  the  fractions  may  be  formed  of  a  and  c,  and  b  and  d. 

If  — ■   -   —   =   — , . LxL   _   —  or   —  or  — (7) 

b  d  J       mb-{-nil-\-pj  b  d  f 


I        I  w 

ma-+-'nc-\-pe 


'ij)  +  ""(t)  +  "At) 


mb-{-nd-f-pf  mb+nd-l-jif 

a 

{mh-{-nd-\-2)f)Y  a 

~        mb+nd-^pf  b 


U±   =   ^   and    ^    =   A 
b  d  n  q 

ma  +  pr         pan^mc  ma         pa 

nb  +  gd  qb  +  nd    ~    nb  qb 

■n      ma  DC         ma-\-pc  ■,      ,_. 

For  —    =    :^   =     -r^=-^,  by  (5) 
nb         qd  nb  +  qd 

pa         mc         pa  +  mc 


or  &c       .     .     .     .     C8) 


qb  ')ld  qb±nd 

But  !!^   ^  -^,  hence  the  equality  stated  in  (8).' 
nb  qb 

If  ^  =   _1  =  _!.  and    !!L   =   Z.   =  ^, 
b  d  f  n  q  s 

rr  c-{-pc  +  re  pa -^  re  +  me  „  ma 

-irr-ij—e    =      7.7    .  ,     ,-    =   &c.,   =    —    =   &c.  .  (9). 
nb±_qd+sf  qb+sd  +  nf  no  ^   ' 

If  an  upper  sign  be  taken  in  a  numerator,  the  corresponding 
upper  sign  must  be  taken  in  the  denominator ;  if  a  lower  sign, 
the  corresponding  lower  sign,  otherwise  all  the  signs  are  inde- 
pendent of  each  other. 


124 


a  e 

1-  I^T  =  T 


The  given  fraction  = 


EA'iiOS. 

EXAMPI^ES, 

i.1.  i  ^^  ~  46         Be  —  Ad 

a         ^                c         ^ 

Fjc  -  Ad 

a                        c 

7-r-  +  5           7-7  +  5 
h                           d 

lc+5d 

2-  I^T   =   -J '^'^^  ^^''^'   S^h^Ah^    =    Bc^d-Ad^' 


Dividing  the  given  fraction  by  b^  we  have 

,  and  this  becomes,  on  substituting  for -^  its  equal --7- > 


2p-  +   37J  ^ 


2c3  +  3r2^ 


^3  i.;,R  i^s  \ 

3.  If  3a  =  2&,  findthe  vahie  of  ^^^^^^^.    This  =   (rj   +   l)  r^ 

—   _   — -)     [by  dividing  both  numerator  and  denominator  by 

b^  b  I 

a       2 
o^j.     But  from  the  given  relation  —  =  -3-  we   have,  by  substi- 


a 

tuting  for  -t-» 


4-  I^  T  =  T-    ^^'^^^  *^^*  03  +  ^3    X    -J   =     (^;:p;/j    • 

a  b  a+h 

We  liave  ^   =    T  =  c^TT/-      ^^'^^ 


— ^- —   =   -TTTT   +   1M--^+   M=  Ti-,  and  this  muUiphed 
by  y  gives  ^=1^-/;. 


RATIOS.  *■-'*' 

„     r^    x^+rix^  -bx  +  c         x^  +  ax—h     ,         .    ,  ^ 

o.   if   _- -„-— 7 — -—   =  -;; --,  shew  that  x  =  — ■-. 

ic* -fl,r-  +  to+t'  x-  —  ax-\-b'  a 

Multiplying  both  terms  of  second  fraction  by  x,  it  becomes 

x^-\-ax^  —  bx 

Zz — n^2~L.h^'^    ^<^^   68,ch  of  the  given  fractions  = 

difference  of  numerators 


difference  of  denominators ; 

= 

c 

^.    =1/.  x^+ax-b  -x^  -ax+h 

or  lax  = 

6 
2b  .-.  «=   — 
a    - 

6.    If 

ace                       ac  +  ce-\-ea         a^  +  c'  +«• 

For 

ac      c<;       ea     ac-\-ce-\-ea 

bd-df^fb-hd+df+fb'   ^y  (7)makingm=r.=i,  =  l. 

Also 

a2        c2        ^2        a,2^.c2_j.^2 

fcs  -</2  -yg- ^2  4.(73+^2-  By  (7). 

afl     0.2 

But    73"  =  T^  hence  the  required  equality. 

The  problem  is  a  particular  case  of  (9),  with  all  the  signs  + 
and  a  for  m,  b  for  n,  c  for  p,  &c, 

(If  the  fractions  given  equal  to  one  another  have  not  monomial 
terms,  instead  of  seeking  to  express  the  proposed  quantity  in 
terms  of  one  fraction  and  then  substituting  an  equivalent  frac- 
tion, it  is  often  better  to  assume  a  single  letter  to  represent  the 
common  value  of  the  fractions  given  equal,  and  to  work  in  terms 
of  this  assumed  letter.) 

„    J.    a+b  h-\-c  c4-a 

prove  that  32a+85i+27c  =  0. 

Assume  each  of  the  given  fractions  =  «,  so  that  a-{-b  =  3(a  —  b)x, 
b-\'C=i{b~c)x.  c-\-a  =  5{c-a)x, 


126 


RATIOS. 


01'   -^-  +    --J-   +    -^^  =  x{a-b-\-h-c-^c-a)  =  0. 

/.    adding  these  fractions  we  have  32rt+356+27c  =  0. 

This  example  might  also  be  worked  as  a  particular  case  of  (7), 
thus  ■ 


a+b 


h  +  c 


B{a-b)  Mb-c] 


5(c  —  a). 


20(a+Z))  +  15(6+c)  +  12(c+a)    _    32a  +  35b  +  27c 


60(«-//)  +  6U(6-c)  +  60(c-a) 

a-\-b 


0 


32a  +  B5b+27c  =  0  x 


3{a-b) 


=  0. 


8.1f^  +  ^    -    '^i 
a  +  c  + 


a 

d  I  17 


c 
If 


e 

+  7 


prove  that 


«2  4.^24., 2 
b  +  d-hjj       "^     W-^Ij2~J^}^' 

Transposing  terms,  &c.,  we  have 

a2  2ac     .     c2  t'S  2ce 


fe2 


C2  t'2 

i(Z  "*"  ;^  "^  7^ 


^/■^"^    £f2      "^     ^' 


or 


la  P    \3  /   /?  c   \2 


that  is,  the  sum  of  two  essentially  positive  quantises  =  0  j 
.'.  each  of  them  must=:0  ;  heuce  we  have 

a  c  e  c 

-J-  —  -y   =   0,  and  -—   —  —    =    0; 

b  d  '  J  (jf 


c 
~d 


e 

7 


Also 


«2     ^     a2_|.c2+g2 

/a+c+e\-. 


RATIOS.  127 

Exercise  xlvi. 

,     Tf    «  c  a"'^  —ab-\-b^  c^  —  cd  +  cl^ 

1.  11  —   =   — ,  prove ^ —    =    — ; — - . 

b  d  ab-W^  cd-U^ 

2.  It  —   =   — ,  prove  =     , — ,       _   I— ^|. 

b  d^  b-'-d^  \b-dj     -   \b+dl 

3.  Given  the  same,  shew  that  each  of  these  fractions 

=    yXb-^  +  d^l' 
i.  U  2x  =  Sy,  write  down  the  value  of 

x^:/  +  xj/-^  +  2ii^'  [x^-ifi)^ 

5.  11  —   =   —   =   — ,  shew  that  —   = —. 

b  d  J  b  mb  —  nd  -  pf 

?}.  From  the  same  relations  prove  that  —   =    ( '■ j  • 

^3  Kb-md—n/J 

7.  If  J±^   =   AfJ±^+^\  then  2.3  =  (6_«)-(6+«). 
1  —  x  a\l—x-t-x'-'/ 

o-  ^     /,'     ,    \       ■// \    =   «,   prove  that  a;  =  ^i — o' 

f.    Tr   rnx-\-a-\-h  mx—c—d  b  —  c 

9.  11 ^ = ,   prove  x   = . 

nx-{-a-T-c  nx  —  b  —  d  n—ni 

in    1"     '^■^^  b  —  c  c—a  a-\-b-{-c 


ay-j-bx         bz-\-cx         cy  +  az  ax  +  by-\-cz 

then  each  of  these  fractions  = ,  «-f-/*+c  not  being  zero. 

x+y+x 

11.  K«t^   =    Jp^^    =    -^±^,then8a  +  9b+5c  =  0. 

a-b         2[b-c)  S[c-a} 

VC-\-  \/{a-  x)  1  a  —  x         /I— a\* 

12.  If  \  , r   =    — >    shew  that =   hn- 

ya  —  Y{a  —  x)  a  ■     a  \l+a/ 

x^  —  yz  7/2  —xz 

13.  If     ..  _"'T   =   ."{iTZ — V   ^^"  ^'  ^'  ^  "®  unequal,  shew 

x[i     yz)         y^i.  •    xz) 

that  each  of  these  fractions  is  equal  to  x+y-rz. 


128 


RATIOS. 


x^-\-2x  +  l         //-+2//  +  1  ,        .     1 

14.  If    ^     „     ,  o    =      ..     b    .  o'    shew   that   each    of    theso 

x-—2x  +  o  y^—2,i/-{-^ 

fractions  =  {xy  —  1 )  -f  {xy  —  8 ) . 

25x2-16         8(a;2J,4)       ,         ,       a;-4  8 

15.  If  —,(. — ^~Q     =     o r~'    shew  that  -   ~p  ■ 

10a; +8  2a;  — 4  x  +  O;  5 

ibc                       V  +  ^b         y+2c 
IG.  If  7/  =   T~-r~  shew  that  ' ^  +       ~o    =2. 

25rt2+2762  4-22t;2=o. 

18.  If -^~     =    -T, =    -^ — ,  shewthat((2^_|./,2^+c32, 

x^ -yz         y- -zx        z^-xy 

19.  n   -r^ —   =  ,-— —    =       ■  t     .'    then  wiU  (.(  -  h)x  + 
[b  —  c)?/  +  (c  —  a)z  =  0. 

^°-    -^  T  =  T  "=  7    *^®^    (//^  +c/3  +  /•2/       -    M  -4-  ,/4  4-/4  ■ 

hx  +  ay  cy-\-hz         az-+cx       ,         ,,     , 

21.  If  --~    =    4-^      =    '   shew  that 

a—b  b—c  c  —  a 

{a-'(-b-{-c){x-\-y-\-z)  =  ax-\-by-{-cz. 

22.  If  --0  ,    2    , 2-r-r    =   ^r:.'    shew  that  each  of  these 

expressions  =1, 

23.  If    I  (^^3      =     j(,'~-^)    -     M^)^    and  a,  /.,  o  he 
different,  shew  that  ]6«  + 116  +  15  =0. 

24..If(^-^'f   =  ^-C    pit)vethatx2+^2^22+2a:i/.  =  l. 
\y-\-zx)  l-x-     ^ 

as.  If      ^*       =       ^^       =    _^,  show  that  a.  +  /^  +  c  =  0. 
x  —  y         ?/-2         z  — « 

2G.  If  _   =    — ,  prove  that  — ^--   :^    '  .— f-  /Vrfx- 


RATIOS.  129 

to 


27.  ii  —    - -   — ,  then  each  is  equivalent 

h     -     d  f 

./'x'^rL^.*,  hence  shew  that 

a                         h                         <?  1 
. _      _   when 

2z  +  2a;-?/  2.c+2?/-2         2?/  +  2z-a; 

X  y  z 

2a +  21)'^    ~    26  +  26'- a    ~    2c  +  2a^* 

c.r.    Tx-    <^  c  ,1    ,   /a-6V  //a^"  +  6''"\ 

28.  li  _   =   — ,  prove  that    — -     =    a(-    ^  ,  .  . 

29.  If  ^^^   =    77-^   =   ^-^'  PJ^ove  that 

^{y-z)-{-JL{z-x)-^I^{x-y)^0. 
a  0  c 

30.  If  -       "    -  '=        ,/        ,    =    -       *     ,  ,.  then  wiU 

/x(?i^  —  ///2)         w?/(/z  —  nx)         nz[mx  —  iy) 

31.  If  e  ^    y  ^  -^ •',  aua  y  =    *^-^ ',  shew  that 

y  ^ 


X 

z 

»2 


32.  If  ^--^   =   ^q^   =    '^    =   1,  shew  that 
a3  0^  c^ 

38.  If  !^   =   -!^   =   ^,    and  ^   =   ^   .   ^1   .   1, 
prove  that  __+_   +   -  =3^:.^y3+^- 


34.  If  -^   =    ^  =   7   =    &«•'  ^^'^«° 


J 


130  COMPLETE    SQUARES. 

a-,  a  a^  a„ 

as.  If  T^    =   7^  =   7^    -     .     .     .  =    1^.    then 

*]  ^o  t'3  ^„  

CTlffg  -  g^  rtg  +    .    .    .    (  -  ly-^an-ian    ^    «i  v/fl,a3  -^-a-iV'a^a^  +&C 
&i  ^2  -  ^2^3  +    •   •   •   (  -  1)"~^^»-A  ^l/i'X  +  ^2T/^3^"  +  &<^- 

A+B+C         A      ^     B      ^      C 

36.  If  1 = h   y-   +   — , 

abc  a  b  c 


and  (J  +  5+C)(a4-&  +  ^)  =  ^4a+£&+Cc, 
1+^2    +    i^^2   +   i+c 


then  -will  -,  ,  ,„-   +   i— rr^   +   T~m   ~   0- 


and  also  — - —   +  +  =  0. 

1  11 

'a  b                '    c 

xh         yk  zl               x^          ?/-           ?2 

37.  If  -;■   =   77    =  -v'    and  -^   =   y^  =    -r   =1) 

a2          6^  c^   •            rH          6^           c' 

I X  If              2  \2            a2             1,2             c2 

thenwill^ly   +  y  +  vj     =,)^  +  7^  +  7^ 
7 


Section  IV.  — Complete  Squakss,  &c. 

1.  What  quantity  must  be  added  to  x^  +r>x  to  make  it  a  com- 
plete square  ? 

Let  r  be  the  quantity. 

Then  fl;2+;>a;  4- r  =  complete  square  =  (.c  +  y7~)^ 

Equating  coefficients  we  have 

2|/r=  r 

7'"  /  /.  \  3 


"    =  T   -   \2 


Or  thus:  Since  {a+x)"  =a-  -^ 'lax  +  x^  ;  we  observe,  (See 
Art.  XII),  thsit  four  times  the  product  of  the  extremes  is  equal  to  the 
square  of  the  mean, 

.'.     Ax^r=p^x^  ; 


r  z=  l  —  \    »    as  before. 


COMPLETE    SQUARES.  >  131 

Or  we  may  extract  the  square  root  and  equate  the  remainder 
to  zero .  thus 

P 

X" 


/> 

4 


p2 


ISTow,  if  the  expression  be  a  complete  square,  this  remainder 
must  vanish  ;  hence  we  have 


jr)3  I  P  ^'   ^ 


4  \  2 

2.  Find  the  relation  connecting  a,  b,  c,  if  ax^-\-hx+c  is  a  com- 
plete square. 

Assume  ax'-^  +bx+c  =  (^ a.x+  -i/c  )^  =ax^ +2-i/(ac).x+c. 

Now,  since  this  holds  for  all  values  of  x,  we  have  2  ^/ac  —  h,  or 
6-  =  iac,  the  relation  required, 

3.  Determine  the  relation  amongst  a,  b,  c,  in  order  that 

a'^x^+bx+bo+b-  may  be  a  perfect  square. 
As  in  Ex.  1,  we  have  4:a'^x-{bc  +  b-)  =  h^x-^  ; 

•      i      _   -1    -    1 

4«3  h     ~ 

Or  thus : 

Assume  a^x^+bx+bc+b^  =  (ax+l/'oc  +  b■^)'' 
=  a^x2+2ayb^b^  +  bc+b^ . 
Equating  coefficients,  we  have  b  =  1a^bc-\-b'^  ; 

.♦.    —    —  , —    =   1,  as  before. 
.  .    4a"  b 

The  same  result  may  also  be  obtained  by  extracting  the  square 
root  and  equating  the  remainder  to  zero. 


132  COMPLETE    SQUARES. 

4.  Show  that  i{  x*-\-ax^+hx^+cx-\-d  be  a  complete  square, 
the  coefficients  satisfy  the  equation  c^  —a^d  =  0. 

Is  it  necessary  that  the  coefficients  satisfy  any  other  equation  ? 
Extracting  the  square  root  of  x'^+ax^+hx^  Tcx+d   in   the 
usual  manner,  we  have  for  the  final  remainder 

Now,  if  the  expression  be  a  complete  square,  this  remainder 
muist  vanish  ;  and,  that  it  ma^  vanish  for  general  values  of  x,  we 
must  have 

a  I         a^\ 


1   /         a*\ 


(2); 


^3 

Eliminating  *  -   x'    we  have  <;- -«3f/  =  0     .     .     .      (3). 

The  coefficients  must  satisfy  the  equations  (1)  and  (2),  an  J 
therefore  either  of  these  equations,  together  with  the  equation  (3), 
which  results  from  them. 
The  same  result  may  be  obtained  by  assuming 
x^+ax'^  -^bx"-\-cx  +  d  =  {x--if^ax-\-  y/d)^ 

=  x^+ax^  +  2x^y/d 

+  ^a^x^  -taxs/d  +  d. 
Equating  coefficients,  we  have  2  v/d+i<*2=  6      ...      (1) 

and      a^/d=c       .     .     .     (2). 
From  (2)  we  have  c^  —  a-d  =  0,  as  before. 

5.  What  must  be  the  value  of  in  and  n  if 
4x*  —  12a;3  +25a;2  —  4???a;+8w  is  a  perfect  square  ? 

Assume  the  expression  =  {(2aj2  -  3x4-  \/{Sii)}^ 

=  4:x^-12x3+4x"  y{8n)-\-9x-'  -6xy/  {8n)+Sn. 
Equating  coefficients,  we  have  Gs/{Sn)=Am      ....      (1), 

and4N/(8w)  +  9  =  25       .     .     .'  .      (2); 
.-.     »=2, 
»i  =  6. 


COMPLETE    SQUARES. 


133 


Or  thus :  Extracting  the  square  root  in  the  ordinary  way,  the 
remainder  is  found  to  be  (— 4m  +  24)a;  +  87i-16  ;  /.  -we  must 
have  4?/;  +  24  =  0,  or  m  =  6, 

and     8m- 16  =  0,  or  ?i= 2. 

6.  If  ax^-\-hx^+cx+d  be  a  complete  cube,  shew  that  ac^  =  db^f 
and  b^  =  'dac. 

Assume  ax^  +bx^  +cx+d  =  (x+d\).  ' 

Equating  coefficients, 

b  =  3a^d^  , (1) 

c  =  3Jd' (2); 

b  ^3 

dividing  (l)by(2),_   =    —  ; 

c  d'  ■< 

Also,  h^=^aU^ (3); 

dividing  (3)  by  (2),^  =  3a.; 

c 

.-.    b^  =  3ac. 

7.  Find  the  relations  subsisting  between  a,  b,  c,  d,  e,  when 

QX*  +  bx^  +  cx-  +dx+e is  a  complete  fouich  power. 
Assume  ax*  +  bx^+cx^+dx-\-e  =  (a^x-\-^)'*' 

=  ax'^+4:a^e^x^  +  Ga^e^x^  +  4^a*e'x-[-e. 

Equating  coefficients,  we  have 
b  =  4aT  cT, 

d  =  Aai  ei  ; 

whence  fc(/  =  16«t' (1). 

bc  =  2iaie^  =  QaAa^ei  =  Qad (2). 

€d  =  1ia^ei  =  QeAa^e^  =  Qhe (3). 

8.  Shew  that  x^^-fx^  +  qx^  +rx+s  can  be  so  resolved  into  two 
rational  quadratic  factors  if  ^  be  a  perfect  square,  negative,  a,nd 

,.2 

equal  to  —- —. 


134  COMPLETE    SQUARES. 

Since  — s  is  a  perfect  square,  let  it  be  n^. 

Assume  x^  +px^-\-qx^-'rrx-n^ 

=  (x^+'mx-{-n)(x" -^vi'x -7i) 

=  X*  +  (in.-\-m ')x^  +  mm 'x"  —  n(m  —  m ')x  -n^. 

Equaticg  coefficients,  we  have 


mm '  =  o 

■J- 

r 

m—m'  — 

n 

ni^+2mm'+m'^=p^ 

4:mm'*=4.^    ; 

;.  {m-mY=P'^  -"^q    = 

r3 

r2 

9      .      =n-'  =  —s. 

p^  —  ip9 

Exercise  xlvii. 

1.  What  is  the  condition  that  {a  —  x){b  —  x)  —  c^,  may  be  a  per- 
fect square. 

2.  Find  the  value  of  n  which  will  make  2x"-\-8x-{-n,  a  perfect 
square.. 

3.  Find  a  value  of  a;  which  will  make  x^+dx^  +  Ux'  +  dx  +  dl, 
a  perfect  square. 

4.  Extract  the  square  root  of 

(a-b)^  - 2(a3  +63)(a _  b)^+2{a^+b*) 

5.  Find  the  values  of  m  and  n  which  will  make 
4a;*  — 4a;3_{-5a;3  ^ffix+n,  a  perfect  square. 

6.  What  must  be  added  to  x^~V{ix^ -lGx^  +  16)-^x^  in 
order  to  make  it  a  complete  square  ? 

7.  The  expression  x'^-^x^ —IQx^ —  4:x  +  4:8,  is  resolvable  into 
two  factors  of  the  form  x^-{-mx-^6,  and  x^-^nx-^8;  determine 
the  factors. 

ex 

8.  Find  the  value  of  c  which  ^Yill  make  -ix^  —  cx^  +  5x^  -f  .y  + 1, 

a  complete  square. 


COMPLETE    SQUARES, 


135 


9.  Oouiiii  ihe  square  root  of 

10.  If   {a-b)x^  +  {a  +  b)-x+{a^  -  b^){a+b),   is    a    complete 
square,  then  a  =36,  or  b=.'da. 

11.  Find  the  simplest  quantity  which,  subtracted  from 

a^x^  +4tnbx-\-4:acx+5bc  -{-  b^c^ ,  will  give  for  remainder  an  es.act 
square. 

12.  a*— 4a;3—a;3 4- 16a; -12  is  resolvable  into  quadratic  factors 
of  the  form  x"  +mx-rp,  and  x-  +nx-^q  :  find  them. 

13.  Find  the    values    of    m    which  will    make   x^+niax  +  a^ 
a  factor  oi  x^  —  ax^-\-a^x^  —a^x-{-a*. 

14.  Shew  that  if  x^+ax^+hx'' +cx+d  be  a  perfect  square,  .the 
coefficients  satisfy  the  relations 

8c    =a{4:b  —  a^),  and 
64r/=    (46-ftS)2. 

15.  Investigate  the  relations  between  the  coefficients  in  order 
that  ax^-{-bii^+cz'^+dx)j-]-eyz-\-fxz  may  be  a  complete  square. 

16.  If  a;  3  +  ^2 -f  to +c  is  exactly  divisible  by  {x  +  dy,  shew  that 

1(5    -d-)=-^=d{a-2d) 

17.  Determine  the  relations  among  a,  b,  c,  d,  when 
ax^  —  bx^-\-  cx—d,  is  a  complete  cube. 

18.  The   polynome   ax^  +  Shx^  +  Sex  +  d  is   exactly   divisible 
by  (a-x)^;  shew  that  (arf- 6c)3  =4(ac-i3)(fcd_  cS). 

19.  Find  the  relation  between  p  and  q,  when  x^-'rpx^+q,  is 
exactly  divisible  by  {x  —  a)^, 

20.  If  x^+nax+a^  is  a  factor  of  a;4+ axS-J-aSaj^+a'aj+a"^, 
shew  that  n^— 7i— 1  =  0. 

21.  Tix^-^-ax^  +  bx^ +cx-\-d,he  the  product  of  two  complete 
squares,  shew  that 

(46-a2)2  =  64(?,  (46-fl2)rt  =  8c,  as/{3u^  ~m  =  Sb. 


130  '  RELATION    IN    INVOLUTION* 

22.  Prove  that  a;*  +  px^  +qx^  -\-rx+s  is  a  perfect  square,  if 


,2 


9  P'' 

p^s  —  r,  and  q  =  —  -{.  2^s, 

■  ■.23.  If  ax^  4-36.C-  4-3cx  +  f/  contain  ax'^-\-2bx-\-c  as  a  factor,  the 
former  will  be  a  complete  cube,  and  the  latter  a  complete  square. 

24.  If  m"x^  +2^x+pq  +  q^  be  a  perfect  square,  fiud^  iu  terms 
of  m,  g,  and  x. 

25.  Find  the  relation  between  p  and  g'  in  order  that 

x^+px^+qx-\-r  may  contain  (a;4-2)2  as  a  factor. 

26.  If  x^  +px^  -{-qx+1-  be  algebraically  divisible  by 

Sx--\-^px-{-q,  shew  that  the  quotient  is  a;  +  ^. 

o 


Relation  in  Involution. 


Art.  XXXVIII.     If  «a'  =  W/  =  cc',  prove  that 

1.  (a  +  b'){h+c'){c  +  a')  =  (a'  +  b)ib'  +  c){c'+a) 
a+b')xa'  =  aa'  +  b'a'  =  bb'  +  b'a'={b  +  a')xb' 
b+c')xb'  =  bh'+c'h'  =  cc'-^c'b'  =  {c4-l')xc' 
c-{-a')xc'  =  cc'  +  n'c'  =  aa'  +  a'c'  =  {a-{-c')  xa' 
a-\-b'){b-irc'){c-\-a')xa'h<c'  = 
a'-{-b){b'-\-c)lc'  +  a)xb'c'a' 
a-\-b'){b  +  c')lc  +  a')  =  {a'  +  b){b'+c){c'-\-n). 
a  +  b){a-\-bi){a'-c){a'-c')  =  {a'+b)[a'  +  b'){a-c){n-    c'). 
a  +  b)xa'  =  aai  +  a'b  =  bb'  +  a'b  =  {b'-\-ai)xb 
a+b')xa'  =  aa'  +  a'b'  =  bb'+a'b'  =  {h  +  a')xb' 
a'  —  c)  X  a  =  aa'  —  ac  =  cc'  —ac  =  (c'  —  a)xc 
a'  —  c')xa  =  aa'-ac'  =  cc'  —  ac'  =  {G  —  a)xc' 
a+b){a+h'){a'-c){a'-c')x{aaY  = 
b'-{-a'){b-i-a'){c'-a){c-a)xbb'.oc_' 

But  bb'.cc'={aa')^, 

and  (c'  —  a)(c  —  a)  =  (a  —  c){a  —  c') 
:.  {a+b){a  +  b'){af - c){a' -c')  =  {al  +  b)(a'+b'){a-c){a-cr). 


RELATION    IN    INVOLUTION.  137 

Exercise  xlviii. 

If  rta'  =  66'  =  cc' prove  that 

1.  {a-b'){b-c){c'-a')  =  {b-a'){a-e){c'-b'). 

2.  (b-c'){c-a){a'-b')  =  (c-b'){b-a){„'-c-). 

3.  (^c-a'){a-b){b'-c')  =  {a-c'){r.-b){h'-ai). 

4.  (^a-b'){b  -c'){c-a')  =  {a-c%b-  a!){c  ^b'). 

(a-b){a-b^     ^     (a-c){a-c') 

(6_c)(6-c')      _     (^>-a)  (/;-«') 

^-   (;/_,,),,,_«')'    -    (c'-6)>'-fc')' 

8.  Shew  that  the  seven  preceding  relations  may  be  derived 

trom  the  single  relation 

(a-\-a')(bb!-cc')  +  ih-^b'){cc'-aa'}  +  {c+c'){aa'-bb')=^.(i. 


138 


SIMPLE    EQUATIONS. 


CHAPTEE  Y. 


Simple  Equations  of  one  Unknown  Quantity. 


Art.  XXXIX.  Preliminary  Equations.  AHhnnofh  the 
following  exercise  belongs  in  theory  to  tliis  chaptei%  in  practice 
the  numerical  examples  should  immediately  follow  Exercise  I., 
and  the  literal  examples  Exercise  III.  Likd"  those  exercises,  this 
one  is  merely  a  specimen  of  what  the  teacher  should  give  till  his 
pupils  have  thoroughly  mastered  this  preliminary  work.  But 
few  numerical  examples  are  given,  it  being  left  to  the  teacher  to 
supply  these. 

Exercise   xlix. 

What  values  must  x  have  that  the  following  equations  may  be 
true  ? 

1.  x-b  =  0.     a;-3i  =  0.     x-a  =  0.     x+3  =  0. 

2.  a;4-4i  =  0.     x-]-a  =  0.     a;+3  =  5.     a;-4  =  6. 

3.  x  —  a  =  b.     x  +  a  =  c.     x-b=—c.     6— a;=3.. 

4.  8~a;=10.     5  +  a;=ll.     9+.t  =  4.     7-a;=-5. 

5.  8+a;=-6.     a-x  =  3b.     2a  =  x  +  3h.     8a  =  5b~z. 

6.  2a;- 6  =  8.     3a;+8  =  20.     ax  =  a^.     mx  =  bm. 

7.  Sx  =  c.     ax  =  5.     ax  =  0.     (a  +  b)x  =  b-^a. 

8.  {a-b)x  =  b-a.      (a  +  bx)  =  (a  +  b)^.      {a—h)x  =  a'^-b9. 

9.  {a  +  b)x  =  b^-a^.      (^a^ -ab  +  b^)x  =  a"^ -\-b^. 

10.  {n^-b^)x^a-b.      (a^-b^)x  =  a  +  b.      {a-'+b^)x=l. 

11.  (a+x  —  b)  =  {a-{-b).     x  —  a-{-b  =  b—x  +  a. 

12.  2a  —  x  =  x  —  2b.     ax-\-bx  =  c.     ax  —  b  =  cx. 

13.  ax~b  =  bx  —  c.     ax  —  ab  =  ac. 

14.  ax  —  a^  =  bx  —  b^.     ax  —  a^  =  bx-b^. 


SIMPLE    EQUATIONS.  139 

15.  ax  —  a^  =  h^  —  bx\     ax+b-\-c  =  a+hx-\-cx. 

16.  a  —  bx  —  c  =  b  —  ax-\-cx;     a  +  hx-{-cx''^  =ax  —  h-\-cx^, 

17.  hx  -  ex-  -\-e  =  ex-b-  rx~  ;     3a;  =  | ;     4a;  =  f . 

18.  10;c=i  — 1;     rta;=  — ;     ax= — . 

c  b 

in      I  (i  '''         i  «c^         ab^ 

19.  aoa;  =  —   +  • — ^3     oca;=  —   _i,    — .. 

b  a  be 

20.  |a;  =  5;     |:e=:8;     -5.^  =  2;    -30;= -06. 

21.  020;  =  20:     -80;= -2;     •4a;=-6. 

a;  ax 

22.  •lBa;=l-8;    —    =.  6 ;     t   =   c. 

ax  h  X  ax      .. 

a  +  ft  a  a  —  h         a-t-b 

*>4  a;  = X  = 

"   '    a-  b  b   '        a-hb  b~a' 

a  a  b  —a         a  —  b 

b—a         a — 0  rt  +  o  b-\-a 

a-^b         a  —  c  112  3 

a-f-c         rt+o  a;  2         a;  o 

1  11  an  hi  1.1 

27.  —  =  -7-5   —  =  -T'   —  =  — ;    —  =  -^  +  -^ 

X  ab      ^^x  b  X  c  X  3  4 

«^      -^      ,      4  33  1  «      ,      ft       n 

28.  H7T  +  —    =   ^ IT'     —  +  —  =0. 

yu         oa;         oa;  o         a;  c 

29.  A    =   6-JL;         ^^      =7+-^-. 
a;-7  a;-7-       3a;-4  4-3a; 

30.  (a;-4)-(a;4-5)+a;  =  3;     2x- (a;- 5)- (4 -3a;)  =  5. 

31.  2(3-a;)  +  3(a;-3)  =  0;    2(3a;- 4) -3(3 -4a;) +9(2 -a;)  =  10. 

32.  a(l-2a;)-(2a;-a)  =  l  ;     a;- 5(a-a;)  =  te- 5a. 
38.  wa;(3a-4)+3w<a;-3a+l=0. 

34.    a{bx- c)+h{cx  —  a)-'rc{ax-h)  =  0. 
85.    a{ax  —  b)-\-h{cx  —  c)-\-c{cx—a)=;iQ, 


140  SIMPLE    EQUATIONS. 

86.  a{bx-a)-\-b{,:x-h)  +  c{ax-c)=-0. 

37.  a(x-2b)-\-h{x-2c)+c{x-2a)  =  a^-\-h''i+c*. 

38.  3(3{3(3a;-2)-2}-2)-2  =  l. 

39.  9(7{5(8a;-2)-4}-6)-8  =  l. 

40.  i{iaU(^+2)  +  2}+2)+2}  =  l. 

41.  i{iaU(x  +  2)  +  4}+6)  +  8}=l. 

42.  UUUh^-^)-i}-i)-l=^- 

43.  m{m^~H)-n}-n)-n=o. 

44.  f|{-A(f{f(!a;+4)  +  8}  +  12)  +  20}+32  =  58. 

45.  |{|(f{M-^-+7)-3}+6)-l}=4. 

46.  r{5'(jLi{w(?/ia;  — a)  — 6}  —c)—d\—e  =  0. 

47.  (l  +  6a;)2  +  (2  +  8a;)3  =  (l  +  10x)2. 

48.  9(2a;- 7)2 +(4a;- 27)2  =  13(4ic+15)(.'c+G). 

49.  (3-4a;)2+(4-4a;)2  =  2(5+4a;)3. 

50.  (9-4a;)(9-5a;)-f4(5-a;)(5-4a;)  =  36(2-a;)'.    ' 

Art.  XL.  In  order  that  the  product  of  two  or  more  factors 
may  vanish,  it  is  necessary,  and  it  is  sufficient,  that  one  of  the 
factors  should  vanish.  ThuB,  in  order  that  (x  —  a)(x  —  b)  may  =0, 
either  a;— a  must  =  0,  or  a;  — 6  must  =0,  and  it  is  sufficient  that 
one  of  them  should  do  so. 

Hence  the  single  equation  {x~a){x  —  b)=0  is  really  equivalent 
to  the  two  disjunctive  equations,  either  x  —  a  =  0  or  x-b  =  0,  for 
either  of  these  will  fulfil  the  condition  of  the  given  equation,  and 
that  is  all  that  is  required. 

Similarly,  were  it  required  to  find  what  values  of  x  would  make 
the  product  {x—a){x  —  b)(x  —  c)  vanish,  they  would  be  given  by 

a;  —  a  =  0,  or  a;  —  i  =  0,  or  «  —  c  =  0    .'.  a;  =  a  or  6  or  c. 

Hence  the  single  equation 

(x  —  a){x—b]{x—c)  =  0 
is  equivalent  to  the  three  disjunctive  equations 

X  —  u  =  0,  ox  x  —  b  =  0,  or  x  —  c  —  0. 


simple  equations.  .    141 

Examples. 

1.  Solve  j;2-x- 20  =  0, 

The  expression  =  {x  —  5){x-\-4),  which  will  vanish  if  either  of  its 
factors  does,  that  is,  if  ic  -  5  =  0,  or  ic+4  =  0, 

.".    x=  5,        ov  x=  —4- 

2.  Solve  a;4-j;3-a;2  4-^  =  0. 

This  gives  a;3 (a; -l)-a;(.«-l)  =  <x-l)(a;2_l) 

=  x{x—l)(x  +  l){x—l),  which  vanishes  fo» 
x  =  0,  x=  1,  x=  —1. 

3.  Solve  aj^+a-it;^ —ax- aS  =  0. 

This  =  x{x^  -  a)-\-a "-  {x""  -  a) 

=  (x  +  a*)(x^  —  a),  which  vanishes  foj 
x+a^  =  0,  and  a;*  —  rt  =  0,  or 
x= —a",     anda;2=a. 

4.  Solve  x^{a~h)  +  a^-(h-x)  +  b^{x-a)=0. 

The  factors  of  the  expression  are  (Ex.  2,  page  79) 

»  —a,  x—b,  a  —  h;  hence  the  expression  vanishes  if 
x—a  =  Q,  or  a:  — 6  =  0. 

5.  Solve  221a;2-5a:-6  =  0. 

Here  we  have  the  factors  17a; -3  and  13a; +2  ; 
.•.    the  equation  is  satisfied  by^  17a;  —  3  =  0,  or  x  =  ^, 

and  13a;+2  =  0,  or  a;  =  -  /j. 

6.  Solve  2a;4+2a;3  + 6a; -18  =  0. 

In  this  case  we  have  2(x4-9)  +  2a;(a;3+3) 

=  2{x^-\-3){x^ —S-{-x},  which  vanishes  for 
a;3+3  =  0,  ora;2+.c— 3  =  0. 

7.  Solve  (.r-a)3+(a-6)3+(6-a;)3=0. 

The  expression  is  equal  to  3(^x  —  a)(a  —  b){b  - x), 
ftnd  therefore  vanishes  for  x  —  a  =  0,  or  a;  =  a ; 
and  for  a;  —  i  =  0,  or  x  =  6. 


142  SIMPLE    EQUATIONS. 

Exercise  1. 

1.  If  an  equation  in  x  has  the  factors  2a;  — 4  and  2a;  — 6,  find 
the  corresponding  vakies  of  x. 

2.  If  an  equation  gives  the  factors  2a;-  1  and  3a?  — 1,  %vhat  are 
the  corresponding  values  of  a;  ? 

3.  If  an  equation  gives  the  factors  dx^  — 12  and  4a;-  5,  find  the 
corresponding  values  of  x.  ' 

Find  the  values  of  a;  for  which  the  following  expressions  will 
vanish ; 

4.  a;2-2a;+l;  4a;2-12a;+9. 

6.  9a;-^-4;  x-  —  {a+bY;  x^-2ax  +  a». 

6.  a;8-0a;+20;   4a;2  -  ]8a;+20. 

7.  x^+x-6:  x2-a;-12;   Oa;^  ...9a;-28. 

8.  6a;3-12a;+6;  6a;2_13a;+6;  6a;2-20a;+6. 

9.  6a;2- 5a;- 6;  6a;2-37a;+6;  6a;3 +a;- 12. 

10.  A  certain  equation  of  the  fourth  degree  gives  the  factors 
x^  —x  —  2,  and  4.<;3  —  2a;— 2,  find  all  the  values  of  x. 

Find  values  of  x  in  the  following  cases  : 

11.  x^-2bx''-Sb^x=i). 

12.  x^-ax^+a^x~a^=^0. 

13.  a;3-2a:  +  l  =  0;  a;3-3a;  +  2  =  0. 

14.  x^  —  2ax^+2a^x—a'^-0- 

15.  x^  +  {b  +  c)x'^-bcx-b^c~bS^=0. 

x-a         x-b    _        {a -by a;«-a^ 

x  —  b         x-a         {x—a){x  —  b)         {x~a)(x  —  b)* 

17.  x^-bx^-a^x-a-b  =  0. 

18.  3.c3-f4rt6a;3-6«263a;-4a363=:0. 

19.  x^{a-b)  +  a3{b-x)+b^{x-a)  =  0. 

^^-    (^a-b){a-c)   "^   (6-cj(6-«) 


/a;-2a\  3  /2a;- o\ 

21.  X    -.—     +  <^\^n-i 


=  a;2-as 


SIMPLE    EQUATIONS.  143 

ah  ,  hx  ,  ax  1 

{b-a){x-a)   ^   (x-a){a-b)   ^   {a~i){b-x)  a~b 

24.  Form  the  polyuome  which  will  vanish  for  x  equal  5,  or 
-6,  or  7. 

25.  Form  the  polynome  which  will  vanish  for  x  =  a,  or  4a,  01 
3a,  or   —  4fl. 

26.  Form  the  equation  whose  roots  are  0,  1,  -2,  and  4. 

27.  Form  the  equation  whose  roots  are  l-}-\/2,  1—  v/2,  1  -  ^.'S, 
and  1  +  /3. 

Art.  XLI.  In  solvino;  fractional  equations,  the  principles 
illustrated  in  the  section  on  fractions  may  frequently  be  applies; 
with  advantage,  as  in  the  following  cases. 

When  an  equation  involves  several  fractions,  we  may  take  two 
or  more  of  them  together. 

Examples. 

1.  Solve  ^J^   +  'L^-ll^   =   '^^±^. 
14      ^   6iB4-2  7 

Here,  instead  of  multiplying  through  by  the  L.  0.  M,  of  the 
denominators,  we  combine  the  first  fraction  with  the  last,  getting 
at  once 

7a;-3  7  1 

g^2   ^   n   ^   T        ■'•  '''x- 3  =  3a; 4-1,  and  05  =  1. 

2     2^+8|   _     lSa;-2  ^   _   Z^         a;+16 

~9  17a;   -  32  "^¥"12  "36^' 

In  this  case,  taking  together  all  the  fractions  having  only 
numerical  denominators,  we  get 

8a;-f34+12a;-  21a;4-a;+16   _    13a; -2 

36  17a;  -  32  '  °' 

25         13a;-   2 


18         17a; -32' 

.'.   425a;— 800  =  234a; -36,  hence  a;  =  4. 


144 


SIMPLE    EQUATIONS. 


It  is  often  advantageous  to  complete  the  divisions  represented 
by  the  fractions. 

4a;-  17    _    3|  -  22a;  6  /         x^ 

^-         9  —33"      =   *-  ^i^-    54 

Here,  completing  the  divisions,  we  have 

4x         17  1  2a;  &  x 

9    ~ir~'9""^y^~"^''"'9"' 

10a;  a;  6  6 

-9--2  =  a;+-   -   -    .-.    -2  =  --,    ora;  =  3. 

ax-\-h         ex  -\-d 

ain+h              cn-\-d   . 
a  -\ +  r  +  =  n-\-c 

X  -  VI  X   —   11 

{a7n  +  h)(x—')i)  +  {cn  +  d)(x  —  vi)=^0 
{ani-\-h  +  c7i  +  d)x={(i+e)mn-i-bn+din. 

6.  Similarly  may  be  solved 

ax+h         cx-\-d  cx^-^-fx—g 

+    + TT \  =  a-\-c+e- 

x  —  m  x  —  n  \x  —  m)[x  —  n) 

am+b         cn-{-d  {e{)i}+n)+f}x—emn—g 

x—m  x  —  v  {x  —  m){x  —  n) 

{am-^h){x  —  n)^{cn-\-d){x  —  m)->r{e{m.-^n)-\-f]x  —  rmn—(i  —  0. 

{{a-^c)m-\-b  +  {c-{-e)n-\-d-\-  f]x  =  {a-\-h+e)mn-\-Jin-\-dt)i-^(j. 

6.    il^^+l         8x+5   ^ 
3a;  +  l      ^    x-l 

43  13 

•••  44  -   3^1  +  8   +  — 1    =    52,  or 

;    .-.    39x  +  13  =  43a;-43,  anda;=14. 


a;-l  3a;+l 

-     25 -Xa;  16a;+4^         ^   ^      23 

a;+l      ^     3a;+2  a;  +  l 

Taking  the  last  fraction  with  the  first,  and  multiplying  the  re- 
sulting equation  by  15,  we  have 


SIMPLiE    EQUATIONS.  145 

210x4-08  r-r     .     5a,- -30 

■ —     11.    75   +  ■ ; 

Sx+2  x+1 

:.    80   -      ^^      =   75  +   5   -     ^^,  or 

'^L_    =    ^  ;    .-.   8a;  =  27,  and  *  =  Sf. 
3.f+2         a;  +  l 

8.  J-  -    4-       —    =    d. 

.,   «^»  _   1   +  ^-1   _   1  +  ^^   _   1   =  0; 

6+c  «+c'  6-|-ii 

fe+c  "*"  rt  +  C  "^  6+rt 

which  is  satisfied  by  a;  —  (a  +  /y  +  c)  =  0  ;     .-.  a;  =  (^/  +  A  +  c. 
+ 


10 


x-\-(i         x—b  x  —  c 

vilx  —  r)         nix  —  f) 
x—a  x—b 

which  may  be  solved  as-in  Ex.  1. 

3x+5  4^+9    _    15a;+7    _   12a;+17 

a;+T    ~   2x+4    ~     3.K+1  3a;-|-4  ' 

•    3  +   — -    -  2  -    -1-   =  5  +    J"-  -  4  -    -J_  ,  or 
"        ^  x+1  2a;+4  ^   3a;+l  3a;+4' 

2  1        _      ^_         _  1  _ . 

:^    ~   2a;  +  4    "    3a; +  1    ~   3a;+4' 

3a;+7_     _     _    3a; +  7 

"      2x3+6a;+4    ~    9a;2  +  15a;+4' 

This  can  be  divided  by  3a; -I- 7,  giving  3a;+7  =  0.  or  x=  -|. 
The  r^siilt  of  the  division  is 

1  1 

— ,  or 

2a;3  4-6.i-+4  9x2  + 15a; +  4 

9a;2+15a;+4  =  2a;2  +  6a;-f  4,  or  Ix^  =  —9a;,  whicl'  we  can  divide 
by  a;,    /.  x  =  0  ;  the  result  of  the  division  is  7a;  =  —9,  or  x=  —%■ 


1. 


146  SUJIPLE    EQUATIOirS. 

Exercise  li. 

lQa;  +  17         12a;  +    2   _  5x-4: 

18        ~"   13a;- 16   ~  ~ir" 

6x-\-lS     '    9a;+l5      ^  2a;+ 15 

^'  15  5a;- 25+^=  ~5 

7a;+l  35         a;+4 

3       ■ —    =    —    X    — ' —    4-    31 

4a;- 7         2- 14a!         3^ +a;  _   10  -  3fa;         19 
^-      2a;-9  "^        7        +  "IT"    "^         2  ~  21* 

2x  +  a  3x  —  a 

^-      3(^^  "^  2(^2M^"^^' 
a;-4         3a;— 13    _     1 
^'      fo+5  "^   18a; -6    ~   T* 

3x+  1  a;— 11_  a;— 9         a;— 5 

^-      2a; -15    ~    2a;-10"     '     ^^  "^  aT^"^' 

8       ^~^^  4.  ^~   ^  _9      _1_.  Sa;-19         3a;- 11 

a;-   7   "^  .^■-12  -""^^--v'     a;- 13  +     x+  7"^- 
a;-2  a;-l      _   _5_ .       a;+l  J^+l.  _     ^ 

^-      2a;+l    "^  3(a;-3)    "    6  '    4(a;+2)   +   5a.-+ 13  ^    2u' 
5(2a;^  +  3)         5-7a;     ,       /       3      ^      1  4 

^^__^   _    3^^         3a;^±^.      17  _15  32 

^^-    9..:!.    ^  2       -^      2a;   ~4'  a;-lG  "^  a;-18  =  ^-1? 


12. 


15. 
16. 


_     3-2ia;     _    28-^  _    10a;  -  11  x 

15  14(a;-l)    ~         3  30  ^  Y' 


_L    _  ^+2|a;g-a^3    _   J^    _    _5^ 

^'^    a;-2  G-5a;  +  a;2   '           2*"~a;- 

1,      30  +  Oa;  G0+8a;            48       ,     ,. 

14. _L.     —     ' _ +    14. 

a;-fl  ^     X+-6           x  +  1 


5a-2+a;-3    _    7a;3-3a;-9 
5a;-4        ^        7a; -lO"' 

X  x—9        x  +  1        x  —  8 


a-- 2  "^  a; -7        a;-l   "^  a;- 6' 


17. 
18. 
19. 
20. 
21. 
22. 
23. 


25. 


26. 


28. 


SIMPLE    EQUATIONS. 


a;2-3a;-9         a;2-7a;-17    _   x^-6x-U 
x-B'"    "^  a;-9  ~  x—Q 

4a;-f7         4a;+9   _   4a;-f-6         4a; +10 
Ax+l  "^  4x  +  7    "^    4x+i   "^    4a; +  8' 

2a;- 3         2a;-4    _    2.r-7         2a;-8 
2a; -4   ~   2a;- 5    ^    2a; -8   ~   2a;- 9* 

7a;+6         2a;  +  4|  x     _    11a;         Xj-^ 

28      ~   23a;-6    "^   T    ~    "21    ~     42  ' 

x'^-5         a;3-ll     _   a;2-7  a;2-9 

^2^^   "^   a;2-l2     ~    a;3-8   "''  a;^- lO' 

a;  -  Iff  2 -6a:    _  5a;-i(10-3a;) 

2 '  '    ~      13       ~  ^  ~  39 

l-2a;  \+x  1 


U7 


8(a:3-a;  +  l)   ^  2{a;3  +  l)   ^  6(a;+l)         9(a;2+l) 


o.     2a;3+a;-30         x^+^x-A.        x"-ll         2x^+7x-V6 


2a;-7        ^        a;-l        ~     x-4     ^       2a;-3 

x  —  a         x  —  h  {a  — by  1{a  —  x) 

x—b     '     x-a         {x  —  a)(x  —  b)    ~      a-{-x 

12a;+10a   ^   28a;+117rt   ^    ^g 
'3a;+«  2x+9a 


07     l^|a;-5  13|a;-ll     _    ISjx-l  13ia;-9 


13i':e-6     '     13|a;-12    ~    13^a;-8     '     13^a;-10 
1  1.x  16a; 


+ 


2(a;-l)2    "^   2(a;-l)  2(.t3+1)    "    ^x-l){x^  +  l) 


29.    i(|.^+4)  -    Zi^^   =   ^  (-1  _  1 
o  2  \  a;  / 


3a; 


81a;2-9  3      2a;2-l  57 -3a; 


30.   _  - ^^^ ^    =  3a-  - 

2  (3a;-l)(a;+3)  ^       2        a;+3 

31     1  +     ^^  +  1  "^^"fl^,   _   a;2  +  ^^  +  ^  _ 

2(a;-l)    ~    %+l)    ~    a;3-2a;+l  ~     ' 


148  SIMPLE    RQUATIONS, 

7a: -30  5x-7         2 -21a; 


32. 


83. 


10^  ^x-B  21 

42a;- 171  2a;-9  1 

63  ^^'^  63-14^  ~   Y^^  ^ 

18^^2  l  +  Hx  101 -04, 

13    -2x   +^^+    ~8~  =131-        'q  — 


4 -9a;         5  -  12x  24x2  -  5 

^^-    l-'6z   ~    7-   4x'^^"  7^  25iB+T2a;2* 

8a; +25         16a; +93         18x+86         6a;+26 

qe      I ' - I      . 

*^^-    2a;+    5   ^     2a;+ll    ~      2a;+    9   ^   2a;+    7 

1  1-1  1  ^ 

x+a  +  w  a;— a  +  o  x-+a— <>  a;  — a— o 

Art.  XLII.     The  results  deduced  in  Section  III.,  Chapter 
IV.,  may  often  be  applied  with  advantage. 

Examples. 
ax  -i-b  m 


(page  123). 


ex  +  d  n 

{ax-\-h)d  —  {cx-\-d)h         md  —  nh 
(ex +d)a—  [ax  +  b)c    ~   na—  mc 

md  —  nb 

X  =       • 

na  —  mc 

ox^-\-hx-\-c  a 

mx^-{-nx+p  VL 

(rta;2+5a;+c)-r/.r;3  n 

/- — iT . —      o    =    "■     (page  122). 

{mx^ +nx+p)  —  mx^  m      ^^    °  ' 

hx  +  c  a 

:.     — r      =  —  &c. 

nx-\- II  III 

3a;+J    _    ?>x-  13 
.r4-4    ~      X—    4 


SIMPI.K    EQUATIONS.  149 


By  (5)  eacli  of  these  fractions  = 
difference  of  numerators  20         0«+7 


difference  of  denominators        "     8  a;+i  x+i' 

or   -^   =    ---7,     ••  x  =  G, 

vrx-\-  a  +  h  mx  -\-  a  -\-e 

4.  .    = ] r» 

nx  —  c  —  (I  vx  —  o—  a 

mx-\-(i  -}-  b         nx  —  c  —  d 

, r—   =    7 — -, ;  or  By  4,  page  122, 

mx-\-(t  -\-  c  nx—b-  a  j     ■>  i    o  » 


or  (»  —  >?/)» 


vtx  -\-a  -{■  h         nx  —  c  —  d 
h—c  b—c 

—      a-\-h-\-r-\-il,   .'.   x=kc. 

^        l/{n+x)  +  i/(>i-x)  ^ 
''•       A^'Jrx)-^/{a-x}        • 

Here  by  (0),  page  122,  we  have 

r"  ^-r^ — —,   =    :: ;  or,  Cancelling  the  2  in  left  hand  mom* 

2p/{a  —  x)         a—1  '^ 

ber,  and  squaring, 

n+x  (a+iy  ■     -u     /n^ 

=    / ,^rj,  whence,  again  by  (6), 

a — X         {n  —  ij 

2x   _    (rt  +  l)--(«-l)3    _    4a 2a^. 

2a2 


6 


^/{x-a  +  b)  -  y{x  \n-  h)    ^   n-b^ 

y^^x-a  +  h)-^\7{x-{-a-h)    ~    a+b 

l/{x  —  a+h)    _    ^. 
V%«+a-6)    ~     b  ' 
squaring  and  again  applying  (6), 

1x  ^'=+&i         1      _         ^"Vh^ 


150  SIMPLE    EQUATIONS. 

Exercise  lii. 


1+x 


2.    -  ~ 


6. 


2x^-lx+B   ~   a;2-9x-|-2 
az+b  —  c  (b  —  c)^ 


ax~h  +  c  {h  +  cY 

7.  If  i/{x+n)+^/{x-y)   ^    x_^  ^^^^^  ^^^^  x+y^ 
V'{x+y)-v'(x-y)         y  x-y 


=   1. 


8. 


0. 


2a:- 7    _    x+l_,    4a- -5_    _    lOx-32 

57a; -43    _    ^x-1  .    23a;+5i    _      36a;-- 7 
19a; 4- 13    ~    18.r  +  2^5 '   115.r-29    ~    180a;  +  23' 

210a;-73  21a;+7-3  .  v»a;  — a  -  i         mx-a-c 


3i0a;-86  3]a;  +  8    '    nx—c-d  nx-b  —  d 

Zx+^jix-x'^)    _    ^      ./(12.e  +  l)+y(12y)    _ 
■     Sx-^/{4x~x^}    ~    ■^■-    {/(12;c+l)-|.^(12.tj    "    ^^ 


12. 
13. 


x^-{-ax-  —hx+c         x~+ax—h 
x^  —ax^  +  hx+c    ~   x^—ax+b 

^/{2a''-x-)  +  b]/(2a-x)    _     /a+b 
|/(2a2_a;3)_&^/(2a-a;)    ~     ^a-b' 


'     V(x-+a^)-V{x^-a^) 


15. 


8a;3+12x3-8.'c+r)    _   4x^+6x-A 
8x3 -1 2.1-3 +Ba;;+ 5    ~    4a;2  -  6a;  +  4 ' 


SIMPLE    EQUATIONS.  151 


16 
17 
18 


28-K/a;   _    9  +3yx 
28 -ya;   -    9+2i/.f' 

a^x^+a^bx^  —acx  +  d  a^x-+abx—e 

a^x^—a^bx^+acx+d    ~  ci'x''^  -ahx+e 

5T/(2a;-l)  +  2t/(3a;-3)  _ 

°-    4i/(2a;-l)_2i/(3aj-S)  "    '^^• 

/2a;+y^(3-2x)    _     3 
72aJ-i/(3^;^-)    -    T" 

2£(3aH-3)+jf (7x  +  8)  _ 
"  ■    2^(3a;+3)-i^(7:c  +  S)  "  ^• 

22 
23 


20 


24. 


33{13-2/(a:-5)}-3{13+2|/(a;  -5)}. 

(l/«+l){l/(».6-  +  l)  -  ^oix]  =  {^/n-l){i/{nx+-i)  +  y'nx] 

l/{x-hc)  +  -y/b  _   ^x  +  i/a 
l/{x+c)  -yb   ~   ^x  -  i/a* 

o"     V^J^^    _    lAjf38.     ^2x  +  17  _   f2x+^l 
l/ic+   4   ~    ^x+  G'     ■^2a;4-  9   ~   -^2^  +  ir/ 
y»+2a   _    -1/5+ 4a.         3a; -1    _    1+j/dx 
l/'x+   b   ~    -j/aH-^i'     |/3a;  + 1    ~  2 

ya—  |/(a  -  a;)  ^/x  +  ^/6 


26 

27 


31. 


■\/a+  ^/{a—x}        '     s/x  —  \/b  b 


ax  +  l  +  ,/{a^x-^-l)  _ 
^^-    «a;-t-l  -N/(a3.t-3-l)  -"' 

on  '«+a;  _    H-1  .     l-fx+a;2    _    62         14a; 


v/(2aa;+u;3)    "    6-1 '     l^^+a;^    ~    63         T~£ 
5a;4  +  10a;2  +  l    _     a^ -\-lQa-^ -\- 1 
a;^4-10a;s+5^    ^    Sa^  +  lOa^  +  l* 


Art.  XLIlI.     Various   other  artifices  may  be  employed  to 
simplify  the  solution  of  equations. 


152  simple  equations. 

Examples. 

1.  Solve  2-f  v/(-J:Cc2-9a3-f  8)-2x  =  0  :  here  there  is  but  one 
surd,  aud  it  is  convenient  to  make  that  su^d  one  side  of  the  equa- 
tion and  transpose  all  the  rational  terms  to  the  other ;  this  gives 
>/(4cc2  -  dx  +  8)  =  2x—  2  ;  squaring  both  sides, 

■la;2  — 9,t+8  =  1.^-2 -8x-+l,    .-.    x  =  4. 

2.  \/{a  +  x)-{-V{a~x)  =  2Vx.  We  might  square  this  as  it 
stands,  but  the  work  will  be  simphfied  if  we  first  transpose,  thus 

\/(a+x)  =  2  \'^x—  \/{a-x) ;  'low  squaring, 
a-\-x  =  4-X  +  (i- x  —  -i  \/{ax—x^),  or 
ic  =  2  v'  {((X  —  x^ ) .     Again  squaring, 

x^=A(ix  —  4:X-,  whence  x  =  0,  or  — -. 

5 

3.  Clear  of  radicals 

-^x  +  f///  +  ^2  =  U.     Transposing, 

^x-^  {/y  =  -  f^z ;  cube  by  formula  [0] , 

ic  +  y  +  of^xy{^x+^y)=  -z;  and  subsiitwtoig  wr 

^x-V-^'y  ''-3  val".  •  —^2,  t^is  becomes 

iK  +  ?/  —  3  -^^xyz  —  —z,  ur 

xi-y-\-z  =  o^''x!jz;    :.  out  lug  again, 

{x+y  +  z)^  =  21xyz. 

.     a+x+Vj^ax+x^) 

a  +  x  ~ 

Dividing  and  transposing,  we  iiave 

a+x  a-'  +2ax+x' 

division  in  left-hand  member, 


fl  =  (6-l)    .-.  -^=  v/{l-(/.-l)^},or 


(a-i-x)^  a-\-x 

^  +  "  =  1      v/il+(fc-l)n.  or 

—  -f  1  =  "fee. 


SIMPJ^E    EQUATIONS.  153 

5.  Solve  v/(4.r3-fl9)+A/(4x3-19)= -/dT+S. 
We  have  the  identity 

(4;t3  4.i9)_(4a;2-19)  =  38  =  47-9. 

Now  dividing  the  membera  of  this  identity  by  those  of  the  given 
equatioju,  we  have 

V(4a;3  +  19)  -  v'(4x2  - 19)  =  a/47  -  3.  Adding  this  to  the  giwB 
equation,  then 

2A/(4a;3  +  19)  =  2A/47,    /.    4^2+19  =  47,  and  a.  =  ±  ^7. 

6.  if(25+a;)+^(25-a;)  =  2. 

Cubing  by  formula  [6] ,  (See  Ex.  3),  we  have 

25+x  +  25-a;  +  6if(253-a;2)=.8,  or 
T^(f525-x3)=-7,  or  (625-x2)=-343; 
.-.     a;3 -525-1-453  =  908,  and  a;  =  ±22|/2. 

Exercise    iiii. 

2.    y(3:e  +  l)+i/(4.r  +  4)  =  i. 
S.  V(2.f-t-10)  +  i/(2a;  -  2)  ^6. 
4.   i/{i)ix)~i/{nx)  =  m  —  n. 

G.    l/-+/(x+3)  =  ^|^). 

7.  A/(aa;+a;3^^(1.-|-a:). 

8.  ^(17a;-26)=   A. 

,  a 

10.  6-i-a;-i/(63+a;2)  =  c2. 

11.  i/(8  +  a-)--/a;  =  2i/(l+a:). 

12.  ■/(2a;-27rO  =  9/a-/(2.r). 


154  SIMPLE    EQUATIONS 

13.  ^(l-^cyi-f{H-x)  =  ^3. 

14.  f'i:>+.c)  +  f/{'d~x)  =  ^l. 

15.  ^(.c  +  l)_,.X(.t— l)  =  ^lh 
IG.  ^[a  +  x)  +  f(a-x)  =  -^b. 

17.  r(l  +  iA)  +  ir(l-l/^)  =  2. 

18.  |/.^;-/{a-|/(a:c+a;2)}  =  i/«. 

19.  Clear  of  radicals  ^a+^^/  — ^6-, 

20.  Solve  aj+,/(a2^x2)  = 


7ja* 


l/(a2+a--) 

21 .  Clear  of  radicals  yx+  \/tj+\/x  —  |/m. 
Solve  the  following  equations  : 

22.  v/(i-^x)  +  ,/{l-l-.c^-|/(l-a;)|•  =  l/'(l-»>. 

23.  -/(x-+  i/x)  -  Vi-'-V-^)  =  '*Vx  +17i* 

24.  ^(l+x-+u;2)+/(l-.t;+.7;3)^/y,x. 

25.  ■i/(a3-.<;2)+.c/(^<3  _!)  =  ,( -2(1  „^.'). 

2o.     —7", — " —     = <^' 

•     i/{hx)+c-  n 

27.  i/C-..;-+5)+\/(2a;3-5)=  v^l5+ v'5. 

28.  -s/(3u;3  +  10)4-a/(3x3-10)=  ^17+ /a. 

29.  v^(3a;2+'J)-  i/(3x'3-9)=  \/31  f  4. 

30.  A/(3«-o/>+a;3)-t-  ^(2(i- 2/^-^x-)  =  V a+  v  js. 

81..   V(4«2_ai-i-2.c2)+;/(3«2_3/.2_^3)=,,4.^. 

32.  Clear  of  radicals,  ]^(2x-)  -  ^{%i]  -  ^(2z). 

83.  |/(«+.t)  +  /("  - ^)  - 2x  -:  -,/{ -t  +  i./(a:^ 4-^-) K 

84.  y(..  +  2..)  +  ^(-^-2«.)=    -^^ 

85.  J  f<-^+^-i-v  (-"?-)  ==^(^^^^^^^^ 

86.    ,/{(2«+x)--^-h/'-}  f/U2<t-^)-+^-;-  =  2a. 


SIMPLE    EQUATIONS.  i£)& 

Art.  XLIV.     Sometimes  a  factor  can  be  discovered,  and  tbe 
principle  of  Art.  XL.  applied. 

Examples. 

1.    t+a^^^ji^  =  x^Jr{ci-b)x^  +  {a^  -ah)x-an. 
x  —  a 

Factoring  we  have 

x  —  a 

ox  x^ —ax-{-a^  =  {x  —  a){x—h)  \ 


fl2 


.'.    {a-{-h  —  a)x  =  ah  —  a'^,?in<lx  =  a—   — 

Transpose    '—  and  factor,  then 
a 

•+'4  (o+hy)  ^1  a\  (a-Vh)-]  \ 


«6 


a~+b    -    ^• 

x+a  a;  — 6  a; — r  ^'4-c 

^'  (^^^)(7^c)  "  {a^b){b-c)  ~  {b  -  c)(^-^)  "  {a-o)[b'c){c-a) 

Add  term  by  term  the  identity  (Th.  iii.,  page  54). 
x—a  x—b  x—c 


(ir-b){c-a)  ^.  {a-b){b-c)   ^  {b-c)(c-a) 

2x  _                h  +  c 

*•      {^b){c-a)  ~  {a-b){b-c){c-a) 

1       h+c 


156  SIMPLE    EQUATIONS. 

Tlie  left  hand  member  vanishes  for  x  =  0,  and  /.  oy  symmetry 
for  a  =  0  and  b=0;  :.  it  is  of  the  form  mabx  in  which  m  is 
nume7-icaL 

Put  x  =  a  =  b,  and  m  is  found  to  be  6, 

.*.   the  equation  reduces  to 

6ahx  =  ahc,   :.  and  x  =  ^c. 

Ix  —  a\  3         x  —  2ia+b  ■ 

5.       r)       =    oz.  I     '>    let   x  —  b  =  m,  x  —  a  =  n,    and    .*. 

\x  —  bj  x  —  ^b  +  a' 

m—n  =  a  —  h,  then  we  have 

m^  n—{m  —  n)         2n—m 

«3    ~    m-\-{m — n)    ~    2m  —  n 

2m*  — 7i?»  3  =  2^4  —  w^m,  and 

2,{m^—7i^)  —  nin(m^  —  w*)  =  0,  which  is  divisible  by  7n^  —  n^, 

.•.     mf —n^  =0,  or  m-\-n  =  0; 

Bnt  m+n  =  2x-a—b  =  0,    :.   x  =  l{a-\-h). 

1      a;2-4a;  +  2       1      x^-Ax+S     _2_     a;g-4a;+3      5 
6-      y    a;2_4a;ri  +  (5      ^2^  4^~3  ~  9  *    x^-4x-6  ~  18* 

Let  ?/  =  a;-— 4a;,  then  this  equation  becomes 

1      ?/+2      1      y+3      2      y+3      5  ,      ,.  .  . 

— •    q+-7r- ^ — TT"    ^e  =  T3'    or  by  division, 

3      y  —  1      6      2/  — 3      9      ^  —  6     18  •' 

1      .    J_         1  1  2  2      _    6 

■3"  "^  2/"^  "^  T   "^   1/-3   ~  "9"   ~  y-a    ~    18'°^ 

_^  : -  =  0  ;  this  may  be  written 


^—1  y-S         y  —  (j 

1111 
y  -  I         y  —  b         y  —  o         y  —  o 

^  +  ^  =0,    .-.    5y-16+32/-3  =  0,  or 

y  =  2i  .-.  a;2-4a;=2^,  ora;2-4a;  +  4  =  4+2^, 

and  a;  -  2  =  + 1.     We  might  assume  (as—  2)^  =  y,  when  the  eriven 
equation  would  take  the  form 


SIMPLE    EQUATIONS.  157 

3'    y-o   '^    a'    y-7  9*    2/ -10         18' 

And  reducing  as  before,  we  should  find 

y  =  6l  =  (x-2y,  .'.  3^-2= +f,  as  before. 

Exercise    liv, 

2.    ?l±i^*   =  x^  +  2a{a-b)x+{2a-b)x^  -2a2b  . 
x  +  b 

o  a^  a^x       _      2c 

'    a^^ab+b^    ~    a^-b^    ^    ^^   ~      ^^' 

4.    ^- _   i-   _   J-   _    i-    =    2ab\x^b)x'^. 

a-\-b-\rX  a  b  X 

1  1 


{x-b){x-c)  "^  {a+c){a+b) 

1  1 

+ 


{a-\-c){x-c)   "^  {a-\-b){x-b) 

^     bz          Bab            an^  b^x      2a -b 

9.   —  —    — -7    +  7 TT^  =   3x  —    —  • 


a  a-b    "^   (a-6)3  "•*'  a        (a-b)* 

x^-{-2ax  x—a 

'    a;*  -lla;2a2+«*    ^   X'-Bax-a^' 


2  \a;+a/  x+a 

=  (.c  —  rt )  (x — h)  {x  —  c). 

aa;         bx  ex  2  \bcx     acx     abxj 

,,      1-ax         l-bx         l-.cx  12  2  2\ 


(a  —  b)^  a  a^-b^  I 

12.  y^L-^  _  1  +  _  =  __ +     1  + 

abc  0  abc  \ 

19,,  x^J^{h->rcY-\-Bb{b-^f)x  =  b^. 


a 

.  X, 

b 


158  SIMPLE    EQUATIONS. 

14.  x—a-B-^(ahx)  =  b. 

15.  lla;4 +10x3  _40.r  =  176. 

in  X  ac  c  ax 

Id.  , ^—  4- 


-„                 «  — &  2c.k2                   «.— &  \  —  cx 

a+b  '  1+cx                  a-\-b  l+cx. 

lo     4x*+4rt4— 33a;2a2  w^    ,  o   «       o      s  o   "\ 

19-      „.         -..,.,00      + 


a;3-lla;  +  28    ^  x^-llx  +  lO         a:2-14a;  +  4U 

20  8  ,  8  ^ ^ 

x'^-Qx+5  "^  a;2-14x+45         .r^-lOx-^O 

{a-b){c-a)    ~    {a-b){b-c)   "^   (fe_c)(c-^,) 
a  +  c 


~    {a-b){h-c){c-a) 

22.  (a;-a)34-(a-&)34-(6-a:)3=a;2-a2. 

ix+  2a\  3  /a+  2x,  3 

23.  X    +   « =2m. 

\.r  —   aj  \a  —   xj 

24.  {x+ay  -(a  +h)s  +  {b  -x)^  ={x  +  a){x+b){a-\-b). 

25.  a;3-(x-6)3-(a;-a+6)3-rt3  +  (.c-a)3+(r/-i)3+&3 

=  (a-'))c2 

26.  {x+a)^-{x+b)^-{x-by'-(-2ayi-{x-a)^+{a+by  + 
(a-by  =  {a^-b»)c. 

x-\-a  x  —  a  (7* 

28.    (a;  +  a-t-5)4  .-{x  +  n)i-{x  +  b)*+x*-{a+by  +  a^  +  b^ 
=  12ab{x-+{a  +  by}. 


a  —  x 


b—x  c  —  x.  3.r 


29-    ^m^c   "^    /)2— ca  "^   c2"_«6    ~    a/;+^c  +  c^" 

30.    x^{b - a^)-\-a^{x-b^) ^b^{a-x-)  +abj{nbx--  1) 
=  («-a;2)(62_«4). 


SIMPLE    EQUATIONS.  159 


81.   (l[x+x^-)"    =^~-..{l  +  x^+x^). 

CttLf         J. 


•;>o        I    aj-l-a  a  —  h 


^  1. 
o. 


x  +  b    ^     \2,.c-\-b  +  c 

34.  x/(x3 -f27.j  +  l 80) --p/(a;3 +26.^  +  168)=  J^±l^\. 

35.  {{,■  +  a  +  y' (x^  +2ax  +  b-^)}s  +  {x+a-  ^/{x^+^ax  +  b^)}^ 

=  14(./j+a)3.     (See  page  17,  Ex.  1). 

36.  {x+u+  y{x^  -2ax-2b^)}-  +  {x  +  u-'/{x'  —  2ax-2b^}}^ 

=  .^-b-^+2a{a-b). 

orr     ,  x+a  1^       :c4-2a  +  6 
■  —  ^  /         ;<;  —  «— 2/*  ■ 

38.  (5a:-7)'-(2.c-4)3^27(a;3-l). 


39. 


_L      '^-<^'-l  1      :t:3-6a;-4  2       a;2-6a;-7 

3  *  .c2-6x--4   "^  T  '  X--6X-9    ~   T  '  a;--6x-16 

14  4 


40. 


15         x'^  —  &X  —  9 

1_     :<;^-2a;-3         J_     a;^  -  2a;  - 15         _2_     x^ -2x- 35 
2.(;-8   "^    9   *a;2-2.r-24    ~   13  *  a^-2x-~48 


I 
I 


'_2 
~    585"' 

41.    {r-{-a-b+^     (.2^^43_J3)i3_ 

a;+a  - 6-  -/(.i^s  ^  a2  _  ^2) ;.  3  -_:  s(x  +  a  -  6)3. 
42 ^1  ,  1 

|.f+a)3   -h~     "^    (.,+6)2  _«3     - 

1 1 

a;3_(a  +  6)2    +  x^-{a-by 

„      ,./U       45          7.C  +  67  \      ^.^^ 
*3.    41    — — -   4- ^     -r  130  =- 

\    X  +  1  .c-r-i      I 

^^   8a;-;- 57          9.v-^68i 
39   ^    H 5- 


t.60  SIMPLE    EQUATIONS. 

44.    51    1?£±15    _?£t^)+e63  = 

.         X—1  X—4:       j 

45.   {x+a){x  +  da){x+ia){z+Ga)  =  x'^  +  6a^{x'  +7a«-f 6aS). 

4«         1  2  3  6     ' 

4b.    +  + 


x+Ca         X  -  da         x+2a         x  +  a 

Exercise  Iv. 

1.  a{b  —  z)'ih{c  —  x)  =  b{a  —  x)"{-cx. 

2.  (a-i-/;.c)(a— i)  — (flx  — 6)  =  «6(a;+l). 

3.  (a-6)(a;-c)  +  («-f-6Xa;+c)  =  2(Z^a;  +  arf). 

4.  (a-')(a;-c)-(a  +  &)(a;+c)  +  2a(6+c)  =  0. 

5.  (a-6)(fi-c)(a-}-a;)  +  (a+6)(a  +  e)(a-a;)  =  0. 

6  (  .-6)(a-c4-x)+(a  +  6)(a+c-j;)  =  2a2. 

(solve  in  {a;— c}). 

7  {m+a(a  +  b  —  x)  +  {a  —  m){b  -x)  =  a{m-rb). 

8.  m(a  +  L  — a;)  =  n(a;— a— ^). 

9.  (7n+n)(m  —  7i—x)  +  m{x—n)  —  n(x  —  m)  =  m^  —  »*. 

m  —  a;         n  —  x         v—x 

10. +   S-  ^^ —  =3. 

m  n  p 

a^b  —  x         b^c  —  x         c^a  —  x 

11.      -f-    7 +     " =0. 

a  0  c 

a  —  x         b  —  x         c—x     ^ 

12.    -1 V +  — T-  =  0. 

be  ca  ab 

1  —ax         1  —  bx         1—  ex     ^ 

18.    -  -r 1 1 r     =0. 

be  ca  ab 

(Deduce  the  solution  from  that  of  No.  12; 

a—hx         h  —  ex         c—ax 

14.  — , —  H 1 r-  =  o. 

be       .  ca  ab 

15.    {a  +  b  +  e)x-—^    =       ,  .    +   —J.* 
^  '  u—b  a-\-o         a—o 


SIMPLE    EQUATIONS.  ItJl 


Sahc  a^b^  '  {2a-hb)b^x   _    {b  +  3ac)x 

^^-    a+b  "^   (a^^   ■*"     «(rt  +  6)2'     ^  a 

10         4  9  2       /^  ,      .      1  > 

17.    —  +  —  = h  -5-      Solve  m  —  • 

X  y  X  o       \  xj 

^^-     X    "^    3     ~      3a;      +   12   ~   ii* 

I.         1^   _    2(5a;-12)    _   17         10 
^^-     3    "^   oa;   ~  3a;  ~   20  "^  T* 

10-a;         13-fx   _   7^+266         4a;+17 
20.-3       "^       7       ~     x+21     ~."2i 

6  3  17 

21.    -T^  + 


22. 


23. 


a;+3  ^  2(a;^3)  2  2(a;+3)* 

6a;+5  l  +  8a;         1-a;        x-% 


8a;-.  15    ' 

15 

3 

1 

1 

a  — 

X 

1      *- 

a 

1 

1 

a; 

a 

a2 

X 

2.5.  (a;-l)(a;-2)-(a;-3)(.r-4)  =  3; 
(a;-3)(a;-4)  =  (a;-2)(a;-6). 

26.  2(a;-4)(3a;+4)  +  l2a;-3)(3a;+2)-6(a;-2)(2a;-3)=0 

27.  {a-x){b-x)=x^  ;  {a-x)(z-b)=x^ -c^. 

28.  {a-x){b-^x)  =  b^-x^.;   {x-a){x-b)=x'' -ai. 

29.  {a^x){b+x)  =  {a-x){b-x); 
{ax-\-b){bx+a)  =  {b-ax){a-bx). 

30.  (if-a;)(6-a;)  +  (a-c-a;)(a;--6+c)  =  0. 


IR2  SIMPLE    EQUATIONS. 

31.  {a~x)(h  —  x)  —  (c  —  x){d  —  x)  =  (c  +  d)x -  cd. 

32.  {x~a){x-b)-(x-c)(x-d)  =  {d-^a){d-h). 

S3.   {(a2  -h^)x-ah}{a—(a+b)x\  +2ah^x  = 

{{a+b)2x+ab}{b-{a-b)x}. 
34    (ar-f l){x+2)(x-\-B)  =  (a;- 3)(a:+4)(a:+5). 

35.  (a;-fl)(a;+2)(a;+3)  =  (:«-l)(a;-2)(a;-3)  +  3(a;+l)(4.r-?-l) 

36.  (a;+l)(a;+4)(x  +  7)  =  (a;+2)(a;+5)2. 

37.  {x  +  2)(x+5y'={x+Sy-{x-^-6). 

38    (a;-l)(.r-4)(a:-6)-a;(rK-2)(x-9)  =  13G 

39.  (a+x)(7;  +a;)(o+a:) - {a~x){b-.r){c -x)  =  2{x^'  +abc). 

40-     (3^-^)(^-^)(^-^)-(^-^0(^-^)(^^-g)     ^    (a;-rf)=. 

a;  — (i 

4 1    .^•(x•  -  a)  2  -  (ic  -  a  +  t)  {x  -  a-^-r)  (x  -  b  -  c)  =  (a  2  +  he)  (b  + '). 

42.  {x-fi  +  b){x-  h-}-c){x-c  +  d)  -  x-(x~  a  +  d)  =  bc{d  -  a). 

43.  (x  -  a  +  b){x  —  b  +  c){x-  c  +  cl)  - x{x  -  a  +  c){x- c+d) 
=  bc\d  —  a). 

44.  {x-2(,Yx-2h){x-2c)-{x-a-b){x~b-c){x-c-a) 
=  {a-k-b  +  c){a^-\-V~+e-)-%abc. 

45.  x^  —{x  —  a-\-h)(x—b-\-c){x—c+a) 
46..L_i)(„_l)f._i)+i,."+M:£. 

■     \  X  I     \  X  I     \  X    I  X^  X 

47.  (x  +  a){x  +  b)+{x+c){x+a)  =  {x  +  b){x  +  d)+(.T  +  d){x+c). 

48.  (aa;+/0(«-«-O-«{^-ic)(«a;+&)=a2(a;-c)(a;-i)_ 
a((7a;  — c)(c— a;). 

2a; -3         3a; -2   _    5x2-  29a;-   4 
*^'    ~x-l  "^   "ai^   ~    "a;2-12a;+32' 

60      ^'^^  3a;+2     _   a;^- 30.x- +2 

8(i-fl)    ~    2(a;-l}    ~        6^21^6 


SniPLE   EQUATIONS.  163 

3x-7         3(^--fl)    _         lla;+3 

7x-5        8^-7         _J-^±1 n 

^^^-    3x--2  "*"   'dx-1   "^  9a;3-9a;+2~    • 

2a:+7     3x-Q     5(:k- 1)     3x-2     5a;-   8     2^+2 
^*    'Sx-7^-Ix-o  '  Qx-'Io  ~-Zx-6^'J:c-25^-6x-l 

^^        „^       4^3 +2x.  x  — a    a; -6 


„  -  ,     a         ex         c         au) 


66. 


2(a;-l)  «+8  _  3(5^+16) 

^"^^    ~a;-7  "^  a;-4  ~      5a;-28  " 

_^         ax  ex               a           c 

58. +    = ^    _; 

mx—p  nx  —  q            m            n 

ax+b  cx-\-d           a            e 

nix—p  nx  —  q         m           n 

„„     6  — a;  c-^x         a(c-2x) 

a+x  a—x           a- —x-' 

a+b  b+c         a+c  +  2b 

x—a  x  —  b            x  —  c 

ax+b  bx                ax            (ax^  —  2b)b 

ax~b  ax  +  b    ~   ax—b           a^x^—b'^ 


60. 


a 


rt-j      ux  —  b  cx  —  d  (bn  +  dm)x  —  {bq-\-dp) 

mx—p  nx  —  q  {mx—p){nx  —  q)        ~    m    "*"    n 

^c,        w.  n  p               m              n               v 

62.    -1. +   — ^—    = -t.    4.    -±L^. 

x—a  x  —  b  x  —  c         x  —  c         x  —  a         x—b 


164  SIMPLE    EQUATIONS. 


fi8 

1          1 

ax— 2a          ax  — 2b  ^    a           «     _ 

1 

X 
X 

ax— 2b     ~   ax-^2a  '    1            1 

1 a  + 

a          X 

2a;2-3a;+5          2 

7a;2-4a;  +  2   ~    7' 

64 

ax'^—bx-\-G           a.    ax^ —hx^  ■\-ax  —  <L 

az- 

-h 

mx^—nx+p         to'    mx^ —nx^ +  >}tx  —  q 

mx- 

-n 

65. 

i-a;         1             aj     .        1  . 
i+a;    •"    4          ^+a;         4  ' 

%^-i         2           2         fx+l        . 
l-a;          3           3    "^    a;-|' 

66. 

21               71               21               71 

X— 98         a;- 94    ~   a;+44   ~  a;-52' 

67. 

7                3                9                1 

- 

x-6  +  a;-ll    ~  aj-7    ^   x-12' 

9                9                2                2 

x-51        x-16   ~   x-Ql        a;+81 

5.4'  8  1 

x- 


68.  T^ze  +  ^  =  ^  +  ^3io' 


70. 


1 

aj-6 

+ 

8 
x-3 

B 

X 

5 

-2 

+  i- 

4 
-5' 

m  —  n 

- 

a  — 

X  — 

h 

m 

= 

m  - 

-n 

a- 

X- 

-b 

x  —  a 

a;- 

-b 

■n 

a-\-h 

a  +  c 

b  +  d 

69. 

x  —  a  X  —  m  x  —  o  x  —  n 


x—b         x—c         x—{a  +  b+2c  +  d)         x  —  {a-\-2b+c-\'fi 

71.  (x-a  +  b)^  -{x-a)^+{x-b)^  -x^  +a^  -(a-b)^  -b^ 
=  {a-b)c^. 

72.  (x+a+by  -  (a+by  -  (x+by  ~{x-\-ay  +x'^  -\-  a"  -hbii 
=  10ahx{2x-\-a  +  b){x+a+b). 

(m-n){x-a)         {n-p){x-b)         {p-m)(x-c) 
o+c  c-f-a  a'f6 


SIMPLE    EQUATIONS.  165 

ax—1  bx  —  1  cx~l  Sx 

74.  7Tnrrh\  +  h2(^si7,\  "t" 


a^{c+b)   ^  h^{c+a)   ^   c^{a-\-'h)         ah  +  bc  +  ca 

x—2a  x—2b  x—2c 

75.    ,— -f r   H -,  -  =3. 

b-\-c  —  a         c+a  —  b         a  +  b — c 

X—  2a  X  —  2b  x  —  2c  dx 


b+c-a         c-jra—b         a-{-b—c         a-^b-\-c 

a  —  x  b  —  x  c  —  x  6 

77.    r,-^-    +   T-, + 


a^  —  be  b^  —  '  c  c^  —  ab  a+b  +  c 

x-\-2ab  2aO-x  x-2ab  x->r2ab 

no       '_ I       ^ 1 

'    a  +b  -\-c         b  -{-  c  —  a         a  —  b-\-c         a-{-  b  —  c 

a  b  a  —  c         b+  c 

x-{-b—(i         x-\-a  —  c         x-\-b         x+a 

m^(a-b)         n^(h-c)  ,    pHc-d) 

80.    — ^ +  — ^ +  -— + 

x—m  x—n  z—p 

q  {pd+{n—p)c-{-{m  —  n)b— ma} 

x  —  q  ~ 


81. 


(a;-2)(a;-5)(a;-6)(a;-9)  +  (a+2)(a-4)(a-5)(a-ll) 


X 


+ 


(,+l)(6  +  5)(64-8)(6-fl2)    ^    (,_43(,_7)(,_ii)_, 

X 


(a2-l)(a-8)(a-10)  +  (&+2)(6+3)(6+10)(&4-ll) 


X 


Art,  XLV.  Employing  the  language  of  algebra,  the  princi- 
ple illustvated  in  Art.  XL.  may  be  stated  as  follows  : 

Definition. — Any  quantity  which  substituted  for  x  makes  the 
expression  f(x)  vanish,  is  said  to  be  a  root  of  the  equation  f{x)  =  Q. 
Thus,  if  a  is  a  root  of  the  equation /(a;)  =  0,  then /(a)  =  0. 

By  Th.  I.,  if  x  —  a  is  a  factor  of  the  pohjTwme  f{xY,  then 
/(a)"  =  0,  and  a  must  be  a  root  of  the  equation  f{xY  =  0  ;  hence  in 
solving  the  equation  we  are  merely  finding  a  value,  or  values,  of 
X  whicli  will  make  the  corresponding  polynome  vanish,  ^u-^. 
pose/(a;)"  =  (a;— a)i?>(a;)"~^  =  0,  we  are  re(j[uired  to  iind  a  value,  or 


166  SIMPLE    EQUATIONS. 

values,  of  x  which  will  make  (x  —  a)(^{xY'~'^  vanish.  The  poly- 
noma  will  certainly  vanish  if  one  of  its  factors  vanishes,  whether 
the  other  does  or  not,  and  will  not  vanish  unless  at  least  one  of 
its  factors  vanishes.  Hence  {x—  a)(p{x)"~^  will  vanish  if  a;-  a  =  0, 
quite  irrespective  of  the  value  of  (p(ic)"~^  Also,  if  (p{x}''~^  =  0,  the 
polynome  will  vanish,  irrespective  of  the  value  of  x  —  a.  It  fol- 
lows, therefore,  that  if  /{x)"  can  be  resolved  into  two  or  more 
factors,  each  of  these  factors  equated  to  zero  will  give  one  or  more 
roots  of  the  equation  /(x)"  =  0. 

When  there  can  be  found  two  or  mor?;  values  of  x  wliich  satisfy 
the  conditions  of  given  equations,  they  are    sometimes   distin- 
guished thus  :  Xy,  iCg,  x^,  &c.,  to  be  read   "  one  value  of  a;,"  "a 
second  value  of  x,"  "  a  third  value  of  x,''  &c.     Thus,  if 
(x  —  a){x  —  b){x  —  c)=^0, , 

Examples. 

1.  Solve  2a;3-13a:2-f27a;-18  =  0. 

Factoring, 

(a;  -  2)  (a; -3)  (2a;- 3)  =0, 

2.  x"-(a  +  b)x+{a+G)b  =  (a-\-c)c, 
...    x^~{a  +  b)x+{a  +  c)(b-c)==0, 

:.    x^-{(a+c)-{-{b-c)}x  +  ia+c){b-c)  =  0, 
...    {x-{a-\-c)}{x-(b-c)}=0, 
.".    Xi  =(t+c,  x^  =  b—c. 

3.  x-{a-b)-{-a^{b-x)  +  b^{x-a)  =  0. 
.-.  x^{a-b)-x{a^-b^)-\-ab{a-b)  =  0, 
.-.  {x-a){x-b){a-b)  =  0. 

li  a  —  b  =  Q,  the  given  equation  holds  irrespective  of  the  values 
oix  —  a  and  x  —  b,  and  therefore  of  the  values  of  x ;  but  ii  a~b  is 
not  zero,  x^=a,  x^  =  b. 


z  = 


SIMPLE    EQUATIONS. 

a;-l    ~    b^-{x^l)      ■'■  U-i/  ^- 

aJi  +  l  a  ^  a+f) 

-^ — -   _  -—   =    0    .'.   X,    =  r» 

Kj-l  6  '  «-« 

+   -—    =    0     .'.2:2    = 


167 


iCg  +  l    ,     ^    _   n  -   ^^~^ 

(a-xy  +  jb-x)^ 34 

{^^^+{a-x){b-x)+{b-x)^    -    49* 

(a-x)^+2{a-x){b-o^  +  {h-x)^    _      2(49) -34      ^ 
{a-x)^-2{a-x){b-x)  +  {b-x)^    ~    3(34)-2(49) 

((a-a;)-(6-a;)j  "      ' 

(a-a;i)  +  (6-a;i) 


a  —  b 


-4   =   0,    .-.  a^i  =  ^(5&-3a); 


(a-a,,)  +  ib-x^    +  i   =   0,    ...X,   =  ^(5a-86). 
a  —  0  ■*         ■" 

(a;-«)(x-6)         (y-5)(a;-r)    _ 
"*•    (c_a)>-Z>)    ^  (a_6)(«-c)    "    ^• 

Subtract  term  by  term  from  the  identity  (See  page  53), 

{x-a){x-b)  {x-b){x-c)         {x-c){x-a)    ^^ 

[c-a){c-b)   "^  {a-b){a-c)    "^    {b-c){b-a) 
:.    {x  —  c){x—a)  =  Q,    :.   x^—c,x^=a. 
7.  Find  tbe  ra/Kma?.  roots  of  x* -12x-3+51a;2-90a:+56  =  0. 
Factoring  the  left-band  member  by  the  method  of  Art.  xxviii., 
(a;-2)(a;-4)(x2-6rc  +  7)=0 
.-.      0^1=2,  2:2=4,  or  a;2-6x-}-7  =  0. 

Since  x^  —  Qx-\-l  cannot  be  resolved  into  rational  factors  we 
know  that  it  will  not  give  rational  roots,  .".  a;^  =  2,  a;,  =  4  are  the 
only  vftluefl  that  meet  the  condition  of  the  problem. 


168  SIMPLE    EQUATIONS. 

Any  literal  equation  of  the  second,  third,  or  fourth  degree,  and 
many  equations  of  the  higher  degree  can  be  resolved  tnto  a  series 
of  disjunctive  equations.  A  full  analysis  for  the  first  four  degrees 
will  be  given  in  Part  II.,  meanwhile  the  following  special  forms 
of  the  Theorem  in  Art.  XLV.,  will  enable  the  student  to  solve 
nearly  all  the  equations  commonly  proposed. 

(A).  In  order  that  two  expressions  having  a  common  factor 
may  be  equal,  it  is  necessary  either  that  the  common  fiictor 
should  vanish,  or  else  that  the  product  of  the  remaining  factors  of 
one  of  -the  expressions  should  be  equal  to  the  product  of  the 
remaining  factors  of  the  other  expression,  and  it  is  sufficient  if 
one  of  these  conditions  be  fulfilled.     In  symbols  this  is 

li  {x-a)f(x)  =  {x-a)(p{x),    :.    Xi=aoYf{x)=(p(x). 
[B).  If  an  equation  reduces  to  the  form  (^nx+n)^  =c* 
(mx-\-n)^ —c^=0, 

()"a;j+w)— c  =  0  and  .'.  x^ 


or     {mx2-\-n)  +  c  =  0  and  .*.  x^  ~ 
(C).  If  an  equation  reduces  to  the  form 


ni 

—  c  —  n 
m 


mx-hn]  *  a^ 


■px  +  q)  &3 

qa  —  nb  —qa  —  nb 

tlienx,=  ^-^— .   X,    =    -^:,:^-    (See  Exs.  4  and  5  above). 

(Z>).  If  an  equation  appears  under  the  form 

(a  —  x){x  —  h)  =  c,  (1) 

then  x-i  =  l{a  +  b  +  r),  aJs  =  K^+^-»')» 
in  which  r^  =  (a  —  6)  2  —  4c. 

From  the  ideu tity  {a  —  x)-i-{x-b)=a  —  b 

vfe  get  (a-x)^  +2{a-x){x-b)  +  {x-by  =  (a-b)^  (2) 

(2) -4(1)  .-.    (a-x)^  -2{a-x)(x-b)  +  (x-h)^ 

=  (a-i)2-4c  =  r2  say 
.-.    {[a-x)-{x-b)}^-r^=0, 
:.    {{a—xy)-(xi-b)}+r  =  0,  anxd  .:  Xi=^{a  +  b  +  r); 
or  {{a-x^)-(xr,~h)}~r  =  0,  &nd  .-.  Xg  =  l{a+b-)). 


SIMPLE    EQUATIONS. 
fill  1  ^ 


169 


x  —  a         x  —  a 


•    •  -I  ^^ 

1  aa; 

.•.  ic  — «  =  0,  or  aa;=  1, 
_    1 


Applying  (.4), 


rt 


9.   {:c-^i-\-mx^h^c)  =  {x-^a^h){^x-?>a^1h-c)\ 


3a  +  c 

^[x,~a-\-h)     _   x-a-hb 
''    ^x'^^+V    ~      3a+c  ' 
.♦.    {A)     x^=a-h 


Page  122.     (5). 


^'^-    ic3-2a;~    h'     "  m{x+2)^ ^n{x^ -'Ix)         mn-\"nh      ^> 
But  (C)  can  be  applied  if  vi  and  ?i   are  so  determiued  that 
m(2;+2)2+7t(a;2  — 2ie)  is  a  square. 

This  requires  that  ^m{m-Jrn)  =  (2m - n)^, 

Assume  ?m  =  1,  then  ?i=8,  and  (1)  becomes,  on  substitution  and 
seduction, 

fa  +  2)2       _      -J" 2 

(3:c-2)3    -    a  +  86-'"  '  '""y 

2(14->-)  2(r-l) 

3r-l  '  '*'2~'l  +  3/-' 

(a; +  1)4  _     a_  (a;2+2a;-(-l)g _   _« 

^^-    (a;3+l)(a;-l)2    "   T"  •*•   (a;--^+l)(a;3 -2a;+l)    "     6* 
For  X-  -j-l  write  a^z 

"*•     .c^(:r»-2a;)   ~     6       ***     2(z  -  2)  6 


.".  a?!  —    ._,  ,   X.J—  -I 


170  SIMPLE    EQUATIONS. 

This  '»quation  was  solved  in  Ex.  10,  hence  z  may  be  treated  as 
known. 

i5ut    -^-      =z,      :.     ^^^^   =   ^32' 

[x-\-\\  P  2-1-2 

UITil       ~   '•  —  9?  ^  formed  solved  in  (C). 

12.  {a-xY^{h-xY^c. 
In  the  identity 

Letw  =  o— a;,  v  =  x  —  b,  .•.  n+r  =  a.  -  &  andii^+v*  =c, 
.-.  (a-i)4  =  c  +  4(a_^,)2(„_a;)(a:_6)-2(a-.r)2(a;-/;)2 

Write  2  for  (a—x)(x  -  h) 

.'.    z2_2(a-6)2z+(a-/;)4  =  i{c+(a-6)4}=f2,  say, 

.-.      {2-(a-6)2}3=t2 

.*.    by  (Z>)         2,  =(rt  — 6)2— i;  Z2=(a— &)'  +  «, ',*.  gis  known; 

But  (a—x){x  -h)-z 

.-.    by(Z>)         a;i=i(a  +  i+r);£C2  =  l(«+6-r)  (1). 

in  whicli  r^  =  [a  —  b)^- 4z, 

=  (a-Z/)2-4{(a.-i)2-«}=4«-3(a-fc)2 
or    («-i)2-4{(a-ft)2+£}  =  -4i-8(a-6)2 
and^2  =  i{c  +  (a-/))4}.  (3) 

Hence  x  is  expressed  in  terms  of  a,  b,  and  r, 

r  is  expressed  in  terms  of  a,  b,  and  t, 

t  is  expressed  in  terms  of  a,  b,  and  c, 
4nd  the  expressions  for  r  and  t  are  cases  of  {B). 

13.  (a-jc)(6+a;)4  +  («-a;)4(i+3;)=fl&(rt.3+/,3) 

Let  a— a;  =  ?? -2  and  i+a;  =  w4-2    .".    n  =  ^  (a  +  b)  (1). 

The  equation  reduces  to 

(n2  _22)r(„4g)3_^(TO_2)3}  =ab{a^-\-b»)  ' 

:.    {n^  -z^)(2n^-[-Gnz^)  =  ah(a^  +b^) 


(2) 


SIMPLE    EQUATIONS.  171 

«2  may  now  be  found  by  {D),  and  from   the  resvilt  z  may   be 
found  by  (/>'),  and  from  (1)  x=-i^{a-h)  +  z  ; 

3z2  =  3(rt  _ ly  or  i(iO«6 - ^<-  - h^) 
.'.  x  =  0,  or  a-b,  or  -^(«-i)+i\/(30a&-3«2  _  3^3). 

14.  {Via-{-x)+  y(a-x)mV{a-\-x)+  ^{a-x}^^cx. 

Divide  the  terms  of  tbe  identity 

\/{a+x)'^  -  y(«-K)*  =  2x 
by  the  corresi^onding  terms  of  the  equation, 

•■•    \  (a-u:/     -    c-r     ••    a-"- a;    "     \c-l/     ' 

(C4-I)4_(c__l)4 

'        •••     *•     -     «-(,,4:i)~4  +  (c_l)4- 

15.  ir(.^-x)2  +  ir{(*-^)(^-^)}  +  l^('^--^)'  =  ^(«^+"^+^') 
Divide  the  terms  of  the  identity 

f/{a~x)^--^{b-xy-=a-b 
by  the  corresponding  terms  of  the  equation. 

.-.  ^(.-.)-r('-)=  ^(,,^|-+-,.)- 

Cube,  using  the  form  (u—v)^  =  u^ -v^—3itv{u-v). 

(»-.)-(6-.)-  8#'U«-K'-)[  •  ^(^+,,) 


.-.  -^{(a_^)i6-a.-)}  = 

.-.  (a-x){b—x)  = 
a  form  solved  m  (D). 


a6 


a363 


(rt«+flZ;-i-62)3 


Assume -i/("  -  x)  =  2  y  (as  -  6) 

a   —  b 
,'.  x-  b  =  glTpi 


1 72  SIMPLE    EQUATIONS. 

The  proposed  equation  now  becomes 

^/(x-b){z+lr' 


2-1 

(^_6)(2-l)4 


l/« 


=  c. 


a  form  solved  in  Ex.  11. 


17.  {x-2){x-5){x-Q)(:x-d)  +  (u+2){y-4:){ij-5){y-n)  + 

{z+l){z  +  5){z-^S){z+i2)=x{z~4.){x-l){x-ll)  + 
(2/+l)(y-l)(z/-8)O/-lO)  +  (z+2j(z-|-3)(z+lO)(0  +  ll). 

Let  x'  =  x^-llx,  y'  =  y'^  -%  and  a'^^^+lSz, 
.-.  (a;'4-18)(u;'  +  30j+(//'-22)0/'+20)+(z'+12)(2'+40)  = 

a;'(a;'+2B)  +  (y'-10)(/y'  +  S)+(2'  +  22)(^'+30) 
.-.  a;'3+48a;'+510+?/'2-27/'— 4'i0+z'2  4.52<;'+480  = 

ic'2  +  28a;'  +2/'2-22/'-    80+2'2+522'+660, 

.«.  20x'  =  0,     .-.     a;2-lla;  =  0,      /.     a;i=0,a;3  =  ll. 

Exercise    Ivi. 
What  can  you  deduce  from  the  following  statements  ? 
1.  A'B  =  0.     2.  A-B-C==0.     3.  (a-i):t;  =  0.     4.  12a;]/  =  0. 

5.  What  is  the  difference  between  the  equation 

{x-5y){x-iy-\-3)  =  0 
and  the  simultaneous  equations 

x  —  5y  =  0  and  a;  — 4^+3  =  0. 

What  values  of  x  will  satisfy  the  following  equations  ? 

6.  x{x-a)  =  0.     7.  ax(x+h)  =  0.     8.  (a;-a)(&a;-c)  =  0. 
9.  ax^-=Sr,x.      10.  a;2  =  (a+6)a;.     11.  a;(a;2 -a3)  =  0. 
12.  a^x^^b^x.     13.  x-2+(a-a;)2=a2, 

14.  ;i;3  4.(«_.^)2  =  (a_2.r)2.      15.   (a-x)3  +  (^ -6)3  =a2a./,2 

16.  (rt— ic)(3:-&)  +  a&  =  0. 

17.  {a-xy  -  (a  -  x){x  -  h)  +  {x  -  hf  =  «=  +  o6  +  6^ 

18.  x^-{a-'b)x^nb  =  Q. 

19.  x3-(rt+6+c)a:2  .    ah->rhc+ca)x—abc  ^0. 


SIMPLE    EQUATIONS.  178 

If  X  must  be  positive,  wiiat  value  or  values  of  z  will  satisfy  the 
iollowing  equations  ? 

20.  (.'e-5)(;f+4)  =  0.     21.  xS-H 29a; -30  =  0. 

22.  x--17x-84:  =  0.     23.  3a;3 +10x+3  =  0. 

i4.  :<;4_i3^3_f_36  =  0.     25.  x^ -^x^  -  5x+6  =  0. 

■m 

Solve  the  following  equations  : 

26.  (a-a;)2+(x-6)2  =  («_A)3. 

27.  (a-a;)3-(a-a;)(x-6)4-(a;-/))2=(a-5)2, 

28.  «2(«_^j3=i2(6_a;)2.     29.  a^{b~xy  =  b^{a^xy. 

30.  (a;-a)3  +  (a-&)3  +  (i-x)3=:0.     31.   (a;-l)- =«(.x-3 -1). 
09     (i  —  x         x  —  a         „£.     a-j-6  — a;         a—c-{-x 
x  —  li         c-\-x  a  —  c  —  x   ~   a-\-c—x 

84.  (a;-a+fe)(a:-a+'')  =  (a-6)3_a;2. 

85.  (a;-a)2-62  4-(rt4.6_a;)(6+c-a;)  =  0. 

36.  (a+fc+c)a;3-(2^-l-5  +  t>+«  =  0. 

Q_     a  +  6 — x         a-\-h  —  G 
01.     ^     . 

C  X 

38.  (a - x) 3  +  (a -  &)3  =  (a+J -  2:.)*. 

39.  a;(a+&-!B)+(a+6  +  c)c=0. 

40.  (n—p)x^+(p—7n)x+m  —  n  =  0. 

.-.      ax^ —bx-\-c  c  .0      ax^  —  hx+e    __    a  —  h-^e 

v}x^  —nx+p  p  mx^  —nx-\-p         m—n-\-p 

43.  4a;-^+a2-i3_2(a  +  ft)a;  =  (rt-x){6+a;)-(rt+x)(fc-x). 

44.  (2a-t_a;)3+9(a-&)3  =  (a+i-2a;)3. 

45.  (2rt+2c-x)2  =  (26+£c)(3r(- J-f3c-2a;). 

46.  (3a-5&H-a;)(5a-3&-a;)  =  (7a-6-3a;)3. 

47.  {%a-b+x)(9a-\-h-x)  =  (5a+3i  -  3ic)3. 

48.  a{a-b)-h{a-c)x-\-c{b-c)x^  =Q. 


174  SIMPLE    EQUATIONS. 

-r{a  —  c}^x. 

50.  {■'•  +  l){x  +  S){x-i)(x-l)  +  {x-l)(x-3){x-lr4:){x  +  l)  =  9G. 

51.  (j--l)i.':  +  3){x-^)[x+'J,  +  [^^:j{x-3){x+o)'^x-^Q}±  IS 
=  0. 

52.  ^-1-    —   =:  8i.  53.    ^  +  _   ^    _J]_   _^    

■'■  ic  « — o  a-\-b 

KA                '^            ^            ^           .„     C'  +  x          b+x     ^, 
54.    X—  —   =    —    -   —        55.    -, + =2},. 


X 


1         —  tyc.       ,     ,  -r- 

o  a  0-\-x         a  +  x 


m 


KG     ^~^         x—b  13                  a-x         b  +  x          in.        n 

Ob,           .    + =  — -.  57.       - —    —    =    — 

x~  0          a—x  ()                   b-^x         a  —  x          n 

«  a;  m  x^+nx+a'^ 

5b.    —   +    —    =    —  59.    -, ,    =    c. 

X  a  n  X'  —ax  +a^ 

{a-xY^+ix-by-  5                 a~x    ^    x-b          m 


64. 
65. 
66. 
67. 


{a-x)(x  —  b)  2  ■    a;  — 6         a— iC  jj 

(a;  +  a)2-(a;-i)2    "       2«6     ' 
(a-a:)3-(a;-Z*)2  4.ab 


{a~x){x-b)  {a^-b'') 

(a-x)^  +  {a-x)(x--b)  +  (x-b)^    _    49 
{a~x)^-{a-x){x-b)i-(x-b)2    ~    lO' 

2a2  +  r/(a-a;)  +  (a  +  a^)2  r+l 

2a^+a{a+x)  +  {a-xr'    "    c^l*    (^sofo^^  =  5)- 


68.  (5-x)^+{2-xy  =  17. 

69.  a;44.(«_,j.)4=c.  a;4_{.(_j._4)4  ^  82. 

70.  {a-xy  +  {x-b)^  =  (a-b)4'.     71.   (rt-a:)5^-(a;-5)5=r, 

72.  af^+(a-r)»=rtS;  a;' -L(C-a-)'^  =  1056. 

73.  (a-»)'(x-i)  =  +  (<f-a;)-(x-6)-'=«-//2(„_/,). 


SIJIPLE   EQUATIONS,  .175 

74.   {a-x){h+x)'^-\-(a-x)^{b+x)^  +  {a-z)-{b+xy^-^ 
{a-x)^{h+x)  =  {a  +  b)c. 

(a-x)^Jx-b)^   _    41 

_     (a-xV+(x-hY    _   211 

'    .    (a-a;)4-t-(a;-i)4   "    97    ('^~'')' 


77. 
78. 
79. 
80. 
81. 
82. 


85. 
86. 


(a-x)^  +  ix-b)^  _  a4  +  ft4 

(a-a;)2+(x-/;)3  ~  a^  +  b^' 

(a—x)^(x-^)i  _  rt4  +  J4 

(a— a;)54-(aJ— &)^  a^  — 6 


(a -a;)  3         (Z»-a;)3 

<7  — re              a;— ^ 

a           h 

(a;- 6)--^   +  (a-x)^ 

~    b2         a^ 

(^y-.r)4  +  (a:-t)4         rt4+54 


(a+6-2a;)2  ~    (^f+fe)^ 

g.j     {a-x)^  +  {x-b)^  a^-b* 

(a+T-2a;)2  =    (a+l^a' 

84     (^i:^'+(x-i)-5 


(a-a-)-(a;-Z))  («-a;)(x— 6) 

(a-x)^+(x-b)^ 

}^ i —  -  ■ L      =  c(a—x)(x—b). 

{a-x)^+{x~b)^  ^         '^         ' 

(a-a:)4  +  (a;-6)4    "  (a-;c)(a;-6)' 
88.    ^l+a;2)3  =  (a;3_3)2. 


176  SIMPLE    EQUATIONS. 

89        ^*  +  ^       ^  ±        90        (a;  +  l)-(a;=^  +  1)  a_ 


ix-l)^x  a  (a;3+a;+l)2 


a 


f7 


93      ('^'+^)'      ^   iL         94      (^+1)'     _    ± 
a;(a;+l)3  6  '  '    x{x'+l)     ~    b  ' 

96.    ^(•'^+^)^ ^  96    a;-  +  .r  +  1     icH-^-1 

(a;-l)4     -ft*         '    ■     j^x+1)-'     ■    (x-ip"    -    T 

97    ^IZj^m   _   JL        98    ^J^ii+ll   _    « 

99     (^  +  1)(^'  +  1)         ^         100      (^+l)fa''^-l)    _   _^ 
(a;-l)(a;3-l)    *    6*  '     (x-l)(a;''  +  l)    "    6' 

101.   (^+11^  =  JL  10^    (x+iy  ^   a_ 

x^  +  l  &■■  "■     x'^  +  1  h' 

103.  2(«.-.7-)4-9(a-a;)3(3;-/;)  +  14(a-a;)2(x-6)3--' 
9(a-a;)(a;-6)3+2(a;-/>)4  =  0. 

104.  4(a-a;)4-17(a-a;)-(a;-6)2+4(x-/;)4  =  0. 
Find  the  rational  roots  in  the  following  equations  : 

105.  x4-12a;3+49.r2-78x+40  =  0.  [Let  2  =  a;2  -  Oa;] . 

106.  x^-Qx^+lx^+Qx-Q. 

107.  2;4_i0x3+35a;2-50a;+24  =  0. 

108.  32a;4-48a;3_i0a;2+21a;+5  =  0. 

109.  a;3  -  6x2 +5a;+ 12  =  0. 

5  4  9  4 


—  -j_ —   _    0 

X  x  —  a         x  —  ^2a         x—  3a     '     x—4a 

14  5  4  14  5  4 


a;+20   ^  a;+5         ar-4         x-s>5    ^  x-40         'x-2{', 


110. 
111. 


j-|o     2a;  +  5a  a;+8a 

iC  a;  — a 

x  —  a  a;+5«         2*  — 5a 

+ 


+ 


9  — 3a         x-4.a  x—C 


ilMPLE    EQUATIONS.  177 

ii„     x+i         x+2         ic4-4        'x  +  3  z-1  x-3 

lis.      — _1_    1—     _|_    _J_      a-     '       — ^_     < 

x  +  2            X            x  —  1         x~2  x—'d  x  —  5 

...      1            31             20              8  20  31 

114.    —   _    _j_   4-    -^ _L    _    — —    J- 

X          x-l   ^  a;-2  ^  x-3  ^  x-4:  x-5  ^ 
'      =0. 


X—  6 

^^g     ^(a^+2x)  +  i/{a2-2x)     _ 
■^/{a^-i-2x)-^{a^-2x)     ~ 

m^       V{m^x-\-2)  +  y/{m^x-2) 
a2      •  7/(m~'x'+2)--/(w3a;-2) 

117.  a/(.t2  -a2)  +  ^(ajs  _i2)  +  ^(a;2  -c-S)  =.r. 

118.  {\/{a-x)-\-\/{h-^)]{y\a-x)-W{lj-Ji)\^fi* 
119     ■^(«  — a;)  — 1^(0;  — 6)    _   a+h-'lx 

f{a-x)-\-f/{:x-b)    "   ~^u^-b 

120.  V(a+a;)4-V(«-a;)=y(2a). 

[Write  u  luJ.'  |^\a  -x>,  and  v  iov  i^(.c  — 6)], 


178 


SIMULTANEOUS    EQUATIONS. 


CHAPTER  VI. 


Simultaneous  Equations. 


Art.  XLVI,  There  are  three  general  methocls  of  resolving 
simultaneous  linear  equations,  1°  by  substitution,  2°  by  compar- 
ison, 3°  by  elimination.  The  last  is  often  subdivided  into  the 
method  by  cross-multipliers,  and  the  method  by  arbitrary  multd- 
pliers. 

In  api^lying  the  elimination -method  the  work  should  be  done 
with  detached  coefficients,  each  equation  should  be  numbered, 
and  a  register  of  the  operations  performed  should  be  kept. 


Ex.  Resolve 


u+i+x+y-\-z  =  15. 
u+2o  +  4:X+8y  +  iez  =  57. 
u-\-Sv+dx+  27?/+81z=  179. 
u+4:v  +  IGx-f  G-1//+2563  =  45S . 
M -f  5  <.■ + 2  5x-+ 1 25.// -f  G252  =  97  5 . 


Register 


(2)-(l). 

(3) -(2). 

(4) -(3). 

(5) -(4). 

:7)-(0). 

(B)-(7). 

(9) -(8). 

(11) -(10). 

(12)-.(11). 

(14)-(13). 

(15)-- 24. 

ii(13)-G0(W)}. 

A[(10)-{12(17)  +  50(16)}]. 

(6)--{3(18)+7(17)  +  15(16)}. 

(l)_|(19)  +  (18)  +  (17)-i-(16)! 


u 

V 

X 

y 

z 

1 

1 

1 

1 

1  = 

=  15   (1) 

i 

•2 

4 

8 

16 

57  (2) 

1 

3 

9 

27 

81 

179  (3) 

«. 

•i 

16 

64 

256 

453  (4) 

1 

5 

25 

125 

625 

975   (5) 

1 

3 

7 

15 

42  (6) 

1 

.5 

-19 

65 

122  (7) 

1 

7 

37 

175 

274  (8) 

1 

•J 

61 

369 

522  (9) 

2 

12 

50 

80  (10) 

2 

18 

110 

152  (11) 

2 

24 

194 

248  (12) 

(5 

60 

72  (13) 

6 

84 

24 

I 

96  (14) 

24  (15) 

1  (16) 

1 

2  (17) 

1 

8  (18) 

J 

1 

4  (19) 
6  (20) 

SIMULTANEOUS    KQUATIOXS.  170 

An  eXcamination  of  the  Register  will  sliow  how  easy  it  ■would 
i;ave  been  to  shorten 'the  process,  thus  (lO)'is  (7)  — (6)  which  is 
(:))  +  (l)-2(2);  similaily  (11)  is  (4)-i-(2)-2(3)  ;  .-.  (18)  is  (4)-^- 
H(2)-3(3)-(l),  &c. 

A  ojeneral  systematic  arranfrement  of  the  ehmination-metliod 
will  be  given  in  Part  II.  For  two  or  three  simultaneous  equa- 
tions it  may  be  stated  as  follows. 

r/2.'K-fA._,//-f  (-2  =0. 
Arrange  the  coefficients  thus — 
^1     ^1     ^1     ^i 

«2        '^2        '^2        "s* 

Form  their  products  diagonally  from  left  to  riglit  downwards, 
thus —  a^feg     ^'i^2     '*i'^2' 

Form  their  products  diagonally  from  right  to  left  downwards, 
thus —  ^if<2     ^J'2     "^i^a- 

Subtract  the  latter  products  in  order  from  the  former,  thus — 
a^b^  —  b^a^,     b^c^  —  c^b^,     c^a^—OyC^. 

Divide  the  2°  and  3'^  remainders  by  the  1°  remainder,  the  first 
quotient  will  be  the  value  of  w,  the  second  quotient  will  be  the 
value  of  y.       > 

[Writing  i?^,  B.^,  7/,  for  the  three  'remainders'  respectively, 

the  general  result  is  [iiix  +  v7j)li^  =  niE^  +nR^] . 

Ex.  1.  Solve  lla;+57/-68  =  0 

6a;-7?/+31  =  0 
11  5        -08         11 

6       -7  31  6 


-77 
30 

155 
47G 

-408 
341 

-1U7) 

-321 

-749 

3 

! 

X 

7 
v. 

1^0  SIMULTANEOUS    EQUATIOKfc 


Ex.2.   12 

X 

25 

y 

=  1. 

22 

X 

^80 

y 

=  17. 

12  -25 

-  1    12 

22    30    - 

-17    22 

360 

425 

-  22 

-550 

-30 

-204 

910) 

455 

182 

1 

1 

2 

6 

II 

H 

1 

i_ 

X 

y 

.'.  x  =  2 

and  y  = 

-.5. 

2°  Let  the  equations  be 

a^x+h^if  +  CjZ+d^  =<> 

Arrange  the  coefficients  thus 


«1 

6i 

"i 

-<^I 

-«i 

-*I 

«2 

^2 

^'2 

-t/.. 

-S 

~6, 

«3 

h 

H 

-dz 

-^3 

-\ 

«I 

K 

<^i 

-d, 

-«1 

-*, 

«2 

\ 

^2 

-d. 

-«2 

-^2 

Selecting  the  first  three  cohimns  form  the  diagonal  product** 
from  left  to  right  downwards,  thus  : 


SIMULTANEOUS    EQUATIONS.  181 


«i     ^1     Ci  giving   a^b^c^ 

\ 
\ 

ao        ^2        Cj  a2*3''l 

\  \ 

«3       ^3       '■$  ^S^l'^fl 

\    \ 

<*1       ^1       ''l 


^2  •     ^'2       ^2 


Form  the  diagonal  products  from  right  to  left  downwards,  thus: 
«i     ^i     c,  giying  c^h^al 


«3        ^3        ''2 

/    / 

Cg&ga, 

a,      ^3      c. 

Cjjftja, 

a,      b-i     Cj 

/ 

«2         ''2         '^2 

From  the  sum  of  the  former  products  take  the  sum  of  the  latter 
products  obtaining  a  remainder,  which  call  R^. 

Similarly  form  a  2°  remainder,  Eg  from  the  2°,  3°  and  4°  columns 
a  3°  "         i?3         "        3°,  4°  and  5° 

a  4°»        «         i?4         "        4°,  5°  and  6"        " 

Then  a;=7?3  4-Z?i,  y^R^-i-Ri,  z=R^—R^, 
and  generally  * 

(wa;-{-wt/+;5z)Ei  =mR^-\-nR^-^pR^» 

Ex.  8.  3a:+2y- 42+20  =  0 
5x-ly-Qz-  1  =  0 
7a;+5y  +  5i!-24  =  a. 


:82 


SIMULTANEOCS    EQUATIOIiS. 


a 

2      -4      -20      - 

•  > 

—  ^ 

5 

-7     -G           1      - 

5 

7 

7 

5         5         24      - 

7 

-5 

3 

2      -4      -2!)      - 

3 

-2                                ' 

5     • 

-7      -G           1     - 

5 

7 

1 

-105 

-288       28    -500 

(3x  -7x5= -^105 

-100 

700     432         14 

5  x5x  -4=--  -100 

-   84 

-    20     500   -504 

7x2x  -6=-   84,  &c) 

-289 

392     9G0   -990 

19G 

GOO   -   15       240 

(-4x-7x7  =  19G 

-90 

10       480       980 

-6x5x3  =   -90 

50 

G72   -840         15 

5x2x6  =  50,  &c.) 

156 

1282  -875     1235 

-445) 

-890     1335-2225 

X  y  t 

Exercise  Ivii. 
Solve  the  followiug  systems  of  equati  ns  : 


1.     2x+Btj  =  4:l 
Sx+2y^S9 

3.     11a; +  12?/ =  100 
9a;+87/  =  80. 

5.     nx+ly  =  7 

5a;  +  3?/  =  -  36. 

7.     5a:+%+2  =  0 
3.^;+2?/  +  l=0 

9.     10a;+77/ 4-4  =  0 
.Gx  +  5//  +  2  =  0. 

11.    ^x+hj  =  ^' 

3a;-4y  =  4. 


2.     5a;  +  7?/=17 
lx-5y=    9. 

4.     18x-?j5y+ld  =  0 
15a; +28?/ -275  =  0. 

G.     3a;+lG/i-5  =  0 
28//  =  5a;+19. 

8.     21a;  +  82/+66  =  0 
23?/ -28a; +  13  =  0. 

10.     23a;+15?/-4i  =  0 
32.r+21?/-6  =  0. 

12.     lx-ly  =  l. 


SIJIUI^TANEOUS    EQUATIONS.  183 

13.     ly  =  lx-l.  14.     §x+§y  =  n. 

15.     l-5x-2y  =  l.  16.     lx=10y  +  -l. 

2-5x-dy  =  G.  Ux=lGy  +  -l. 

17.     5x-iy-{-l  =  0.  18.     ■lGx--Oi>j  =  l. 

l-7x-2-2y+l-d  =  0.  •ldx--lly  =  l. 

19.  3-5.c  +  2^v/  =  13+Ua;-3-57/. 
2i*-+  -8^  =  221+  -Tu;  -  3|!/. 

20.  1  +  JL  =  A. 

ic  y  6 

Jl        J_   _   J_ 

a;  2/  6 

22.  i:?  =  ?:!  _  1. 

«  y 

•8         3-6 

—  +    —    =    5. 

X  y 

24.    |,   A.,,.        . 

IT  +   y    -  '^^• 

20.  ix-\{:y+\.)  =  \. 
k{x+l)+l{^y-l)^'d. 

1  o 

28. 


3x-+l         5// +  4 
1  2 


4a;- 3    ~   7//- 6 

80.    l^^^±i   ==   8. 

45-1/ 


21. 

^    +    '^    =   3. 

X           y 

15          4 

—  —  —   =   4. 
X          y 

23. 

•3 
17a;    -    —   =   3. 

y 

•4 
IGa;    -     —   =    2. 

2/ 

25. 

';=  +  ■'  =  6. 

lOa;           9 

-   -    -j =    31 

5                  7 

27. 

.^+2^/    ~    2a;+2/ 

7     ■             5 

3a;-2   ~     G-y' 

2'J. 

x+^y  _  g 

7a;-13 
3//- 5 

31. 

3a;+l           4 
4-2y    -     3* 

{B+|/=l. 

=  4. 


J  84  SIMULTANEOUS    EQUATIONS. 

32.    1--^  =   1.  33     ^^±^1±}   _   2 

5-3?/  2  2a;-?/  +  l    ~ 

7-22/   _   _2  8.r-y+l 

5 -3a;   ~    3*  «-!/  +  3     ^    ^' 

3^        a;+3y+13                                    x+1         y+2         2(a;-y) 
*"  •    •4X+-52/-2-5   =   ^^'  ^^-     ~3 r    =  ~5^ 

•8a;+ajH--6    _   J_  ^-3         y-3 

6a;H-%-23    ~   T'  4      ""       3       =   ^^  ~  '■ 

36     ^^-y+^         a;-2y+3 
3  4^~ 

3a;- 47/+ 3         4a;-2?/-9 

i —  +  — y—  =  ^• 

87.    20(.c+l)  =  150/  +  l)  =  12(a;+7/). 

38.  (a;-2)  :  (y+l)  :  (a;+.v-3)  ::  3  :  4  :  5. 

39.  (a;-5)  :  (y/+9)  :  (a;+^  +  4)  ::  1  :  2  :  3. 

^^-    ^    =    ^-  ^1-  (^-4)(.'/  +  7)  =  (^-3)(i/+4). 

X+i  2/r" 

^)   =   5W  (.+6)(,-2)  =  (.+2)fa-l). 

42.    (a;-l)(5y-3)  =  3(3x+l).     43.  (a;+l)(2//  +  l)  =  5a;  +  92/  +  l. 
(a;-l)(42/  +  3)  =  3(7a;-l).  (a;+2)(3//+l)  =  9a;  +  13y  +  2. 

44.  (3a;-2)(5i/  +  l)  =  (5ar-l)(2/+2). 
(3a;-l)(s^+5)  =  (a;+5)(7y-l). 

45.  a; +  7/ =  37.  46.     2a;+2z/  =  7. 

?/+z  =  25.  7a; +192  =  29. 

z+a;  =  22.  i/+8«=17. 

47.     l-3a;-l-9</  =  l.  48.     5x-+3i/+2z  =  217. 

17//-Mz  =  2.  5a;-3s^        =   39. 

2-9z-2-lx  =  3.  3?/ -22=   20. 

49.     \x-\xj  =  ^.  50,     l^a;+l|?/  =  10. 

^a;-:|z=l.  2§x+2|0  =  2O. 

^«-jy  =  2.  3i-s/+3iz  =  30. 


SIMULTANEOUS    EQUATIONS.  i^" 

//+z-a;=l3.  x+2y  +  iz=li. 

z+x-y=    7.  a;+3/y+y2  =  ^B. 

53.     ^.a.^^^^=    3.  54.     7x+6?/  +  7i  =  100. 


x-y 


=  0. 


2^-+4?/-f-  F3=13.  :c-2(/+  z  = 

3a;+%4- 272  =  ;U.  ?ix+  y  ~2z  =  G, 

Sr,.     3j5+2!/  +  82-110,  60.     .c  +  y+z  =  d. 

5x-\-  ?/-42  =  0.  a;  +  2y  +  3c-14. 

2x-3y+  z  =  0.  a:+3.v  +  Gz  =  '20. 

57.     aj+2//  +  82  =  32.  58.     x  +  y  +  2z  =  %4t. 
2^  +  3y-(-z  =  -42.  «+2.v+z  =33. 

3x+   ?/  +  22=10.  2a;  +  ^i2^82. 

59.     3.C+3//+  z  =  17.  60.     c+'ly-z=   4-6.. 

3a;+    y4-Cz=15.  y  +  -2z~x=l(yi. 

x+Sy-\-Bz  =  lS.  z  +  1x-y=    5-7. 

61.       x4-2i/--73  =  21.  62.     x+^^l^z  +  S. 

3a;  +  -27/-   z  =  24.  </+z  =  2f/y-U. 

•9;c  +  7^- 2^  =  27.  z +a;  =  3|.,--82, 

63.     ix+il/  +  |3  =  30^.  64.     %x  +  ?>\y^A\z=\m 

ix  +  iv  + 12  =  27.  3^x  +  4iy/  +  .^^z  =  ?  75. 

1:^  +  ^2/ +  ]2  =  18.  2§.c  +  3|^+l?2=iuV. 

65.     -+i    =    2.  66.    -^_p^-    =    ^ 

z+3    _    ,  gg  +  ^    ^.    9 

:^    -    ^'  2/  +  1     "    ' 

67.     -_±^   =    10.  68.   ^^   =   2. 


a  +  z 


186 


SIMULTANEOUS    EQUATIONS. 


69.    i  -  A  ^  3.       y     70.  —  +  :l  +  :l  =  i 


v  y 

2  '6 

X  z 

y  g 


=   4.  _  +   —   +  —   =   4. 

X  y  z 

^  12         10 

V.      0.  '-  + =      ^r 


71.     ^-   .    4-  72. 

J/2      _     1 
j/+z   ~   "6" 

za;  1 

2+a;   ~    7  ' 

73.  (.r+2)(2?/-i-l)  =  (2a;+7)//. 

(a:  -  2)f32  -^  I)  =  (x+3)(3z  -  1). 
(y  +  l)(2+c     =(2/+3)(  z  +  1). 

74.  (2a;-l)(2/+i;=2(a;+l)(v/-l). 
(a,+4)(z+l)    =(a:+2)(0  +  2). 
(2/-2){z+3)    .-=(y-l)(2  +  l). 

75.  (rc-ui)(%-3)  =  (7.t-+l)(2//-3). 
(4a;-l)(z+l)  =  (a;+l)(2z-l). 
(i/+3)(2+2)    =(3^-^)(3z-l). 

76.  21a;+31?/+42z=n5. 
6(2aj+i/)  =  3(3a;+2)  =  2(^+2) 

77.  15(a;-22/)  =  5(2a:-3;?)  =  3(?/+2). 
21a;4-31^  +  dlz  =  lSrj. 

78.  6a;(i/+2)  =  4?/(z  +  x)  =^  32(x  ^  vj. 
1  1  1 


79. 


6 

4 

y 

6 

+  — ■ 
z 

—   + 

X 

8 

5 

ft/ 

'-   + 

X 

12 

10 

y 

z 

xy 

iy  —  3x 

= 

20. 

xz 
2x  -  Si 

.-= 

15. 

yz 
4»/  -  oz 

= 

la. 

a;           2/           ^ 

=    9. 

^ 

3aj +7/4-2  =  20. 
3«+a;  +  4?/  =  30. 

3?f.  +  6.c-fz  =  40. 

6u-f8;/-f3^  =  50 

60. 

a-+;'-f  S//  =  33. 

5»  f  ?/-f-2  =  ii, 

i^z+A-f  2  =  11. 

3?.  ^  a'+^  =  J.l. 

ST:\ItTLTANEOUS    EQTTATIOXS.  187 

81.      a-{-x  +  i,  +  z  =  lU.  82.     ■u  +  .r-\-jj-^z  =  ^4:. 
It  +2j;-{-2ij  +  2z=-2,67.  ti  +  2x+  3i/  -  L'z :-  0. 

n  +  2x+Bu  +  3z—SrA).  Su-x-5ij-^z  =  J. 

7(  +  2x-^3i/+iz  =  110.  2u+3x-4:y -  5z  =  0. 

38.     u-{-x  +  y+z  =  GO.  84.      :i-{-x-\' ,/-\-z  =  l. 

u-\-2x+3ij+-iz  =  100.  2u-\-4x  ^  8//  +  IQz  =  G. 

u+Sx+G>j-\-10z=150.  32i  +  9x+27.(/  +  8l2  =  lo. 

?t  +  4.«;+10//  +  202=210.  4?H-16x-f  04//  +  25Gz  =  35. 

85.     ^x+y-U  =  l.  85.     ht-^x-\-iy-:^.^Vi. 

lx-^P-lu  =  l.  ^u-Sx+l-,j-l^^l7. 

f.'/- 12-^(6  =  0.  pi^}^x-y  +  i?=n. 

Art.  XLVII.  Tlie  principle  of  symmetry  is  often  of  use  in 
the  solution  of  symmetrical  equations.  For  from  one  relation 
which  may  he  found  to  exist  between  two  or  more  of  the  letters 
involved,  other  relations  may  he  derived  by  symmetry  ;  also, 
when  the  vahie  of  one  of  the  unknown  quantities  has  been  deter- 
"lained,  the  values  of  tlie  others  can  be  at  once  written  down,  &o, 

1  •  (x4-  y)  {x+z)=a. 

{x+y){y+z)  =  /^- 
(x+z){y+z)=c. 

Multiply  the  equations  together  and  extract  the  square  root. 

•■•    {x+y){y+z){z+x)  =  i/{ahc). 
Divide  this  equation  by  the  third. 

.'.   x  +  y  =  }— i— — i-,  and  therefore,  by  symmetry. 


.*.     J/+Z    = 

.".    z-\-x  = 
Hence  we  get 

X  = 


c 

1 

a 

V[ahc) 


nh  —  bc-\-ca 


2Viabc) 
wlienee  y  and  z  may  be  derived  by  symmetry. 


188  SIMULTANEOUS    EQUATIONS. 

2.         x.+v-^-z-O (1). 

vx  +  by  +  cz  =  0    (2). 

br.r+caij-^ahz-{-{a-b){b-c){c-a)  =  0: (3). 

cx(l)-(2)  gives  {c-a)x+{c-b)y  =  0. 

.'.  y  =  ^ L,  and  similarly, 

b  —  c 

_  (a-b)x 

—      r • 

b  —  c 
Substitute  in  (3)  these  values  of  y  and  z,  and  reduce, 

,-.  x{a  —  b){c —a)  =  (a  —  b)(b  —  c){c  —  a), 

.'.  or  x  =  {h  —  c),   .'.  y  =  c--a,  z  =  a  —  h. 
8.  a{yz—zx—xy)  =  b{zx  —  xy  —  yz)  =  c{xy  —  yz  —  zx)  =xyz. 
Divide  tlie  first  and  the  last  equations  by  axyz  ; 

.-.   —   —   —   _   —  _  — ,  and  hence,  by  symmetry, 
axyz 

J-  _  i-  _  -1  _  i- 

b     ~     y  Z  X 

1  _  JL  _  1  _  1, 

c  z  X  y 

,♦.  _)_  —    =    —  —,  and  by  symmetry, 

be  X 

L     i_  -  _  A 

c  a  y 

!_     J_  _  _  A. 

a            b                   z   ■ 
i.         ax -Y  by -\- cz  =  \  (1 ) 

«2.f+A3^+r32  =  l (2). 

a^x  +  b^y-\-c^z  =  l     (3). 

<;x(l)-(2)    gives    a{r-a)x+b{c -h)y  =  c-\ (4). 

cx(2)-(3)        "       a^-{c-a)x  +  b^{c-b)y  =  c-l (5). 

4x(4)-(5)       "       ah{c.-a)x-a^{c-U)x  =  h{c-l)-{c~l), 
ox  a[a  —  b){a  —  c)x=-{G-l){b—l), 

...  X  =    (1-^Hi-c). 

a[a — b)[a  —  c) 
wlienae  y  and  z  may  be  derived  by  symmetry. 


SIMULTA?^SO'JS   EQUATIONS.  189 

6.  Eliminate  a;,  y,  z,  u  (wLich  are  supposed  all  diflferent)  trom 
the  foUowiug  equations  : 

x=^hy-\-cz-\-du. 
y-cz  +  du+ax. 
e  =  du-\-az+by. 
u  =  az-\-hy+cz. 

Subtracting  the  second  equation  from  the  first» 
.•.  x  —  y  =  hij—ax,  or 
(1  +  a)x  =  (1  +  h)y  =  (by  symmetry)  (1  +  '^)z  =  (l+dju. 

These  relations  may  be  &lso  obtained  by  adding  ax  to  both 
members  of  the  first  equation,  by,  to  both  members  of  the  second 
squation,  &c. 

Now  divide  the  first  equation  by  these  equals, 

1 b__  c_         _d_ 

"    \  +  a    ~    i  +  6   "•"    r+c   "*■  \-ird 

And  since  =    1  — we  have 

1+a  1+a 

^   _      a  b  c  a 

Exercise  Iviii. 

b-c-bc' 

1 .  (jriven  ax  -\-  bi/  =  c  and  that  x  =  ,— t~i* 

''  b  a  -ba' 

a'x+h  'y  =  c'  derive  the  value  of  jr 

a(dm—  en) 

2.  Given  bx  =  ay  and  that  x  =  — ] t — » 

"^  be  — ad 

dx + ""?  =  cy-\-n c  derive  the  value  of  y. 

8.  Given  ax-{-bi/-^cz  -d.  and  that  a;  = 

a(d-b)(d-c) 
a^x-{-h^ij-\-c^zz=d-       "-T pr/ \.  write  down 

<3iSx-\-b^y  -\-  e^z  =d^      the  values  of  y  and  z 


190 


SIMULTANKOUS    EQUATIONS. 


4,  There  i?  a  set.  of  equations  in  x,  y,  z,  u,  and  w,  -witli  corres- 
coucling  coefficients  {a  to  x,  &c.),  a,  b,  c,  d,  and  e ;  one  of  the 
equations  is 

x  =  by  +  rz-\-du-\-ew,  write  down  the  others. 
Solve  the  following  equations  : 

K      ^     ,     y  V  z  ^     X  2 

m  n  n  p  m  p 

6.  x-\-a)/  +  J>z  =  7n,  y  +  az  ^hx  =  n,  z  +  ax-\-hij  =p. 

7.  x  +  rtv  =  /,  y  +  bz  =  )n,  z  +  cu  =  ii,  u-\-dw=p,  w  +  ex  =  r. 

8.  Eliminate  x,  y,  z,  (supposed  to  be  all  different)  from  the 
following  equations : 

z=by-\-cz,  y  =  cz  +  ax,  z  =  ax-\-liy. 

9.  Eliminate  x,  y,  z,  from 

X  y  z 

=   a,    — . —   =  i), =  c. 


y-]-z  '   z-\-x  *    x+y 

10.  Having  given 

x  —  hii-\-rz-{-dv.-\-ruj, 
y  =  cz  +  dn-\-en--\-((x, 
Z  =dit  +  (ii-\-ax  +  by. 
u  =  ew  ■]-ax-]'by  +  cz, 
w  =  ax-^by-^cz-\-dii, 

abode 

Shew  that    ,-7—    +   ip- —7   +   tT'  +   i~r~)  +  rT~   =    ^^ 
1+a         1  +  0         1+c         l+(t         1+^ 

Art.  XLVIII.     Eesolution  of  Particular  Systems  of  Linear 
Equations. 

Ex.   1.                    •           x+y-\-z  =  a  (1) 

y+z-^u  =  b  (2) 

2-fH  +  . r  =  c  (3) 

ii-{-x+y  =  d  ^        (4) 

(l)  +  (2)  +  (3)  +  (4)              B(u  +  x-\-y-^^)=r>  +  h+r+d  (5'] 

8(1)                                            3(a;+7/+z)  =  3«  (C) 

|t{(5')--(C')}  tt  =i(-2rt4-AV  +  ''-) 


SIMULTANEOUS    EQUATIONS.  191 

The  values  of  x,  y  and  z  may  now  be  written  down  by  sym- 
metry. 

Tiie  following  is  a  variation  of  the  above  method,  applicable  to 
a  much  more  general  system. 

A-Ssume  the  auxiliary  equation 

u-\-x-\-y-{-z  =  s,  (5) 

.-.  (1)  becomes  s  —  a-.a,  (6) 

(2)  "  s-x=^b,  (7) 

(3)  "  s-ij  =  c,  (8) 

(4)  '•      .  s-z  =  d,  (9) 
(5)  +  (6)  +  ^7)  +  (8)  +  (9)                     4.s  =  s  +  a  +  h^-c+d. 

:.    s  =  \{a ->!■■}> -\-c-^d), 

s  is  now  a  known  quantity,  and  may  be  treated  as  such, 
in  (6)  giving  u  =  s  —  a 

(7)  "  x  =  s-b 

(8)  "  y  =  s~c 
"  (9)     "  z  =  s-d. 

Ex.  2.  yz  =  a{y-\-z),  (1) 

z.c  =  b{z+:^,  (2; 

xy^c{x-\-y),  (3) 

111 
(l)^ayz,  ^   -\ =    — , 

y      .      z  a 

/o^      /  111 

Z  X  o 

111 

{Sy-i-cxy,  1 =   — . 

X  y  G 

This  may  now  be  solved  like  Ex.  1,  using  the  reciprocals  of  a 

b,  c,  X,  y  and  z  instead  of  these  quantities  themselves. 

Ex.3.  ai«+ii(a;+7/+2)  =  r,  (1) 

a„x-\-b2{y+z  +  u)  =  c^  (2) 

'^3.'/  +  ^3(2+   i  +  'i^)  =^^3  (3) 

a4Z  +  b^{u-^x+y)  =  c^  (4) 

Assume  the  auxiliary  equation 

u-tx+y+z  =  s.  (5) 


V\;iJ  SIMULTANKOUS    EQUATIONS. 

\i;  becomes  b^^s—ib^— a ^)it  =  c^ 

by         '  r, 

«i  — «!  Z>i  -  a,  \  / 

Similarly  from  (2)  ,     ^     s  -x  =  j — ^  (7) 

2        "2  2  ~  '2 

^    '  ^B-'^3  ^       h-^3  ■  ^    ^ 

From  (10)  we  can  at  once  get  the  value  of  s,  which  may  there- 
fore be  treated  as  a  known,  quantity. 

bjS  —  Cy 
in  (6)  giving  u=,    _ 

"\      "i 

'and  the  value  of  a;,  y,  and  z  may  be  obtained  from   (7),  (8)  and 
(9),  or  they  n'va^y  be  written  down  by  symmetry. 

Ex.  4.                                  ax-\-b(}j-^z)  =  c         '  (1) 

ay+-h{z  +  n)  =  d  (2) 

az-\-hl(ii-\-x)  =  e  (8) 

au-irb{xJf-y)^f  (4) 

Assume                           tt+.T  +  i/  +  z  =  s  (5) 

(l)  +  (2)  +  (3)-f(4)          (a  +  26)s       =c  +  ^  +  r+/  (6) 

Hence  s  is  a  known  quantity  and  may  be  treated  as  such. 

From  (1)  and  (5)  hs—bu-\-{a  —  h)x  =  c, 

:.     bi(  ~(a  -  b)x=bs—c,  (7) 

Similarly  from  (2)  and  (5)    bx -{a  —  h)y  =  bs—d,  (8) 

"     (8)     "     "      by-{a-b]z^bs-e,  (9) 

*•  (4)  "  "  lz~{a-b)n^bs-f,  (10) 
b{7)  -{-{a-  b){8)  i"  a  -  (a  -  b)  -y  =  abs  -bz-  {a  -  b)d,[l  1 ) 
6(9)  +(^i  -  b){\i))            b-^y  -  (a  -  h)"- a  =  ab>i -  hi,^  {a-Oy,  (12) 


SIML'LTANEnuS    EQUATIONS. 


1'15 


-«(/>2./  +  ("-6)3/-}-6{63(c-fZ)  +  (a-6)2(.-/)}    (13) 

The  values  of  x,  y,  aud  z  may  now  be  written   down  by  sym- 
metry. 

Ex.   5.  a^  +  a^.v-\-ai/+z  =  0, 

b^  +  b^x  +  by-\-z  =  0, 

The  polynomc  t^  +  .<;;-  -{-yt+z  vanishes  for  i  =  a,  t  =  h,  t  =  e, 
.'.  by  Th.  II.,  p.  4G,  for  all  values  of  t. 

t^  +.rJ2  +yt+z  --=  {t-a){t-  h)(t  -  r\ 
=  t^-{a  +  h-\-c)t-+{ah  +  hc  +  ca)t-ahc. 
,-.  Th.  III.,  p.  53,  x=  -{a-\-h-\-c), 

y  =  ah-\-bc-^ca, 
2=  —ahc. 


Ex.  6.  a;+?/-|-2  +  ?/  =  ], 

ax  +  /^^  -\-fZ-\-  du  =  0, 

Employing  the  motliod  of  arbitrary  multipliers, 


(4)  +  /(3)  +  »»(2)+?i(l)         a^x+h^y+  c^ 

+  ma     +  ?.'fi     +  VIC 
-\-n  I    4-  71  I    -{■  n 

To  determine  x  assume 

63-f /Z/2-l-H?7>  +  n  =  0, 


0  +  d^ 
+  ld'^ 
+md 
+  n 


u  =  n 


(2) 
(3) 
(^) 

(5) 


iC  = 


(7) 

{^) 

(9) 

But  the  system  (6),  (7),  (8)  has  been  solved  in  Ex.  5,  from 
which  it  is  seen  that 

1=  —{h-\-c-\-d),  m  =  hc  +  cd  +  db,  n= —bed, 
and  a^+a~l-]-ain  +  n  =  {a  -b)(a-c){a- d)  i 


'i;H  SIMULTANEOUS    EQUATIONS. 

•.  using  these  values  in  (9) 

-hcd 


( a  —  h){^a—  c){<:i  —  d) 

The  values  of  ?/,  z  and  u  may  now  be  written  down  by   sym- 
metry. 

Ex.  7.  ^^  +  _L  ^  _^   =  1.  (1) 

VI  — a  m  —  0  m  —  c 

-       +^J'+^^^l,  (2) 


n  —  o  n  —  h  n~c 

-f-  — ^V  +   —  =  1-  C'^) 

p  —  a  p—b  ]j  —  c 

Assume  1  -   ^_-  _    .  L'  _   _^_  _   t^+Bt^~-\-Ct+D 
t  —  a  t-h  i-c     '     (t-a){t~b){t-c) 

But  in  virtue  of  equations  (1),  (2)  and  (3),  the  first  member  of 
(4)  vanishes  for  t  =  m,  t  =  n,  and  t=p,  and  .-,  t^  +  Bt-  +  Ct  +  D 
♦janishes  for  the  same  vahies  of  t,  and  .•.  Th.  II.  p.  46, 

ti-\.Bt^  +  Ct  +  D  =  {t-m){t-n)(t-p), 


.'.  (4)  becomes  1  — 


X  y  z 


t  —  a  t  —  h  t  —  c 

_    (t-m){t~H){t-p) 
'"  {t-a){t-b){t-c)  ' 

To  obtain  the  value  of  x  multii^ly  both   sides  of  this  equation 
hj{t-a), 

t-a-x-  y^^~^)    _    z{t-a)     ^    {t-m){t-n)(t-p) 
t-b  t-c       ~  (t-b}{t~c) 

Now  t  may  have  any  value  in  this  equation  ;  let « =  a. 

(a  —  ni)(a  —  n){a  —  p) 
~  {a—b){a  —  c) 

The  substitution  (xyz\abc)  will  give  the  values  of  y  and  t, 

Ex.  8.  ^^t^   =   ?^   =    ^±i  .  (1) 

p  q  r 

lx  +  7Hi/4-nz  =  s^  (2) 


simulxa! 


By  Art.  XXXVII., 

x+a         y  +  b  z-i-c 


??5 


v 


(2) 


q  r 

s^-{-la  +  mb-\-nc 


Ix-\-))iy-{-fiz-{-ia-{-miy-\-nc 
lp-\'-viq-^nr 

=   II,  say 


lj)-\-niq-\-nr 
,'.  x=pR-a,  y  =  qR  —  b,  z  =  rR—c. 


Ex.9. 

yz+zi«+xy  =  (a-fi  +Cjxyz 

(1) 

yz +zx         ZX+  xy         xy + yz 
a        "^        b         ~        G 

(2) 

{l)-xy» 

11,1 

—   +          +          -    a+b  +  c. 
X           y            z 

(3) 

{2)^xyz. 

11111 

—   +  —         —   +    —          —  + 

X        y    ^    y         z    _    z 

y 

X 

(4) 

a                       b                        c 

Page  122  and 

(3)                          "    4-    2    +    2 

-   X        y        2    _ 

a-\-b-\-c 

2 

c^) 

(4)  and  (5) 

11                      11 

.-. h  —   =   2rt,    —    +   —   = 

X            y                     y            z 

20, 

1            1 

—  +  —  =   2c. 

Z              X 

(6) 

(3) -(6) 

111 

—   a  —  b  +  c,          —a-\-b—c,          — 
X                          y                        z 

—  a 

+  h+c. 

Ex.  10. 

aJ  +  c     ,    y+b 
a+b         a-\-G 

W 

"-''  +  ^'-f   -   2. 
«— c          a—b 

(2) 

(1) 

x-\-c                              y  +  b 
a-\-b                              a  +  c 

x—a—b+c           a+c — b—y 
a+b           ~          a+G 

(3) 

1C6  SIMULTANEOUS    EQUATIONS. 

Similarly  from  (2)      5Z/^1±£   ^    a-h  +  c-y_  ^^^ 

a—c  u—b 

(3)  and  (4)  ...     x-a-h-\rc  =   '^-(a-h-\-c-y) 

a-\-G 

But  unless  ^- —    —   ^  ~^.,  this  cannot  be  the  case  except  for 
a  +  c         a  —  b 

a-b+c-y  =  0, 

in  which  case  z—a  —  b-{-c  =  0  also, 

giving  x  =  a  +  b  —  c  a.Tady  =  a  —  b+c,  (5) 

Ti;      a  +  b  a~c 

If     ---—    =:   r-  .-.    a3-£3  =  ,,2_,.2  (6) 

a.+-c  a  —  b  ^  ' 

63-c2=0,  or  (6  +  c)(5-6-)=..0, 

&  =  c,  or  b—  —c. 
But  if  &=  +c  or  — e,  (1)  and  (2)  are  one  and  the  same  equation ; 
hence  if  (1)  and  (2)  are  independent,  (6)  cannot  be  true,  thus 
leaving  only  the  alternative  (5). 

Ex.11,  "lax^ib  +  c-aYii-^z),  (1) 

2by  =  {c  +  a-b){z^x),  ■     (2) 

(a;+?/  +  z)-'+a;2  +  ?/-'+23=4(a2+62  +  f2)     (3) 

(1)  and  page  122  (5)^_    =    Ip-   =   ^+l±f  (4) 

b  +  c  —  a  2a  o+c+a 

(0)     u  u  y         _     x+z     _    x  +  !j-\-z  .gv 

c  +  a-b  2b       ~    c  +  a+b  '  ^  ' 

^4),  (5)and     "    ...^+?/+^   =.         ^         =         ^         =    __J__ 
«  +  /'  +  (^         6+c  — «         c+a-6         rt-|-6-c* 

«3  (a;+?/+z)2+x2+?/2_(_22 


Reduction  and  (3)  =   4(^24.^,2^^2) =   ^* 


SIMULTANEOUS    EQUATIONS. 


1&7 


1  1  1  ... 

Ex.  12.       .  ax   =by  =  cz=   ~   +  —  ^-   ^  \^) 

.a  h  c  _      ff+6+c 

{l)^xyz         :.     -  =   ^   =  :^  •  •   -  ^H^+---»  ^^ 

a  1     /I       1       1\         a;(/  +  ?/z  +  z.c    ' 

(2)x(3)  •      4\-    =   ^i^     • 

:.    a^x^    =    a+b-\-G. 


Ex.13. 

a         ~          b          ' 

c 

(n 

xyz  =  m^ 

(?) 

(1) 

z               X              y 

a+  b    ~   b+c   ~   c  +  a'  ~ 

m 

suppose 

(3) 

then 

xyz                      m^ 

{a-\-b){b  +  c){c  +  a)          rs 

.-.     r^  =  (a  +  b){b  +  c){c+a) 

Hence  the  value  of  r  is  known  and  from  (8) 

rx  —  m{b-\-c). 

Ex.   U.  y+z  =  2axijz  (1) 

2+u;  =  26.<;^z  (2) 

a;  +  ^  =  2rjc2/«  ,:  (3) 


y-\-z        z+x        x-\-y        x+y  +  z 

•*•  ^y^^  ~2^  =  ~2r  "^  ~2r  "^  ^+y+o 

a;  7/  z 


b  +  c  —  a         c-{-a~-b         a-{-b  —  G 

xyz 
:.  x^yH^  =  (^f)^c-a){c  +  a-l)){a+b-c) 

1 

.-.  x^yH^  =  ^^r^  a) (c -I- a-  b){a + 6  -  c)/ 


(-t> 


t9B  8IMULT4NE03S    EQUATIONS. 

Hence  the  value  of  x^y~z^  is  known,  call  it  —^  and  substituto 

in  (4)         • 

1  X 


r     ~    b  -{-  G  —a 
.-.  rx  =  b  +  c—a, 
in  which  r^  =  [b-{-c  —  a)(c-^a  —  h)[a  +  b  —  c). 

Ex.15.                       y^+z^-x{2j  +  z)  =  a  (1) 

z^-+x^-y{z  +  x)  =  b                _    .  (2) 

a;3+2/3-2(.x-+^)  =  t  (3) 

(l)  +  (2)+(3)  2{x-+y^+z^-xy-yz-zx)  =  atb-\-c    (4) 

(1)  may  be  written ,      x'-^  +y-  +2-  —x{x+y-]-z)  =  a  (6) 

(2)  "            "              x^  +  y"+z-^-y{x  +  y-\-z)  =  b  (6) 

(3)  "           "             x^+y"+z^-z{x+y+z):^c       -  (7) 


.•.  x  +  y+z  ■= 


a  —  b  b  —  c  c  —  a 


y  —  x         z  —  y         x  —  z 


\y  -x)-  ^{z  -yy  +  {x-z)^ 

a'i^h^j^c^-ah-bc-ca 
x^+y'^-{-z^  -xy  -yz  —  zx 


2(a3+&3_f.c3_3aJc) 
Write  r2  for  2(a3  +  &3+c3  _3^,;,c^^ 


(9) 


(9)  ....+,+.  =  __L__  (10) 

Eeturning  to  (8)  {x-^y^zY  =  ^^"^  +  ^^t+r+t  "'^"''^       <^) 

(4)  2(x-^  +r"+2"-^i/-2/2-2-0  =  ^^^±^        (11) 


BlMUL'rANEOUS    EQUATIONS.  1'^ 

i[(8)  +  (n)}        x^'-^^r-^z-^  =  ^l±}l±^  (12) 

(5)  ami  (10)  x-+7/-^z"^-  — ^_    =    a 

a  +  b-\-G 

(12)  ^a^j^h^-j^c"_a{a  +  h-\-c) 

-h^'+c^  -a{h+c). 

(5),  (6),  (7)  are  symmetrical -with  respect  to  (.-ryzjfiroc);  (10)  sliowg 
this  substitution  does  not  affect  r,  and  consequently  the  values 
of  2/  and  z  may  be  written  down  at  once  from  that  of  «» 


Exercise  '. 

lix. 

1. 

az+h]/  =  c, 
mx+ny  =  d. 

2. 

(iz~\-hy  =  c, 
mx  —  ny  =  d. 

3. 

ax  +  by  =  c, 
mx-\-ny  =  c. 

4. 

-2-  +  ■'   =  1. 

a           b 
x+y  =  G. 

5. 

a    '^    b 

1, 

6. 

a     '     b     -   ^-^ 

X           V 
b     ^    a 

1, 

X           y 
b           a 

7. 

ax+bc  =  by-{- 
x  +  y  =c. 

ac. 

8. 

a           h 

b            a 

—   +  -=«. 
X            y 

9. 

{n-^c)x-{a- 
(«  +  %-(a- 

■b)x  = 

■-  2ab, 
=  2ac. 

10. 

x—c          a 
y-c   ~     b' 

x  —  y  =  a  —  b. 

ll" 

X           a 

-     /■' 

y         '' 

x+m          c 

1 

12. 

x+y         a+h-{-c, 
7/-|-l      '    a  —  b+c' 
y—1         a  —  b^c 

y  +  n  d  x+l         a+b  —  c 


200  SIMULTANEOUS   EQUATIONS. 

y-a+h           c  '     a+b   ^   a+c            ' 

y-^b          c  +  a  x  —  b         y-c 

"     "    =    7  — •  4- —    2 

x-\-c         h-\-a  a  —  c         a  —  b            ' 

vi-a  '^  m-b  ~      '                      (b-^c)x-\-{a-\--)//~\-{u-{-b)z 

X              y  ^                       =0, 

+    -    ^  =    1 

«  -  (7          n-b  '                        hex-\- acy  +  ahz  =  1 . 

17,     x-{-y+z  =  l,  x-a         y-b         z-c 


15. 


-» 

r 


19. 

;s-a         y-b         z-c 
P               q      ~       r    ' 

hc-{-my-\~vz  =  l. 

21. 

x-\-y-\-z  =  a-\-l>JrC, 

hx\cy  +  nz  =  a^-\-b'^+c^. 

cx-\-ay-\-hz  =  a^+b^  -f  c2. 

ax  +  hy  +  cz  =  m,  ^^-     'p  q 

Jl_     s V_      ,        «  ,  l{£-a)^m{l^-b)^-n{z  -c) 

I- a   ■•"   Z-i    +  TZTc    "    ■^-         =1. 

20.     rf(a;-^)=%-/j)  =  r(2-c), 
ax\by-\-cz~  in'^. 


22.     a;+?/+2  =  0, 

ox  +  6y  +  c«  =  rt/)+Sr  f  r/7, 
(^;  -  c).«  +  (c  -  «)^  -f  (a  -  b)z 
=  0. 

23.     a;+^-}-2;  =  /n,  21.     ax+by  +  cz  =  r, 

X  :  y  :  z  =  a  :  b  :  c.  inx=  <ny,  qy=pz 

25.     xy  +  yz^zx  =  0,  ayz-\-hzx^cxy  =  0, 

hcyz+a.cxz-\-ahxy  ^{a  -b){b  —  c)  {c  —  a)xyz  =  0. 

26.  (a  +  6)x4-(?'+c)?/  +  (c  +  a)z  =  a6  +  6c+ca, 

{h-irc)x-\-{a  +  c)y-\-(a  +  b)z  =  a^+b^+e'». 

27.  *ox  +  ny+jiz  +  qu  =  r, 

X  y  z  u 

a  b  c  d 


SIMULTANEOUS   EQUATIONS.  JOl 

a  b  c  o  ^b-c 

1  1  1 

—   +    —   +   —    =■-    a-\-h-\-c. 

X  y  e 

29.  {<i-b){c-\-c)-a:j-{-hz-={c-a)[ij -{■}))  -C2-faaj  =  0, 

x-\-ij  +z  =  '\'<  +^  +  c). 

30.  a:'-+h//  =  l,  31.         lf/+mx  =  n, 

b//  +  cz  =  l,  '       vx  +  lz  =  m, 

cz  +  ax  —1,  viz-\-  ny  =  L 

82  x  +  y  =  a,  33.    ^+2-r  =  '-^^ 

,  /)i 

2/+2  =  o,  H-.C  -y  =  — f 

x-^-z-c.  z-\-y~z  =  — , 

n 

84. 1 =    2a.  85. ^  — , 

y  Z  y  z  X  a 

1  ^  o.  1112 

«  X  z  X  y  b 

a;  2/^'  ^  y  z    ~    c 

86.  (a  +  ?>).r4-(a -5)^  =  2ic, 
(6 + c)7/+ (&  —  c)a;  =  2ac, 
(c4-n)z4-(c —  «)?/  =  2a6. 

87.  a:  +  f  -   -    =    a.      ^8.    ^  +   -^    =    6-.,, 

/j  c  b-\-c  c-\-a 


Z  ^       _     -L  ^  JL     ?- 

T  ~  T  "    '  T+7  "''  ~^b 


y  +  —    -   —    =    ^y  -TTT    +    TTTIT    =    ^-^* 


XV  y  z  , 


a 


b  '  c+a  a +  6 


'202  SIMTTLTANEOUS    EQUATIONS. 

39.    a:+Z/  -2  =  ",  40.    jt-f-v-x  =  a, 

y  -^-z  —  v  —  h,  v+ X  —  y  ~  b, 

z+v  —  x=c,  x+>/-z  =  c, 

v-\-x-y=d.  y+x-u  =  d, 

z  +  u  —  r  =  e. 

Exercise  Ix, 

Eesolve 

1.  {a+b)x+(a-b)y  =  2{a''-\rb^)  2.  x  +  y  =  a, 

{a-b)x  +  {a  +  b)y  =  2{a^-b2)  x- - y^  =b. 

3.     2x  —  3y  =  m,  4.     {a  —  b)x-l-(a-\-b)y  =  a  +  h, 

2x-  —  3y^  =  n^  -{-xy.  x  y  \ 


a  +  b  a  —  b         a-{-b 

K     V        7\     .          a^-h  +  1  {a-irb-c)x-{a-b^c)y 

5.     [a  ~-  b)x+y  =  — — rr  '  "•          i    n       \ 

a  —  b-^1  X          a-\-b—c 

Z-\-ia-\-b)y=  T~  '  —   =    T~7~' 

'  V    '    /:/         a  —  b  y          a  — 0+0 

x-\-ii            a  x  —  a         a  —  b 

7.     — ^    =-    j .  8.     =    — T- 

x+c         y-\-^  ^          a^~b^ 

a  +  b    ~    a-\;-0  y           a^  +  b^ 

x-y+1  _  .Q  ^+y_±l  _  "+} 

x  —  y—1  z—y  +  1         a — 1 

x+y+1    ^   ^               '  x+y  +  1  _  1  +  h 

x+y-1    ~     '  x  —  y-1         1  —  b' 

x+y—1  a+b         a—b         '    * 

x+y+1         ,  ^     ,     ?/          o 

x—y—i  a           6 

13.     {a  +  c)x+{a-c)y  =  2ah,  14.     rt^+r/.r  4-7/  =  0, 

(a+6)?/  — (a-fe)jc  =  2ac.  b'^  +  bx  +  y  =  0. 


*  SIMULTANEOUS    EQUATIONS.  IZHB 

15.       y  +  Z  — :f  =  rt.  10.      7:r-{-ll;/  +  z  =  a, 

j:  +  i/—z  =  g.  7z-{-11x  +  ij  —  c. 

X  1/  z         '^'    '  •  . 

—     <     —   _  IL   ^2hc  (c-a){>/-rb)-ci,+ax  =  0, 


X 


y 

c  a  b  x+y^z  =  2[a  +  h^c). 

—   +   —   —    —    =  Aca. 


z  X  tj 

•0     ,     y           -  %->n     ^           y          ^ 

19.    T—-  +  -r—.  =«+^      20.  — --  -4.  ^ ^^0, 

0  +  c         c  —a  o-\-c         c  — a         a—  b       * 

y           ^__  _  /  ^           y           ^ 

c-\-a         a — b               '  h  —  c         c  —  a         a'^b~   * 


.     Z                     X  X                   If                   ^ 

21.    ^  +   -J^  +  -^    =    1,  22,    ^    .   „. 

a           a  —  1  a  — 'A                                 x+y 

T  +    6-1  ■*■  &-2     -    ■"'                    i/+s     -   *» 

c  ^    c-1  ^  c-2     ~    ^'                   z+x     ~   "" 

23.    ±  +   f  +  i--+^  +  A^ 

rt            0  c  oca 

a;           ,y  2  1  1  1 


04    ^       •'/       ^ 

""   '      rt             b             c 

u 

d' 

25.  ax  =  bt/  =  cz:r=ifi 

mx  +  ny-]-pz-\-qu  =  r 

7/3  —  23  _^„..jj. 

26.    ?/+z  =  att, 

r 

27.  x+!/  =  m, 

x  +  z  =4)U, 

y-^z  =  n. 

x+y  =  cn, 

z+^i  =  a, 

1-x          a 
1  —  u    ~     b 

u  —  x=:b. 

•201 


SIMULTAXEOUS    EQUATIONS.  • 


28.    Ux+9i/-\-z-u  =  a,  21J.  x  +  ay  +  a^z  +  a^u-^a*  =  0, 

Uy  +  Vz+u-x  =  h,  x-{-bi/-i-b"z-^L^u+ij'^  =  0, 

11z+9u+x-y  =  c,  a;+«/  +  c22+6-3w  + 6-4  =  0, 

nu-{-9x+y-z^d.  x+dy  +  d-zi-d^H  +  u^  =  Q. 

30.        .x  +  y  =  a,  31.          x  +  Iy  =  a, 

v+z~b,  i^^niz  =  b, 

ii-\-i-  =  d,  u^pi:  =  ,i^ 

v-\-x  =  e.  v+qx  =  e. 

82.,        xf//+3  =  a,  33."        x- !i-\-z  =  a, 

y  +  Z  +  H=:b,  y^z-\-H=b, 

u  +  V'\-x  =  d,  u—v-{-x=d, 

v-\-x-\-y  =  e.  v—x  +  y  =  e. 

34.         x+y+z~u  =  a,  35.     ,    x+y+z-u  —  v  =  a, 

y+z  +  u-r  =  b,  'f^'  y^z  +  u-v-x  =  b, 

z  +  u  +  v  —  x  =  c,  z-\-u  +  v-x-y  =  e, 

u  +  v+x  —  y  =  d,  u  +  i-  +  x-y~  z  =  d 

v+x  +  y-z  =  e.  v  +  xj-y-z-u  =  e. 

86.    2x-y-z+2io     v  =  3a,  37.   v~  2:c  +  SH-2y  +  z  =  a, 

2y-z-  H  +  2v -x=  ob,  x—-2y^Sv -'■22  +  11=  I,, 

2z ~  u  —  c -\-2x- y  =  3c,  y-2z  +  3x-2u+v  =  c, 

2u-v-  x  +  2y  —  z  =  3d.  z-  2u  +  oy  —  2v+x  =  d. 

2v-x-y+2z  -  u  «=  3e.  u—2c+'dz -  2x+y  =  e. 

Exercise   Ixi. 

Eesolve  the  following  systems  of  equations  : 

J     l+x  +  x^ 

"  i+u  +  y' 

1  +  y+x^    _  x-+x-{-\          ,„,.c-l 


i+*-+^^         '  y'-^y+^  \y-l 

(\+x)a+y)    _   l±a 
(l-x)(l-y)   "    l-a         4. 


x+1 

ix-l\ 

i/+l  - 

X-+X+1 

i/'-i-y+i 

:*       b2 

x  +  y 
1+xy    ~ 

■SIMULTANEOUS    EQUATIONS.  205 

(l+a;)(l-//)    _    1+6  x-y     _    b^  -  g- 

a;+?/  a  ,,      x-\- y  2a 

1+a;//  ~    6+6-'  1—^   ~"  l  —  a^ 

x—y  b  —  c  -^  ^—y  25 

1— icy  ~       a  l  +  a;y    ~  1  -  6' 


1  — a;//    ~  a^  — a^'  ^  +  y  l  +  -<^//  "'- 

a;  -  ?/    _       2.''/3  1  -  a;//  x  -  //  2/^ 

l+'V/       ~  62_^3"  ;j;_^        "f"      1_     ^y      ^         ,t 

9.    :yU+-^')  _  10     Z/  +  z  =  2rta;;v2, 

a;(l  +  ?/2j  ~      »  a;+z  =  2ia;?/^, 

i/(l-a;3)  x  +  y  =  2cxyz. 
=    0, 


11 


41 -r') 

?/+5  — .1-         z-{-x-y         x+y-z     12.   ax-hy  =  cz, 


a         ~         b         ~         c       '  I  1  i 

=  —  4-  —    4-   . 

xyz  =  m^.  '  X      '     y  z  ' 

13.    y-  +z'^  -x{y  +  z]  =  a, 

x^+z^-y(x  +  z)^b, 
x^-ty^-z{x-\-y)  =  c. 

14  2ax  =  {b-\-G  —  u)(y-^z), 
^hy  =  {c  +  a  —  b){x-\-z), 
{x+y-\-zy  +x^+y3-^z^  =  i{a2  +b^  +c^). 

x^  +xy-\-y^     x^  -\- 11^     xii 


f 
15. 

x—1         a  —  1 
1/-1    -    6-1' 

a;3-l         a3_  1 

j,3_i    -    53_i- 

17. 

x^  \-x^y^+y^  =  a, 

x2+xy+y^'=b. 

19. 

xy    +    ^     -   «(a;2  +  j/2) 

16. 


x'^  —  xy-\-y'^  a 


IQ 


'-•    c^  +  fr    = 


3    _ 


a 


"> 


a;^y  -  r//2  = 


x  —  y 

'    cl 


20.    x^=a{x'^-iry^)-b.nj. 


206  SIMULTANEOUS    EQUATIONS. 

21.  4.c[x-^  +  l)  =  {a  +  b){x-ii)2, 
4.c{y^-l)^{a-b){x~y)\ 

22.  x^-ti^    =    -^  {x'^xy  +  y^'){x-^y), 

23-    r^   =   a,  24.    ~^^     =   a, 

ll-ry  xy  ' 

ic+2/2    -    ^-  xy~    =   *• 

25.    %  +  2)  =  a.,  26.    (x-+7/)(a;+0)  =  a, 

«(a;+ //)  =  (%  (z^x){z^y)=c. 

"in.    x{x+y+z)  =  a-yz,  28.    a;^  -  (3/- «)2  ^«^ 

^  V{x+y-rz)  =  b-,-:x,  y2_^z-x)-=b, 

z{x-}-y+z)  =  6-  -  x^/.  z'^~{x-  y)  2  =  c. 

29.    a;2+^2^,,2  1            1           2a 

30.    ^   +  —    =    — . 
X-  y  22 

a-  +  ?/==/^z,  1  1  26 


^3         0,2 


-» 


ai^*         ?/^         z3 
x-y  =  cz.  l^  1^1 

X  y    ~    c' 

32.    xy    =   ^--i, 

x+y  =  I'z,  .  (a; -  y)(3 + 1 )  -  2a,. 

x-y  =  cz,  {x^-y-'jiz+iy-i^iht. 


EXAMINATION    PAPERS.  207 


CHAPTEE  VII. 


EXAITINATTON    PaPERS  :       EdUC\TION     DEPARTMENT    AND     UxiVSRSITT 

OF  Toronto. 


L 

1.  State  the  rules  for  the  addition  and  subtraction  of  Algebraic 
riiiautities.     Express  in  the  simplest  form. 

(b+c  —  a)x+  {c-\-a.  —  b)y-{-{a-\-b  -  c)z 
{c+a-h)x  +  {a  +  b-c)y-\-{b+c-a)z 
(a-}-b  —  c)z-{-{b +c  —  a)y  +  {c+a  —  b)z 

2.  State  and  prove  the  Index  Laws.     Assuming  these  to  be 
general,  interpret  a;"™. 

Find  the  products  in  the  following  cases : 

(1)  (x^  +  6x^y  +  12xy^  +8y^){x=  -6x'^y+12xy^  - 8yS). 

(2)  {a  +  b-\-c){b-\-c-a){c+a-b)(a  +  b-c). 

3.  Prove  the  rule  of  signs  in  Division. 
Divide  :    [Apply  Horner's  Method  to  (1)] 

(1)  a;6_22x4+60a;3-55a;3  +  12a;+4  by  x^-^Gx+l. 

(2)  a;4+9-l-81a;-4  by  a;2-3-f9a;-2.  (3)     a;"' -  1  by  a;*  - 1, 

4.  Find  the  square  roots  of 

4  ] 

(1)    4a;4™-— a;^"*    +  —  x""" 

•       o  J 

b^  c^  a^  c  b  a 

5.  Distinguish  between  an  algebraic  equation  and  an  identity. 
Solve 

(1)     1^/(1 -2.1)4-1^(1  + 2a;)  =  3. 


808 


EXAMINATION    PAPERS. 


(2)     '-^2    _^    .r^    ^    ^.+, 


x+1     '    x-2   ~       x-3 

6.  A  person  bought  a  certain  number  of  oxen  for  $320.  If  he 
had  been  able  to  i^urchase  four  more  for  the  same  sum,  each 
would  have  cost  him  $4  less.  Find  the  number  |of  oxen.  Ex- 
plain the  negative  result. 

7.  (1)  If  ^   =   ^  shew  that  «_!±M±!^'    _   ^i^J^ 

b  d  c2  +  2c(/+3d2    -    ^(c-3f/)* 

(2)    Find  the  value  of  a;6  -200a;^-fl98a;4+200a;3  -197;r2 
-397a;  when  x=199. 

S.  Three  towns,  A,  B,  C,  are  at  the  angles  of  a  triangle.  T'rom 
A  to  C,  tlirough  B,  the  distance  is  82  miles  ;  from  B  to  A,  through 
<;  is  97  miles  ;  and  from  C  to  B,  through  A,  is  89  miles.  Find 
the  direct  distances  through  the  towns. 


11. 

1 .  Prove  x^  -=-  a;"  =  x"'~". 

Simplify  {a+h  +  c)^  -S(a-{-h-\-c)'^c  +  ^a-\-h-]-c)c^  _fS. 

2.  Prove  the  rule  for  finding  the  L.  C.  M.  of  two  quantities. 
F«a  theL.  C.  M.  of 

a^-\-h^  +  c^-dabc,  and  (a  +  b)^ +2{a  +  b)c+c^. 

^  a  c  ac 

8.  Prove  -^    x   —r  =  —;• 
b  d  bd 

Pimphly    [^^^  +  -^— ^,)  .  (^-^_^-^-,    _    ^-^^. 

4.  Eoduce  to  their  lowest  terms  — :r„7-; — ;;; — ir,  and 

(I-  '-{-a.    —  2 

a(a^.2h)-^b(b  +  2c)  +  c(c  +  2a) 
a2_^2_(.2_26^ 

5.  (1.)  li  a^-pa^+ga- r  =  0,  then  x^-pxl+qx~r  is  exactly 
divisible  by  a;  — o. 

(2.)  Prove  that  {n  +  b  +  c){bc-{-ca  +  ab)-{b+c){c+a){a^b)is 
divisible  by  abc.     Is  there  any  other  divisor  ? 


6.  Ux  = 


EXAMINATION    PAPERS.  209 

la  +  6\  2^  ,  a  -  -b-  /« +  b\  "+" 


7.  Solve  the  equations — 

^  ^^  l-"2x  ~   7^^2i   ~  -^  ~  7^1fe+lx--^* 

(3  )  •^•+^         ^+1    _    4x-l-9         12a; +17 
a;  +  4    ~   x+2    ~    2uH^7   ~      Gx'-PTu' 

8.  A  pei'sou  going  at  tlie  rate  of  ^j  miles  an  hour,  and  desiring 
fco  reach  home  by  a  certain  time,  finds,  when  he  has  still  r  miles 
to  go,  that,  if  he  were  continuing  to  travel  at  the  same  rate,  he 
would  bo  g  Inufs  too  late.  How  much  must  he  increase  his 
speed  to  reach  home  in  time  ? 

9.  Of  the  three  digits  comprising  a  number,  the  second  is 
double  of  the  third  ;  the  sum  of  the  first  and  third  is  9,  and  the 
sum  of  the  three  digits  is  17.     Find  the  number, 

10.  A  owes  B  $a  due  in  months  hence,  and  also  .$6  due  n 
mouths  hence.  Fiud  the  equation  which  determines  the  time  at 
which  both  sums  could  be  paid  at  once,  reckoning  interest  at  5 
per  cent,  per  runum. 


III. 

1.  If  3;=  10,  ?/=lL  2=12,  find  the  value  of 

\  x'^  —  (i/-[-z)-      X   — , —  ;  and  subtract 

(  V^   '     '     j         x+)j  +  z' 

(ij  —  2)a2  +  (z  -  x)ab  +  [x  —  j'/)b'^   from 

{u-x)u^ —  {y—z)ah~{z  —  x)b^. 

2.  Divide  a  +  (a  +  6).f  +  (a  +  i  +  c)a;2  ^^a+b+c)x^  +  ib^c)z* 
+  rx^by  l+x-{-.i^ -\-x^  ;  and  find  the  square  root  of 

9  -  2-l.iH-58.c2  -  llC./;3  +  120^1  -  UOr^  +  lOf 'a;«. 

c.    r.  ^         ■-,     •^•c+5         a;  +  5  2x-\-') 

3.  Solve    (1; ^T    +    ^7~(    = .>    -    TT" 


eio 


EXAMINATION    PAPERS. 


i.  A  boy  bought  a  number  of  oranges  at  the  rate  of  45  cents  a 
dozen  ;  if  he  had  received  20  oranges  more  for  the  same  money 
the  whole  would  have  cost  him  only  40  cents  a  dozen.  How 
many  did  he  buy  ? 

6.  A  farmer  took  to  market  two  loads  of  wheat,  amounting  to- 
gether to  75  bushels  ;  he  sold  them  at  difierent  prices  per  bushel, 
but  received  on  the  whole  the  same  amount  for  each  load  ;  had  he 
Bold  the  whole  quantity  at  the  lower  price  he  would  Lave  received 
$78.75  ;  bnt  had  he  sold  it  at  the  higher  price  he  would  have  re- 
ceived $90.     Find  the  number  of  bushels  in  each  load. 

6.  Show  how  to  find  the  square  root  of  a  -h  y^b. 
Find  the  square  root  of  1  +  |/(1  —  w^) 

^^+^         4x— 1         7a; -1-1 

7.  bolve   ^ — "^  +  X   = ^  ;  and  find  the  value  of  i 

^x  —  /  X  —  S  X  —  o 

when  ax^—^Qx  +  Sl  =0,  has  equal  roots. 

a+b    _    \/{ac)+  \/{bd) 
*^a*a_i    -    -/(rtc)-  V{bd)' 

9.  Show  that  a3(6  —  c)+i3(c  —  «)+c3(a  —  i)  is  exactly  divisible 
by  a-{-b  +  c  ;  and  resolve  the  expression  into  its  factors. 


IV. 

1.  Multiply  a^-f  5=- c2  +  2a6  by  a^ -b'^ +c^ +  2ac,  and  divide 
the  product  by  a^  —  6^  -  c-  -f  26c. 

2.  Simplify 

18a362      _      ^3ab{x-y)  li(c-d)  S(c^ -d^)\ 


■u 


]  ~1(^4     "     \  2T^3~   -^    a{x^  -  j/^^l   r 


EXAMINATION    I'ATl^RS.  211 

3.  Find  theL.C.M.  of  4x•--9//^  4x2-10x'y  +  G»/e.  and  6.r'- 
13a;?/+6?/3,  and  the  G.C.M.  of  l+x^+x{-x^  and  2x  +  2x'  + 

Sx'^  +  Sx^ 

4.  Obtain  the  square  root  of  i  — fj/A^,  and  find  the  value  of  c 
when  4:X^  —  12x^u+cx^l/^  —  12x7/^+4^*  is  a  perfect  square. 

5.  Distinguish  between  an  equation  and  an  identity.  Give  an 
example  of  each.  What  value  of  m  makes  (a;— 3)2  —{^x  —  V){x—'6) 
=  m  an  identity  ?     Can  any  value  of  m  make  it  an  equation  ? 

6.  Eeduce  to  its  simplest  form 

l/(2  +  :e)-T/(l+x)     ^     l+y^ll -1 -f- (l+a-)\ 
1/(1+0,-) -v'x        "^    l  +  i/'{l  +  l-(l+x-j[ 

7.  Solve  the  equations 

(2)    73^-5rr=:.;-5^)(.T  +  3^), 

2  '         S        _     7 

X— 5p         x  +  '^!/    ~    33 

8.  A  person  performed  a  journey  of  22. V  miles,  partly  by  car- 
riage, at  10  miles  an  hour,  and  partly  by  train,  at  36  miles  an 
hour,  and  the  remainder  by  walking,  at  4  miles  an  hour.  He 
did  the  whole  in  1  hour  50  minutes.  Had  he  walked  the  first 
portion,  and  performed  the  last  by  carriage,  it  would  have  iaken 
him  2  hours  30^  minutes.  Find  the  respective  distances  by  car- 
riage, train  and  walking. 


U^^,       ^^^v 

° 

9.  Solvo 

• 

a:  +  3 

x+1 

4a--f9 

12a;+17 

x+4 

ic  +  2    ' 

"    2x-+7 

6x+16 

10.  What  value  of  y  will  make  2x*-^Sxy +Qy^  eracily  divisible 
by  a:- 3? 

If  a  and  ?^  are  lae  roots  of  the  equation  x^-i-x  +  l  =  0,  show 
that  fl3_?,3^0. 


212  BXAMINATION    PAPERS. 

V. 

1.  Multiply 

Ax^'-^.x+\\  by  2ar  +  |. 
Prove  that 

{h''  —  '>j)^  —  (x  —  ^y)^  is  exactly  divisible  by  x+y. 

2;  Express  in  vrords  the  meaning  of  the  formula 

{x  +  a){z  +  b)=x"  +  {a-\-h)xi  ab. 

Retaining  the  order  of  the  terms,  how  will  the  right-hand 
member  of  this  expression  be  affected  by  changing,  in  the  left- 
hmd  member  (1)  the  sign  of  b  only,  (2)  the  sign  of  a  only,  (3) 
the  signs  of  both  a  and  b  ? 

C.   S:m\My  {a  +  by-\-{a-h)^-2{a^-b^)»  ;  and   show  that 
{a+b  +  c){b  +  c -  a]{a-\-c  -  b){a-{-b  - c)  =  iaH^ 
when  a^+b^  =  c». 

a  c  (id 

4.  Prove  that  -, — =---■   =    -r— 
u  a  he 


Simplify 


a^-\-h^  \   I    ah^     \  ^      4«(a+6) 


2a6      ^     I   \a^  +  b^j    '    a^-ab+b» 

5.  I  went  fi-om  Toronto  to  Niagara,  35  miles,  in  the  steamer 
"  City  of  Toronto  "  and  returned  in  the  "  Eothsay,"  making  the 
r:>und  trip  in  5  hours  and  15  minutes;  on  another  occasion  I 
went  in  the  "  Eothsay  "  (whose  speed  on  this  occasion  was  1  mile 
an  hour  less  than  usual),  from  Toronto  to  Lewiston,  42  miles,  and 
returned  in  the  '•  City  of  Toronto,"  making  the  round  trip  in  G 
hours  and  30  minutes  ;  find  the  usual  rates  per  hour  which  these 
steamers  make. 

6.  Solve 

^  '     X  y  a  X  y  a 

(2)    a;3  +  5a;  =  5^/(^2+5x+28)-4. 

7.  Find  three  consecutive  numbers  whoso  product  is  48  times 
the  middle  number. 


EXAMINATION    PAPERS. 


£18 


8.  If  m  and  n  are  the  roots  of  ax^  +bx  +  c  =  Q,  then 

ax^ -}-hx  +  c  =  a{x  —  vi){x -n). 

Show  that  if  ax^  +  bx-i-c  =  0  has  ecjual  roots,  one  of  ihem  is 
given  by  the  equation 

{2a^-2ah)x^ab-b"=0. 

m  n        ,  x"^         y^ 

9.  If  —   =   — and -r-  +  -r-^  =1,  prove  mat 

??)3  7;--  III- -\- 71^ 


VI. 

1.  SimpHfy 

2.  Divide  a^—h^  —  c^—Sahc  by  a  —  b  —  c,  and  show,  without 
expansion,  that 

(1  +^+a;2)3  _  (1  .-x-{-x^)^-Gx(x^  +a;2  +  1)  -8a;3  =0, 

3.  Eesolve  into  factors  x'^—^x-y^-i-i/^,  and 

7^2  -  G;/3  -  x;/  +  19x  +  33//  -  36  ;  and  prove  that 
b^{c-\-a)+c^{a-\-!j)  —  a^{b-\-c)+abc  is  exactly  divisible  by 
fc  +  c  — a. 

4.  Apply  Horner's  method  of  division  to  find  the  value  of 
5a:»+497a;*  +  200a;3  +  19Ga;^ -218a; -2000  when  a:=-99,  and 
the  va  ae  oi  Gx^  ^Sx"^ -llx^ -Qx^+lOx-2  when  2a;- =  -3^;  +  !. 

6.  Find  what 


^\  -r-c)-]- — 1^ -J  becomes  when  x  = -. 

V{a+x)-  V{a-x)  1+6* 

6.  If  a  and  b  be  any  positive  numbers,  prove  that 
la  a  b 


214  EXAMTNATION    PAPERS. 

7.  Solve  the  equations — 

(])    ,.*  +  7/-  =  5, 


.1 


«        +2/ 


K' 


(2)  a;+27/+3z=i4, 
2a;  +  3?/  +  2  =  ll. 
3a;+?/+2z  =  ll. 

(3)  (x+l)(x  +  3)(.T+4)(«+6)=.iG 

8,  There  are  three  consecutive  numbers  such  that  the  sum  of 
their  cubes  is  equal  to  16|-  times  the  product  of  the  two  higher 
numbers  :    find  the  numbers, 

9,  (1)  Form  an  equation  three  of  whose  roots  are  0,  \/{  —  2), 

and  1  —  ^/2. 

(2)  If  one  of  the  roots  of  the  equation  a;-4-i''C-f-g  =  0,  is  a 
mean  proportional  between  p  and  q,  prove  that 
p^=q{l+py. 

10,  Two  trains  start  at  the  same  instant,  the  one  from  Dto  A, 
the  other  from  Aio  B;  they  meet  in  1^  hours  ;  and  the  train  for 
A  reaches  its  destination  52^-  minutes  before  the  other  ti-ain 
reaches  B  :  compare  the  rates  of  the  trains. 


VJI, 


1.  Give  some  application  of  tlie  '•  rule  of  signs  "  in  Algebraic 
alultiplication  and  Division. 

2.  Find  the  numerical  value  of  the  quantity 

hc(c  —  a){a  —  h)  —  cii[((,  —  b)(b  —  c)  +  ab{b  -  c)(c  —  a), 
when  a  a  10,  b  =  -01,  c  =  0;  and  prove  that  if 

X  =  ,  then  will  (a-+o) . — ; 

a+b  a+b—c+x 


EXAMINATION    PAPERS.  215 

8,  Investigate  a  method  of  finding  by  inspection  the  remainder 
after  dividing  any  rational  and  integral  function  of  x  by  x+a. 

Show  that  the  quantity 

a^b^-ab^x-{a^+2b^)x^+ax^-{-2x'^ 
is  divisible  by  each  of  the  quantities  x-\-a,  x-\-b,  a—2x,  b-x. 

4.  Investigate  the  rule  for  finding  the  H.C.F.  ef  two  algebraio 
quantities,  showing  under  what  limitations  factors  may  be  intro- 
duced or  suppressed  at  any  step. 

Find  the  H.C.F.  of 

(1)  6.c4-7x-3_l3«3  +  I9a;-G  anda;3  +  2x3-l.       ' 

(2)  {x-\-y){ax^-bir)-xy{a-b){x-\-y),  and 
{x-y)[ax^-by^-)+x>j{a-b){x-y). 

6.  Prove,  by  general  reasoning,  that  the  value  of  a  fraction  is 

not  altered  by  multiplying  or  dividing  both  the  numerator  and 
denominator  by  tlie  same  quantity. 

13  7  X     \ 


Simplify       (1) 


12(2a;-3)  12(2.f+3)         4:X^-\-% 


^^^     \{x-\-a){x~b)   +   {x-a){x-^b))      ' 


1  1 

+ 


Xx+a){x  +  b)         {x  —  a){x  —  h) 

6.  Solve,  with  respect  to  x,  the  equations 

a;-18         2a;-24  lla;-34    _     1_ 

->    '^T'  ■'"   ^Tl^^   "^         22         "    44" 

5.^2 +x- 3         7a;2-3a;-9    _  a;-3   

^^1         5a;_4        ~        7.C-10        ~    35ar3-78a:+40' 

(3)     X-  =  ax-^by,   and  y^  =  bx-\-  ay. 


VIII. 
1.  Definfi  the  terms  "power,"   "root,'*  "  index,"  and  "coeffi- 
cient ;  explain  also  the  reasoning  by  Avhich  it  is  shown  that 

<t  ~  (6  —  c)  =  a  —  6  +  c. 


219  EXAMINATION    PAPERS. 

2.  Multiply  (a;3+x?/ -1-2/2)2  ^y  (x—y)'*. 

■    Find  the  values  of  a  and  b  which  will  make 

x^+ax-\-b  divisible  'byx-\-p,  and  also  by  xi-q. 

a.  Divide  x^ +7/^+2x^,0^  by  (.c-(-//)^  and 

4.  Investigate  a  rule  for  the  extraction  of  the  square  root  of  any 
algebraic  quautit}',  and  deduce  the  rule  for  the  extraction  of  the 
square  root  of  a  number. 

If  to  any  square  number  be  added  the  square  of  half  the  num- 
ber immediately  preceding  it,  the  sum  will  be  a  eomplete  square : 
viz.,  the  square  of  half  the  number  immediately  following  it. 

6.  Find  the  square  root  of 

(1)  a2a;6  +  2a6x4  +  (62-f2a,)„2-j-c3a;--+2/jc. 

(2)  ix^ - ix'  -h ix^  +  lx^ - lx'^  +  ^\J. 

6.  If  x^+ax-\-b  and  x^+a'x-b  have  a  common  measure,  it 

will  be  a;+  — k — ,  and  the  condition  that  they  may  have  a  com- 

mon  measure  is  Ab  —  a^—a'^. 

Find  the  H.  C.  F.  of  x^ +p^ x^ -\-p*  and  x"^ +2px^-\-p^x^ -p^. 

Find  the  L.  C.  M.  of  2^{x^  +  x-20),  d^{x^ -x-BO),  and 
mx^-lOx  +  2,4). 

7.  Find  values  of  a  and  b  which  will  render  the  fraction 

3x^-{'ka  +  b)x-\-a-{-2b^ 
5x-  -{Sa-fb)x-a  +  4:h3 

the  same,  for  all  values  of  x. 


8.  Solve  the  equation  12  -  i/{x+])[x+(})  -  i/{x-'i)[x-{-5}  =  0. 
and  account  for  the  circumstance,  that  the  values  of  a;,  determined 
from  it,  apparently  do  not  satisfy  the  equation, 


BXAMINATION    PAPHRS.  217 

IX. 


1.  Prove  thata(2?i-}-l)(a2+7i-M  +  l)-n(2</-f  l)(7i2+«-a-i-l; 
=  (a  —  n)^. 

2.  If  a,  b,  and  s  are  positive  quantities,  and  if  a>fe  and  c>a  — 6< 

prove  that 

e  —  {a  —  b)=c — a  +  b. 

Assuming  this  equation  to  hold  good  when  a,  b  and  c  are  unre- 
stricted,  prove  that  the  expression-( -a),  occurring  in  an  algeb- 
raic operation,  is  equivalent  to  +a. 

3.  li  x^-\-ax^+h  and  x^+px+q  have  a  common  measure  of 
the  form  of  x^  +  ni,c-\-n,  then  a^bq-{b  —  q)^ 

4.  Find  the  H.  C.  F.  of 

a--b--abxy  +  abx~^y~'^,  and  a-x^ -b^y-^ +a'bx^y-b^xij''i', 

5.  A  and  J5  are  two  numbers,  each  of  two  digits.  The  left- 
hand  digit  of  A  exceeds  that  of  B  hy  x;  the  excess  of  A  above  B 
is  y ;  but  the  sum  of  the  digits  of  B  exceeds  the  sum  of  the  digits 
of  Ahy  z.  Prove  that  y-\-z  =  2x;  and  give  an  example  of  two 
such  numbers  as  A  and  B. 

a  b  c 

6.  If  -r-   =    —    =    -r,  prove  that  each  of  these  ratios 

^a  a+b-^-c 

=     „  r-,  and  also  =    ,— — :-,♦ 

7.  Solve  the  equations 
x  +  a         x  —  a         b->rx         h  —  x 
x  —  a         x-\-a   ~    h  —  x         b+x 

(2)    a{x'-+y^)-h{x^-y^)  =  ^a 

(fl3-63)(a;3-7/3)  =4rti. 

8-  A  farmer  buys  a  sheep  for  $P  and  sells  h  of  them  at  a  gain 
of  5  per  cent.  ;  at  what  price  ought  he  to  sell  the  remainder  to 
gain  10  per  cent,  on  the  whole  ? 

9.  The  sum  of  three  numbers  is  70  ;  and  if  the  second  is  divided 
by  the  first,  the  quotient  is  2,  and  the  remainder  1  ;  but  if  the 
third  is  divided  by  the  second,  the  quotient  is  3,  and  the  remain- 
der is  3 ;  whg^t  are  tlie  numbers. 


(1) 


218  EXAMINATION    PAPEB3. 

X. 

1.  Divide  ax^  +  1cxijz-\-hij^  +ax- {y-\-z)  +  hij- {z-\-x)  +  2cxy{x  +  y) 
by  x+y-\-z. 

2.  Prove  that  if  x*^-{-p.v--^qx+a^  be  divisible  by  x^ —1,  it 
is  also  divisible  by  x^  —a^. 

3.  Explain  the  reason  for  introducing  or  suppressing  factors  in 
the  process  of  finding  the  H.C.F.  of  two  algebraical  quantities. 

Why  is  the  name  "  Greatest  Common  Measure  "  objectionable  ? 
Find  the  H.C.F.  oi  x^-x^-x^ -x-2  and  Qx^ -lx^+dx-2. 

4.  A  traveller  leaves  A  for  B  at  the  same  time  that  another 
leaves  B  for  A ;  the  former  walks  at  the  rate  of  3  miles  an  hour 
till  he  has  performed  half  the  distance  ;  he  then  rests  for  an  hour ; 
after  which  he  resumes  his  journey,  walking  now  at  the  rate  of  4 
miles  an  hour  ;  the  second  traveller  goes  at  the  rate  of  4  miles  an 
iiour  till  he  has  got  over  one-third  of  the  distance  between  B  and 
A. ;  he  then  rests  for  40  minutes ;  after  which  he  resumes  his 
]aurney,  walking  now  at  the  rate  of  3  miles  an  hour.  The  tra- 
vellers reach  A  and  B  respectively  at  the  same  time.  Find  the 
distance  between  A  and  B. 

6.  Show  by  examining  the  square  of  a-\-b  how  the  square  root 
of  an  algebraical  quantity  may  be  found. 

Find  the  square  roots  of 

(1)    25a;4_30rtx3-f49a3a;2-24a3x+16rt4,  and 


(2)    ^+Kl    _    (^  +  JL)^2  +  i. 
2/2         a:-  \y  xj  2 


6.  Show  that  a  =  \/aJ^,  when  m  and  n  are  integers,  and  m  is 
divisible  by.7i;  and  state  the  principle  on  which  you  would  main- 
tain the  truth  of  the  equation  for  all  values  of  m  and  n.  « 

7.  Solve  the  equations 

^  ^        5a: -4        ~    '    7a;-- 10 

(2)     (y.c- 1)2  -f  (-U-  -  2)-  =  (ox  -  3)3, 


EXAMINATION    PAPERS. 


219 


B.  Two  regular  polygons  are  so  related  that  the  number  of 
their  sides  is  as  2  to  3,  and  the  magnitude  of  their  angles  as  3  to 
4  ;  find  the  figures. 


XI. 

1.  State  in  words  the  several  operations  to  be  performed  in 
order  to  obtain  the  result  expressed  by  the  following  algebraical 
expression : 


V 


'ma^  +nb^ 


Also  find  its  value  when  a  =  b  =  4:. 

2.  Two  men,  A  and  B,  dig  a  trench  in  3f-  days.  If  A  were  to 
do  more  work  by  one-third  than  he  does,  and  B  more  work  by 
one-half  than  he  does,  they  would  dig  the  trench  in  2|f  days.  In 
what  time  would  each  dig  it  alone,  at  his  present  rate  of  work  ? 

3.  Perform  the  multiplications  in 
(1) 

/  2a;^-f  3/  j  /  2x^-2/  I  (  4x^-\.  6xV+9z/'  )  (  ^^^  "  ^^V  +  %*  1 

(2)     ax^+\xy^pj^^){ix^-lxi/+iy^). 

4.  Divide 

(1)  a;4-f9+81a;-*  by  x^-3  +  9x-^. 

(2)  x^-{a-{-h+p)x^-{-(^ap-\-bp  —  c+q)x^-{aq-{-bq-C]>)x  —  qc  hj 
x^  —px+q. 

5.  Show  that  x~"'+^-x'^''~^  is  always  divisible  by  x±l,  m  and  w 
being  any  positive  integers. 

6.  Define  a  fraction  ;  and  from  yonr  definition  prove  a  rule  for 
adding  together  two  fractions  with  different  denominators. 

Add  together  the  fractions, 

a^ —be  b^  —  ca  c^—ab 


(a  +  b){a+c)     (b  +  c){b+a)     {c.  +  a){c  +  b) 


220  EXAMINATION    PAPERS. 

7.  Solve  the  following  equations  : 
,^.    3;2^2x+2         x^  +  8x-{-20         x^+4tx  +  6         x^  +  Gx+12 


x+1        ^         x+i  a;+2         ^         a;-f3 

X  100     ,  ^        o.    ?/ 


(2)    (x^+y^)-^   =    ^^,    (X3--/2)   ^  21 


XII. 

1.  When  OT  and  n  are  whole  numbers,  and  m  greater  than  n, 

a™  1 

show   that  —  -  a'"-"  and  that  -^  is  correctly  symbolized  by  a~" . 

2.  Multiply  (a '-6)(a+^)(«-+^^)(«''+^'')  •  •  •  to  (n  + 1 )  factors. 

3.  Divide  1  —  x  by  1  — 2a;,  to  5  terms,  and  write  down  the 
<'7-+l)th  term,  and  the  remainder  after  (r+l)  terms. 

4.  If  the  number  three  be  divided  into  any  two  parts,  show 
that  the  difference  of  the  squares  is  three  times  the  difference  of 
the  numbers. 

5.  Find  the  L.  C.  M.  of  l-8x+llx^+'^x^-24^xA,  and 

^  1 -2a;- 18;<;2 +38.1-3 -24x4. 

6.  What  relation  must  there  be  between  the  coefficients  m,  n, 
p  and  q,  in  order  that 

(x3  4-wx+7i)2  -f  pa;2  +  gx 
may  be  an  exact  square  for  all  values  of  a;  ? 

7.  Solve  the  following  equations : 

(1)    (T+^ys  +  (l-x)2    "   "• 
ox  —  h^  ^/{ax)  —  b 

(2)  v{ax]-\-h  =      ^r~  -'• 

C3^    — "-    =  1,  — ^-   =  2,  and      —   =   3. 

8.  Given  x4-?/+2  =  «a;  =  %,  find  (x+?/+z) -z. 

9.  Find  a  number  expressed  in  the  decimal  notation  by  two 
dio-its,  whose  sum  is  10  ;  and  such,  that  if  1  be  taken  form  its 
double,  the  remainder  will  be  expressed  by  the  same  digits  in  a 
reversed  order. 


EXAMINATION  PAPERS.  221 

XIII. 

1.  Find  the  value,  when  a  =  2  J,  b-.3^,  c-  4+  of 

2.  Show  that  the  vahie  of  the  expression,  in  the  preceding 
question,  is  not  altered  by  changing  a  into  a-^x,  h  into  b-\-x,  and 
c  inte  c-\-x. 

3.  Multiply  (1 -fa  ia;)(l  4- rt  2^)  (1+^3^)  •••  (l+«„a;)  to  3  terms. 

4.  A  speculator  borrows  a  sum  of  money  at  the  yearly  interest 
of  7  per  cent. ;  part  of  the  amount  he  Invests  at  8^  per  cent.,  and 
the  remainder  at  9 ;  and,  at  the  end  of  the  year,  he  finds  that  he 
has  made  a  profit  of  $75  ;  but,  had  the  former  part  been  invested 
at  9  per  cent.,  and  the  latter  at  Qh,  his  profit  at  the  end  of  the 
year  would  have  been  only  $65.     Find  the  whole  sum  borrowed. 

5.  Given  ax+hy  =  c,  a'x+h'y  =  c',  determine  the  value  of 
nix-\-ny,  and  find  the  conditions  under  which  the  value  becomes 
indeterminate. 

(I  I  da  0„     1 

6.  If-!-  =  ^= =   -^^, 


then  will  a, +^2 +^3+  .     .     .   +  a„-=-^ 
7.  Eliminate  x  and  y  from  the  equations 

S  ,  -7.  5 

X    -^r  y     =    a 

«   =  x-k-^x^'y 


a\-a^a^ 


«!  —  aj 


1     a 
5    ■^ 


P   =   ?/+3xSA 


8.  \iax- ■\-'bx-^-c  =  ^ -A   d  .<i»^  +  6ia;-fc,  =0,  then  will 

(«5j  —a^b)(bci  —b^c)  =  (aCj  —  a^c)^ . 

9.  Find  that  number  of  two  figures  to  which  if  the  number 
formed  by  changing  the  places  of  the  digits  be  added,  the  sum  is 
121 ;  and  if  the  same  two  numbers  bo  subtracted,  the  remainder 
is  9. 


222  EXAMINATION    PAPERS. 

XIV. 

1.  SimpKfy 

a(i  +  c)3  +  6(r+rt)2  +  c(a+&)2-{(a-?;)(a-c)(64-c)  + 
{b-c){b-a){c+a)  +  {c-a)(c-h){a-j-b)}. 

2.  State  the  law  of  Indices,  and  prove  it  for  positive  integral 
indices  ;  and  assuming  it  to  be  general,  interpret  the  expressions 

z~^,    X  ,  where  m  and  n  are  positive  integers. 

3.  Having  given  the  equations, 

prove  that  a^ [yz  -y'z ')  +  ''^ [z-c- -z'x')+c'^{xy-x 'y ')  =  0. 

4.  A  traveller  P  sets  out  to  walk  from  A  to  B,  proceeding  at 
the  rate  ot  3  miles  an  hour ;  and,  32  minutes  afterwards,  another 
traveller  Q  sets  out  to  walk  from  B  to  A,  proceeding  at  a  uniform 
rate.  They  meet  half  way  betwixt  A  and  B.  P  then  quickens 
his  pace  by  1  raile  an  hour ;  and  Q  slackens  his  1  mile  an  hour. 
Q  reaches  A  at  the  same  time  that  P  reaches  B.  Find,  the  dis- 
tance between  A  and  B. 

6.  How  are  equations  classified  ? 
Solve  the  equations — 

(1)  mnx+a'm.n  =  n^x-\-aiu^. 

(2)  x^-x^+y*-y^  =  84:, 
x-+x^y^-\-y'^=49. 

6.  What  two  numbers  are  those  whose  difference,  sum  and 
product  are  to  each  other  as  the  three  numbers  2,  3,  5  ? 


XV. 

1.  What  is  the  meaning  of  the  symbols  a,  a-,  a^     .     ,     ? 
Show  a  priori  that  a°=l ;  how  do  you  know  that  ab  =  ba  ? 
How  is  it  proved  that  the  multiplication  of  hke  signs  gives  a 
positive,  and  that  of  unlike  signs,  a  negative  result. 


EXAMINATION    PAPERS.  223 

2.  Find  the  value  of 

(^b~c)^+2{c-ay  +  {a-h)^-S{b-c){c-a){a~b) 


when  a  =  l,  b=  —^, 


3.  Simplify  the  following  expression  : 
{ac-b^){ce-d2)-\.{ae-c2){bd-c^)-{ad-bc){be-ca) 

4.  P  and  Q  are  travelling  along  the  same  road  in  the  same 
direction.  At  noon  P,  who  goes  at  the  rate  of  m  miles  an  hour, 
is  at  a  point  A ',  while  Q  who  goes  at  the  rate  of  n  miles  in  the 
hour,  is  at  a  point  B,  two  miles  ha  advance  of  A.  When  are  they 
together  ? 

Has  the  answer  a  meaning,  when  m— n  is  negative  ?  Has  it  a 
meaning  when  jn  =  )i7  If  so,  state  what  interpretation  it  must 
receive  in  these  cases. 

5.  Show  how  to  find  the  Least  Common  Multiply  of  two  or 
more  algebraic  quantities, 

(1)  x2—ax~2a^,  x^+ax^  andax^  —  x^. 

(2)  x^  —x^y—a^x+a^y  and  x^+ax^  —xy^ —ay^. 

In  what  algebraic  operations  is  the  Lowest  Common  Multiple 
of  two  or  more  quantities  required  ? 

6.  State  and  prove  the  principle  upon  which  the  rules  of  Addi- 
tion and  Subtraction  of  fractions  are  founded. 

Simplify  the  following  expressions  : 

(a _!_&  _  c)2  -  ^3  (^b+c  -a)^-d^  (c+a-by  -  d^ 

(1)  (^^6)2_(c+d)2  +  {b  +  c)''-W-fd)2    +    (^+u)-^b+d)-' 
J.2  ^y2  -z^  +  2xy     a^-\-a^h         a{a—b)  2ab 

(2)  ^2_\j2-z-  +  2yz     a^b  -  i>3    ~    {a+h)b    ~    a^-b^' 

7.  liax-by-{-c{x-y)  =  {a-b){a  +  b-c}, 

by  -  cz+a{y-z)  =  {b-  c){b  +  c-u), 
cz--ax+b{z-x)=^{c-a){c+a  —  b) 
then  will  a^{b-c)+b^{c-a)  +  c2{a-h)  =  0. 

8.  P  is  a  number,  of  two  digits,  x  being  the  left  hand  digit,  and 
y  the  right.  By  inverting  the  digits,  the  number  Q  is  obtained. 
Prove  that  11  {x  +  y){P-Q)  =  i)  {x-y)  {P+Q). 


224  EXAJIINATION    PAPERS. 

XVI.    ' 

1.  Show  that 

{(ax  +  by)2  +  (ay-bx)^}{{ax-{-hy)^-'{a!/+bxy}  = 
(a4-64)(:<;4_,^4) ;  and  that 

2{a-b){a-c)+2{b-c){b-a)-i2{c-b)ic-a) 
is  the  sum  of  three  squares.  ' 

2.  It  s  =  a  +  b-{-c-\-&c.  to  n  terms,  then 

s  —  a  s  —  b         s  —  c,j,  " 

+     + +     &C,      r=     n  —   1. 

s  s  s* 

3.  Show  that  a  —  b,  b  —  c,  and  c~a  cannot  be  all  three  positive' 
or  all  three  negative. 

4.  Extract  the  square  root  of 

Ax^  +  dx^  -  V2x^ +16x- +9 -2x{Gx<^-8x* +9x^-12). 

5.  Given  ab - ^{a+b){p  +  q)-{-2jq  =  0, 

cd-i{c+d){p-{-q)+2)q  =  0, 

find  the  value  of  p—q,  and  show  that  if  either  a  or  6  is  equal  to  r 
or  d,  then  p  is  equal  to  q,  unless  a+b  =  c  +  d. 

or  * 

6.  Find  the  value  of  — ,  having  given 

y 

ic^"  —  ay^"  x^  —  Mx—yY 

7.  Prove  that   {a  —  b){b—c){c  —  a)  is  a  common  measure  of  the 
quantities 

c4(a-6)  +  a4(/v-f)+i4(c-a). 

8.  Find  the  conditions  that  a^x+h\y  =  Cy,  cf^x+h^y  =  c^,  aocl 
aoX-\-b^y  =  c^  may  be  satisfied  by  the  same  values  of  x  and  y. 

9.  Two  persons,  A  and  B,  start  at  the  same  instant  from  two 
stations  (c)  miles  apart,  and  proceed  in  the  same  du-ection  along 
the  line  joining  tlie  stations  with  velocities  (a)  and  {b)  miles  per 
tiour.     Find  the  distance   {x)  from  the  stations  where  A  over 
bakes  B,  and  interpret  the  result  when  a  z  b. 


EXAMINATION    PAPEKS. 


225 


XVII. 

1.  Ex^Dress  in  symbols  the  result  of  subtracting  from  unity  the 
quotient  obtained  by  dividing  the  sum  of  a  and  b  by  their  product. 

2.  Multiply  together  x  +  '^/a  +  b,  x— Vn-\-b,  x+^/a-h  and 
X-  Va-h\  and  divide  24a3-..22«2^,  +  2fl2c_5a^,2^27a6c-34ac2 
4-6^3 _22i2c-fl6&c2  +  8c3  by  '6a-2b-\-4c.    . 

3.  If  x-\-a  be  the  H.  C.  F.  of  x^+px+.q  and  x^+p'x+i', 
their  L.  C.  M.  will  be  {x-\-a){x-\-p-  a){x-\-p'  -  a). 

Show  that  the  difference  between 

X              x  X  a  b  c 

+   1  +   z — lana- — -  + 


x—a         x  —  b         x  —  c         x  —  a         x—b         x  —  c 
is  the  same  whatever  values  be  given  to  x. 

4.  Prove,  if  the  four  fractions  "" 
bx-\-cy-\-  dz       cx-\-dy  +  az       dx+ay '+  bz       ax-{-by  ■\-cz 
h^c  +  d — a       e-\-d-\-a—  b'      rf-^rt-f-Z>  — c'      a-\-b+c  ~d 

are  equal  to  one  another,  their  common  value  will  be  equal  to 

x  +  y+2 

— ^ —  as  long  as  a  +  h-\-e-\-d  does  not  vanish. 

5.  What  do  you  mean  by  solving  an  equation.  Show  that  3  is 
a  root  of  the  equation 

T/(a;3-3x+4)  =    -^^^ 

6.  Eliminate  x  between  the  equations 

a;3  4--^  +  3  Lc  4 ]=m,  ani 

a;  *  \  X I 

a;3— -r  — 3  a; =  n. 

x^  \  X I 

1111 

7.  If  —    +  -—   _  —   =    — -—r ,  a,  6,  c  are  not  all  different. 

a  b  c  a-\-  0  —  c 

8.  A  cask,  A,  contains  m  gallons  of  wine  and.  n  gallons  of  water ; 
an  another  cask,  B,  contains  p  gallons  of  wine  and  q  gallons  of 
water,  how  many  gallons  must  be  drawn  from  each  cask  so  as  to 
produce  by  theii-  mixture  h  gallons  of  wine  andc  gallons  of  water  ? 


226  EXAMINATION    PAPERS, 

XVIII. 

1.  Multiply  together  the  f.^rtors 

1-x,  1+x,  1+x"-,  l+a-*,  arc!  1+x^. 
and  show  that  if  n  is  any  uneven  number,  the  s  im  of  the  nth 
powers  of  any  two  numbers  is  always  divisible  by  the  sum  of  the 
numbers. 

2.  Find  the  numierical  value  of  the  expression 

c       \'n  +  \/c 
b       \/a—  \/c 

where  a,  h,  c  are  connected  by  the  equation  a'b  —  c)^—  c(h+c) '^  —  0. 

3.  A  has  a  younger  brother,  B.  The  difference  between  their 
ages  is  ^  of  the  sum  of  their  ages.  By  adding  twice  B's  age  to 
5  times  A's,  we  obtain  the  age  of  the  father;  and  by  subtracting 
twice  B's  age  from  5  times  A's,  we  obtain  the  age  of  the  mother. 
Show  that  the  age  of  the  mother  is  -^^  that  of  the  father. 

4.  Find  the  II.C.F.  of 

x^-(2a+b)x^+a(2a+?>)x-a2(a  +  b),  and 
X*  - {2b+a)x^  +b{2b+a)x-b2(b+a). 

114 

5.  If  —  4-  =   — .  shew  that 

b  c  a 

6.  Show  fully  how  the  rule  for  finding  the  square  root  of  a 
«flven  number  is  obtained.  If  w  +  1  figures  of  the  square  root  of 
a  number  have  been  obtained,  prove  that  the  remaining  n  may  be 
obtained  by  division. 

Extract  the  square  root  of 

7.  Find  the  value  of  the  expression 

t ^  when  X  =    ,  y  =  — 

l+xy  «— «  a 


EXAinNATION    PAPERS. 


227 


8.  Solve  the  equations  : 

(1)  ),{.v-'la)  -l{x  +  Ba)  +  l{x-6a)=0. 

(2)  V(2x2+l)+v'(2.;3+3)  =  2(l-a:). 

9.  Divide  21  into  two  parts,  so  tliat  ten  times  one  of  them  may 
exceed  nine  times  the  other  bv  1. 


XIX. 

1.  Multiply  together 

X'  +  \^ax-a2  -  |.«+|«-i. 
Divide  this  product  by 

lx^+^ax-2a^-^x-{-2a-^; 
and  extract  the  square  root  of  the  quotient. 

2.  If  x+y+z= 1 1 =  0,  shew  that 

•^  X  y  z 

(x'^  +y^ -\-z^)-^  [x^  +y^ -hz^)  - xyz. 
8.  Find  the  H.  C.  D.  of  20a;*+.r- -1  a.nd'ii>x^-i-lox»-3x-3; 
also  of  (x+y)'  -ic^  -y'  and  {x^-y^}'. 

4.  Given  that  ab -  (a-^b)(x+y)-^4:xy  =  0, 
cd  —  {c+cl){x—y)->r4:xy  =  0, 
find  the  value  of  (j"-i/)^. 

6.  Having  given 

x^=y^-\-z-  -2ayz 

y^=z^-{-x^-2lzx 
z^=x^+y^-2cxy, 
x"-  y""       _       2^ 

Show  that  i-a^    ~   1-b'^    ~   1-^c^' 

6-    ir^+^{2x-\-x'^)  -      ^^• 

7.  Determine  a;  in  terms  of  a  and  b  in  order  that  x^-\-2ax^  + 
3523.3  _  4^ 3a._j. 454  may  be  a  perfect  square. 

8.  A  company  of  90  persons  consists  of  men,  women,  and 
children  ;  the  men  are  4  in  number  more  than  the  women,  and 
the  children  exceed  the  number  of  men  and  women  by  10.  How 
many  men,  women,  and  children  are  there  in  the  company. 


228  EXAMINATION    PAPEB3. 

XX. 

1.  Divide  (l-{-m)x^ —  (in-{->i)xy{x  — y)  —  (n—l)y^  hy 

x^  -xy-\ry^. 
•I.  If  x'^+px'^^+qx+r  is  exactly  divisible  hy  x'^-\-mx-\-n^theti 
nq  —  n^  =  rm. 

3.  Prove  that  if  m  be  a  common  measure  of  j?  and  q,  it  will  algo 
measure  the  difference  of  any  multiples  of  ju  and  q. 

•Find  the  G.G.M,  oix^-px^  +  {q-l)x--\-px-q  and 

x^  —  qx^-{-[p  —  VjX^+qx  —p. 

4.  Prove  the  rule  for  multipKcation  of  fractions. 

simphfy  -IjiS^yz^  X  T-(?-^  ^  z^^ziix-zy):  , 

(2/+2)'--^2         {^+^Y-y-         (a:+^j2-23 
,         a  a  a^  2a^~b^  —  ab^ 

5,,  What  is  the  distinction  between  au  identity  and  an  equation  / 
li  x  —  a=:.y  +  h,  -piOYQ  x  —  u  =  y-\-a. 

Solve  the  equation 

16a;- 13         40^-43         32^-30         20a; -24 


4a;-3       '       8x-9  8x'-7       '      4a;-5 

6.  What  are  simultaneous  equations  ?  Explain  why  there  must 
be  given  as  many  independent  equations  as  there  are  unknown 
quantities  involved.  If  there  is  a  greater  number  of  equations 
than  unknown  quantities,  what  is  tlie  inference  ? 

Eliminate  a;  and  ?/ from  the  equations  ux-tliy  =  c,  a'x-\-b'y=.c^. 
a"x  +  b"y  =  c". 

7.  Solve  the  equations — 

(1)  ^{n  +  x)+^{n-x)  =  m. 

(2)  3a;4-Z/+z=13,  3//-f z-|--j;  =  15.  32  +  x+z/=17.  ' 

8.  A  person  has  two  kinds  of  foreign  money  ;  it  takes  a  pieces 
of  the  first  kind  to  make  one  £.,  and  b  pieces  of  the  second  kind  : 
he  is  offered  one  £  for  c  pieces,  how  many  pieces  of  each  kind 
must  he  take  ? 


EXAIMINATION    PAPEES.  229 

9.  A  person  starte  to  walk  to  a  railway  station  four  and  a-half 
miles  off,  intending  to  arrive  at  a  certain  time  ;  but  after  jvalking 
&  mile  and  a  half  he  is  detained  twenty  minutes,  in  consequence 
of  which  he  is  obliged  to  walk  a  mile  and  a  half  an  hour  faster  in 
order  to  reach  the  station  at,  the  appointed  time.  Find  at  what 
pace  he  started. 

10.  (a)  If   —  =  —  then  will     — ' —     =    —    . 

(b)  Find  by  Horner's  method  of  division  the  value  of 
a;«+290a;*+279a;3-2892a;2- 586a;- 312  when  x=  -289. 

(c)  Show  without  actual  multiplication  that 
{a+b-\-c)^-(a  +  b+c){a^-ab+b"-bc-{-C'-ac)-3abc  = 
S{a-\-b)[b-\-c){c-ra). 


Note. — In.  Ex.  6,  p.  87,  after  proving  that  a+h+c  is  a  factor, 
we  may  proceed  as  follows  to  discover  the  remaining  quadratic 
factor : 

The  quadratic  factor  must  be  of  the  form 

m{a^  +  b^+c^)  +  n{ab+bc+ca), 
in  which  m  and  n  are  independent,  being  either  zero,  or  a  positive 
or  negative  number.     To   determine   them   put  <:-  =  0,  then  the 
given  expression  gives 

{aS  +  b^-{-3ab{a+b)}^{a  +  b)  =  a2-^b^-+2ab, 
but  also  =  m{a^-\-b^)-\-nab.      :.  ?n  =  l  andw  =  2. 
,*.    a^  +  h^  +e^  +S{a+p)(h  +  c){c  +  a)}  ^  {a  +  h+c)» 
a^+b^-i-c^  +  '2{ab  +  bci-m)  =  {a-i-b  +  c)''(. 


230  EXAMINATION    PAPERS. 

.     XXI. 

1.  ?ind  the  value  of  a:3-  I— 1 r]  ^' +  \~u ~)*+"p 

when  a  =  ^,  b=^,  x  =  2.     SimpHfy 

2.  Fiud,  by  symmetry,  the  sum  of  {a-{-b-\-G)^ —{a  +  b  —  c)^ - 
(a-b-\-c)^-{b-a+cy,  and  of  (a^-^r/^.t-f  Sa^ajS  -  2«x3  +  3a;4)2 
and  {a'>=  +4:a3x+2a^j;-  +  2ax^+Sx^)-. 

3.  Exphiin  and  illustrate  the  signs  > ,  < 

Prove:  x^+y^'>2xi/,  (x+y+z)- >3(.c//+//z+zaj),  and 

s  J  i  4       J      ^ 

4.  Determine  the  value  of  a;  4-2/  — ^  + 3a;' //"z  ,  when  a;  +y  —  z    — 

0,   &c. ;  of  a7+7f/x-^  +  8a;--3a3-(a;*  +  7«a;3-8a;3-3a'-'),   when 
x=  —1. 

p  Tnp 

5.  Show  that  (a"") «' =  a7  . 


SimpUfy  I  (-—J  *|     '  X  (-^)  ^  X  t/(256),  and  divide 

a;  —Qax  -f  5a  x+la^x  —2a   by  a;   —2a  a;+a  . 
0.  If  u  =  ^  ix-^—\  and  v  =  jl  y  +  ~)  pi'ove  that 

7.  Gold  is  19j  times  as  heavy  as  water,  and  silver  10^  times. 
A  mixed  mass  weighs  4,160  ounses,  and  displaces  250  ounces  of 
water.  What  proportion  of  gold  and  silver  does  the  mass  con- 
tain ? 

8.  Shaw  that  l+px+qx^-i-rx^^is  a  perfect  cube  if  p-=dq, 
and  (]'  =3pr. 

9.  Solve  the  equations  : 

ix-2  lx+2 

m    ^.+-2  +   ^'.-2  =  *- 


(3)    —  +  --p^  =20  - ,     x+Q  =  ^y. 


* 
«XA.l\nNATTO^    PAPBR3.  231 

10.  A  person  buys  two  bales  of  clotb,  eacb  contaiaing  80  yards, 
for  $240.  By  selling  the  first  at  a  gain  of  as  maeh  per  cent,  as 
the  second  cost  bim,  and  tbe  second  at  a  loss  of  as  mucli  per 
cent.,  be  makes  a  profit  of  $16  on  tbe  whole.  Find  tbe  cost 
price  per  yard  of  each  bale. 


SECOND  CLASS  TEACHERS,  1880. 


XXII. 

1.  Find  tbe  value  of  x'^+x^ -IQGx'^ -lCn).c^ +81x  +  81  when 
x=  —'•7  ;  and  the  value  of  x^  —  dpx-  +  {3p'^  +g)x — pq  when 
x=:a+p.     (Arrange  the  latter  result  according  to  powers  of  a). 

2.  What  is  tbe  condition  that  x-\-b  shall  be  a  factor  of 
ax^+hx-\-c? 

Find  the  factors  of 

(a).      (a^^-ah)  +  2{l>^-ah)^3(a^~h2)-{.4:(a-h)^  ;   and 

(b).      (ax+b){bx  +  c){cx-\-cb)  —  {ax  ■}-  c)(bx  -{-  a)[cx  -j-  b) . 

3.  What  must  be  the  relation  among  a,  b,  c,  chat  a.v-+bx-\-c 
may  be  a  perfect  square  ? 

(a).  Extract  the  square  root  of 

(a-b)i-4:{a^+b^){a~b)2+4:(a4^+b*)-r8aH-\ 

(b).  If  5  be  subtracted  fi.'om  tbe  sum  of  tbe  squares  of  any  four 
consecutive  numbers,  the  remainder  wiU  be  a  perfect  .square. 
(Prove  this.) 

a  c  c  h  In 

4.  If    T~  =  ~r  =   "I'  a^f'l  IT   =   —    =  — ■ 

b  a  t  «  in  p 

(a+c  +  e){h  +  L-^n)  ah-\-cl-\-en 

Pi-o^etbat    |6  +  J,-J:/-)(A;+iM+;;;)    =    bk+d^^' 

ab{x^-y^)+xy{a^-b^) 

5.  (a).  Reduce  ^^^.^^^2^^_ry(^a2+b^)  *°  ^*^  ^°^®^*  *^^°^^- 

(6).  II  xi/-\-yz  +  zx  =  l  prove  that 

X  y  z  4,xyz 

+   1-^  + 


l_a;2    -r   i_y2    -r   i_^i    -    ^i^^2>^i^i_yi)^i_z^) 


232  EXAMINATION    PAPKB8. 

6.  Prove  that 

2{x-+2+4/(x2-4)}  . 

(h)     {b+c-a)a''  +  {c  +  a-b)b^  +  {a-^h~.c)c   a 

a         A         -1-  -^         ■■<  a 

{a  +  h-4-c){a''  +  //'  +  c')-2(a'  +  6'  +c')., 

7.  Solve  the  equations — 

(a),     (b  -  c){x  -  a)^  +  {c  -  a)(x-b)3  +{a~b){x  ~c)^  =  0. 
(b).    x  +  ij  =  4:Xi/;  )j-\-z  =  1yz;  z+x-'dzx. 
(r).     x+y+z  =  0. 

ax-\-bii-{-cz  =  0. 

bcx  +  cay-{-abz-\-(a  —  b)(b—c){c  —  a)=^(j. 

x  —  1-       »;  — 3 
(^)    ,1^    +    .  +  1  +  2  =  0. 


FIRST-CLASS  TEACHERS,  1876. 


XXIII. 

1.  Investigate  Hornei-'s  method  of  division. 

Divide   x^  ~dx^  -Six'  +'2rjx<'  +dx^  -8x^  -\-19x^  -{-Sx-^IO  by 
3.**  — 21a;3-f  9a;— 6,  showing  the  "  final  remainder." 

Find  the  value  of  2x^-^803x^  -3d^x^  +  1605x^-  -1204x+i22, 
when  x=  —402. 

2.  Ii/{x),  a  rational  and  integral  function  of  x  is  divided  by 

X'  +px  +  q,  the  remamder  is a'—B ' 

where  a,  (3  are  the  roots  of  x^  +px+q  =  0. 
Examine  the  case  where  p-  =4:q. 

3.  Show  without  actual  expansion  that 

a^{b-c)  +  b^{c-a)  +  c^{a-b) 

(a3  _  /;2  )3_|_(i2  _ c2)3  +  (c2_a2)3 

(a-i)3+(6-c)3  +  (c-a)3 


SXAMINATION    PAPERS.  233 

4.  rind  the  value  of  x  aud  y  that  will  render  the  fraction 

o  9  ,  , n 5^;^ ^TT  the  same  ior  all  values  ot  z. 

^«  +  (2/  ~  '^)'^  +  ^^'(i/  "■  ^^) 

5.  Show  how  to  find  the  suro  of  n  terms  of  a  series  in  Geo- 
metric progression. 

(1)  Show  that  the  sum  of  n  terms  of  the  series 

l  +  r  +  (l4-2r)(H-r)  +  (l+3r)(i+r)2  4-  .  •  .,  isn  (l+r)". 

11  1 

(2)  Sum  to  infinity  the  series  o.^.p  +  4^./-.o  +  p  Q.in+  •  •  •  • 

6.  Explain  the  notation  of  functions  :  prove  that  if 

f  (,„)  =  i  +  „,a;+  ^^^.7     •^'  +<^c-»  then/  (w)  x/  (?i)  =/(w.-}-70. 

Show  that  in  the  expansion  of  (1 -[-«)"  the  sum  of  the  squares 

1-2-3  ....  %i 

ot  the  co-efificients  =    ._,  ^^  .-, ,-^« 

(l-2*3  •  •  •  •  ny 

7.  Solve  the  equations — 

,,N    x  -  «         x— i         x  —  c 

^  '    h~-^c   "•"   l^c  +  a-f-A  ""  ^• 

(2)     a;4-10.6-3  + 35x2 -50.^  +  24  =  0. 

1  1 

(^)    21x2  -  13x+2   +    28x8  -  16x4-2  "■^^•^^"'^■''■^^' 

8.  Give  a  brief  account  of  mathematieal  induction,  and  show 
that  a  square  of  a  multinomial  is  equal  to  the  square  of  each  term 
together  with  twice  the  product  of  each  term  into  the  sum  of  all 
that  follow  it. 

Find  the  sum  of  the  products  of  the  first  n  natural  numbers 
taken  two  and  two  together  ? 

9.  If  — -  =  ?/  +  «,  ^=  2  +  X,  — -  =  X  4-  y.  pi'ove 


(1) 


1        1  1+a       1+i        1+c 


a         1)         G  \  —  ah\  —  6c     1  —  ca 


^^    a{l-bc)~b{l-ca)~c{l-ab) 

VT^^     Vl-ca  .   V\-ab      ^1—bc    Vl-ca     VV^ 


234  EXAMINATION    PAPERS. 

10.  AB  is  divided  in  C,  so  that  AB,  BC  =  AC^  ;  from  CA  is 
cut  off  a  part  CD  equal  to  CB ;  from  DC  is  cut  off  a  part  DE 
equal  to  DA  ;  from  £1)  is  cut  off  a  part  equal  to  UC,  and  so  on 
cd  inf.  Sbow  that  the  points  of  section  continually  approach  a 
point  C  such  that  AC'  =  BG. 

14.  Eliminate  x,  y,  z  and  n  from  the  equations 

a^x+hiy  +  c^z+d^u  =  0. 

a2X-\-h2y  +  c^z-\-d.2U—0. 

a  ^x+  h  ^y  -{-c  ^z  +  d  ^n  =  0. 

a^x-^-b^y+c^z  +  d^K  =  0. 

12.  A  rail^vay  train  travels  from  Toronto  to  Colling-wood.  At 
Newmarket  it  stops  7  minutes  for  water,  and  two  minutes  after 
leaving  the  latter  place  it  meets  a  special  express  that  left  Colhng- 
wood  when  the  former  was  28  miles  on  the  other  side  of  New- 
market ;  the  express  travels  at  double  the  rate  of  the  other,  and 
runs  the  distance  from  Collingwood  to  Newmarket  in ,1^  hour; 
and  if  on  reaching  Toronto  it  returned  at  once  to  Colhngwood, 
it  would  arrive  there  three  minutes  after  the  first  train  ;  find  the 
distance  between  Toronto,  Newmarket  and  CoUingwood. 


FIEST  CLASS  TEACPTEES,  1877. 


XXIV. 

x-{y--z)+y''{z—x)-j-z^{x-y) 


x^y^  -\-x^y^+x'^z-  -j-xH^-\-y'^rJ+y''z^  +  2x^y^z^ 

ax+m-{-l         nx-\-n  ax  +  m         ax-hn+1 

2.  Solve  (1.)  ^^^^—i  +  ^;^qr7i32  =  ^  +  ^  -  2  +  ax^n:^! ' 

(2.)  iri:^7^+rr:^y^=2. 

3.  A,  B,  and  C  start  from  the  same  place  ;  B,  after  a  quarter 
of  an  hour,  doubles  his  rate,  and  C,  after  walking  10  minutes, 
diminishes  his  rate  one-sixth  ;  at  the  end  of  half  an  hour,  ^  is  a 
quarter  of  a  mile  before  B,  and  half  a  mile  before  C,  and  it  is 


EXAMINATION    PAPEKS.  235 

observed  that  the  total  distance  walked  by  the  three,  had  tLey 
continued  to  walk  uniformly  from  the  first,  is  6J  miles.  Find 
the  original  rate  of  each. 

4.  {1)  investigate  the  relations  that  must  exist  between  the 
constants  in  order  that  Ax'^-\-Iiij- +Cz- ■^-aijz  +  bxz  +  cxy  shall  be 
a  perfect  square. 

(2)  Find  the  conditions  that  the  values  of  x  and  y  derived  from 

the  equations  ax+hy=  — -4-  —  =  c^  maybe  rational. 

X       y 

5.  11  x~+px+g  and  x^-\-mx+n  have  a  common  factor,  then 
will  {n  —  q)--\-ii{m—p)-  =in{m—p){it-q). 

6.  Prove  («"*)"  =  «"*",  whether  m  and  n  be  positive  or  negative, 
integral  or  fractional. 

Show  that  (x--'"+ar")»»  =a;"»  ^  "  x  (af-" +  -*;"-'")«» 

7.  (l.)Ifx   =    4-then  V^^5?!   =   {-^X^ 

1 

of  these  fractions  =  —  (a"4-i"+c"+t^). 

8.  If  a;  be  very  small,  show  that — 

, —  =2  -  4..!-,  very  nearly. 

2+ox-(l+4.«) 

^2(^2  _  12)        7l2(,j2_12)(-^2_23) 

9.  Prove  that  l  —  n^  +  "Ts  "  02 12 — 2^ — P —  +  -.  =  0 

10.  If  a  debt  $a  at  compound  interest  be  discharged  in  n  years  by 

.  a 
annual  payment  of  $ — ,  show  tliat  (l+rj"(l  — m/-)  =  1,  where  r 

is  the  interest  on  $1  for  a  year. 


236  EXAMINATION    PAJfUAa, 

11.  Solve— (1.)  8a;2-2a:2/-55. 

x--5xu  +  8y^  =  7. 


5  5 

3     a     p  +  ,j  1 


i,  5 


(8)  a^b'^x"  -4:a  b'xm  ={a-by-x 


V 


FIEST  CLASS  TEACHERS,  1878. 


XXV. 


2  ..3 


1.  SimpHfj  W'^-±f_V-4-)'-  (V^-V^r-7-"---, 

■         a;  a  +  a'  \      «  x  ■         a{a-x) 

x^-ii/-zr-       y^-Jz-x]^-       zl-J^-][Y 

^""^  {x-\-z)^-y^-    ^   '{x+y)--z'   +    (y  +  ^P-^c^ 

X  b        b^       b       h'i 

2.  Divide  — —  1  —  — —-7,+  —  +— ,  bvic— a; 

a  a       a"       X       x^     "^ 

shew  that  (-9a2)*  =  i{  ^(6rO-t-A/(-6rt)}. 

m       n        r  x^     v^      z- 

8.  If  —  =  — = — and-5r+7-^+^7  =  l,  prove  that 
X       y       z  a^     b^  '  c'  ^ 

4.  Fiud  the  relations  between  the  roots  and  co. efficients  of  the 
equation  ax^+bx-\-c  =  0. 

If  m  and  n  are  the  roots  of  the  equation  ox^  -bbx-\-':--0,  show 
that   the   roots   of  the    equation   acx'--i-{2ao— b'-^)x+ac  =  0    are 

ni  n 

—  and  — • 
■II  m 

6.  Solve  the  equations  : 

(1)  a-24-2l/a2_2a;=2x+8. 

,^,     x^     y^  X       y 

(2)  — -— =  10|,  — -  — =  4. 

y       X  *'    y       X      ^ 

(3)  X2  =  i/S  x+y+z=12,  x^-^y"'+Z'=[n. 


EXAMINATION    PAPERS.  287 

6.  Two  men  start  at  the  same  time  to  meet  each  other  from 
towns  which  are  28  miles  apart ;  one  takes  five  minutes  longer 
than  the  other  to  walk  a  mile,  and  they  meet  in  four  hours.  Find 
each  man's  rate  per  hour.  » 

7.  If  P,  Q,  R  be  respectively  the  pth,  ^th,  rth  terms  of  a  G.P., 
shew  that 

12       3 
Sum  to  infinity  the  series  —  +  773  4-":^+  ^^' 

JC  tL-  Jb 

8.  Find  the  amount  of  $f  ufc  compound  interest  for  n  years,  r 
being  the  interest  on  $1  for  one  year. 

Supposing  %2^  to  be  withdrawn  at  the  end  of  each  year,  what 
will  be  the  amount  at  the  end  of  n  years  ? 

9.  Determine  the  number  of  combinations  of  n  things  taken  r 
together. 

The  number  of  combinations  of  n  things  taken  two  togethej; 
exceeds  by  6  the  number  of  combinations  of  n  —  1  things  taken 
two  together  :  find  n. 

30.  (1)  Find  the  limit  of  (l-f^)"^  when  x  increases  without 
''imic. 

(2)  Find  the  (7*+l)th  term  in  the  expansion  of  (3  — 6a;) 

x^  -  3a;  -  8 

11.  Determine  the  limits  between  which  lies  o  .3  lo^ii  for  all 

possible  values  of  x. 


FIEST  CLASS  TEACHEES,  1879. 


XXVI. 

7.    Prove  that   ^{{0 -hy  ■{■(h-c)-' +  {c- ay]  =l{a-b){h-c) 
{c-a){{a-bY->r{b-cy  +  {c-ay). 

2.    Extract  the  square  root  of  ^<6—2a-j/(r/&  —  a8),  and  find  the 
simplest  real  forms  of  the  expression 

,/(3  +  iA/-l)  +  v/{3-V     1). 


238  EXAMINATION    PAPF.K». 

8.  Solve  the  equations : 

(1).  2a;4-f^3_ii^s_l_^4-2^-0. 

(2).  u;2+.y2+2-^=«3 
yz-\-zx  -^x^  =  h'^ 
z-f-  y—   z—c. 

(3).    y (»3  +  5x+4.)  +  -^/(x-  -f-Sx-  -  4)  =  x+i. 

4.  Prove  that  the  number  of  positive  integral  solutions  of  the 

c 
equation  ax+hi/  =  c  cannot  exceed  — r  +  1. 

In  how  many  ways  may  £11   15s.  be  paid  in  half-guineas  and 
half-crowns  ? 

5.  If  xy  =  (il>{a  +  b),  and  x'^ —n-y  +  i/^  =:a^-\-b^,  shew  that 
!^     j[\   I  ^       V 


\  a       o  I   \  o       a  j 

6.  Given  the  sum  of  an  arithmetical  series,  the  first-term:  and 
the  common  difference,  shew  how  to  find  the  number  of  terms. 
Explain  the  negative  result.  Ex.  How  many  terms  of  the  series 
6,  10,  14,  &c.,  amount  to  96  ? 

7.  Find  the  relation  between  p  and  q,  when  x^+px+x  =  ('*  has 
two  equal  roots,  and  determine  the  values  of  ni  which  will  mane 
u^  +max-{-a^  a  factor  of  »■*  —ax^  +a"x^  —  a^x-\-a^. 

8.  In  the  scale  of  relation  in  which  the  radix  is  r.  «hew  that 
the  sum  of  the  digits  divided  by  r  —  1  gives'  the  same  remainder 
as  the  number  itself  divided  by  ?•— 1. 

9.  Assuming  the   Binomial  Theorem  for  a  positive   integral 

index,  prove  it  in  the  case  of  the  index  being  a  positive  fraction. 

« 

Shew  that  the  sum  of  the  squares  of  the  co-efficients  in  the  ex- 
pansion of  (l-fx)"  is  [2n-j-d  w  )2^  n  being  a  positive  integer. 

10.  Sum  the  following  series  : — 

'  (1.)  l  +  3x-\-5x^  +lx^  +&C,  to  n  terms. 

(2.)   o — o+Q — i'Q+  &c.  to  n  terms,  anil  to  infinity. 


11.  Shew  that 


EXAMINATION    PAPESS.  239 

be,  —  nc,  —  ah 

b^^c^,  a^  +  2ac,-^r^  -2nb       is   divisible    hy 
c2,  0^,       (a+i)3 


FIRST  CLASS  TEACHERS,  1380-~Grade  C. 


XXVII. 

1.  Kin  ax^+2bxy+ci/^,  hi+lv  be  substituted  for  a;  and  W7?(+n^; 
for  y,  the  result  takes  the  form  Au^  +  IBnv  +  Gv'^.  Find  the  value 
of  (£2  —  ^Q'^.^(j)2  _ac)  in  terms  of  k,  I,  m,  n. 

2.  Eesolve  a{b—c)^-\-b(c  —  a)^+c{ri-b)^  into  factors. 

Prove  that = ^ 

«i;i/;  xi/z 

i{  It  =  x{-By^  —  Cz^) ,  v=-y(Cz^—Ax^),  w  =  z{Ax^  —  By^). 

5.  Extract  the  square  root  of 

(a-i)2(6-c)2  4-{&-c)2(c-rt)2-f  (c-«)8(fl_/,)3, 

and  tiy,  cube  root  of 

4.  Eliminate  a;,  y,  e  from 

'    -^  '  X        y        z 

k{x"-i-y-  +z^)+2[Ix-\-my+7iz)  +  /i  =  0. 

^'  Shnphfy   ''|^1;|;^'^^  {l/(4  +  3i)+i/(4-3i)}2, 

/-1+.?V3\^     -14-JV3,  ^ 
and    (^ 2 j  + 2 +^' 

in  which  j=  \/{  —  l). 

6.  Given  the  first  term,  the  common  difference  and  the  number 
of  terms  of  an  arithmetical  progression,  find  (i.)  the  sum  of  the 
terms,  (ii.)  the  sum  of  the  squares  of  the  terms. 


240  «XA5fD:s,TI0N    PAPEES. 

7.  Solve  tbe  equations 

ah 
(11.)     aa;+%=— +  — =1. 

—1  -1  —1 

(iii.)     oc{y-\-z     )=a,  y(z  +  x     )  =  h,  z(.r  +  y     )  =  ^. 

8.  What  value  (other  than  1)  must  be  given  to  q  that  one  of 
the  roots  oi  x^  —2x+q  =  0,  may  be  the  square  of  the  other. 

If  a,  b,  c  are  the  roots  oi  x^  —  fx^  -\-qx  —  r,  express 

'iah  +  26c+ 2ca -a^-h^  -c"^ 
in  terms  of/-",  q  and  r. 

9.  A  vessel  makes  two  runs  on  a  measured  mile,  one  with  the 
tide  in  m  minutes  and  one  a.gainst  the  tide  in  n  minu^es.  Find 
the  sjjeed  of  the  vessel  through  the  water,  and  the  rate  the  tide 
was  running  at,  assuming  both  to  be  uniform. 

10.  Five  points.  A,  B,  C,  0  and  P  lie  on  a  straight  line.  The 
distances  of  ^,  B,  and  C,  measured  from  the  point  0,  are  a,  h, 
and  c ;  their  distances  measured  from  the  point  P  are  x,  y,  z. 
Prove  that  whatever  be  the  positions  of  the  points  0  and  F, 

«*  {b  -  c)  +2/ -'  {c-a)-^i^{a-b)-^{b-c)l^c-u)  {a  -b)  =  0. 


APPENDIX. 


Section  I. — Elementaey  Theorems  on  Polynoaies. 
(See  page  39,  et  seq.) 

Theorem  I-  If  the  polynome/  (ic)"  be  divided  by  x  —  ti,  the 
remainder  will  be /(«)". 

D'Alembert's  Proof.  J\xY  is  the  dividend,  a; -a  is  the  divisor  : 
let /i  (a;) "~^  be  the  quotient,  which  is  necessarily  a  polynome  o* 
degree  n—1,  and  let  R  be  the  remainder.  Then,  since  the  pro- 
duct of  the  quotient  and  the  divisor  added  to  the  remainder  re- 
produces the  dividend, 

But  R  does  not  contain  x,  hence  it  will  remain  the  same,  not 
merely  in  form  but  in  actual  value,  whatever  value  be  given  to  x.. 
Take  the  case  x  =  a,  then  («— «)/i  (a?)"~^  vanishes  for  its  factor  x—a 
does  so.  heucB  R=:f{ay\  Thus  the  remainder  is  the  value  of  the 
dividend  when  x  has  the  value  which  makes  the  divisor  vanish. 

It  has  been  objected  to  the  above  proof  "  Division  can  be  per- 
formed only  when  there  is  an  actual  divisor,  therefore  in  assum- 
ing R  to  be  the  remainder  ol f{xY -i-{x  —  a)  it  is  assumed  that  a- is 
not  equal  to  a,  and  although  R  will  remain  unchanged  for  all 
values  of  x  that  fulfil  this  assumption,  it  cannot  thence  be  inferred 
that  it  will  do  so  if  the  contradictory  assumption  be  made.  In 
such  case  the  only  legitimate  conclusion  is  that  there  being  no 
divisor  there  is  neither  quotient  nor  remainder.  Therefore, 
although /(a)"  may  be  the  remainder  in  the  case  in  which  x  is 
not  equal  to  a,  yet  the  above  argument  does  not  prove  it."  This 
objection  confuses  arithmetical  or  numerical  division- with  alge- 
braic or  formal  division,  division  by  a  definite  quantity  with  divi- 
sion by  an  undetermined  or  variable  quantity.  The  following 
proof  does  not  involve  the  assumption  x-a,  and  consequently  is 
not  open  to  the  foregoing  objection. 


242  APPENDIX. 


Lagrange  s  Proof.     Lemma,     af*  -  a"  is  divis\ble  by  .*     -i,  if  « 
be  a  positive  integer. 


By  actual  division   ——zr~'^  -_^----     -     , 

.-.  «"  -a»  is  divisible  by  x.—a  if  a;"-^-«»-i  is  so  divisible, 
hence  u;'-^ - a"^^  "  "  x-a  "  ^''-^-^"-^  .... 
Thus  we  can  reduce  the  exponent  unit  by  unit  until  at  last  we 
arrive  at,  x^  —  a"^  is  divisible  by  x-a  ii x—a  is  so  divisible.  But 
x~a  is  certainly  divisible  by  Itself,  .-.  x^-a^  is  divisible  by  ic -  <*, 
.-.  x^-a^  is  also  divisible  by  a; -a,  .".  so  also  is  x^—a'*'  and  thus 
we  may  go  on  to  any  positive  Integral  exponent  whatsoever. 

Theorem.  Writing  /(re)"  m  polynomial  form  arraugpa  in 
ascending  powers  of  x, 

f{xy=A^+A^x+A^x^+A^x^-h  +A,a^, 

.-.  /(a;)"-/(«r=^i(a;-«)  +  /l2r.-/;3-a2)_^.^g(^3_a3)+  .... 

+.l„(a;"-a"). 

But  every  term  of  this  polynomial  is  divisible  by  a;  —  a,  and  the 
highest  power  of  x  in  the  quotient  is  a;"~^  got  from  the  term 
A^{j:"  —a"),  so  the  quotient  may  be  represented  by/i(a;)"~S 

fix)"  f(a) 

x  —  a     -'  1  ^  '        'x  —  a 

Theorem  II.     If  the  polynome /(^)''  vanish  on  substituting 

for  x  each  of  the  n  different  values  a-^,  a^,  a^,  .  .  .  .  «.„, 

i\\Qnf{xY  =A{x-a^){x  —  a^) {x-a„), 

in  which  A  is  independent  of  x  and  consequently  is  the  coefficient 
of.7f  in/(.t;)". 

Since /(ai)=0,  ■•  fixY  ={:x-a^)f  (a;)"-^  In  this  substitute 
a,  ior  X,  :.  ^ineef{a^Y  =0,  it  becomtrf  0  =  («., -rtj)/i(a2)"~^-  Of 
this  product  the  factor  a^  —a^  does  not  vanish  since  by  hypothe- 
sis a^  is  not  equal  to  a^,  therefore  the  other  factor  fiia^Y"^  must 
vanish  that  the  product  may  vanish,  and  consequently/j(a;)"~^  is 


APPENDIX.  243 

divisible  Ly  x—a.,.     Let  the  quotient  I)e  denoted  by /2(.'»)''~^,    .. 
f{xY=(x  —  a^){:x~a,^f^{xY~".     Substitute  tig  for  ./•  and  proceed 
as  before,  and  it  will  be  proved  that  x—a^  is  a  factor  of  /(jc)"  . 
Continuing  to  n  factors  we  get  a  quotient  independent  of  a;,  since 
each  division  reduces  the  exponent  of  a;  by  unity,  .-.  finally 

fixY  =  Ai;x  —  a^){x-a^^ (./j  -  a"  ). 

Cor.  \ij\xy-  and  <p(x)"'  both  vanish  for  the  same  r>  different 
values  <Ax,j\xY  is  algebraically  divisible  by  (^{xY^. 

» 

•Let  «j,  «2>  ^3' «m  1^6  t^6  "'■  different  values  of  x  for 

which  the  poiynome.s  vanish, 

.'.J{xf  ={x-a^)[x-a^) {x-a^F{xY-^- 

and  ^(./')"'  =  .i(»;  — «i («—<<„ ) (.c  — a„) 

.-.  1\xf  -=-(Z)(.r)"'  =  F(a;)"-'"-4-J , 
w.'iich  is  an  integral  function  of  a  siuce  A  does  not  contaiu  x. 

Theorem  III.  If  the  polynome/(a;)"  vanish  for  more  than 
n  different  values  of  a;  it  will  vanish  identically,  the  coefficient  of 
every  term  being  zero. 

Let  (7 J,  Oa,*a^ a„,  «„+i  be  v+1  different  values  of  a; 

for  which  /(a;)"  vanishes, 

.-.  f{xY  =A(x-a^)(x-a2)(x-a^) (x-a„) 

Substitute  (1,1^1  iov  X,  and  since /(a„+i)"  =0, 

.-.  0=  ^l(«„+i  -  rt J )(«„+!  -  «._,)(rt„4.i  -  u^)  ....   (a„,+i  -  n„ , 
But  none  of  the  factors  On+i  —  a^,  a„+i  —  a^,  &,e..  vanishes, 
,'.   A  must  be  zero,  or 

f(xY  =0{x-''ii)(x  —  a^)(x-a^) (x  —  a„) 

and  the  factor,  zero,  will  be  a  factor  in  the  coefficients  of  every 
term. 

Theorem  IV.  If  the  polynomes  f{xY ,  (p(.r)"'  (n  not  less  than 
m)  are  equal  for  more  than  n  different  values  of  x,  they  are  equal 
for  all  values,  and  the  coefficients  of  equal  powers  of  x  in  each 
are  equal  to  one  another. 


244 


APPENDIX. 


f(xY'^-A.,-i-A,x+A.,x^  +  A^x^+  ....   +A„x'' 
(p{x)^  =B^-^B^z  +  B2x''  +  B^x^  +  .  .  .  .   +5^ic^ 
.'.f{xr-<i>(x)"^  =  A,-B,  +  {Ai-B,)x  +  {A^-B^)x^-{- 
{A^-B^x^^+  ....    +(J„-i?>- 
+  J,«+ia:"^+i-f.^^+2a;"'+2 -]-A„x\ 

aud  this  is  a  polynome  of  degree  n  at  most.  i>nt/(a;)"  =(p(a;)"'  foi 
more  than  n  different  values  of  x,  that  is  /(a;)"  —  ^(a;)'"  vanishes 
for  these  vahies,  .-.  hy  Theorem  III. /(a;)"  -  (p{x)"'  vanishes  identi 

cally,  aud*the  coefficients  yl„  —  5o,  ^ J— 5j,  A^—B^, 

A„- B^,  A„i^'i,  Am+i, ^„  are  all  equal  to  zero, 

.*.  Aq  ~  Bq,  a  j  =B^,  A^  =  lJ2,.--Aj„=  B^,  Afn^i  =0,  Afn4-2  —  ^  ■■■ 

Note  to  Art.  XVII.     To  find,  where  such  exist,  the  factors  of 

ax~  +  bx;/  +  Cxz+ey^  +fnjz+hz^. 
Multiply  hy  4a 

4:a^x^  -^iaJ>x>j  +  4:'^icxz+4:riey^  -^4:ar/yz-}-4:nhz^ . 
Select  the  terms  containing  x  and  complete  the  square,  thus 
Aa^x^  +4:aJ)xy-i-4:acxz  +  b^ij^  +2bcxz  +  C^Z' 
■^(b2-4:ae)tj2-2{bc-2arj)yz-{c^-4.fth)2^  = 
(2ax+b]/  +  Cz)^  -  {(62  -4:ae)y^  +  2(bc-2ag)i/z  +  {c^-4:ah)z^} 

If  the  part  within  the  double  bracket  is  a  square  say  {my  +  nz)^ 
the  given  expression  can  be  written 

{2ax+bii  +  Cz)^  - (mf/+nz)' 
which  can  be  factored  by  [4J .  Factor  and  divide  the  result  by 
4a.  If  the  part  within  the  double  bracket  is  not  a  square,  the 
given  expression  cannot  be  factored.  If  b  and  c  are  both  even, 
multiply  by  a  instead  of  by  4a  and  the  square  can  be  completed 
without  introducing  fractions.  If  e  is  less  thau  a  it  will  be  easier 
to  multiply  by  4e  instead  of  by  4a  and  select  the  terms  containing 
y.     A  similar  remark  applies  to  h. 

This  method  can  evidently  be  extended  to  quadratic  multino- 
mials of  any  number  of  terms. 


appendix.  245 

Examples. 

1.  Eesolve  x-  ■{-x)/  +  2xz-2i/-+7ijz-iiz'^  iuto  factors. 
Multiply  by  4 

4x^  +  4:X>j+8>jz-S!/^-{-^S>jz-12z^    - 
Complete  the  square  selectiug  i-nns  m  x, 

4a;2  +  4<:.y+8x-2+.'/2+4?/s+4z3-9^2+24j/z-l(;z3« 

(2x-  +  7/  +  2z)"-(32/-42)2  = 

{(2x+v/  +  22)  +  {3?/-4^)}{(2a;+7/+2z)-(3^-4r)}=- 

(2x-l-4?/-2z)(2x--2^+6z)=.4(^4-2^-2)(a;-^+3.^) 
.  • .  the  f iictors  are  (u;  +  2//  —  s)  (a;  —  //  +  3z). 

2.  6rt2  -7rt/;+2^tc-2062  +  64Z;c-486-2. 
Multiplygby  4  x  6  =  24 

14ia2-168a6  +  48^c-480/^2+1586&c-1152c5=s 

(12rt-7i+2'-)^- 529^2  _^1504/>c-1156c2  = 

(12„— 7i+2f)2-(236-34f)2= 

(12rt+16?^-32r)(12a-306  +  36r)=B 

24(3«+4&-8c)(2«.-5/^  +  6c), 
.'.  the  factors,  are  3fl+4&  —  8c  and  2a— 5/>  +  Gc. 
3=     a;2_|.i2.r;/+2x2+26^2_8^2_923  = 

(a;2  +  l2x//+2x-2  +  3Gy2 4.12//Z  +2-') _  10^2  _  20^2-102'*  = 

(x+6y  +  z)--{0/+2)v/10}2:= 

{:r+(6+  v/10)^+(yiO+l)z}  < 

^a:  +  (6-A/10)y-(i/10-lM 

4.     3a2+10r^6-14«p+12«rf-8/>2_8W+8';2_8(;f/. 

Multiply  by  3,  not  4x3,  since  the  coefficients  of  the  other  terms 
in  a,  are  all  even, 

9rt2  -i-  30ai  -  42«c+36a6i  -  246^*  -  24^*d  +  24c 2  -  24efi. 


246 


APPENDIX. 


Select  tho  terras  containing  a  and  com^Jotc  the  square 

706c -  MIkI  -  25c '  +  GOcd  -  3Gd^  - 
(3a  +  5h-7c-{-Gd}^  -  {Ib-iic  +  Gd)^  = 
{3a  +  12b-12c  +  lM){'da~2b~2c)  = 
ii(a-^U-4:C  +  4d){3a-2b-2c), 

,•-  the  factors  are  a  +  ib —4c-{-id  and  da  — 2b  — 'Ac. 

Work  lixercise  XXIX  by  this  method. 


m             p                   m   _ 

_  P 

q 

m 
.   b~n 

nt               m 

{a  -f  h)Ti  =  an  - 

m 
-ITn 

m     p                mp 

(a'n  )'q     =  am 

Section  II. — Indices  and  SuPvDS. 
The  general  Index-laws  are 

in  p  m     i_  p 

an  ,  a g   =  a  n      V  (1) 

(3) 

(^) 

{-) 
Tho  law  connecting  the  Index  and  the  Sura  byiubjls  is 

m 

oJ  =  ^"/(a™)  (G) 

[The  indices  1-,  i,  i,  &c.,  are  generally  used  to  denote  '  either 
square-root,'  '  any  of  the  cube-roots,'  '  any  one  of  the  fourth- 
roots,'  &c. 

The  surd  symbols  -j/,  -^,  V,  &c.,  are  by  some  writers  re- 
stricted to  indicate  the  arithmetical  or  absolute  roots,  sometimes 
called  the  positive  roots.     Thus 

^4  =  2,  but4*=±:2,    .-.  4^=±y4 
Also,  V{{-2f-}=  i/4  =  2. 

lf27  =  3,  b'lt  27^=  3  or  3/ ^       j  /.  S  '  =(]/')-3/^': 


J. 


i     ,X 


4/16  =  2,but  IG  =+2or  ±2j,  .-.  16"  =  {r)t/16 


APPENDIX.  24V 

With  this  restriction  the  geueral  connecting  formula  -wonltl  be 

a7  =  (111)  ;'(«"•) 
In  the  following  esercises  this  restriction  need  not  be  observer,] 

EXEKCISE. 

1.  What  is  the  arithmetical  value  of  each  of  the  following  : 

8G*,  27^",  IG*,  Q2\  4^  8',  27*,  G4^,  ?.2',  64*,  81*,  (3f)^ 

i  I  h  I         -5         -2         -75 

(5:Vr)  ,    (IfV)  ,  (-20)-,  (-027)%  49   ,   32   ,   81 

2.  Interpret  ar'^,  «»,  a^' ,  («2)    ^,  aS    ",   r^,   («"')-%  a^  a"*. 

3.  What  is  the  arithmetical  value  of 

36~',  27~*,  (•1G)~^  (-OOIC)^,  (1)~^  {4^)~^',  {^%)~\  {5^\)~^ 

1  -L 

4.  Prove  {iry  =  (a")" ;  [cC^f  =  (a" )"^ ;  a"^  =  (a-^)"* ; 
and  express  these  theorems  in  words. 

6.  Simplify  a  J,  c^ x^ ,ni  .vi'^n  .n~^^\  (7i)*-(2^)*-(Bl)* 
a       c        a        e  e  5  1  \h 

1'  T  ^'  "T"'  "^'  (2fr  .(G^r-^(i) 

a      c      a  ex 

6.  Eemove  the  brackets  from 

(ae)^  (Z;)"^,  (cJ)~^  ('^V',  («~^A  (.r^)~^ 

{an--)\  {al}y,  («.^c-i)~*,  (a--'^/')~^    (a;'^/"')""^ 

7.  Eemove  the  brackets  and  simplify 

1    .13-  1   1      2  1    1       i  /  1    I  \  2         / 1    1^2        / 1   1  \« 

(.«  )    (x*  )    [X  )     ',     X  X  X  ; 

r  A'  ;  a?  ;r  : 

{x''~' -^x-^~'  ]  {x''--''-^x--  ^}. 


248  APPENDIX. 

8.  Sim^\i{j-x{x~^{-x)-^}^,  x{{-x)~^{-x)-'-}~^, 

{  —  x)       {x~^x      } 

9.  Determine  the  commensurable  and  the  sua.-cl  factors  of 

12%  24^  18"^,  (-81)*,  12^  64^  {^^J,  {Gif^. 

(The  surd  factor  must  be  the  incommensurable  root-  of  an 
integer.) 

10.  Simplify  8*-fl8*-50^72V(T¥;5/-(To5)'"^i 
{(6  +  2")(6-2^)}S  (2V3V+(^'-3S'; 

(2  +3^)(r+9^-6^);  (7  -3^)-(7"  +  3-)-. 

[{{a-^x)[x  +  h)y'-  {{a-x){x-h)}^]  3  ;  ' 

{a'+(a3     a;3)^[*  .   {J -{a'-^ -x^)^}^ 
Express  as  surds, 

i.1.  a  ,  x  ,  jj    ,   c      ,  h 


n  +  ^      _^  +  §       .25      -«  +  ,^ 
12.  a;        ,  ^/  ,  a     ,  b 

a  m—H  n~^ 

18.  (ax-bf,{x^-4:x+l)   4    ,  (p -7:^)      *• 
Express  with  indices, 

14.  if  a-,  Vc^   "/a;"',   if/r-",   Vf^/.-K),  7/1-* 

15.  -^(a^+iB)^  ^(^34.^3)2,   {ir(«3+63)p,  .-y|^^_;).^>, 
;/(a-ia;)"-\   V  (a" -Z>" )'"-'" 

16.  (rt  )  ,  (6      )  ,  (c      )      ,  (x-  )      ,  (a^x)      ,  {a   ^x      )      , 

1   — i  14 
(a?  v/     ')     . 

Simplify   the   following,   expressing  the  results  by  borb  v^^-Vk- 
tions, 


APPEND1.V.  240 


1  X  ■  t  2  1 


17.    ii-(i'     ",a^.a     ',  a    .a  ,  a.a      ,  a  '-^(1,0,'  ■^"'n^ ,  a  ^a 

a  h  c    .  a      b      c 

•^          "^          *^           a;             y  oc-  c[ab) 

X  i  3  3  -^n  Sn 

It' +  a     "        a   —a    *  a~  ^  —  (t^  a'-+l  +  a~'^ 

1Q     -      - ,       .,  I 

4       —h          h       —h        _"~        -"  «  +  l  +  «~^ 

a   —a  a   —  a  a      '^  -\-a'^ 

1            1          *          3-2-  ■            ,■'          \    I          ¥ 


20.  Divide  x-y  by  a;"  —  ;v^;  ic  -\-a  x"  +  a  by    "-+-«  .c  -f'*  ; 
ic+.'/H-^— 3.C // 2    by*  +?/  +;s 


i       .\      i 
2a6+2/;(;+2c;a-a!»-fe2  -q-  by  <("  4-^''+f 

ExERCISE. 

1.  Express  the  following  quautities  i.  as  quadratic  smds,  ii 

as  cubic  surds,  iii.  as  quartic  surds. 

'  J.         __ " 

a,  3a.,  2a2,  a-x,  a;",  y  .  a~"\  -,  mx     p,  •!, '01,  I'l.c'-. 

y 

2.  Eeduce  to  entire  sui'ds, 

xVx,a^a,   l>^^h\   3if3,  4if2,   V^,  ilT^,  if%,  ^\ 


n  + 


xJ^y    I  /j-  +  ?/\ 


.'//. 


(x-y)   ''■^{x^+2xy  +  y^}  ^    (x-x   ')f{x^  +  iy, 


250  APPENDIX. 

3^  Eeduce  to  their  simplest  form 

1/12,  1/8,  ^,^50,  -^16,  4if -250,  Vh  ^h  i/A.  5ir(-320), 

,/{a3(l-^^)[,    ^r,,2(,,._i)4|,    ^(,,i),    »^,,.-.i^     »,^»+»^ 
y«^«-^^  I'ya'™-,    V('f2x  +  «3),  ^(a34.2.,4^^,,5^2)^ 
l/{(x-l)(^2_i)|,  -^{(a2  +  2ffa;+;c^)(«3+;c3)}, 

,/(8.c3 -16^+8),  ^{{x^-^+x-^){x'-'lx^-^l)f, 

N  \~^+2+.<;-i   /  N  \27j;2  +18u;  -r  3  N  (  ^^  j 

4.  Compare  the  lollowing  quantities  by  reduciug  tlicm  to  the 
same  surd  index : 

2  :  ^/3  ;  2  :  -jf  9  ;  1/2  :  -^3  ;  ,/10  :  -,^30  ;   2  ^/2  :  ^^22  ; 
a2  :  Va^  :   V-^  :  iy//  ;  ^x  :  ^// ;  '"^£'  .  y.c^;  i/a  : 'yb  :  ::/c  ; 

5.  Reduce  to  simple  surds  with  lowest  integral  surd  index 

Vif'a),  f^{i/b),  ^{^c),  ^{i/x%  t/ifx'),  4/(r-^")» 
'    f'it/'^''),  1^(1/27),  A/(ir81),   irif/81),   yi.n/a), 
■     f{aya),    VWx),  iTC^'-yf),   iflov/S),  |/{3-^'3), 
V(3#/3),   e'(^C/-^),   V{a^'{h:yc)],  xV{x-Wx-'), 
y^{y-'fr''),z^/{z-''-fz-^-),y{x:y{yiyz)],x-^f{x"^Vx^) 

6.  In  the  following  quantities,  combine  the  terms  involving  the 
same  radical ; 

3|/2+5v/2-7a/2;  -/8-'/2;  ^nQ+Qf^; 

^16+ \/ 2;  cq/aj-i/ic;  a'2/x—bl/x; 

8^/a  +  5Vx-7i/a+  V{4.a)-3y{Li-)  +  iV{dx) ; 

/.c;+3/(2.f)  -  Zy'{dx)+y^{ix)  -  v'  (S^j  +  v'(12..) : 


APPENDIX, 


251 


4.\/{a^x)  +  2^(b^x)-Sj/{{a  +  h)^x\; 

^/{{a-b)^x}+^/{{a+by-x\-V{a^:^^  +  l/{{l-a)^x}-Vx; 

^/ (a _ 0)  +  ■,/ {lQa-Wb)  +  i/{ax^-bx^)-  V { 9 {a - b)}  ; 

l/{a3+aV>)—i/{b^+ab-^); 

j/{a^+2a^b  +  ab^)-  ^(^a^ -2a"'b-Jrah^)  -  i/{AaJr-). 

7.  In  the  following  quantities,  perform,  as  far  as  ]DOSsible,  the 
indicated  multiplications  and  divisions,  expressing  the  results  in 
their  simplest  forms : 

^/2.|/o;    a/3,  a/12;  |/14.  a/S-j.-j/IO;  j/a.y'[3a); 

l/c.A/(12c);   V{Gx)y.{8x);   Vy^Vu^ ;  -^y' -V^u' I 

fa.^a^yb:  iA.a/(^);   Va.vi}];  l/-3^'- y  (£)  ' 
|/r/"+i.-i/«"+^ ;  f/b^^\^lr-^^ :  v/12--  A^3  ;  ^/{Qx)  -  x/(2a;) ; 

{a^x)~V{a-\-x);  (a2-a;2)_=.v(a-aj);  (.^^  _i)_^^y(a-^l)3  . 

(3v/8-6s/2+>/18+\/32+x/72-2v'50).i/2; 

(7i/2-5  A/r)-3v/S-}-4  a/ 20)  '/18) ;  (v'5+i/3)(^5-v  3)  ■ 

(i/2+1)(s/6-a/3)-  (3-V2)(24-Bv/2); 

(5V3+v/6)(-V2-2)  ;  {^n-  Vb){ya-\-y^b)  ; 

(«y'/;4-?,yrt)(/;  Va-aVb) ; 

{  ^/(:/;-hl)  +  v/(a;- 1)}  {  a/(^;+ 1)  -  n/(x- 1)[  ; 

{A/(3rt-/>)  +  |/(3&-a)}{v/(3ff-&)-  A/(36-rr)}; 

V(<'  +  ^/b).V{a-  Vb);    V{^/x-\- ^ ij).  V^x- Vy) '. 


252  APPENDIX. 

{a  +  y\l  -«2)13  .    !y^a  +  b-.r)-y'{a-h  +  T)}^  ; 

[{V(«+^)(x-Z>)}  +  v/{^z-a:)(;.+i)}]2;|4i^)-4|^)[' 
W {{a-^x){x+h)]  +  ^  {{a-x){:c-h)]Y^  ', 

{V^V10+1)-V(V10-1)}2; 

■\y{a-\-^{a"-  -a:-')}4- V{^/  -  V(«2  _a-2)l  j^  ; 

(V./;+Vy)*  +  (V.^- v/.v)4  :    [a^+ah^2+h2){a^ -ah  s/'l-^-b^); 

B.  Find  rationaliziug  multipliers  for  the  following  expressions, 
and  also  the  products  of  multiplication  by  these  : 

a-{-Vh,   \/a-\-l\/c,   aVb  —  bVa,   a  +  V{a^  —x^), 

V(a-x)  -  V(ri+x),     V(a^  +  Vr)+\/(a2  -Vc), 

V{8  +  V(24  +  Vo)}-V{8  +  V(24-V5)},   V-v  +  V^  +  Vc, 

3  +  V2+V7,  v'6  +  V5-V3-V2,  ^a  +  J-b  +  ^c  +  ^d, 

y{l+a)~^{l-a)  +  y(l  +  b)-^{l-c),   f/a+f/c, 

yx  +  l  +  ^x-^',   y{aI-')-^{a-'b),   f-^  +  fS-fB, 
fu  +  f'b  +  ^c,  a  +  ^b  +  fc. 


APPKNDIX.  253 

« 

9.  Eationalize  the  divisors  and  the  denominators  in  the  follow- 
ing, and  reduce  the  results  to  their  simplest  form: 

l^(2-^/3),  3--(3+i/6),  5-^.(■^2  +  V7), 

(,/3+v/2)-=-(v/3-i/2),  (7|/5+5|/7)--(|/5+V7), 

«-=-(s/rt4-a),   {x  —  a)-^{-i/x  —  i/a), 

aVx  +  bVii  2V6  1  +  3|,/2-2a/3 

TVsT-^^'   1/2+1/3-  V6'   1/2'+ 1/3+1/6' 

a/6 -1/5 -1/3+^2     2 

•     V'6+i/5-a/3-V2'    i/(fl  +  l)- A/(a-l>» 

2c a.+a;+\^(a2+a;2) 

|//(rt-j_c)  +  i/(a-c)'    a+x-v\a^-\-x^y 

^/{a-\-x)-^V^a—x)  1 

V(a+a;)-V(a-a;)'    ai/(l  +  h^)+b ^Jl'^aFy 

\/(l-62)+   i/(l-a2)'    av'(l-c-^)  +  cV(l-a2)' 
l/{(l+a)(l  +  Z>)}-A/{(l-a)(l-6)} 
.    l/{(l  +  «)(l  +  ^)}+l/f(l-«)(l-i)}' 

(a-x)i/(&^  +  //2)-(6-y)i/(a2+a;2) 
(«+a;)i/(62  +  2/2)  +  (6  +  7/)i/(a2+^2)» 

A/(l  +  a)-l/(l-a)+i.l/(l+//)-vXl-^>) 
l/(l  +  a)+ v'(l-a)  +  s/(l+/>)+V(l-^)» 

%/(x  +  fl)-N/(y-a)-  ^/{x+b) -\-^{x -  ft) 
l7(u;  +  «)+  V{x-a)-\-y{x+h)+  V{x-~by 

V6"^l/a'     ^U-W       ^U+W'    Na-V-C'     ^Vj^Vy' 
1         1       a/«     \/a; 

•4r-i/(a3-l)'      1         1  '     A/a     v/a;' 
\/j;     •«/ V     \/ic     A/a 


254  APPENDIX. 

10.  Find  the  values  of  the  following  expressions  for  n=l,  2, 
3,  4,  5,  respectively. 

V5\  \      2"    /     -  \      2       /  [• 

1     f(2+i/6)"+^-(2+V6)     (2-\/6)"+^-(2-VG)] 
2761  1  +  1/6  ~  1-V6  I 

11.  Show  that 

2(x  — 1) 
is  a  square  for  n=l,  2,  or  3  respectively. 

12.  Extract  the  square  roots  of 
x+y-<2.A/{x}j),   a-{-c+e+2V{ac-\-ce), 

a+2c  +  e  +  2^/{{a+c)(c+e)},   2a+2V{a^  -c2), 

2{a2-|-63-^/(rt4+a2/>2+64)},  x-^  +  x'^, 

^x-\-2  +  Vx-^,  x+3x^  +x^+2x^/x-^2x^ y/ X, 

x^-xi/  +  iy^-hV{Ax^y-8x^-ir'+xy^),  2x+^  {?,x^ -y^), 

5-2i/6,  10+2\/21,  9+4a/5,  4-v/15,  7+4  v'3, 

12-5V6,  70  +  3;/451,  4-x/15, 

9  +  2;/6+4(i/2+v'3),  15.25- 5  V.6. 

13.  Find  the  value  of 

•     ^^¥+te^'gi-ent.l=:j^^^^^andy=^,^, 
y(a;2+i/2),  given  x=^{a^c)  y  =  ^{a^e)-y 


V(l+x)-V(l-a;)      .  _     2ah 

V,  r+;^V(l  -a;)'  ^^^^  ^  -  «3  +/;2  J 

2«V(l  +  a^)  _J    |l_    l£l. 


APPENDIX. 


255 


14..  JiV{x+a-{-b)-rV{x  +  c-td)  =  y/{x-i-a-c)+V{x-b-\-cl), 

*(1.4-V5)a;-2  ^(l-V5)a;-2 

15.  Simplify  :,y^^i^v5)x+l'^^-h[l-Vo)x-^l' 


Complex  Quantities. 


Quantities  of  the  form  a+bV  —  l  in  which  neither  a  nor  i 
involves  \/  —  1,  are  called  complex  quantities.  The  letter  J  (or  i) 
is  frequently  used  as  the  symbol  of  the  ditensive  unit  •/  — 1,  so 
that  rt  +  ij/  — 1  would  be  written  a  +  hj.  So  also  V  —x^JVx, 
y/-x.V  -y  ^p_ V {xy)  =  -  Vxij,  and p  =  -j 

EXEKCISE. 

Simplify  the  followmg,  writing  j  for  >/  — 1  iu  any  result  in 
which  the  latter  occurs  : 

1.  y_4,  |/-3G,    1/-81,    V-8,    i/-12,    1/-72,    ^-8, 
v/-5.i/-G,     V-QV-8.     V-8.VI-2,     ^/-8.lr-8, 
l/-5.j/-20. 

2.  /-x,    i/-a;^    v/-a3,     ^-a^"*,     V{-a)^     ■/(-a)^, 

■«/5.  \/  —  rt. 

Q      o2       ;3      i4      iS      ,;9      -;i5      ,*1G       ,-17       ,'18       ,'4'>      ,'ln+l      -/te+2      „'4n+3 

4.  cj-bj,  jVx.Ji/y,  5j,  j-^i/5,  ji/~a,ji/-a^,  J-i/a.-i/  -a. 

6.  V-r,  V-p,  i/-j\  l/-i^  -/-i^",  i/-p\ 

^_6        a/-6         |/G  i/a         |/     a  1 


6. 


V 3  '      V-3'      i/-3'      i/-/.'      -i/-6'     ■/-!' 
a  a^  \/{—ax)        —V-1  a^ 


y/ —a         v/— a^  y'  —  x  V  —  a    '     f/ — 


a^' 


256  APPENDIX. 


J_     J_     J_      -JL     2_     _J_     ^iL     ^ 

^'   j  '   P '  F'     i  '   j' '  i*"+''  i^"^''  i^"-'' 


2 


8.    A/(a-Z,)V(/-'-a),    a/(3*-4?/)V(47/-3x),   (3  +  5;-)(7  +  4j), 
tS-9i)(8-7;),  (7-jV5)(7+jVlO),  (v'3~iV6)(V2-jVU), 

(/a4-il/6)(l/a->^/c)  (a+/i/'j  («-/;;'),  («;'  +  &)(«y-&), 
(  v'"+y|/^)(/a-iV'6),   {(iVl)+GJy/x){a]/b  —  (jVx), 
l/(l+yjV(l-yj,    V(3+4i)V(3-4i), 
v/(12+5;jVil2-5;-),   (l+i)3,    (ya-jV6)3,   (5-2JV6)", 
{a  +  hjf+[a-hjy\   ^a  +  bjy^-ia-bjr-,    (a+l>jy  +  {aj-by, 
{^/(4  +  3Jj+v/(4-3i)}^   {^(3-4;-)-v/(3  +  4i)}^ 

[i{y(30-6N/5)-l-i/5}  +  i;{N/15+  ^/3+ ./(10-2|/5)}]» 
tor  all  positive   integral  values  of  n. 

4  64  21  5  1-20;V5" 

1+JV3'    1  -iV7'    414- 3;- V6'    V2+Jv3'     7-2;V5  ' 

i+yv3'  1-/    i+i'    i-i'  (i+j)3'    i-y'  u;-^/ 

a+jy/x    JV(i+V-b     a  —  hj     (i-j-jv'(l-£2) 
a—[y/x     \'  -<t-j\/b     aj-{- 1)     a-jV\^  1  -  x  - )' 


9. 


APPENDIX.  257 

V{x-y)-V(>/—x)        1  1        l+i     1—,; 

v{y^x)+v[x~^y  1+7+ r^''  i-j+i-h/ 

1,1  1  1        x-^yj    ^'  —  vj 


(l+^-)2-^(l-j)3'    (l+ij_4      (i__^-)4'    «_|_j;-r^,  _ij 

a;  +  yj     a;  —  yj     Vx  +JV1J     Vjz+jV^ 
a -\-hj     a  —hj     Vx—j^/ij     ^y—jVx' 

y(l+a)+7V(l-a)  _  y(l_--«.) -|-^-y(l+^) 

y(i+«)  -Ml  -  a)    y(i  -rtO  -jV(i  +rt)' 

10.  ^/(3+4y)+|/(3-4i),     ^(3  +  4i)-A/(3-4y), 

v^(4  +  3i)  +  v/(4-3i),    |/(l+2;V6)±v/(l-2;V6) 
V(5+2;V6)±a/(5-2jV6), 
v/(2,/15  +  3q;)±v/{2v'15-30j), 
A/{x/3+iv/105)+  A/(i/3-ii/105), 

11.  Prove  that  both  i(-l+j|/3)  and  i(-l  -jV3)  satisfy  the 

a;^-l 
equation  .    _^  =0, 

that  {x-\-icy  -^-w-zY  =  x^  ^-y^  -\-z^  +  ^{x  +  wy){y\-  wz){z-{-n-x) 
and  that  (a^  +  ^ +«)(«+ it;^-}- it; ^z) (a; +m;^ ?/+«;«)  = 
x3-j-?y3_}.23  —  'dxyz,  vo.  which  10  represents  either  of  the  pre- 
ceding complex  quantities. 
Hence,  prove  that 

(i)   {2a-&-c+(5-c];V3}3  =  {2&-c-a+(c-«)j/V3)}3=: 
{2c-a-6+(a-%V3}3; 

(ii)  u^-\-v^-\-w^ —Quvii:=-{a^ +h''^-\-c^ —  ^ahc)x 

(a;3 4-7/3  ^gS  -3a;!/2),  if  u  =  ax-{-by+cz,    v  =  a^-f  &2'+c3«j 
u;  =  az  +  bx-\-cy,  or  if  w  =  rtx-f-c?/+/;2, 
t;  =  ca;+%  +  rt2,     w  =  hx-\-ay-\-cz. 


258 


APPENDIX. 


12.  Prove  that  i  U/^  +  t+j  v'  (10  — 2 -/S)}  satisfies  the  equation 

x^  +  1 
x+l 

Writing  v  for  tlie  preceding  complex  quantity,  prove  that 
(7+'r+?<'2_|_:3„,3)(7_ti.4_„;3„3^(.2)  =  71^ 

and  {x  ■}- >/ -\-z){x+w^y  —  ir3z){x  - iv^jj  —  icz)(x  -  icz-{-rv*^z) 
(x-^w^ij  +  ic-z)  =  x'^  +1/'''  +z''  -  5x'''yz+5xy^z^. 
Prove  that  {4«  +  (/;-,-)(,/5  -  1)  +  (6+c)jV(10  +  2  \/5)}5  = 

{[{a  +  b){-l+jVii/5  +  2)}  +  ia-b){V5+jy'(V5-2)}] 
X  y5-4c}^ 


Section  III. — Pure  Quadratics. 


Examples. 

J       x  +  S(a-b)  _  a{Zx  +  %a  -lb) 
x-S{a-b)  ~ b{3x^a+\9b)' 

m       p  m-\-n    X''^^ . 

Apply,  if  —  =  — ,  .".   = 5 

^••^  •"        n         q  m  —  np  —  q 

X  3.r(ff  +  ^>)  +  9«2_i4,,?;  +  9J2 

'*•  B[a-h)^       3a;(a-6)4-9(a2Tr^ 

Dividing  the  denominators  by  3(a  —  b) 

.-.  a;2  =  9a2_  14^6  +  9^3 

'a;_2ff-f-4M  2     5a;-9r7+3& 


x+4a-26/         bx  +  'da-Sib 

m      p  n  —  m     q—p 

Apply,  if  —  =  — ,  .'.  — - —  =  — —  ,  and  factor  the  numerator 

(a;+4a-2i)3-(a;-2fl  +  46)2, 


APPENDIX.  25i) 

mx-\-aA-h){a-h)        \^a-h)_ 
(a;+4«-2i)3       -5^+ 3a- 95* 

x+a-\-b         .T4-'4a-26       ^a-'h) 

:. 7^  =  T- — ,—7^ Ki  =  "a ^.   oy    taking   differ- 

••  x-{-^a-^h     5a;+3a-9Z;     4ai-a-lV     ^  ° 

ence  of  numerators  and  difference  of  denominators.     To  the  first 
and  third  of  these  fractions,  apply  if 

m       p  m  p  ^ 

n    ~  q  '    "  **  —  m~~ q—p 

x-\-a-\-h  ^a  —  b) 

''•  S{a  -  b)^'4x--4:a  —  A¥ 

.-.  x2^x^i(^a  +  b)"}+d{a-by^}. 
l/{Sx-'-l)-\-V{B-x'- )  _  a_ 

3a;- -1     a-{-b' 


4 


3_a;2  -a-b' 

•       3a;2-l     (a  +  b)^ 
"       3-x'   ~{a-by' 

S{a  +  h)^  +{a-b)^     a^^ab  +  b^ 
•■•      ^^  =  '(^j^Ij)2  ^  s^a-by-^a^  -ab  +  b^' 

4.  7nV{l+x)-nV{l-x)=V{m^+n'i)  (1) 

Square  both  members  and  reduce 

.-.  {m^-n^}x-'imnV{l—x-)=0.  (2) 

Transfer  the  radical  term  and  square  both  members, 

.-.      (m3_^3)3a;3=4^2„2(l_a;2)  (3) 

.-.    {m^-{-n^)^x"=4:m^'fi^  (4) 

The  above  follows  the  usual  mode  ol  solving  equations  involv- 
ing radicals,  viz.,  make  a  radical  term  the  right-hand  member 
gathering  all  the  other  terms  into  the  left-hand  member,  square  each 


260  APPENDIX. 

member,  repeat,  if  necessary,  until  all  radicals  are  rationalized. 
This  method  is  convenient  but  it  does  not  explain  the  difficult^ 

that  only  one  of  the  values  of  x  in  (4)  satisfies  (1)  viz.  ~jT~Va 

—  2m7i 
The  otber  value,      „  ,     ^  satisfies  the  equation 

w  \/(l  4-a;) +?i  a/(1 -ic)  =  l/(m2  + /,  3). 

The  exi")lauation  is  simple.  Squaring  both  members  of  (1)  is 
reully  equivalent  to  substituting  for  (1)  the  conjoint  equation 

{wV(l+x)  +  /.V(l-a;)- V(/?t2+n2)|^0  (5) 

which  reduces  to  (2)  above. 

Treating  (5)  or  (2)  by  transferring  and  squaring  is  equivalent 
to  substituting  for  it,  the  equation 

{m  V  {'i- -i- x)  -  71  V  {1  -  X)  -  V{>ii'^+n")^-  X 

{7«|/(1  +a;)+»  \/{l  -x)-  v/(m3  +^2)}  x 

{mA/(l  +  x)+W|/(l-a;)+i/'(m3+w3)}  =0  (6) 

^s'hich  reduces  to 

Urn^  -n^)x-%nny/{l-x^)]  {m''  -n^).c-\-2mnVil-x-}  =0       (7) 
which  further  reduces  to  (3) 

Thus  the  whole  process  of  solving  (1)  is  equivalent  to  reducing 
it  to  an  equation  of  the  type  ^  =  0  and  then  multiplying  the 
member  A  by  rationalizing  factors.  Thus  instead  of  solving  (1) 
we  freally  solve  (6),  i.e.,  a  conjoint  equation  equivalent  to  four 
disjunctive  equations.  (See  page  140,  Art  xl  )  Now  the  values 
given  in  (4)  will  satisfy  (6),  the  positive  value  making  the  first 
factor  vanish,  the  negative  value  making  the' third  factor  vanish, 
while  no  values  can  be  found  that  will  make  either  the  second  or 
the  fourth  factor  vanish. 


APPENDIX. 


261 


Hence,  if  one  of  such  a  set  of  disjunctive  equations  is  proposed 
for  solutiou,  the  conjoint  equation  must  be  solved,  and  if  there  be 
a  vakie  of  x  which  satisfies  the  particular  equation  proposed, 
that  value  must  be  retained  and  the  others  rejected. 

(This  process  is  the  opposite  to  that  given  in  Arts.  XL.  and 
XLV.  :  there  a  conjoint  equation  is  solved  by  resolving  it  into  its 
equivalent  disjunctive  equations.  The  two  processes  are  related 
somewhat  as  involution  and  evolution  aie). 

Further,  it  should  be  noticed  that  just  as  there  are  four  factors 
in  (6)  while  there  are  only  two  values  in  (4),  it  will  in  general  be 
possible  to  form  more  disjunctive  equations  than  there  are  values 
of  a;  that  satisfy  the  conjoint  equation,  and  consequently  it  will 
be  possible  to  select  disjunctive  equations  that  are  not  satisfied  by 
any  value  of  x,  or,  in  other  words,  whose  solution  is  impossible. 

This  will  perhaps  be  better  understood  by  considering  the  fol- 
lowing problem. 

Find  a  number  such  that  if  it  be  increased  by  4  and  also  dimin- 
ished by  4  the  difference  of  the  square-roots  of  the  results  shall 
be  4. 

Keduced  to  an  equation  this  is 

^(a;+4)--/{a;-4)  =  4  (8) 

Eationalizing  this'becomes 

{4--/(a;-H4)+v/(a:-4)}{4-/(a;+4)-i/(x-4)}x 
{4+v/(a;+4)  +  i/(a;-4)}{4  +  i/(a;+4)-V(;c-4)}=0     (9), 
which  reduces  to 

{24-8v/(a;+4)}{24+8i/(a;-f4)}  =0 
ie.  9— (a;-+-4)  =  0,  oric  =  6. 

Now  »=  5  satisfies  (9^  because  it  makes  the  factor 
4-|/(a;+4)-A/(a;-4) 
vanish  and  it  is  the  only  finite  value  of  x  that  does  satisfy  (9),  or, 
in  other  words,  there  are  no  values  of  x  which  will  make  any  of. 
the  factors 


262  APPENDIX. 

*  4-i/(a;+4)  +  |/(a;-4),  4+  V{x^^)+  \/{x-4.), 
or  4  +  A/(a;  +  4) -V  (^  -  4) 

vanish.     There  is,  therefore,  no  number  that  will  satisfy  the  con- 
ditions of  the  problem. 

[It  will  be  found  that  as  x  increases,  ■j/(.'c+4) -|/(a;— 4) 
decreases,  hence  as  4  is  the  least  value  that  can  be  given  to  x 
without  involving  the  square-root  of  a  negative,  the  greatest  real 
value  of  -i/(x+4)  —  -/(a; -4)  is  -j/S  which  is  less  than  4.  We  see 
by  this  that  our  method  of  solution  fails  for  (8)  simply  because  (8) 
is  impossible] . 

5.  V.{{a  +  x){b+x)}-i/{{a-x){b-x)}  = 
^{{a-x)(b+x)}  -  V {{a-hx){b-x)}  (1) 

Collecting  the  terms  involving  i/{a-{-x)  and  i/(a-x)  respec- 
tively the  equation  becomes 

{l/(a  +  x)-y/{a-x)}{^{h  +  x)-i-V{b-x)}=0  (2) 
This  is  satisfied  if  either 

l/(a+x)-i/ia-x)  =  0  (3) 

or   Vib  +  x)+\/{b-x)  =  0  (4) 

The  rational  form  of  (3)  is  (a  +  x)  —  {a-x)  =  Q  which  is  satisfied 
by  a;  =  0  and  this  also  satisfies  (3). 

The  rational  form  of  (4)  is  {b  +  x)-(b—x)=0  which  requires 
x=0,  but  this  does  not  satisfy  (4).  Hence  the  second  factor  of 
the  left-hand  member  of  (2)  cannot  vanish. 

Therefore  the  only  solution  of  (2)  and  /.  of  (1)  is  a;  =  0,  derived 
from  (3). 

6.  f{a+x)-]-^ia-x)  =  f{2a) 

Cube  by  the  formula  (u+v)^  =  u^ -\-v^ -\-Snv(u-{-v) 

:.    {a+x)-h{a-x)  +  3f'{2a{az-x^)}='2a. 
.-.    2a{a^-x-^)  =  0, 
:.    x=±a. 

Both  these  values  belong  to  the  proposed  equation. 


A1>1'ENDIX.  i 

The  rationaliziiog  factors  of 

f/^a+a-)-{-^{a-x)-f/{2a)  =  0 
are  -^{a  +  x)-\-o>]V{" -x)-(^)^-^{2a), 
and  i/(a4-a;)  i-o^-f/{a  -x)  -(^^y{2a).    See  page  257. 

The  remarks  on  Ex.  4,  will  apply  wwfaiis  mutandis  io  equation  •; 
of  this  type. 

^'      f/(a+x)^  -f{a2-x^-)^+^{a-xy     "'  ^^' 

Assume  -^{ft+x)  =  w  and  ]^(a— a;)  =  v 
.'.  u^  +v^  =  2a  and  u^  —  v^  =  2x, 

and  .'.     o   ,  ^-0  =  —-  (2) 


Also  (1)  becomes 

ll^  +  W  +  V^ 


=  e  (3) 


m2  —UV+V^ 

U  —  V 

Multiply  both  members  by   ^— ^ 

^3 — y3         u  —  v         .      ,^^    X  u—v 

Again  adding  and  subtracting  denominators  and  numerators 
in  (3) 

u^  -\-\v^     c+1 
uv      ~  c  —  1 

Adding  and  subtracting  2  (denominators)  and  numerators  in  this 

ti^-2uv4-v^      3-c  /M-r\  2      B-e 

or    ' '     "~ 


M2_j_2Mu  +  t;3     3c— 1'         \u-\-vl        3c- 1 

a-2  3-c 

.'.  substituting  by  (4),  -^  =  c^q^_  i* 


|8-r 


264  APPENDIX. 

8.    ^V{x  +  a)  +  V{x  -a)}^  {r{x+a)  -  \/{x--a)]^1c  {\) 
Assume  u=\/(x-\-a)  and  v='^y{x--a),  and  (1)  becomes 

(?i+?.-)3(M-r)  =  26- or  (?t+i-)2(?i2_r2)  =  2c  (2) 

Also  ?t4  -r4  =  2a             or  («3  +  y2)('u3  -  ^2)  =  2a  ■  (3) 

and  u'^-\-v'^  =  2x.  (4) 

From  (2)  and  (3),  {u -  v) 3 (nS  _  t-2 )  =  4^  _  2c  •  (5) 

.-.   (2)X(5),   (l(.2_t-2)3(^2_^3)2or  (M3-t;2)4  =  4c(2«-c)  [<6) 

Also  (3) 3-1- (6), 

|(M2  +  ,.3)3_^(jt2_^,3)2|(ji2_^3)2=4(«2_|.2aC-c2) 

or  (?(44.^.4)(^i2_^,3j2  =  2(a3  +  2ac-c2) 
Substituting  by  (4)  and  (6) 

^a;|/(2ac-c2)  =  «2_|_2ac-c3. 

Exercise. 

1.  (a;+a  +  6)(x- a  +  &)  +  {x  +  a -  h){x -a-b)  =  0. 

2.  {a  +  bx){h  —  ax)  +  (/; -(-oa;)(c—  hx)  +  {G+ax){a—cx)  =  0. 

3.  {a-^bx){ax—h)-\-[b-{-cx)[bx  —  c)-\-[c  +  ax){cx  —  a) 

=  ^{a^-{-h2+c^). 

4.  (a+a;)(i-a•)-4-(l-fa;«)(l-^*a:)  =  (a  +  ^))(l+•^•3). 

o.  {a-rx){b-{-x){c  —  x)-{-[a-\-x){b  —  x){c+x)  +  {a—x){b+x){c  +  v) 

+  {a-x){b-x){c+x}-\-{a-x){b-^  x)  (c  -x)-\- 

[a-\-x)(b  — x){c  — x)  =  5abc. 
J.  {n  +  x){b  +  x){c  +  x)-\-  {a  +x){b  +  x){G-x)  -^{a+x){b  -  x){c  +  a;) 

+  [a—:x){b  +  x){c-\-x)-\-{a+x){b — x){c — x)  +  {a — x)[b  +  x){c — x) 

-t-[a-x){b  —  x){C'\-x)-\-{a—x){b  —  x){e-x)  =  Qx^ 

7.  (a4-5&+a;)(5a+6+a;)  =  3(a-|-i+a;)3. 

8.  {a  +  llb+x){lla+b-\-x)  =  %a+h+xy. 

9.  (9a-7i+3a;)(9i-7a-f3a;)  =  (3a+36+a;)^ 


APPENDIX.  liuj 

ab                  cd  x  —  a     x-\-n 

a^  —  b^x"     c-^  —  d^z^  x  +  1     x—1 

V 

a  +  x     x+b  ax+b     cx-\-d 

a  —  x~x  —  b'  '    a-{-bx~  c+dx     . 

a—x      1  —  bx  a — x       -k—x 

14.    z =  -, .  •               15. 


22. 


25. 

26. 


1  —  ax      b  —  x'  '    1  —  ax     1-bx 

x+a  +  26     h-%(-\-1x  a  +  46+a;     'db-a+x 

^^'    a;  +  a-2ft^6  +  2a-2.c'  .               a-4.b^x^  2>b  +  ^'^x' 

x-\-6a4-b      x  —  a-\-b  a—lbA-x     a+5b-\-x 

^°-    x-3a-{-b~a-x  +  Sb'  '^'    Ta-b-x     5a-tb-\-x 

3a~b—x     5b-3a+x  Sa-2b  +  Sx       x-a+2b 

20.    "771-—  =  .— oT^..  21 


32. 


a  —  '6b-bx     oa  —  'db+.r'       ^  '       a  —  'Io  +  x        'dx  —  'da-j-'Ab' 

3a-2b+Bx      x-7a  +  8h^ 
a  —  '2>b-\-x    ~  3a;— 5a  4-46 


5a-6b  +  x     da  —  5b-\-3x  a  +  b~x       S(a-h+x) 

23     ! — 24     =  ^^ ' 

a-^x  a+b+x    '        '    Sa—b  —  Sx      a  —  ^b-^-x 

la  +  b  —  x       3{a  —  h+x) 


5a-\-db  —  'dx     a—llb  +  x 

5a-b  +  x        2(2g— Z)+a;) 
2(a  +  26-a;)'^^ll/>-a;  * 


7a  —  b-\-x     a{a-{-5h-[-x)  x  +  a  —  h     a(x  +  a-{-5by 

27-    Tb-a-{-x^b{5a  +  b+xy  x-a+b"^ b{x  +  5a+'Fj' 

.5a-Sb+x\  2     7a-9b  +  Sx 
29. 

30. 


5b-da  +  xl        76 -9a  4- 3a; 
la  +  5b+x\  "-      a+17i+a; 


\5a  +  b-\-xj         \la+b-\-x 

na-h-\-x-  3  ^  lla  +  b-x 
31-     [rjJZ-^x]         rfb  +  a-x 

lla+b-x     a^(a+nb  +  x) 


a+llb—x     62(17a+6-i-aJ) 


son  Arpr.Nnn:. 


33. 


{5x  +  'Sa-llb){:v-a  +  nb}     bx+la  —  bUb 


g^     (1  +  Sx+]5x^){x^  +  3a;+5)  _  ^ 
(l  +  2.c+3a;2)(a;2-|-2^  +  3)~  4" 


35. 

8G. 

37 

38. 

39. 

40. 


y(l+^2)_j_^(l_a;2)         ,, 

V'(l+.f-^)-l/(l-xf)~  ^ 
lf(l+a;2)  +  ^(l-a;3)       q. 

4/(l+.,.2)^_4/(l_a.2)         ^ 

t/(i+«2)-V(i-a;-)~ir 
V(l4-x^)+!y(a;3-l)  _  jt. 

'y(i+x^-)-'^{x-'-i)-  b" 

y(a;2+l)+y(a;2-l)       a 


41.  -,/(4rt  +  6-4.i;)-2V(.r  +  Z/--2x)=v''^. 

42.  -,/(3n-2/>+2a;) - %/ (3a-26- 2.^)  =  2 \/a. 

43.  A/(2a-/)4-2.r)-  A/(10a-96-6a;)  =  4v/(a-6). 

44.  i/{Sa-4:b  +  5x)  +  ^/(x--a)  =  2V(x+a). 

45.  V(3a-46  +  5.ri  +  |/(a;-fl)  =  2v'(2.«-2&). 

46.  i/(5x  --  Sa  -{-ib)  +  -^/{5x-Sa  -  Ab)  =  2V (x  -{-a): 

47.  i/(2a+/^  +  2a;)  +  i/(10^/+9/>-Ga;)=2v/(2a  +  6-2a;). 

48.  2'v/(2r/+6+2.r)  +  ,/(10rt+6-6«)-  N/(10ff+96-6.'c). 

49.  y'{2a-13b+Ux)+V {S{b -2a-{-2x)}  =fiV {2.a-b  +  2x) 
60.  i/{3(7a+6+a;)-N/(a+76-x)  =  2i/(7a  +  6-a:). 

51.  V{{a+x){x+b)}  +  V{{a-x){x-b)}=2i/(ax). 

62.  ^{(a  +  a;)(a;-l-6)}-i/{(a-a:)(a;-6)[  =  2|/'(6a;). 

68.  ^(aaj+ic^j-  -t/(ax-a;2)=  ^{2ax-a^). 


APPENDIX.  207 

1  1  _2_ 

x+V{ax)     a  +  y^(«.c)     x  —  a 

56.    7- — .-{- 77 — r  = ■• 

a  —  V  [ax)     X  —  -i/{ax)         a 

l/{{a+x){x-^b)}+  V{{a-x){x-b)}         U 
''^'     V{{a-{-x){x  +  b)}  -  V{{a-x)i:x-b)}--<b  ' 

tSa-U  +  'i.e     {^/a-^^/(2ft-26)}" 

58.  ^3,f_2i-2.<;~  26-^* 

59.  ^{a-{-x)  +  f{a-x)  =  2^a. 

62.  ir(H-x)--'+iril  -x)^  =  ^h^{l-x-'). 

63.  i^'(3+*-)  +  lf^3-.c)  =  ^6. 

6i.  -,K(i+^)='+r(i--^-)'=5{i^(;i+^)+f/(i--*^)i'- 

65.  f{U+x)^  -f^{196-x-^)  +  f'{U-xy^  ^7. 

66.  {if(9+x-)+i^(9-a;)}if{81-a;3)  =  12. 

67.  {f{u-^.ir-f^{u-xy-}{^{u+x)-f^{u-x)}  =  ie. 

68.  {irt57+.«)^+#/(57-x)3}{if(57-a;)  +  iK(57+a;)}  =  100. 

69.  5{t/(-il+'-^)  +  V(41--^-)}'=8{^/(41  +  '^-)+v/(41-^)}- 

70.  {t/{x+5)+t/i'^-5)}m/(x+5)-t/{x-5)}=± 

71.  {V(-*^+i)+M^-i)}{-/(-«+i)+v/(-«-i)}  = 

26{V(^+l)-t/(a;-l)}. 

72.  r\^^  +  r\^^  =  a.     [y+y-'=a]. 

73.  2{f'(l+a;)3+^(l-x3)}  =  (c2+l){ir(l^''^)  +  #'(l-^)}^ 


208  APPENDIX. 

74.  f^(a+x)  +  ^{a-x)  =  fc. 

75.  {^(a  +  x)  +f'(a  -  x)}f'{a"--x^-)  =c. 

76.  ^ia+x}-=^{rr-  -x2)^^(a~x)-  =  fcK 

77.  {f'{a  +  xy'-f/{a-xy}{f{a+.v)-f\a-x)}=c. 

78.  {f/[a+xy-+f{a-xy-}{f{a+x)  +  fia-x)}=c. 

79.  [a-^x)-^{a ~x) - ia-x)f{a+x)  =  c{f/{a  +  x)~f/(a-x)}. 

80.  (a  +  x)  f/ia+x)  -  {a-x)f{a-x)  -  e{^(a  +  x)  -  |f  (a  - u;)}. 

81.  {^ia-^xy^-f{a^~x^')  +  f[a-xr-}^-  = 
c{f'{a+x)  +  f/{'i-x)}. 

82.  {V(«+^)  +  M«-^)} '=(«  +  !){  \/(«4-aj)+A/(«-.c)}. 


Sectioi?  IV. — Quadratic  Equations  and  Equations  that 

CAN    BE    RESOLVED    AS    QUADRATICS. 


Examples. 
1.  x^  +  (ah^iy  =  {'i^+b^)ix^+l)+2{a^-b^)x-{-l, 
.-.    x^+aH^'  =  {a^  +  b^yx-  +  2{a^-b^)x+[a-by 
:.    X*  +  2abx^-  +an^-  ={a-{-by-x-  +2{a-2  -b^)x+(a-h)^ 
.-.    x^.^ab=±{{a  +  b)x+{a-b)}, 
or  x"+{a-\-b)x  +  ab-±{a  —  h), 
:.    x^^{a+byx-{-^ia  +  by  =  ^a-by ±{a-b), 
.-.    x^i{a-\-b)=W{['^~byziz^{^a-b)l. 


APPENDrX. 


269 


{a-x)^l/(a-x)+(x-b)W(x-b)__^_f^ 
2k.  ' 


{a  -  x)  i/{a  ~x)-\-)x-b)^^{x~b) 

Writer  — 6  in  the  form  {a—x)-r{x~h)  and  multiply  by  the 
denominator  of  the  left-hand  member, 

.-.    {a~x)^yia-x)  +  {x-by  \/{x-b  = 

{a - x)  V(« - ^) -^  {ci-x)(x-b){^{a-x)+  ^{x - b) }  -L 
(x~hyW{x-h), 
:.    (a~x)(x-b){V{a-x)+V{x^b)\  =  0, 
:.    {a~x)  =  0,  or  x-b  =  0, 
or    \/{a  —  x)-\-V(x  —  b)=0, 

x^  =rt,     .fo  =  ^• 
The  equation  V{a-~x) -\-V[x -b)=0  has    no   solution   for  the 
Bum  of  two  positive  square-roots,  cannot  vanish. 

The  solution  x  =  ^[<i-\-b))  belongs  to  the  equation 
V{a  -  X)  —  V{x  —  b)  =  0. 

ax+b     mx—n 

3.      T—~  = 

bx  -{-  ci     nx  —  m 

Add  and  subtract  Numerators  and  Denominatorg 

{a+h)(x  +  l)  _  {m-\-n){x-l) 
{a  -  b){x  —  l)~  {m-n){^x+V) 

^_l/         {a+h){m-n)  "" 

•'•    ^1  =  s"Zi'   ^-3  -  .s4-l* 

b-\-x         a  —x 
Square  both  members,  subtract  4  and  extract  the  square-root. 


270 


APPENDIX. 


/fg  —X 


a  —  x 


"    b^x 


=  63 


1  -  ^3 


•••    x=h{{a~b)  +  {a+b)  j-^^3^ 

Or  thus,  cube  both  members, 

a—x  b+x       „ 

.-.    -    --  +  3c+ =  c3 

b-^-x  a—x 


{a-xr-  +  {b+xY 


=  C3-3C 


••      ^a-x){b±x) 

({b+x)-{a-x)Y     g^-3c-2     (£+1)2  (c- 2) 


2a;- (g- 6)     c  +  1        |c-2 
a +"6       ~  c  -  i      Nc  +  2' 

Wrt-a;)-V(6-a:)      ^/{{a-x){b-x)] 


■  + 
(Prove  that  ^~  3 


V(<i-x)-j-V(6-a;) 

{v/(a-.r)-V(6-a;)}g      v{(a-3;)(5-a;)} 
{a—b)  -  c  ' 

{ V(a  —x)-V{b-x)]  -     a-h 
V(a-a;)|-V(i-:c))  3_      a-6 


.   Eationalize  Deuom. 


(-1) 


"  iV(a-a;)H-V{Z»-a;))        a-i+4c' 

V{(a-a;)(^-a;)}  _     I  _a-b^ 
c  ~  ^c7^b  +  Jc' 

Also  from  (A), 

a+b  —  2x  a  —  b-\-2c 

V{{a-x){b^^'x)}  "^       c        ' 


(^) 


APPENDIX.  271 

Multiply  (B)  and  (C)  member  by  member 


;.{„+,_(._, .+2.)  4-- 


h 


■     6.  a;4-4=-5^^^;a;e.-2.7;4-  5a;3-12  =  0. 
aj-  —  2  ' 

Find  the  rational  linear  factors  of  the  left-hand  member  by  the 
method  of  Art.  XXVII.,  page  90. 

.-.    (a:-2)(x-f2)(^*  +  2a;3+3)=0, 
.-.    x-2  =  0,  ora;  +  2  =  0,  or  x'^-\-2x^+3  =  0. 
The  last  of  these  equations  may  be  solved  as  a  quadratic  giving 
a;2  =  -l±2A/— 2,   :.  x=±l±V-2, 
,'.  x^=2,  x^  =  -2,  x^  =  l  +  V-2,  x^  =  l-\^-% 
a;5=-l  +  V-2,  a;6=-l-V-2. 

I^.B. — In  solving  numerical  equations  of  the  higher  orders,  the 
rational  linear  factors  should  always  be  found  and  separated  as  dis- 
junctive equatiom,  before  other  methods  of  reduction  are  applied. 
Such  separation  may  always  be  effected  by  the  methods  of  Arts. 
XXVIl.  to  XXX.,  and  unless  it  is  done  the  application  of  the 
higher  methods  may  actually  fail.  Thus,  if  it  be  attempted  to 
solve  as  a  cubic  the  equation, 

a;3-9a;-10  =  0 

the  result  is  aj=  {5+ V- 2}  +{6-V-2}  ,  which  can  be  reduced 
only  by  trial.  The  left-hand  member  can  however  be  easily 
factored  by  the  method  of  Art.  XXVII..  and  the  equation  reduces 

to 

{x+2){x^-2x-5)  =  0, 

which  gives  a;  =  2  or  l±y'6. 


272  APPENDIX. 

7.  {x-2y  -x'-\-2'  =0. 

Factor,  (See  No.  20,  p.  89),  rejecting  constant  factors, 

.'.  x=-0,  ov  x-2=^0,  ov  x^--2x  +  4:  =  0.  * 

The  last  equation  gives  a;  =  Izt  \/  —.3. 

Exercise. 

'  Solve  the  following  equations  : 
1.   {x  +  a  +  b)^=x^+a^-hb3.  2.  (x  +  a  +  b)^  =.r:^ +  a'' +b». 

3.  {a—b)x^  +  {b-x)a3  +  {x-a)h^  =  0. 

4.  {a-b)x^+{x-b)a^+{x+a)h^  =  2abx. 

5.  (x-a)^  +  {a-b)^  +{b-x)^  =0, 

6.  (x-ay  -\-{a-by+{b-x)'  =0. 

7.  {a^-b)x'>^+{x^-a)b^-\-{b»-xyi*  =  abx{a^b^X'  -1). 

8.  {x-a){x-b){a-b)  +  {x-b){x-c){b-c)  + 
(x — c){x^  a){c  —  a)  —  0. 

x^-1  x^^-1 

^^'  x^-1                                              x^  —  1 

13.  a;'' +5a;3-l6x^4-20,x-16  =  0.  (See  Art.  XXII.) 

14.  a;4  -  3.6-3 +  5.C2  +  6.C  + 4  =  0. 

15.  (x-a)4+a;4  +  «4=0.  16.  2.r--' =  (.r-G)». 

17.  x{x-2y{x+2)  =  2.                 18.  (4;.-3-17)x+12  =  0. 

19.  a;*  +  (ai+l)=  =  («"-+^')(^'  +  l)  +  2(«'-&^)-i^  +  l. 

20.  a;2(x-169)2+17x  =  a;3-3540. 


APi-ENDIX.  273 

21.  6x(a;2  +  l)3  +  (2^'-+5)3  =  150x+l. 

22.  2x{x-l)^+^  =  [x  +  l)^.         23.    x^  =  12x+5. 

24.     5x^  =  12x^+1.  25.    (x+4)3  =  3(2:«-l)=. 

20.  V(.c2+w2)-fv[(7i-2;)2+m2}=V{(a;-4«)^  +  (i«'^3-wO}- 


27. 


31. 


33. 


34. 


35. 


86. 


(x  +  iy         .    m  28.    1-^+1)^ ''J^L 


(.6-"  +  lj^.t;-l)~       n  x{x^+l)       n 

(^3  + 1)0:3  +  1)       m  .  (:,3  +  l)(,;."+l)       m 

-^-       {x  +  l){x^  +  l)~  W  "^^^    {x^-l\x-^-l)      n 


x2(u;+l)       ~w*  ''"■    x(x'-^  +  l)(x--l;-~  ?t 

x{x-\-l)'  n{n—m) 


(a;3  +  l)(:c-l)2-2»i(2m-?2) 

(a;3  +  l)2  4m2 

.<;-  -  1)-      7/i-  —  ?4'' 

t>-l)(a;2+l)2     2(»?-  -n)3^ 
(j;3-l)(a;+lj'"~       '"^'^ 

a;^  - 1  2h?. 


(.c+l)(a;^-  l)'"2m- 


71 


^x^-l){x  +  l)^  _  in+n  (x+l){x^  +  l)  ^ m  +  n 

39.      x^  =7 ••  •  40.    x^  =  7 

bx-a  hx-a 

ax-b  ,^       ,     ax3  +  ?»ar  +  c 

41.    ^^=i^i::^-  42.  a.^=^:;-j^qr^- 

43.     a;3  =  (a;-l)2(x2+l).  44.    aS^jS  =  («_a;)2(«2  _^2). 

45.  x^  =  {x-ay{x^--l). 

46.  aA/(2;2  +  l)-a;/(a;2  +  r72W^, 


274  APPENDIX, 

47.  l^(a3-f-.,.-3)4-^(,(3_a;3)^^(a6_a.6)3. 

48.  tn{x+7n~7i){x  —  m  +  ln)^  =n{x  —  7n+n){x-\-lm.—n)^. 

49.  vi^{x  +  m+nn){x-m-  5n)^  =u^{x+nm+7i){x-  5m+n)^. 

50.  )7i^{.r-i-m  +  n7i){x-7n  +  l7i)^  ^n^{x  +  nm+7i]{x+lin  -  ii]^. 

V{x-a)  +  i/{x-b)  _     Yx-a 
V{x-a)--i/{x-b)~^x-b 

V{x  —  a)  +  V(x  —  b)        \a-x 


52. 
53. 


V{x-a)-V{x-b) 

]a  —  X         \b4-x  \a  —  x,\h-x 

\r^~-J =c.  54.        , +V =  ( 

'vj'^+x'     ^u  —  x  ^b  —  x      ^a  —  x 


oyCi—x      o/h+x  3  /  la  —  x\  ^         lb—x\" 


b -\- X         a—x       '  \b —xj       ^    \a  —  x! 

^,a-x         b  +  x  4/«-a;     4/i-a; 

o+u;         a  —  a;  b  —  x         a  —  x 

.,a  —  x      .   b  +  x  riA—x      .,b—x 

59.    y.-— +^ — =c.       GO.  V^ — -e^ ^c. 

b+x         a  —  x  b  —  x         a  —  x 

61.       V7n— +v'^-=c.  G2.     V, -V =  c. 

w+;<c         ax  b  —  x         a—x 

^^        ^{a-xY-^sf(b-xY  __ 
^{a-x)-\r^{b-x) 

^/(a-xy^+i/{b-x)^  _ 
^^'      {^^a-x)+y{b-x)}^-'' 

V{a-x)^+l/{h-x)^ 
-\/{a—x)  —  V{b—x)    ~ 

_       W{a-x)+^/(b-x)y 
'^^'  ^/(^a-x)-V{b-x)     -"' 

y^{a-xy+V{x-by 
^''         V{a-x)+/{x-b)    -"' 


APPENDIX.  27/) 


68. 


70. 


it^. 


76. 


■x/{a-x)^  +  V{x  +  h)5  _  ((?  +  />) 2       _ 

~~V{a  -x)+V{x  +  b}     ~4:V{{a  —  x)  {x + bj}' 

x-+(a-x^)V(a-x~) 
a;+V(a— a;^) 

72.  ^  ,    77-0-—- o-x ''  ^  ^-^"^i^' --x^). 

x+V{a^  —x^) 

73.  if(«-x)-^-ir{(«-.T)(.r-/>)}  +  |3/(a:+6)2  =  ^/(a2-a5+i') 
74        b^/{^-x)  +  aV{x-b) 


75       ^\/(fl-a;)  +  ^l/(a;-/^)   _ 
l/(a-aj)  +  i/(a;-ft)       ~ 


V{x-  a)  +  -/(a;+q)  -  .7(2^)    _    y^'+f 
v'{x  — a)  — y(.x-H-rt)+v/(2a)    ~        a;-c' 


77       -y(^^  -  a:)  4-  y  ■    _    4.^ -a: 

78.  ■|y(rt.  -  a:)2  -  fV  | (a  -x){x  +  h) }  +-^/{x+h)^  =  iy{a^  -  ab  +  h^). 

79.  {^{a-xY-^^[{a-x){x-h)-\  +  ^3/(a;-/.2)2p  = 

(^,-i){r(«-.^)  +  l>n:« -/')[. 

80.  {x1/(rT-a;)2+ir(/;+.'^)2l2  =  (a  +  6){^3/(„_.^)4.^/(/,^^-jj 

81.  f/{a-x)^^{x-h)=f/c. 

82.  -,3/(a+^)2-T^/(^'-^')'=if(2^a;). 

83.  f/{a-x) 2  + 1^/  J  (a.  _ a,) (/,  -  a:l }  +  f{h  -x)^=  f/c^. 

84.  ■,f(a-a;)2--,f{(«-x)(a:  +  o)}4-^(a;-fi)3  = 


276  APPENDIX. 

85.  { ,3/(a - x)  +  f/{x  +  h) )f{{a-x){x-\- h) }  =  c. 

86.  f/{a-xY  +f/{x-hy^  =c{f\a~-x)-{-f{x-h)]'^. 

87.  x+f/{a^-x^)  =  "'  . 

89.  (a+x)V(fl+^)  +  («--''')M«--'')  =  «{t/('-«  +  -'^)  +  M«-2j)}. 

90.  (a+x}^/{a-x)  +  {a-x)t/{a-\-x)  =  a{'^{a+x)  +  t/{a-x)]. 

91.  4/l^G-»^)  +  t/(^-10)  =  2- 

93.  {a- x)t/{(Ji'-x)  +  {x-y)t/i:^  -^>)  = 

{a-b){J:/{a-x)+^{x-h)]. 

95.    {^{a-x)-\-if{b-x)]{s/ (a-x)+  ^ [b-x)y~  ^ 

c{t/{a-x)-if{h-^x)}. 
90.  rt\/(l+.r=)-3;i/(a;2+rt3)^g. 

97.   {a-x)^{x-h)+{x-h)-^{a-x:)=e{^(a-x)  +  f/{x~h)]'-, 

98.  {Ty(a-^)+ir(&+a:)}^=c{ir(«-a?)2+r(/>+^)2}. 

99.   {^{a-x)  +  f{h+x)]'^=cf^{{a-x){h-^x)]. 
100.   ir(a-a;)^-i«/(?>-rc)2=r|r(fl  +  /;-2a;). 

101.  V('^-^)  +  M-'^-^)  =  ^''- 

102.  ^{a -x)  +  '':/{x-h)=  ^c. 

{a-x)^{n-x)  +  {x-lAt/(^--h)  _ 
^^^'      (^.-a;)i/(a;-&)+(^—*)^> -•'«)"''• 

104.  ly^a-x)^i/{h-x)  ^■''• 


105. 


V(a-a;)4-V(->--fc)  ^        g 
4/(a-a;)- t^(3;— i)     a+^— 2a;' 


APPENDIX.  til 

« 

107.  [a-xyi/{a-x)-{x-hy^{x-b)  =  c{:i/{r,-x)-^{x-h)}. 

108.  {a-x)^{x  +  b)  -  {x+h)^{x-  a)  =  c{^y^a  -x)  - i^[x+b)}. 

109.  {^{a-x)^  +i/{x-b)^}-l/{ia-x){x-b)}  =c. 

110.  {:/(a-a;)-y(^-i)|3{5^(a-:c)^-M^-5)2}=c. 

111.  {^y{a-xy-  ^{x-by-}^-  {y{a-x)  +  -Ifix-b)}  =c. 

112.  {^(«-u;)^+:/(x4-Z^)3}2=c{.V(a-a:)  +  y(a;+Z^)}. 


Section  V. — Quadratic  Equations  involving  two  or  more 

VAUIABLES. 


1.  (x  +  y){x^+y^)  =  a,  I. 

X^-^j/+XlJ^  =c.  II. 

I  +  2TI.  .-.  {x  +  yy  =  a  +  2G 

.•,  x-{-y  =  -^{a+2c).     (Any  one  of  the  three  cube-roots).     III. 

ic^  c  '  *  '  U+2//  a+2c' 

Tj    TTT                                         ■i/(a-2c) 
By  111.  a;  —  y   =    -'— ) — -; . 

Ai  .  i/(«+2c) 

^^^°  "+•''  =  ^/I;h:2^)' 

.  ^  ^     y(a+2g)  +  y(a-2c) 
26/^a  +  2c) 

=    A/(a+20 --!/(« -2c) 
^  2V("  +  2c) 

(Not  any  one  of  the  six.  sixth-roots  of  a -f  2c  may  be  used  indiffer- 
ently in  the  denominator,  but  only  any  cube-root  of  whichever 
equare-root  of  a-t-2c  is  used  in  the  numerator.     Thus  if  the  radi- 


278  APPENDIX. 

cal  sign  be  restricted  to  denote  merely  the  aritlimetical  root,  if  it- 
be  defined  by  the  equation  k^-k+l  =  0,  and  if  m  and  n  indicate 
any  integers  whatever,  equal  or  unequal,  the  value  of  x  may  be 
written  * 

{l-"'s/(a+2c)  +  k"'-^  v^(a-2c)}-4-2V(«  +  2c). 

2.       8x^-5x>/+d>/^=9{x-\-y)  !• 

llx^-8xii+5y^=lB{x+7/)  ,  il' 

1st  Method.     Eliminate  {x+y). 

.-.  104^2  _65.r//+397/2=  99x2 -72x!/  +  45;/». 

.-.   5.r2-f  7a;// -6^/2=0, 

.-.  (5x-32/)(:/:+2»/)  =  0, 

.-.  a:  =  §?/  or  -2?/, 
Substitute  these  values  for  a:  in  I. 

.-.  72?/2  =  360.y  or  45//2  =  -9^ 

.-.  ?/  =  0,  or  5,  or  — ^, 
anda;  =  0,  or  3,  or  f . 
2nd  Method.     Take  the  sum  of  the  products  of  I.  and  11.  by 
arbitrary  multipliers  h  and  /, 
k{Qx^-  -5a;2/+3z/2)+^(llx2-8:r?/4-5?/2)  =  (9/,-  +  130(x+?/).     HI. 

Determine  h  and  /.  so  that  the  left-hand  member  of  III.  may, 
like  the  right-hand  member,  be  a  multiple  of  x+y.  This  may 
be  done  by  putting  x=-y  in  III.  from  which 

16/C  + 24^  =  0,    .-.    2k=-dl 
.«.   if  ^  =  3,  /=-2. 

Substituting  these  values  in  III.,  it  becomes 
2.^2 +a:7/-?/2  =«-+•?/ 
.-.  {x-¥y){'ix-y)=x+v,  or  (.r+?/)(2ar-u-l)  =  0. 
.-.    either  x+y  =  0,  or  2a;-?/  - 1  =  0. 
..    t/=  -ic,  or  2a;-l. 


APPENDIX.  279 

Substituting  these  values  for  y  in  I.,  it  becomes 
16^--'  =0,  01-  10a;--'  -  7a; +  3  =  27a;- 9, 
.'.  x  =  i),  or  3,  or  | ; 
and  y  =  0,  or  5,  or  —  ;^. 

a;^  +  ?/3  ~(^3 -j.  ^3  '  1. 

a;*  —x'^y  +  x^y^  —  xy^-{-y* 


I.~I1.,   . 


{a-i  +  b^-y^-a'ib-  ' 

x^y+xy^  a^b-\-ab^ 


•  *      (x^+ir)-  -  x^-y^  ~  (a3+/.3j2  _  ,,2^2 
Write  2  for  -;r-,^ — 5  and  k  for 


z  k  1 

I^i-   ••■    1372  =  1313.  •••  2  =  /'^or— ^ 

a;;/              ab             ct"  -^-b'-^ 
or  i — > 


v/(II.-3IV.)  &nd^x-y=±{a-b), 

M-+ab  +  b^ 

,*.   3;-  ±a,    +6  or 


III. 


a;;/  «6  a^ -{-b^ 

IL,      .-.     x^v  =  a5,or  («2  +  i2)-__X_  iv 

n/(IL+IV.),  .-.  a;4-?/-+(a-fi) 

orV(2a3_«5  +  263)    y^^' +'^ -r^' . 


280 


APPENDIX. 


y=+b;  ±«  or 

.a"--{-ab  +  b^ 


Putz  = 


xy 


1-z 


l-^. 


lU 


lY 


a 

T 


:    %tz^-bz—{a-b)^0 
:    Aaz'^=b±:V{8a^  —S(ib  +  b-)^b-\-y  say. 
xy        _    b+r 

X'  +  ?/3 

.    x  +  y 


4a 


[2a  4-^  + 
--Nl'ia-/;- 


x-y 

.     x_   _    |/(2c/-f/>+r)-}-v/(2«-/'-r) 
"    ~y     ~    ]/(2a  +  /;^+r)— v'(2rt-i  — /•) 


2(6 +  r) 
{x^^-y^--^1xy){x''-\-y^Y{{x^+y^)-xyY^a 

Ua+2b  +  2r  ]   I  4a  \  ^  (ia-b-r^  3 


M' 


b+r 


I  Aa 


b  +  r 


—  a- 


.1  0 


X 


1 
II. 


in. 


IV 


32u2{2a+b+r){4:a-b-ry)    ^   ^ 
.  (b+r)^  '     ~ 


X' 


-      [yl      t32(2a  +  /'+r)(4«-/>-r)2| 


JT/(2a  +  6+r)  +  V^  (2<<  -  6— r)}  \' 
102i(2.t-f  A  + '•)(•!'.<  -  i-r)2 


APPENDIX.  2S1 

_     V{''^'-i-\-lj  +  r)+ V{-2,a  —  b-r) 

in  wliich  r^±s/{Qa--iDab-\-b-). 

The  value  of  y  may  be  deriverl  from  that  of  x  by  the  first  form 
in  lY. 

6.  X'*  =ax  —  bij,  1, 

?/4  =  ay  -  bx.  II. 

X.I.  —  2/.II.     x'^ —y^  =a(x^ —  y^) 

y.l.-x.Il.     xy{x^-y^)  =  b(x^-y^), 

.".  either  x  —  >j  =  0  from  which  x  =  y  =  0,  or  ^(a  — 6)  III. 

or     x^+x'^y-^x^y^  -\-xu^+y*  =  a{x-lry)  IV. 

and  a?^(ie2-[-a;//+y^)  =  i(.«4-^)  V. 

(IV.+V.)                 (^^  +  y)2^^'^+y2'^  =  a.{.b  VI. 

V.        »                      (a,^^J4_(^-2  4._y2)3^4/,(^^.y)  VII. 

l/(VII^  +  4.VI).    (^+^)4  +  (^'-'+^2).^2|(u;  +  y)  VIII. 

in  which  t=  y{ia-\-bp -j-ib^}.  IX. 

i(m.+VIIL),  ..  ^x+y)^^{2h  +  t){x+u) 

:.{x-{-y)^  =  2b+t 

.        :.(x+y)  =f''{2b+t)  X. 

VL-.X^               •■-^+^^-rPT7)                       '  ^^- 
o  VT       V                  /          ^•^        2(a  +  b)                                    2a — t 


x-y     = 


y{2a  -  i) 


282  APPENDIX. 

in  which  t=  V{'-i'^ +  '^ah-\-5h^), 
6.    x^—G*'=m{x-\-y)*y 

Letz= -,  .'.  z-{-l=  audz-l  =  — ^  III. 

x-y'  ^       x-y  x-y  "^* 

I. +  11.  x^-\-y'^  =  m{x-{-y)^-^n{x-y)'^ 

,'.       (2  +  l)4  +  (z  -1)4  =  16(W724+W) 

{3  +  ]/{9-(8/ra-l)(8TO-l)f         , 
•*•      *  -   >J  .      8m- J  ~  J^« 

11.  &  III.         (?-l)*(x--7/)4+lGc4-16w(.c-^)* 

2c 

•■•'^~^~i/li6/i-{2-i)4}  V. 

2f2 

Kz+1) c(g+l) 

'    C(2-1) 

and  2/  =  4^  ( i6«  -  (zHtTT'  ^^^  *^®  ^^^"®  °^  *  ^^  S^^®^  ''^^  ^^• 
7.     a;2 +2/2  =  1  (2»7.+n2), 

and  {x-\-y)  3  —  o.(://(^-  +  ?/)  =•.  mn. 


APPKNDTX.  283 

Let  u  =  x-\-y  aud  v  =  xy,  and  the  equations  become 

U^  —3llV  =  17171. 

Eliminate  r,  .-.  u^ —  {2m+n^)u+2>nn  =  0, 

.'.  w*  -  {2m+n^)u^  +2mn  ti  =  0, 

.*.  u^—%nu^-\-m^  =n'^u^ —  2innu-\-m'^ . 

.'.  u"^  —m  =  ±{uu—in), 

.-.  u  =  n,  (the  value  it  =  0  was  introduced  by  the  multipiica- 
fcion  by  m), 

or  u^  -\-nii  —  2m  ~0, 

,'.  u=l{  ~n±  \/(n^  4-  Fyni)} 

.'.  v=\{n^-m)  or  l{;j3+8/«  +  3;?|/(w24-8w;} 

.%  tt  and  V  are  completely  determined. 

Also  x-\-y  =  u,  x—y=V(u^—4.v) 

r 

If  ?«  =  7  and  n  =  5,  the  above  equations  become 
x-+y^=^13,  and  a;^ +^3  =  35. 

Solving,  as  above,  gives 
u  =  5,  or  2,  or  —7, 
2j;  =  12,  or  -9,  or  36, 

.'.  x+y=:5,  or  2,  or  —7, 

x-y=±l,or±V22,  or  ±ji/2n. 
,:.  a; .-=3,  2,  |('2±s/22)  or  K-7+/\/2B); 

_7  =  2,,  3,  i(2+v/22)  or  i(-7q=,;V23). 


284  APPENDIX. 


8.     x^-\-y  =  U; 


«   +2/**  =  - 


4" 


•■•  H-^/=(f-?/')^ 

Testing  tliis  for  rational  linear  factors  it  is  easily  reduced  to 

li/-l)nz/^+2z/+4-)  =  0, 

.-.  2/  =  l  or^(-2+v/2); 
a;  =  i  or  i(-l±4V2). 

;«.     (2.r— 7/+z)0r+?/+z)  =  9;  ,  I. 

{x  +  2y-z){.r  +  >/+z)  =  l',  n. 

(a;+7/-2z)(x+^  +  z)  =  4.  III. 

Let  s  =  x+!i+z  and  the  equations  may  be  written 

(s  +  a:-2//)s  =  9'  IV. 

(s-\-y-2z)s  =  l  '        V. 
(s-32)s  =  4.  VI. 

IV.  +  8.V.       (4s  +X+.V  -  6z)s  =  1 2,  or  (5s  -  7z)s  =  12  VIL 

R  VII-7.VL     {(15s-21z)-(7s-21z)}s  =  8, 
.-.  8s2=8,    ..  s=±l. 

Substituting  in  I,  II.  and  III.  they  become 

2x-y+z=±9,     x+2y-z=±l,     x-{-y-2z=  ±4., 

.-.  x=  ±4,     y=  +2,     z=  ^1. 

10  x^+y^  =  a; 
u^-{-v  =b; 
xy+uv  =  c; 
xu  +  yv  =  e. 

Let    t  =  xy  —  nv. 

:.  {x  +  yY=a  +  c+t,    .'.  x  =  \{^{a-\-e^t)+y/{a^':-t)} 
{x-y)''=u-c-t,         y=lU/{a+c+t)-  V(a-i-t)\ 


APPENDIX.  285 

(u-\-vy  =  b  +  c-t.         u  =  l{^/(b  +  c-t)  +  ^/(h-r-{.t)\ 
(u-v)^  =  h-c-\.t,         v=l{y'{b  +  c-t)~V{b-c-{-f)} 
Also  2{xii  +yv)  =  (a; 4- ?/)("+■'')  +  (•«— Z/)(«  —■*-')  =  2e, 

•••  ^^W'+<'  +  t){h+c-t)}+^/{a-c-t){b-r-\-t)}=2e, 
.-.   {4:e^-\-{a-c-t){b-c  +  t)-(a  +  c+f){b+c-t)}^  = 

IGe^a-c-tXh-r  +  t). 
,'.  {(a-i)?  +  4r3}i2-2(a2-i2)rt-f 

(rt4_/,)3c2-4e2(a6+c3)+4^'4=0, 

(a^-62)c-j-2cv/  [{ab  -  e^){{r,  -by  ~4:{c^  -e^)\] 
•*•    *  =  (a-Z))'-^+4^  "        "  ■ 

11.  xy  =  uv  I. 

aj+y  +  w+t-^a  II. 

jc9^y3+u^  +  v^=b^  III 

a;S+y5^„6+^;5^c5  IV. 

het  x+y  =  k{a+z).    :.  u-^v  =  l{a-z).  V. 

Also  let  r  =  xy  =  uv  VI. 

.'.  a(302+a2)  =  4(fc3  +  3rt,.)  VII. 

Also    (a;  +  //)-^  =^\+^'^+6u:.v(a;3+,y3)^10^2^2(a.^^) 

.-.  a(5z4  +  10a2z2  4a4)  =  16{c5  +  5'j3,-4.io.ir2}  Vlll. 

Eliminating  r  between  VII.  and  VIII, 

45„224  _30a(fl3+2J-^)52_|_„6  _20a3/;3  _80/;6  +  i44,,cS  =0 
.-.  Ir./22_5(a3+2^j^)=±2v/{5(a34-5^3)2_i80,,c5}  IX. 

. a ^  4  2b^±2y  [^{(gs  +  56^)2  -  SQac^  }] 


2=  V-  3a 


X. 


286  APPENDIX. 

VII.  &  IX.      12ar  =  a^  -U^  +  Saz^ 

5(a^  -  b^)±V{5{a^  +  5b^)^  -ISOac^} 
'''  ''  ~  ^  30a  ^• 

X.  aud  XI.  give  the  values  of  z  and  r  which  may  now  be  treated 
as  known  iu  V  aud  V. 

^+l/~^(a+z),  and  xi/  =  r 

.-.  x-y  =  ^v{{a+zy-16r} 

x=^{a+z±V{.{a  +  z)Z-16r}); 

i/  =  i(a+zqiV{(a+z)2-16r}). 

The  values  of  u  and  v  may  be  obtained  from  those  of  x  and  y 
respectively  by  changing  z  into  -  z. 

Exercise. 

1.  6{(7 -a;)3 +2/2}  =  13(7 -x)?/,    x^- +Ay  =  y^+4:. 

2.  10x3-9?/3  =  2a;^    '8x^ -6y^  =  ldx. 

3.  xy={S-x)^  =  {2-y)K  4.  x^  ^y^  =  8x  +  9y  =  U4. 

5.  x2_|.^2=a;+y+12,    a;?/+8  =  2(a:+y). 

6.  x+xy+y=5,    x"" +xy  +  ij^  =7. 

35         28 

7.  x^+y^  =  7xy==2S{x-^-y).       8.  a;2+cc^  +  ^/»  =^^-^2  =— • 

10.  {x-]-7j){x"-+y'-)  =  17xy,    {x-y){x^ -y^)=9xy. 

11.  25(a;3 +2/3)  =  7(ic_f.^)3  =  175a;?/. 

12.  2x^-y^  =  U{x^-2tj^)  =  U{x-y). 

13.  2a;2-3a;_y  =  9(a;._3//),    3(a;2 -St/^)  =  2(2a;2  -  3a;!/). 

14.  2x^-xy  +  5y^  =  10{x-\-y),    x^+ixy+Sy^  =  U{x+y). 

15.  (2x-32/)(3a;+42/)  =  39(a;-2i/),  {3x+2y){ix-Sy  =  {99(x- '2y) 


APPENDIX.  287 

16.  {x-\-^y){x+^)  =  ^x-Vy),     {%x^y){^x+y)  =  1Q{x+y). 

17.  x+y  =  ^,    a;4  +  i/*  =  706.       18.  x+y  =  5,  .t^  +  z/^  =  275. 

19.  x-\-y  =  %  13(a;S  +  ?/5)  =  121(a;3+2/3). 

20.  a;+^  =  4,    41(a;3 +i/5)  =  122(.c4  +  ?/4). 

21.  x^  —  5xy-\-y^  +5  =  0,     xy  =  x-\-y  —  1. 

22.  x^+y  =  5{x-y),     x+y'^  ='l{x-y). 

23.  3(x^+2/)  =  3(.:c  +  //2)  =  13.c//. 

24.  10(a;3+//)  =  10(.>;+.y-^)  =  13^x--^+^3). 

25.  a;3+.v=y.     ^J  +  y^^V-     26.     9(a;2  4-2/)  =  3(z-+^3)  =  7. 

27.  x4-^//+//  =  5,     a;3+.r//+//3  =  17. 

28.  .«+i/  =  2,     (^-}-l)-'+(i/-2)5  =  211. 

29.  3(.-i)(,+i)  =  4(.+i)(,-i),  ^;,::^:^i  =  ^(^._^;i) 
1         • 

80.     x-+//=  — ,     x-y  =  xy. 

31.  x--F//  +  1^0,     u;6+i/^+2  =  0. 

32.  .^+^  =  1,     3(u;«+^«)  =  7. 

33.  4.f//a=5(5-.«),     2(a;3+2/3)  =  5. 

34.  'ilxy  =  17,     9{x^-\-y-^)=-8. 

35.  (.^.■•^+//^)^  +  4x2^^=5-12r/,     y(aj3 +^2)  43  =  0. 

36.  x-\-y=--xy,     x^+y"  =x^-\-y^. 

37.  x^     Cxx-W{y'-x^)   -16y^  =  9x^, 
{x^~+-A)^=4.{2  +  x^]/{y"+x"-)-y^-}. 

38.  x{y-^+Sy-l)  =  tiy^-\-2y-\-d,     y(x-^ +  ?> r  -l)  =  2x^ +2x+S. 


288  ■      APPENDIX. 

39.  i^  ,  ^  ^  2c3,  ^  +  1-  .  .(-^  _  r\. 

a^    ^    b^  a    ^     b  \a  b  j 

« 

40.  x^+xi/^=a,     y'^  -\-x^y  =  b. 

41.  x+y  =  a, J.  ^    =   c. 

b-y  X 

42.  a;^+«!/2='^,     aa;2+^2  =(a3 -1)^. 

a—1 

43.  a;  +  T/2=aa;,    x^-^y  =  by.         Ai.  x -{■  y- =  mj^ ,    x^+y  =  bxS. 

45.  a;* — y^=a'^(x—y}^,     x^ -x^y-\-xy- —y^  =  b^[x-\-y). 

46.  (a;+2/)(a;3  +  3?/3)  =  »i,     (a;- ;/)(a;2+3//2)  =  n.' 

47.  x^y^  =y(a—x)^  =x{b  —  y)^. 

48.  x3(6-^)  =  2/3(a-.c)^(a-a;)2(6-y)3. 

49.  a3(^t:+^2)  =  i2(^.+^)2^     a^-iy^+e2)  =  c^{x  +  y)^. 

50.  x3-2/3=a(a;2-2/3),     a;3+^3  =  6(u;+«/). 

51.  x+y  =  a,     x^-\-y^^bxy. 

52         I—   -       I—   =   ^~^,     ^(^1+^)    ^    ^3 

53.     a;+?/  =  a;2/  =  a;2+2/3.  64.    a;-2/=  —    =   a;^ -^^ 


55.     x^{l-¥y^)(l-Vy^)^a,     x^{\-y^){l-y^)=h, 

_^      a;'^+a;i/  +  2/2  x^+y"^         xy 

Ob. =-    =    =   —. — 

x^-xy+y^  a  b 

57.     x''y+xy^=  -^^,     x''y+xy^=b. 


58.  x^y-\-xy^=a{x^-+y^),     x^y-xy^  =  b(x^ -y'^), 

59.  (^  +  JL\ix+y):=a,     fl  ,+  HI   ^    t, 
\y     '     xj  y  i'- 


AT^!-  vilX.  289 

60.  x^  +.'/'^  =  nx^]!^  =  a;//(.r+?/). 

61.  abxii  =  (i  {x  •'  +  //  3 )  =  A  {x + // )  ^ . 

62.  xi/(x+y)  =  a,     x^i/^{x^  +1/^)  =  h. 

63.  (1    ^   l)(x-3-yS)=.^,       [1    _    lj(.rs+^3^==ft. 

64.  a;*+_?/4  =7?7(.r2+7/2),     x^ -\-xy+y'^  '—n. 

65.  fl6(a;  +  y)  =  .r;/(a+i),     a;2+v3=a2  +  i^ 

66.  x3+?/3  =  a(a;  +  t/),     .r4+?/*  =  6f:r-l-?..)* 

67.  x^+v-=o.,     x^A-ir^=b(x^-^V^\, 

68.  xy  =  a,     x^-^y^  =b(x^+'i/^). 

69.  (x-?/)(:r=5+?/3)  =  (a_j)(fl3+53)^     a;2_^2=(^2_^.j_ 

70.  x^—y^  =  o,     x^-^7j^  =  h{x-y). 

71.  a;+?/  =  «,     x*+//*=6.  72.  a;+^y  =  a,     a;^+i,'5=ft. 

73.  a;+2/  =  o,     x^-^y- =h^-x-y^. 

74.  a;  +  ?/  =  a+^     (a-i)2(a;*+?/4)  =  fa;-7/)2(a4 +  64). 

75.  x^ry  =  a,     c{x^->ry'^)=xy{.c^+y''). 

76.  (a;4-?/)3=rt(,i;2-f-?/2)^     a;.?/  =  c(a;+i/). 

77.  x^y+xy^  =a^,     c^(x^-\-y^)  =  x^y^.  ^, 

78.  x^  =  a{x^-\-y^)  —  cxy,     y^  =  c{x^-\-y^)  —  axy. 

/  1        1  \ 

79.  a;2-?/2  =  fl2.     3.3  _y3  ^^4  |____  1  . 

80.  ic4-y''=«2a;?/,     (a;- +?/2)3  =  ^2(3.2  _2^2\_ 

81.  [x+y)x^y'^  =  a,     x^-hy'^  =  b. 

82.  {x+y)xy  =  a,     x^+y^  -hxy. 


290  APPENDIX. 

83.  x^-i  y^  =  a{x+y)'^,     x^+y^  =  b{x-\-y)^. 

< 

84.  x*+x-y'^-\-y^=a,     x^-xy+y^  =  l. 

x'^iX+x^y^)      a       1  +  xy 

85.  {x"-+>i^)xy  =  x^  ~2/^      yi{l+xy)^  =X  '  r^y' 

■j/(x^i(/^)— a; 

86.  x+y  =  {x-y)V.{^y),    V(^5y7)+^  =  «- 

87.  ^  +  ^  =  ~      i/{l-x)-V{l-y)  =  b. 

88.  «-+,'/-=«(»;  +  ?/),     a;4  +  2/'^  =  i(a3  +  2/3). 

89.  x3+y^=a,     {x  +  y){x^ +y'^)  =  b{x^+i/^). 

90.  ('a;2  +  (/2)(a;3+,^3)  =  rta;;/,     (:«+?/)(«* +//'^  )  =  ^'-^.V. 

91.  ix+yy'{x-'+y^)  =  a,     {x^--Jry^)\x^+y '>')  =  (>, 

92.  (x-2/)(x2_^3)(a;4_j/4)  =  4aa;y, 

{x+y)(x^+y''^){x^  +y^)  =  K^  -  ?/)• 

93.  x'^y  +xy4  =  a(x3^  -hxy^)  =  h{x'^  +*/*). 

94.  a{x'^  -f  ?/5)  =  ai(a; +?/)  =  hxy{x^  +?/^)- 
a;3  -  y^     a^  - h^         x^  -y^     a^  -  &* 

97.  .r^  =  2a.'<;  -  &?/,     y^  =  2a?/  —  /)»;. 

98.  {x+y){x^+y^}=^'',     {x-y){x^  ■-y^)  =  h. 


APPENDIX. 


201 


100.  {x  +  y){x^-*- ij^)  =  axi/,     {x  —  y){x^-y'^)=bxy. 

101.  (^  +  2/)(^2+i/3)-a(a;^+.V^j,       ^x-y){x^-y^')  =  h{x^-^y-). 

102         {^+y)^{x^+y^) ^\., 

[x^+xy  +  y-){:x'-\-y-) 

'(x-y)^{x^~y^y        ^   ^^ 


{x"'-xy+7j^){x-'+y-') 


105.     xy{x+y)[x^  +  y^)=a,     xy{x-y){x^ -y^)  =  b. 

107.  ^{x-x]f)^V{y-xy)  =  a,     y(^x-x^)^V\y-y-')  =  h, 

108.  (.c4-l)(2/-l)  =  «(a;-l){2/  +  l), 

{xo  +  l){y-lY=b^{x-lY{y-^+l). 

109.  a;+t/  =  rt,     V(iC-^)  +  V(2/-'^)  =  <^. 

110.  x+y  =  a{l+xy),     {x^yY  =  b^{l±x^y^), 

111.  a;+?/  =  «(l+a;?/),     a^^+y^  =  63(l+x'i/5). 

112.  (a;+l)(w-l)=a(a;-l)(y+l), 
{x^-l){y-l)  =  b--{y^-l){x-l). 

11^-      (l-x)(l-y}-"'      (l-a;3)(l-^3) 

(c+a;)(c+2/)_  {c^^x^)(G^  +  y^)  _^        ' 


{x  +  m)iy  +  n)  _        '  (a;^+m^)(y4  4. ?i4)  ^ ^^ 


292 
116. 


APPENDIX. 


119. 


120. 


121. 


(a;+l)(y  +  l) a_       (a;^  +  l)(y^  +  l)   ■'• 


117    i'K-    •    --    /  c/yx-rx-; 

•'"'•   :.(l  +  ^^)=«'   ..3(^+^4)  =  *. 

118.    ,^±^  = «  \^,   yil+^±^)_^ 


414-y-)-''-   x^(r+yio)  =  6. 


122   ^£±J()M±J:)__«'^+^^   :%2-fl)_a-6   • 
(^-2/)(^^^^^   2(<6  '   (/(a;2-lj=.7+d  . 

123.  ^^-^^^^-^\-aHb--l)      ^(i-^"-')-fe 

124  (^+y)(l+'a-y)^    (a;^+2/3)(l  +  a:V)_. 

125  (^+2/)li+^/)^    (a;3+y3)(i^^.3^3^ 

126  C-^'  +  ?^*)(l+^V)      {x  +  y)(l  +  x>,) 

127  (ii±jO(l+£//)^    (a5^+Z/^)(l+a;V)^/, 

128  (^!±^^l^l+a:^+^^2/=')_ 

{^->jY{i-xyr~      ••-*• 

129.  x^    ^x-^u  +  oa^x-\-y"=0,     y--x^~y^-2a'^r  =  0. 


APPENDIX. 


293 


130.      2x(y^-2x)^=a,     yiif- -2x)-^{y^ -4:x)  =  h. 

(Hence  deduce  the  solution  of  x^ — 5x^-\-2  =  0). 
131      2xij(x^+y ')''-=  a,     {x- -y-){x' -^y^)- =h. 
132.      v^x-+//2)  +  V{(a-x)2+^3|=V{(>V3-^)-  +  (ia-a;)^}. 

Q{x-  -//^)  =  «(6x-2^vo  +  -0- 

ExEltClSE. 

1.     (2./-+7/-4z>(.r+7/+c)  =  24,  2.     x^-iz=l, 

{x  +  'hj  -2z)(:c+.V+z)  =  6,  y^-xz  =  % 

(  — 2a; +3^H- 52)1^0; +  2/4- z)  =  30,  z^-xij  =  '6. 

3.  (x-+2//-83)(a;+ //  +  2)-2(a;y+i/s+2.i-^  =  -  12, 
(2a;  -  %+2)(a;+.7 +z)  +  (a;(/ +  2/Z  +  2./.)  =  Gl, 
{;dx-y  +  '2z){x-Ty-tz)  —  o(xij+yz+zx)  =  5. 

4.  a!2_^2,^^  6.     (a;5^^5^.2o)3_j_(a.^.,/)2^31. 
a;+2/+z  =  7,  (a;^+^^4-2^)3  +  u-+^./  +  2)3  =729. 
x3+i/2+22  =  21.  (a;4-i/;2-}-(a;+y/4-z)3---=31. 

6.     a;2-(,'z=0,  7.  x+yz  =  U, 

•     a;+.y-f2  =  21,  ^+za;  =  ll, 

(a;-2/)2+(i/-s)2-i-(2-.tj2  =  126.  z+xy  =  l(i, 

8.     a;+|/  =  Sz,  9.  a;+?/  =  5z, 

a;=*+i/3  =  134.'23,  x^+y''=S9z, 

a;2+-y- +2^  =  134.  a^ +^3  =  105^2. 

10.     x+y  =  7z,  11.     a;+?/  =  78, 

a;2+^3=25z2,  a;3+?/3  =  25z3, 

a;4+7/*  =  674z3.  a;»  +  t/«  =  20272z. 

12.     aj4-2/'.V+2:3+i<'-:^:^.<5»  13.    '■x+y:,/-{-z:z-\-x::a:b:c, 
{n  +  b+c}xyz  =  2,,  {a  +  b  +  c)xyz  =  2{x+y-\-z:) 


294  APPENDIX. 


14.     ax  =  b)/  —  rz—   —    _j_  —   -f.    —         16.     zl —  _[_  — 


Z  X 


—  1. 


15.     {x  +  y  —  z)x  =  a,  ^Jx 

{x  —  y+z)fi  =  b, 
i-x+y^z,.=:c.  ^(f    +    ~)   ^'* 

17.     (y  +  z){1x-h!f-j-z)  =  a,         18.  x{^j+z):y{z  +  x):z{x+y)  = 
[z+x){x+'Iy-'rz)=b,  b-\-c:c-^a:a+b, 

{x+y){x-{-y  +  2z)=c,  xy+yz+zx={a-^b+c)(x+y-rz). 

19.  {a  +  b)x+[b+c)y  +  {c  +  a)z  =  {a-^b+c){x  +  y-i-z), 
a{x+y)  =  c{y+z), 

{x  +  yy'  +  {y  +  zy-  +  {z+x)2=4:{a^^b-+c-^), 

20.  c{x+y)-^b{x-z}-a{y+z)=0, 
b{x-z)  =  {a-c)y, 

21.  x-{-y—az  =  x  —  by-'rz=  —cx  +  y-\-z  =  xyz. 

22.  {a+b+c)(x-y)+a{x+z)-b{y-\-z)  =  0, 
{a  +  b-\-c){x-z)+a{x  +  y)-c{y-\-z)  =  0^ 

ax^  by^  cz^  ^ 


(6+c)3    "^   (c+aj3    ^    (a  +  by^ 

23.  xy  +  —   =   a,      yz  +  1-   =    b,      zx  +   ~   =  c 

z  X  y 

24.  y-\-z'.z-\-x:x+y::b+c:cJra:a^h, 
{x+y-\-z){xyz)  =  {a+b-\-c){xy+yz+zx). 

25.  n^—yz  =  a,     y^-xz  =  b,     z^-xy  =  c. 

26.  a;2+(i/-«)3=a2,     y^^^^.^^.  =/,2^     z'^ +{x-y)^  =c». 


APPENCIX.  295 

28.  x^+y^ —z^+Sxyz  =  a(x+y  —  z) 
x^  —y^-{-z^  +Sxyz  =  b[x  —  y-\-z), 

—  x^  +]/^  +z''-{-3xyz=  c{  —  x+y  -^z), 

29.  x+y+2az=--0,  30.  x-^y-az  =  0, 

«"+?/" +2"  =  <-•".  x^-\-^J^=c^. 

31.     a;0/-l)(2-l)  =  2a,  32.  x{y-l)  =  a{z-l), 

a;3(y3_l)(23_l)  =  Cv22  a;3(2/3_i)  =  c3(z3_i). 

33.     x{y-l)  =  a{z-l),  34.  a;(2/-l)=na(z- 1), 

35.     a;(2/-l)  =  fl(2-l),  36.  {x-y^- =nz{x+y), 

x^{y^-l)  =  h^{z"--l),  {x^~y^=bz{x+y)^, 

x^y^-l)  =  c^{z^  - 1).  (x-y)^  =  cz{x^+y^ 

87.     x—y  —  af  88."  a;-|-2/  =  <*) 

w  — i;  =  6,  w  +  ?;  =  6, 

xy  =  uv,  x^  +  H^  =c^f 

x^  —y^-\-u'^—v^  ^c^a  +  h).  y--\-v^=e^. 

89.     xy  =  icv  —  a^,  40..  xy  =  uv  =  a^, 

x+y+'u.-\-x  =  h,  x-^y+u-\-v  =  h, 

a;3  ^^3  4.,t3  _J.i;3  =  c».  a-4  +7/4  ^„4  _,_j,4  =(.4. 

il.     xy  =  uv  =  a^f  42.  a;^  =  w'y  =  a3, 


296 


APPENDIX. 


43.  xy  =  uv  =  a^,     x-\-y-\-K-\-v  =  h,     (.c  +  u)^ ->r{y +  vY  =c^. 

44.  xy  =  nv,  45.     xy  =  uv, 

x  +  y-}-ii-}-v  =  a,  x-*-y-\-n  +  v  =  a, 

X-  +?/"+"-  +r2  =  h^,  X^  4.,y2  +„2  ^^,2  ^  ^,2^ 

x^  +2/^+"'^  H-*^^  =c^.  X*  -\-y^-Yu^-\-v^  =0"*. 

46.  xy  =  uv,  47.  xy  —  ^tv, 

x-\-y-\-n-\-v  —  a,  x  +  y  +  u-\-r  =  n, 

iC^4-?/^  +M^  +1-^  =C^.  X^+y^  -f-W^  +  f*  =C*. 

48.     a;//-?<?;  =  0,  49.     a:2+?/2  =  fl,3^ 
xii+yv  —  a- ,  ii--^v^  =71(1^^ 

iJC-\-y  +  v+v  =  h,  iix+vy  =  c^f 

x^-{-^^-\-7i^+v^  =c^.  vx  +  uy  =  n^. 

50.     x+y+7(  +  v  =  a,  51.     7/{l+x^)  =  2x, 
xy  +  uv  =  b",  '   u{l+y^)  =  2y. 

u^+v^=n^.  x{l+v^)  =  2v, 

52.     x  +  y-i-ii+v=:a,     (x+y)" +{u+v)^  =  b^, 


53. 


X        la  —  u        y        2b— u        z  '      2e—u 
y  +  z~ a  —  2n     z  +  x     fc  — 2i*'    x+y~ c—2u 


ANSWERS. 


Exercise  i. 

1.     9.  -P.O,  1,  0.  1206,  -29,  1|.         2.     -160,106,41,108 
8>      -^,i-h    -25,   125,  -.v,  -31,   -4^V0.    -1-         4.     9,8 
7,-^1^.         5.     176,  82,  254II-,   -37-=-7^3.         6.     18  each. 
7.     146,14,-72,-270,896.         8.     Eacii  =  0.^ 

Exercise  ii. 

1.     -1.       2.     -166542.       8.     100.         4.     -2967511. 
6.     968.  6.      -162.  7.     10.  8.     -8.  9.     0. 

10.  -20.        11.  706440254900.         12.  0  each.        18.  Each  0. 

Exercise  iii. 

1.  0,  16a4.         2.  a,  a^/B.         3.  2a,  0.        4.  26^«,  -26«^ 
6.     0.         6.  4^7*.         7.     6a4.         8.     #.  9.     c.  10.     0. 

11.  a-f-(a+6).  12.  a2c{b-h2c) -^b^.  13.  a2+/,2_|_c2. 
14.  0.  15.  (12a2i-24rt^2_j.2863)-f-(3/>-a)3.  16.  0. 
17.  0.  18.  -b^c.  19,  20,  21  and  22,  each  0, 
25.     2{b-{-l)h,  4:x^.              82.     <Z2=3i3.              33,     i=  y'^^ojg). 

85.     wr^  <;-+r')(r-r')- 

Exercise  iv. 

1.  2{bx+cy).  2.     3(aa;-6?/). 

3.  a2{x-z)-ab{x-i/)-b^{y-z). 

4.  (a;+2/+2)(a  +  6  +  c).  5.      (a  +  b+c){z^ +y^-i^z^). 
6.  2(a;+?/+g)x{«-+63+c2-a&-6c-ca).  7.     0. 
8.  2/'7;r4-i3/  +  c?).                           9.     a^+^s+c^. 

10.     2a;"(a-2fe).  .  11.     a-[-b-c. 


11  ANSWERS. 

Exercise  v. 

1.     2(r?  +  9//4),  4rt2ft2.  4.     4(«.3-S3)2. 

5.     x^+4:x,    -3ia;4-4a;3.72_|.3iy4.  6.     a2. 

8.  a;2  -6.1-3 +9^4+ 2x//  .-6x//3 -6a;3y  +  18x3//3 +7/3  -6»/3  +9^/4. 
■    9.     ixu{x^-y^),  2(1 +  12a;2 +16.1-4). 

10.  yV^^-        11-    ^'2-2*2,  8«6(fl  +  &)2.        12.     2(a-c)(i-d), 
13.     ia;2+i//=  +  i2=  +  K.^7/  +  Z/«  +  ;^a;).  15.     (l+x--)^. 

16.     A(x;/+yz  +  zx)-2.{x^+y"+z^-).  17.    .x^. 

18.     (a'-^+2Z.2_2c3)2.  19.     16a;2;v3.  20.     -4rt&. 

21.     4(«  +  A-fc)2.  23.     4(1+^2 +a;i+;c^^). 

24.    (rt^X-- +63^2)2. 

Exercise  vi. 

1.  l-4:x+10x^-20x^  +  25x^  -2ix-  -f  16a-.^, 
l-2x+3x^—4:x^  +  3x^-2x''+x^. 

2.  1-4:X  +  8.r3  -  14.c3  ^  14.^*-8.r5  +o.x6  +6x''  +a;% 
l_|.6:c  +  15:c2  +20a^3  r  l^x^  +  (3.c^+x«. 

3.  4a2-+??-  +  c4  +1  -4aZ/  -4af3  -4a  +  2&c2  +  2/)  +  2c3, 
1  +a;2  +  ,/2  ^23  _  2^  4-  2^  +  2z  -  2;cy  -  2xz+2y2, 
kx^+y-  +  362-  -  k^y  +  6x2  -  iyz, 

4.  a;6_2x-'^^  +  3x%3_4.,.3^34.3^3^4._2x7/5+;/6, 

a2a;2  +  2ai;«3^.(2ac  +  &2)a;*  +  2(rti  +  6c)a;^4-(26(Z+r2)^8^ 

2cc/.«^+(i3a;8.         8.     3{a^ +b^ +c^')-2{ab+bc+ca). 

11.  4rt.-+i?>2a;2  ^__i^c3x-  +4:d-x'^  -  2abx  —  acx  +  8adx+  ^icx*  — 
2Wa;2-c(ia;3. 

Exercise  vii. 

1.       (&2_63)2.  2.       ix4  +  ?/4.  3.       rt4  +  3fi253+454. 

4.     a;4_.^4.         5,     ^2^         6.     IGr,?.         7.     U. 
8.     4a4-9&4_16c*4.24^2c2.         9.     5* -9c2 -4n'2+12ac, 
Qc^-4.a^-b^+iab.       10.    a;.8 -^/S.       11.    ajS+x^^^+t/'. 


ANSWERS.  "^ 

12.  a-'-a^b^+b^-l.         14.     x*+.v*  +  iV'Z/'- 

15.  a;S+2.c«+3.K'» +  2.^2+1. 

16.  4:a^x^  -Aa^xy  +  a^i/^  -a^x^  -2a'''.}^y+2ax^  +^ax^y  -x^. 

20.  (a:2+i/2-2x//-z2)3.        ■  21.     x»-.v«. 

22.  i-6«2  +  27a4.         23.     (w+^)3-(w+^)2. 

2i.  2.i;-+a;^+2x°-a;«-l.  25.     6*8-^,16. 


F. 


XERCISE    VIU. 


1.  x^+ix^+Sx^-2x-l-2,    a;-+i/3-2.r//  +  8.c2-8i/2+1622. 

2.  x^  +  12x--i-49.t-2  4-78u;+i0,    x''+6.«3  _a3_|_3fli_2/;^ 

3.  a3+8a6-10a4-104«3  +  105,    a;^4-2.i;S -x^— 2. 

4.  a;44-5x3^^'- 12.^2//^ +5a;?/3+?/4. 

a;2     1/2     2a;     2y 

6.     a;2»-2x"-a2-16fl-63,    —+^+—  +  —-1. 

(/-     X-      y       ic 

6.  7i2a;3_^2na;?/  +  7/2  +  10»a;  +  102/  +  21. 

7.  (^.  +  a)3  +  27/(.i;  +  a)-3y/3.     8.     u;4»  +  2a;3»  +  a;2»(l-a-6) - 
a.«(a+i)+a6.         9.     ia-« -.C^/y^+i/* -a;^  +  22/a -8. 

/I        1  \  2         /I        1  >        5 

11.  a;4-8x3+19a;2— 12a;+2. 

12.  (u;+6)4-(a34-c3)(a;+^')^+»^<''^-         1^-     «^+c<i. 

Exercise  ix. 
1.     2(1+3j;4),  ^xy^{dx^+x''y^).  ,      2.   96(a»  +  i2+a63), 

b{21a^-21ab-i-lh^).        3.    (a;  +  ?/)3.       4.    Sa^.       5.    Sx^. 
6.     8a;3.      7.    aS.      8.    27x3.      9.   {2+x)^.      12.   8{x'+y-')^ 
14.     (a3  4-i3)(;x3  +  2/3).       15.    0.       16.   0. 

Exercise  x. 
1.     l-3a;+6x^-7x^  +  Gx^-dx^+x^,  a^ -h^-c^ -3a^{b-\-c) 
+  3b^{a-c)-^Sc^{a-b)+6nbc,  1  -  6a:+21a;3 -oCx^ -h 
lllx*  -  174x-5  +219x«  -204x^  +144x8  -64x'''. 


IV  ANSWERS. 

2.     ~(x^  4- iar«  +  27.r'  +  2f)x<'  -  lix^  -  36a:*  +  5a;'  -  Sx^  -  2). . 

5.  0.      6.   ■i5x«+16Ssx*//^-432a;3^22a.      7.    (^ax  +  bu  +  cz)K 

ExERcisF,  xi. 

1.  a;«4-6^^?/+15a;*//3+20a;^2/^+15a;3y4  +  6a;y'+2/^, 

a;^+7a;G?/+21a;-5?/2  +  35a;4?/3  +  35x3.y4+21.i-27y«+7ic?/«+i/% 
a;8  +8x^v/+28j.-6^2  +  56s;5y  3  4.  qOx^yi  +  56a;3yfi4-28a;2^6  + 

8^//' +;//»,  a;i  2  4.12x1  i^+66a;iOi/2  +  220u;'^//^  +  495a;Sy*-h 
792x-^^^+924^6;y«4-792^"//'+&c. 

2.  The  signs  will  be  alternately  positive  and  negative. 

3.  a^  -  5a^h  +  IQaH^  -  lOa^b^  +  5ab^  -b^, 

a*  —  8a3^-[.2ia-62  —  32tt/;3  4-iG/>^,  same  as  last,  terms  in 
inverse  order.  4.  l  +  Giii-{-lo)tt--\-'2{)in^  +  15m*-[-6m^ ■+■ 
m^,  )n^+5m'^  -f-  lOm^  +  lQm-+oin  +  1,  6-i/M^  +  192;u--|- 
240;«4  +  1G0m3  +  GUw-  + 12^  +  1.         5.     120. 

6.  x^ -4:X^y  +  &x'^r -ix^ij^+y^,     a^ -10a462_f.40a3^4_ 

160a9^9+240a6/;i--192a3i;i5^ti4^is^ 

7.  49i5a8^*-792rt^i-'. 

Exercise  xiL 
1.     l+x3+a;*+-^•«+a;l^       2.    1 +x  +  a:''+x3+a;4+.BG_|.a.7.|. 
a;«+a;»+a;i«.         3.    x4  +  2a;3-85a;2-86^+1680, 
2^3 -3^6 +4x5+^4  4-^^ -2^- -x+2.         4.    x^-57x^  + 
266x2-1.         5.    18x8  4-21x'  +  8a;6+a;5-f-63a;-^+96a;2  4. 
43ic+6.      6.  l-i^--ix-4.      7.   6a;i3_4^9_5^8_2a.7  + 
9a;°-10x5+a;4-5x3+5x-2  4-:c-f-4.    8.  x^ +dx- -irlQx+ll, 
0      x4  +  3a;3.         10.    x'^-'dx^.         11.   a;4-f-8.c3_8a;. 
12.     (1),   -1,  (2),   -1,  (3)    -4.  13.   -1. 


VNSWERS.  y 

Exercise  xiii. 
1.     Sa-S-2a;*-4r+2.  2.     5x*  -  4x^+Bx^  -2x+l. 

5.  a3+8„2^+8.,:^.3  4..,.3.  6.     4:x'' -\-8x-\-7,    -13a; -20. 

7.  10^-3 +  5a:-+l,   10.^;  +  10.       8.     x^-xy  +  T/''. 

9.  a;2-«3.  10.  x^  +  {l~a)x^  +  {l-a+b)x^+(l-a]x-\-l. 

11.  3a;3+2.c3+a;+Ur3i(a;+l).         12.     5a;2+13.ry+12(/3. 

13.  6x^-x^-x^+x^-x+Q,   -1. 

iJ.  2xi-3a;3+4a;2_5^  +  6.  15.     a  +  b.  16.     «+!/+*, 

17.  10x3,  10(a;4-20).  18.     wrc^  4.^^.2  _}.«. 

19.  i+x-oix^-dx^+Ox^.     '20.     33.         21.      -4. 

22.  -2U.  23.     15^4.  24.     85x+8.         25.     755 

Exercise  xjv. 

1.  ?/3-2y2-4i/-9,  if?/  =  »-l. 

2.  ?/3  +  3?/+5,  if  ?/  =  a:+l-         3.     y^  +  Sl,  i:  y  =  x-2. 

4.  ^^4^1,^3 _432/3+92y_67,  if7/  =  a;+2. 

5.  3.v''+30y4  4-ll9v/3 +  238^3 _|. 249,7 +  106,  if  2/  =  a;-2. 

7.  V^-'-iy+W^  i!iy  =  x-h 

8.  (x-2?/-'')-3//(a;-2y)--18i/3(a;-2r/)-24yS. 

9.  (^-//)^-107/2(a._,y)3_207/3(a:-2/)'2-10^4(a;-_j/). 

10.  (2a:  +  i/)3  +  2//2(2a;+2/)  +  %^- 

11.  5l2(/3-3//--ri/4,  ifi/  =  ia;-TV- 

12.  ■*-24(/-^+492/-28,  if2/  =  ^j;+2. 

Exercise  xv. 

1.     a^b,  +ab^-\-n^c+b^c  +  bc^+ac'>, 

(a-6)3  +  (i-c)2+(c-a)3,    a{b-c)  +  b(c~a)  +  c(a-b), 
ab{x-c)+bc{x  —  a)-^ac{x-b), 
abc{a^b-\-a^c  +  b-c  +  ab^  -tac^-^bc^), 


VI  ANSWERS. 

{a  +  h){c-a){c-b)  +  {b  +  c}{a-b){a  -  ^r     ■■i-a){b-c){b  -  a). 

(a+(;)--63  +  (6+a)3 -c2+(c+&)^ -a^ 
a(b+c)^+b{c  +  ay+c{a  +  b)^. 

2.  cibc-{-bcd-\-  cda-\-dab, 

a^{b+c+d)+b^{c  +  d  +  r,)+c^[d-[-a  +  b)  +  '!"(a  +  h+c), 
{a-  b)  +(a-c)  +  {a-d)  -\-[b -o)-\-{h  -  d)  +  {c-d), 
a^{a-b)  +  b^{b-c)rho-{c-d)+d2{d-a). 

13.     X  and  y.         14.     ax  and  i?/,  x,  y,  z.         15.    /  and  h, 
16.     a;  and  y,  also  ic  and  —  z,  and  y  and  —2.  * 

n.     a,  ^  and  —  e.       ^18.     ic^,    —  ?/-  and  s-.  19.     6  and  c, 

20.     a  and  c.  21.     a  and  6.  22.     a^  and  2a&. 

23.     a-o  and  aic.  24.  a^/^^  abc.         25.     a;^,  x*?/,  and  x'^y-\ 

same  ;  a;^//,  u;^?/-.  26.     Not  symmetrical. 

28.     uS  u^o,  a^bc,  abed;  u^,  a'^u^.        29.     a^,  a^b. 

Exercise  xvi. 
1.     4(a3  4./,2^c2).  2.     3(a24-62_|_c2)^2(ai+k+ca). 

3.  4(a3  +  fe2+c2+rZ3).         4.     ^(a2+63+c3). 

6.  4(x2+?/2_i.22+7t2).  G.    2(a3+63  4-c3)  +  62a3&-12a6c. 

7.  14(a;3+2/3-fz3)+2(a;(/+?/3+2a;).         8.     ^Ubcmnr. 
9.     2ak(a+i+c).  10.     a263+i2c2+c2a2. 

Exercise  xvii. 
1.     115.       2.   ?m»-3<7a2+3m-s.      3.    2.      4.  -17-3533. 
5.     1,  2(3a2+l).         6.    0  or  27/",  2?/",  0.         7.    36. 

8.  -(i2+a2)3_(3&2)3.         9.    -15a*.         10.    StiSSa^b*, 
11.     a2fe2(«  +  ^,),         12.  0.         13.  2a3-3a6(a-6), 

263-i-6«6(a+Z/),  2(«3  +  i3). 

Exercise  xx. 

1.     3.       2.    1.       3.    -l±2v/-2.       4.  2.       5.    36.      6.  11. 
7.     -l^f  13.   ;^=-?,   9  =  6.  ■         14.   /'=-46,  5  =  14. 


ANSWERS.  Vll 

Exercise  xxi. 

1.     ^=_3,  r  =  8?5,  ^=-24.      2.  c=  -20i,  f7=-13-^,  ^=GOf. 

8.  /)= -3,  (■= -10.  4.  a  =  3,  />  =  0,  '  = -57.  5.  a=-2, 
c  =  24i,  ^  =  0.  6.  c=-  100^,  (1  =  202^.  7.  a  =  200, 
6= -810,  c  =  639.  ^    8.    a  =  4,  c  =  — 27,   d  =  7,  e  =  m. 

9.  399.         10.    x^-{p  +  -3)z^+{2p-{-g+3)x-{p  +  q-^r  +  l). 

11.  x3-(/?-3)a;2-(27J-g-3)a;-(^-g+r-l). 

12.  ra;3-(3r-g)u;2  +  (3r-2g+7j)a;-(r-5+;>-  1). 

13.  x^-qx^+prx-r^.     U.  x^ -{p2-2q)x^ -t{(]' -2pr)x  ~r-. 
15.  x^ —  2qx^+{pr+q^)x  +  r^—j)qr.       10.    ra;^  —  (y^y  +  3r)a;2  + 

(p3_2;>g  +  3?-)a;-(/)?-r),      83.  -i.       34.   1.       35.    -1. 
3G.  1.     87.    -1.     S8 -And  Bd.  a  +  b  +  c  +  d.     40.-1. 

Exercise  xxiii. 

1.  5h*+15c^.     2.  6.     3.  3.    4.- {y}(?;+r+(0  +  ---  +  ---  +  ...}. 
5.  0.    6.  5i4_30fl/,3-f80rt3^a-5«36.   8.0.    9.0.    10.0.  11.1. 

12.  {a  +  b-{'r  +  d).         18.    -1.         14.  a  +  b+c  +  d. 

15.  (a+^>+c')(r/2  4-i3_^c"+rf&  +  ftc+m)+^//;6'. 

16.  {a  +  b+cy{a^+b^-{-c^)  +  2abc{a-{-b  +  c).        17-  a  +  6  +  r+(i. 
18.  (a  +  A  +  c+rf)2.     19.   {a  +  b-{-c-{-d){{a-{-I)  +  c  +  d)^  - 

(ab-{-ad+ac  +  bc  +  bd-\-cd)}+abcd.       20.  a  +  6H-c.       21.  3. 

22.  -1.       23.  0.      24.  0.       25.  0.       26.  1+lx-^x^ -{-T^t^x^. 

27.  l-ix-ix^-^'-sx"",  28.  1+x+x^+x^. 

29.  1— 2a;  +  3a;2-4a;3.  80.  l+lx-^x^+^\x^. 

Exercise  xxiii.    (a) 

1.     (p-pf+q)^  =  {p  +  l){p2-pp'-q). 

8.  9(p3  - .7)(r2  -g«)  - (pr-t)^  =  9{3(^3 -^)(gr-p«) - 

(j99-r)(pr-0}x{30)g-r)(r3-90-(??/--?;)(9r-;7/)}- 

9.  af{4:X^-[-Spx^  +Sqx+r)  -r-  (ic4+4i;a;3  +  6^a;2+4ra;+ij. 


VIU  ANSWRRS. 


10.  -ip,    (ip)^~2(Gq),    -(4:p)'^-^B(ip)(eq)-^(ir\ 
(4;^) 4  -4(4/>)--'(6y)+4(4p)(4r)+2(6^)3  -4^, 

-(4/;)^  +  5(4p)3(6y)-o(4y>)2(4r)-5(4j9)(6?)2  +  5(4p)<+ 
5{6q){4r),  (4p)6-6(4;j)4(6^)  +  6(4,v)3(4r)  +  9(4;^)2(G(/)2  - 
6{Ap)H-12{Ap){Qq){4r)-2{6q)^  +  6{6q)t+S{4r)K 

11.  .Sp.s^  -4si.s3+a'?^,  .f^Sp  -e.Sj.s^.+lo-Sj.s^  — lOs^,  where  So,  -"i, 
&c.,  are  the  coefficients  of  the  terms  (taken  iu  order)  of 
the  quotient  in  No.  10. 

12.  a;»(4,x3  _2Sx  +  l) -^  (x^^ -Ux'^-\-x-S8) ;  .s,  =0.  Pj^SS,  s, 

=  -3,    .54  =  544,    .s'5=-70,    6-g=8G83;    2(a-/.j4  =  4526, 
S(r/-6)6=2G4122. 

Exercise  xxiv. 
1.     (3m4-2)-',   (o--l)2.  2.     (y3  _,^3)2,  4,^?(2.r-f-7/)3. 

3.     (3ai  +  2c)2,  4?/3(3«-.7/)2      4.     (^a;^ -47/^)3,  (^a^  _i^2c2)2 

5.      (.,  +  /' +c■)^(3.^•4-l,/3)2.      6.      (2-a;+2/)3,|(-)    -(--]}    , 

7.     (ar2-22)3.     8.     {x-7j)K     9.     (a  +  M-,  (fi-'^—k-^)-. 

10.  (x-//)3.  11.  4(a:3 +7/2)2.         12.      (x-+^)*. 

15.  («2_^3_c2)3.       iG.     (2rt-2c)3.       17.     (2a2_3S  +  4.)s. 

Exercise  xxv. 

1.  (7rr  +  2/*)(7a-26).  2.     {Sa+U){^a-U). 

3.  (3a-2i)(9.^3+4/;3)(;v,  +  2&).  4.     (10.r-6/y)(10.r+G.v). 

5.  5i(a  +  2x;/)(«-2x2/)  6.     (3a;-^-4.y3)(3j534-4//2). 

7.  (ic  +  l)(fc-l).  8.     (27/2-t:f2)(2.v=  +  §x^). 

9.  (3a-l)(3fl  +  l)(9r/3-f-l).    10.     (rf-2h){a-\-2b){a2  +  U^). 

11.  (a-Z-)(a  +  Z>)(a2+Z/3^(a4  +  64)(a8+&8^. 


ANSWERS.  IX 

12.  (a^h-c)(a-b-\-e).     13.   {a  +  2b -•Sx  +  4:y)(n-}-2b-'^x  +  4:y). 

14.  (a;3 -7/3)3     15.  (^x+y-\-2z){x+y~-2z).    16.  XG(.r+l)(l -a;). 

17.  {x+y+z){x+y-z)(z—x+y){z  +  x-y). 

18.  ixy{x  +  y){x-i/).  19.  (a;— 24-?/)(a;-2— i/)(.c+2+2/)(x+z-?/). 
20.  4(a  +  c)(i+^).  21.     24a;(H-2a;2).  22.     8«i(a  +  6)2. 

23.  (a  +  b+c-\-il}{a+c-b-d){a-h-~c  +  d){a  +  b-c  —  d). 

24.  (a;+j/+2)(a;-?/— z)(x+^-z)(.r-.V-l-2). 

25.  8a3Z)3^aG_3a3?>3  +  66).  26.     {a^  +  bS){a^ -b^)». 

27.  (a;2+2/2+«2)(^2_|.^2a-22--2x^-2?/z-22x-). 

28.  {x+2z){x-2y).     29.  («  +  i-c)(rt-6+6-)(i/  +  c+«)(i  +  c-«). 
30.     (a;-2/  +  z)(a;+2/-2)(d;  +  2/-f2)(a;-</-2). 

Exercise  xxvi. 

1.  (a--7)r.r+2),  (a;-7)(a;-2).  (a;+4)(a;  +  3). 

2.  {x-3)(x~5},  (a;-7)(a;-12),  (a;- 12)(.r+5). 

3.  2(2a:-5)(a;  +  2),  3(3a:-20)(a;- 10). 

4.  -^(a;+l2)(ia;-3),  5(x+l){5x  +  S),  {Sx^ -4)i3xS  -  5). 
o.  (ia;+4)(ia;+3;,  4(4a;-5)(a;+l). 

6.  (x-a){x+a)ix-b)(x+b),   {2(a:+?/)- 11}  {2(a;-!-y)  +  9}. 

7.  (a;^ +7/2 -a2)(a;2 +7/2+^2).         s.     {a-tb-3c){a-^b+c). 
9.     (x+y)(l-\-x+2/){x+y+(x-y)^}. 

10.  (a-{-b){l-a-b){a+b  +  (a-b)S}. 

11.  (a;^+x//+y-+2a;+2/)x{.c2+a;//+2/3-(a;4-2y)}. 

12.  (a-86  +  3c)(a  +  &-c).  13.     (x--'+y2  +  aS)2  ^b^  =Scc. 
U.  (a;-  -  10.-c-l2)(a;^  -  10a;+8). 

le.     (a;2-14a;+10)(a;-9)(a:-5).  16.     {x^--y^-)2, 

17.     (z+l)i2-l)(z2-2),  (.«^'-3)(ar3  +  lj, 


ANSWEKS. 


(3.c*  +  5./y2)(3a:4-2?/2).  18,     (L-'"-f  2)(c"' -  1), 

19.     {x"'-ufj{x"'  +  by"]. 

JdjXEEciSE  xxvii. 

1.  {x-hy){l>x-v).  2.     3(ic+2//)(2.c-?/). 

3.  A[Ux-5y){x-y).  4.  4(14.c4-5^)(a;  -  i/). 
5.  (14.c-^v}(x— 20//).  6.  4(7ic-5//)(2a;-2/). 
7.  2(28x  +  7/)(x-10?/).  8.  A{14.x-5y){.r^y). 
9.  (8x-5^)(7x-4y).                 10.     (8x+5//)(7«-4?/). 

11.  2(3x+?/)(a;-3?/).  12.     (3x-2//)(2a;+3?/). 

13.  2(28a;+?/)(^c+10i/).  14.     2(28.6- -5//)(;c-- 2y). 

15.  2(28x+5.v((.«-2//).  16.     {5Qx-5y)[x-4:y). 

17.  2(4,.r-2/)(7u;-10i/).  18.     ^{l4.x-\-y){;£-5y). 

19.  3(3.i;+^)(4.c-5i/),  20.     (8u;+.jy)(9x' -8^). 

ExEKcisE  xxviii. 

1.  (5:i;-7)(2a:+3).      2.   (6:c  +  3)(2x-7).      3.  (5x--3)(2.c+7). 

4.  (2a;-5)(3x-ll).     5.   (4a  +  l)(3a-2).       6.  (dx-T)>yix-o). 
7.  (8a;  +  7)(4.c+3).  8    (.5.,3  _46-2)(3a2 +  56^'). 

9.  (4a;+l)(3a;-l).  10.  S^^^x— y/)(:J.c+2/y). 

11.  (2a;  +  3!/)(2aT4-?/).  12.  xH^b-^x){'lh--dx), 

13.  (3a;3  +  72/3)(2a;3-.5?/3).        14.  (2a;3-9)(j;3  +  5). 

15.  (2a;+?/)(22;-j/)(a;-37/)(x+37/). 

16.  (2.c+4+y)(2x4-4-?/)(.c+2-3?/)(a;+2  +  3?/). 

17.  \mxy.         18.     (19//2  +  G0a;//-6a;3)(3oa;^-12x?/4-yU//3). 

19.  2(4.i;y-3x3-32/3)(61a:2-49a://4-6l2/2). 

20.  2(5^2  +  4.,-y +  10;/3)^x-  +  10.cy  +  2(/2). 


ANSWEKS.  Xi 

Exercise  xxix. 
1.     {7x-^ey  +  S)(x-y-z).         2.     (5a;-5i/-22)(4.r4-?/  +  4). 

3.  (3x2 +4y3  +  13)(x2 -2/3-1).  4.     (4a-_|_52^)(5a;-4?/-«-7\ 
5.     (9x+8?/-20)(8a;-?/-l).  6.     {x+Sy){x-i>/-5). 
7.     (4x  +  3;/-z)(2x+3//+0).         8.    (3a;-22/-2z)(2a;- S^z  +  Jz). 
9.     (3x2  — 27/2  +5z2)(2x3  _|_5y2  _  5), 

10.  (15x-2+8//2  4-5z2)(a;a_2?/2+3a2). 

11.  (2a.-5i-7c)(2a  +  3/i-f3c). 

12.  (rt - b-\-c){a  +  b- c){a  +  b  +  c){a-h  - c). 

EXEECISE    XXX. 

1.     x-+-J+fi/5,  2a;2  +  |±-||/5.        2.    x^ +|^2-^|^/2  ^C    ■ 
j\(6x2-r5y^±y^vlS).  3.  i(4a;2  +  6±  v/13), 

T-V{6(x+y)2  +  523±a2yi3}. 

4.  (a:2  +  iy2)(a;2_|.6^^3),  (.^2 +ii,/5)(a;2  +  |^2). 

5.  (2.f'^+4i^2)(2^3  +  iy2),  if4(V(+/;)2+5ci>/13}. 

6.  ^V(6^^+5^2)(6x3+ll//2),    (6x2  4-5)(6x2  +  ll). 

7.  J(5x2+10  +  3v/10),  i2a2^3-t.2j/2). 

8.  {2(x+yi2  +  (3±2v/2).s^}  ; 

-H  10x2 +  (10±3v/10)!/2}{10x'^+ (20- 61/10)2/2 }. 

9.  J(9.c^+7+v/13),  4{2x2  +  (0.L--i/16)(2/+2;^}. 

10.  ^(2x2+6±  a/6),  1(7x2 +  20±v/;  85). 

11.  i{4x2+(9±y23)//3}. 

12.  i{7(^.-fe)3  +  Sr2+rV29},   i{-)a^±b-]/n}. 

13.  i{3x2  +  (3±v/3)^2|^   ^{3(^a-^by  +  {S±^/S){a^hy}. 
14  {7a--'+(G±v/14)6-},   (5»i3+9/i2)(5m--}-3,i3). 


Xii  ANSWERS. 

Exercise  xxxi. 

1.  (x^±^xj/-\-^if9),  (x^±xy-y''),  {x^ +xj!-\-y^).' 

2.  (x^±2x]/  +  2y^),  (4a;2±3x?/+?/2),  (Xx2±xy+7/''). 

5.  (x"" +i/2x+l),  {x''±VGxy+Sy^),  {l±2y-Ay^). 
4.     {x^±Sx-\-l),  x^±V6x  +  3,  ^,r2±2j-?/  +  //3. 

6.  {2x^+y^±?ixy),  (x^ +y^±ixyVS9),   {2x-  +  l±2x). 

7.  (a:2'"  +  8?/2'"  +  4a;"'?/"'),  x^'^  +  2y-^'"±:2x'"y"'), 
(^a;2  _  |jy2  ^;;£c^/  V5). 

8.  (2d:2-l  +  2a;),   -(^icS  _6?/2 +x?/) (^.7-2 -62/2 -a;y), 
(«-  +a^y^  +  axyi/2). 

9.  mig^—ny^±xyyp),  a;2"«-f 2'"-»y2m^-2'"?/'». 

10.  4x3 -3±a;,  2a;2-2±2a;^/2, 
-(3x2  -2i/2+a;y)(3a:2  -  2;y2  -xy). 

11.  2^2  ±4.1-7/ -3?/ 2,  a;2  +  22-  +  5. 

12.  2{a2  +  ab  +  h^)^,  {2a^-\-a  +  l)2. 

13.  {(a:+2/)2  +  3(x+2/>+z'}{(ic  +  y)^-3(x4-?/)~4z'} 

14.  {a  +  b)^+^^dz^G^V5. 

15.  {4«3  +  6a(6-c)  +  2(fe-c)2}{4a3-5a(6-c)  +  2(6-c)2}. 

16.  4(a2  +  5«i-2/>2)(62_|_5«i_2a2). 

17.  {ix^+y^-xy)^±S{x^+y»-xy){x-[-y) 
+  {x+y)^}. 

19.     (4a2  +  2a+l),  a;2+7a;+4. 

20      (a;2±9x//  +  9//2),  {l±:Sz+^z^). 

21.     4(3a;2_2x+l;(a;2_2a;+3). 

Exercise  xxxii. 
1.      (a;2  +  3)(3:  +  .Sl(.r-l).  2.      2(.)-2-f  3)(r2  +  .r -f?) 

3.  {x^  +  A)(x+i){x-i).         4.     (a;+2)(x-2j(3a;2+x+12). 


ANSWERS.  Xm 

5.  (a;2-B)(5.f^ +4.^+15).  6.  {x^  ■\-G)(10x^  +  ^x-G0). 

7.  (ix2+TV)(i^^+-^0-«-To)-     8.  (5^2  _i)(5^2_  8^+1). 

9.  (5i;2-8)(7ia;3-6x-12). 

10.  (3x2-4)(21.k2_13u;-28).    11.  {18x^-\-l){A5x^ +^x  +  ^). 

12.  (ll.'«2  +  l)(22a;2-8x-2).      13.   (^^^ -f)(^a;--^+ia;-f§). 

14.  8(;r2-2?/2)(10x2-4a;?/  +  20y3). 

15.  (2x^  -5?/^)(12a;2-6:r(/  +  30//2). 

16.  (x^-16y^){2x'-  +  l^y  +  ^^y^}- 

17.  (a;2-|)(llx2+10a;+Y)-      18.  10(.t2+2)(4x2  + 3.^-8). 

19.  (a;2-6//)3(13x3-12a;y  +  78y3). 

20.  («2+4?/2)(3.,.2+3a;y_i2y2). 

21.  (x2-3?/2)(5a;2 +4x^  +  15^2). 

22.  2(x2-2//2)(2a:2  -7x?/+2//2). 

23.  (a;2  4-i.!/2)(ic2-t-80xt/-^//2).  24.  (x^ -6y^)(2x^--xy  ^12ij"-). 

Exercise  xxxiii. 

1.  (x2+3./:  +  27)(:«2_9.^+27).  2.     x2 +a;(l±,/3)+4. 

3.  {.j;2+l+i(l+ a/5)4.  4.     x^+l-x(2±^/5). 

5.  2x2  +  2-3x±a;\/23.  6.     (x^  +  lSx- 5)(.x-2_a;_5). 

7.  (la;2-2)(4x2-6x-2).  8.     {x^-}-8x+4:)l,x-\-3x+^). 

9.  (x--^+7x-2)(x2-a;-2). 

10.  (a;3+5x//  +  3i/2)(a;2-a;2/+3y2). 

11.  (x2+.10x^l)(a;2  +  2a;-l). 

12.  ix^+l.ry  +  y^){x^  -Sxy+y^). 

13.  2x^  +  xy-5y^±xyi/4.6.      14.  [x^-^.'Jxy -y"-){x^ -xy-y^), 
15.  a;2  +  2?/2+3.r?/±a;yi/3, 

10.  (3x2+10a;?/-2?/2)(3x2  -4xy-27/2). 

17.  T-V{lla;^+22//'-'  -h5x(/±ffx!/v/ll}- 


XIT  ANSWERS. 

Exercise  xxxiv. 

1.  {y-z){T:^-y).  2.  {hy  +  c){ax-^hy-c). 

3.  (z-+a){x-\-a){x-a).  4.  (2a;-a)(a;- 2/*). 

5.  (.T+3a)(a;  +  2^).  6.   {x~b-^){x-a){x  +  a). 

7.  (a;-6)(a;  +  ?))(x-fl)(x2+^cG+a2).        8.  (2a;-l-3«)(4.'c+5/;). 

9.  {a-\-hx){a-hx-\-cx'')  10.   (a-&s)(«  +  ^^x+c.x-2). 

11.  (aa;-(^)(5a;3+rz-/).        12.   {px-q^x"'  -x-l). 

13.  („_6_c)(«  +  2A  +  Br).       14.    (ic  +  «)(x^ +x  +  1). 

15.  (wx-n)(;9x2  4-(?a;-r).      16.   (.'B-«)(.'c~/>)('a:- (■). 

17.  {x-\-<i)(x-h){x-c).  18     (xH-ri')(^4-/;)(>c-c). 

10.  {a^-{~z){x-ay){x,^ -y).  20.  {abx+nly -c.fz)(ax+hy).  ^ 
21.  ('/ic  +  c)(^'a;3-/«+c).  22.  (x-y){x-i-y){nix-iiy-\-rz). 
23.  (i»x-w?/)(rta;+/'//  +  ''z)-      24.   (;/'a^  +  «)(«.i-'-6ra-4-'f). 

25.  (c2-a;3)(/>2-|/2)(«--a://).     26.   (x^ -jwSa;^ -«)(.r3 -«  +  »-M, 

27.  {l  +  x-x^){l-ax  +  hx^—cx^y^ 

28.  (fla:-'/'/M^-«— ^.'A)("^+c?/).     20.   (?7?.r+9)(;?a;4-")('"^-»- ")■• 
30.  {mx-\-Hij){:iix-ny}[p'x'^  +q-y-){x+l). 

Exercise  xxxv. 

I  (^a+x){a  —  b).  2.    {ax-{-hy){bx     ny). 

3.  {x-n){x+a){x-  +ox  +  rr-),    -L   .r(n+x){a^ -^ax+x^). 

5.  (aa;-6)(ca;  +  f?).  6.    (5a;2  -  l)(5a;2 -.^  + 1). 

7.  (fl-/*)(rt  +  /^+.i--c).  8.    (« 2 +  7,) („  +  /;). 

9.  (x-?/)(a;+?/)^  10.  (a;-;y  +  l)(a;2 +^;_v-^-_y2^ 

11.  (fc-2.r)(2  +  6.v).  12.    (;r-l)(a;  +  2)2. 

13.  {p-q){p--2g^)-  14.    (a-l)(a2  +  2a  +  2). 

15.  (a/.2_l)(3a/;2  +  l).  16.     ^y-l)'-^y  +  2). 


ANSWERS.  X'V 

17.  (a^h)(2a2-nah-{.'lh^).       18.    (Z^"'-l)(/>2'"4-2?>'"4-2). 
19.     (^"  +  z"j(y-'«_8v"z"+,2-").    20.    {u-b)(a^-i-ab-'db-), 
^1.     (a"'-c")(a"'-2c'').  22.    (aa;-6)(a;- -aa;-/>), 

.28.     (5a;"-3rt2)(7:c"  +  3rt2).  24.  (a6+6c-c«)(aft-ic +  01). 

25.      (w-6)(w  +  M(a-»t).  26.  {^ -Sa-){l-Sa){l  +  3a). 

27.     (a;-y-z)(x3-2.c^-}-//2+2).  28.   {6m-7n){4:m^  +  n^). 

29.     (.6-"' +/)(.*;" +y").     yO.   (.c-+.c//  +  ^;x-+(/2)(a;-^+sr^  -ax-y/)^. 

Exercise  xxxvi. 

1.  {x~y){x-{-7/)(x^+x;/  +  !j^){x^-xy+!/'),      {x -l){x-' +x  +  l), 
{x+2}{x^-2x  +  4:),      (2a--Sx){ia'+(iax-\-dx-), 

{2  +  ax){4:-2ax+a-^x^). 

2.  {x-a^){x'^+.r'^a-+x2a'^-\-xa^+a^), 
(3f(-4)(9aa-[- 12^  +  16),   {a^  -b-^){a^  +  b-'){a<^  +  b^), 
(a;3  - 2//)(u;»  +  2x-«//+4.i;4^iJ +  .-:Jx-' //3  + 16//4). 

3.  (rt-/0.  4.     a:+4.v.  3.     (.c+//){ic- +2/3)(u;4  +  //^) 
7.      i'(a;     ?/)(i/-hl).                        8-      {x-a)[x^+ax+u-)[a  r  b). 

11.  {a^+bc){a^-^a^bc  +  7b2c-2). 

12.  (x-a  +  ^){(.r-(0^-(^-«>  +  ^^}- 

18.  (a;2  _2x-//  +  4]ry3)(.c+2?/+4.r,y). 

!4.     {2x+S,j){2x-3y)^.  15.     (1  -  2.r)(l+4.t3). 

LG.      (a-^ -^ubc  +  b^c^){a+be){a^  -abc  +  b^c-'i). 

Exercise  xxxvii. 

1.     d(x-\-y){y  +  z)(z  +  xJ.  2.   {a-b){b-c){a-c). 

3.     3(aa-/>2)(>-!-6-3)(c2-(,3).  4.   ^^_j.^)^^_|.^)(2_j.a;). 


XTl  ANSWERS. 

5.  8(r;f  +  /;)(/;  +  o)(c  +  a).  6.   («+fc+c)(a-?v)(&-c)(c-a). 

7.  (a+//)(ft  +  c)(f+rt).  8.     („3-Z»)Z.2_(.)(t.3_«). 

9.  (a  +  A)(/;  +  f)(c  +  a).  10.     (a-6)(6-c)(c-a). 

11.  (.r3-f?/2)(//3 +23)^23  4.3,2). 

12.  («3  _f.52  .|.  c^^ab~bc  -  ca)ia  -  b){b-c){c-  a). 

13.  (a3  +  />2^c2)(«4-&  +  c). 

li.  (C-i3)(a-c3)(6-a3).  15.     (^2  _2/2)(y3  _22)(,c3  _2-2). 

10.  (x  +  ?/-f2:)(a;-//4-z)(.7-c+.'c)(z  +  2/-«)- 

17.  (a-b){h-c){a-c).  18.  8(a  +  i4-c)3. 

2i.  (a-6)(6-c)(a-c)(rt2  +  A3  +  c2+<76-f-/;c+ca). 

ExERcisK  xxxviii. 
1.     (r,-2)(a2-7«  +  2).  2.  {x-2){x-S){x-'i). 

3.     (x-3)(a;-2)2.  4.  (a;-2)3(.r  +  4). 

5.     (x+l){x^  +2x-rS).  6.   (a;2+2a;  +  3)(x2+2x  +  3). 

7.     (x+2)(a;-l)3.  8.  {x^+2x+3)(x^ -2x+'d). 

9.     (»t-«)(m2-2wi»— 2?i2).  10.  None. 

11.  (M-n)(»/-2u)3.  .  12.   (6+3e)(i2-26c  +  13c2). 

13.      -(m-w)3(w2-wn  +  >().    14.  (rt+26)(fl— 26)(«3— 7ai+46-'; 

15.     (x-5)(.-c-3)3.  16.  (.«+2)(x3-|-3.c+l). 

17.     (r*-l)(a2_2a-195).        18.  {p  +  2){j)—l){p+4:). 

19.     (a-l)2(«+2)(a+3).  20.   (fl.3"-l)(a=''-2)(a2"-3). 

21.     rt2  +  463+7a&.  22.  (a-i)2(a3+2a6  +  263). 

23.     {jj-2){2J^-2j>+-2).  24.  (.c"-l)(a;'-»  +  ua;" +5). 

25.     (y/-2)(,)/5 -37/2 +2^+4).  26.  None. 

27.     (fl-/^)('t*-  +  2ai+362).       28.  (a''+l)(2«3»_3rt»  +  2). 

29.     (x-2)(x-3){x-6){x-7).  30.  {x-t/){x-2y){x-3i/)^. 


ANSV/F.R3.  XTU 

Exercise  xxxix. 

1.  2(x-l){x"--9x-\-10),  {x-2i/)^x-3y). 

2.  (ix  +  3i/)(3x'-i-x!/  +  y''),  (x-l)(4c-2)(2x+3). 

3.  {x-5ci){3x^+a''),  {2x  +  oy){x^-{-3xi/-y^). 

4.  {b+c){b-4:c){2h^~bc-\-c2},  (5a+4i;)(3a3+76i//-3&3). 

5.  {2p+q){2iJ  +  3q){p^+q-^). 

6.  (lOx-  9v)(15^+  16//}(x-3  -  5xtj-\-8y2). 

7.  (2./:-3//){2x--f3//){3a;+47/)(3a;-5?/). 

8.  (5x  -  2z)<^2.cS  -  3^2^  +  B^^a  _^  12^3^. 

Exercise  xl. 

4.  (^+2^)(a;2+Hv-).  5.  l-2x-+3a;2. 

6.  {a~x)ia-\-x)^.  7.  a;^ +z/^ +s3 +ie^+7/2-2x. 

8.  [a  +  />){da+b}.  'J.   (a;-2/)(2.c  +  3^). 

10.  aa_/,2_f.^3.  11.  7a2-Sab+2b2. 

12.  a -7.  13.   {a-h)(b~c){a-c). 

14.  (x-a)2-ft(.a-_^,)-f/.2.      15.  a;2_j_^2_j_22  ^1, 

16.  x{x--ax+b).  17.  rc3  +  ^3. 

18.  {x-y){x'+y^).  Id.  a^-b^+c'^+l. 

20.  a3-63_c3,     21.  'i-i-x.    22.  (c-i)(a-f-6  +  r). 

23.  ab  —  ca  —  hr.  24:.  x^+y^-\-l-xy'\-x+y. 

25.  (x3-2)(a;  +  l).  26.   a^+5a-\-B, 

27.  (2.<;-t/)a2-(x-  +  //)a.7:+x3.  28.  a(a;2+a;+l)-(a;4-l). 

Exercise  xli. 

1.  j:2-3.  2.  x-\-o.  3.  a;3-.T+l  4.  aa;^'4  6a^4-c 

5.  None.  6.  c^+c*.         7.  (a-i)(.cf«).      8.  6(a;+3/). 


XVm  ANSWERS. 

9.     (a-b){b-c)(c-a),  10.  «.2'"+l. 

12.      5(a-b){b-c)(G-a).  13.   {y-l)(x-l). 

15.     (x+l)(x~  +  l){x~l)^.  16.  {x+l){x+2){x+'6)(x  yi).  17.5 
18.     Same  as  given  quantity.     25.  (a  — 6j((!>-c)(c-a). 
29.     a;^+a;2+2./;-M. 

Exercise  xlii. 

1.  (.r-l)--(a;2+4x-  +  16),  a;(3^-7)-^y{77/-3). 

2.  (^^-ax^a^)-^[x--a^),   (x  +  4)-^U- 1)^. 

8.  (a;-l)(,x+2)-^(a;2-f5.*;  +  5),  ix^  +  2xyd)~{x^--2x~3). 

4.  l-(6-2./:),   l^(ar3_2.<;4-2). 

5.  5a^[a  +  x)-ir-x{a'-'+ax-rX-^},   {4:X-  +l^^{5x^  -i-x+1). 

6.  (a;-7/)-^(;c-L//).  7.   [Sax^  -hl)~{ia^x'^-\-2ax^  -1), 
[ax^bi/)-^{''X  —  bf/).        8.    —1-T-abc. 

9.  -(.,+/'+')-H(a-6)(6-f)(c-«).     11.  5-7(»2  +  a;2/+?/3) 

Exercise  xliii. 

1.  (4-.r)--(5-a-),  [a^ +h-)^2ab. 

2.  X,   2a-(«--M).  3.   r^(l+rO--(l+2r,+.3rt2),   X. 
4.     b^-i-a^,   {h  +  l)-^cib^.  5.  («c--M)-=-(ac+^)c^),  ^n-a. 

6.     l  +  6x27/z(7/+2j--{y-3--s:3(v/  +  2!)3}.    7.  (a^ +63 ^c2}-v-a6c. 

8.     1.  9.    -0/^+a^6^+o4)^rt6(a_6)2. 

/I— a;\  2 
10.     {a  +  b  +  cy-i-2bc.  11.     |j^l    ,    4a2a;2H-(a2+a;2). 

12.     (a.-+2/)-j-{a;-y).       '  13.  (a-6)S-=-(a  +  6)3. 

14.     (a;  +  ?/)^(.<;-2/)  15.   l^a;3, 

m.    1--W.  1/.  ±(l-i)-T-(l  +  o).  18.  l-»-c 


ANSWEBS.  XlX 

Exercise  xliv. 
1.     (x-a)-i-5.  2.     a+b.  3.     16a^x^  (a*  -a:"^)*. 

4.  0.  5.      1h-(5:+2).  (y.      lH-(n4-a;4). 

7.  12^?/^(9.r2-4y3).  8.     (4^2 +2) --.-'•;  10x4-1). 

9.     l~-{x  +  l)(x+2){x  +  B).     10,  4(a;4  +  4^2y2+^4j_j_(.c4__;y4), 

11.  (^a-b)3-i-{x+a)2{x+b)".       '  12.     2,',- --a;. 

13.  mG-77x)^18{llx-  8).  14.     l-^(a-6). 

15.  15a(8a-«)-^(9a4-2a:)(«  +  3a;).      • 

16,  {10x~7)-^{x~l)['2x-5)-l-^{-2x-l',.--4:).  17.     Z. 
18.  y^y^'-K").                      19.     (fl-/>r"+2. 

20.     0.  21.     4.c^~ix'2_i). 

Exercise  xlv. 

1.  a;-y.  2.     a  +  h.  3.     0.  4.     0.  5.     0, 

6.-  {{a-\-h){c+a)x^-\.2iah  +  >'  +ca)ax-%i''hc]  ^      • 
(fl  +  //)(rY+r;)(x  +  a)x(a;  +  />)i>+c.)  7.      1. 

8.  «+i+c.  9.     1.  10.     a;3-.v3  11.     Q. 

12.  (ff-fc)(/>-c)(a,-c)-i-(a+6)(6+c)(c+«). 

18.  a;2^(a;-fl)(a;-6)(a;-o).  14.     1.  IC.     0. 

16.  {h{;x  +  a-b)-\-ax]  ^  {rt64-(o-rt)(x— o)}. 

Exercise  xlvii. 

1.     («-i,)2+4c.2^0.  2      8.  8.     10.  4.     rt2+i». 

5.  7«.  =  2,  ?i=l.  6.     2a;2,  or  5.  7.     7?7=-5.  7?  =  6. 

8.        +12,       9.      (a2+/;3l(c2+d2).       H.      _  3?;C  -  4^2  4./,2^2  _  4^2. 

12.  (a;2-4a;  +  3)(x2-4),    also  (a;^ -3x-4-2)(.r2 -« -6). 

13.  i(-l±i/5).  15.     a-^c  =  d-^  ^e-,  a^b^p  ^e^. 


XX  ANSWERS. 

b^c  =  d^  ^p.  17.     ac^  =  b^d  and  9ad  =  be. 

19.  ip^  +  27q  =  0.  24.     p  =  27n^q±2mqV{m^-^l'}. 

26.  4(jt)-3)=2. 

Exercise  xlix. 

1.  5,  31,  a,    -3.  2.      -4i,    -a,  2,  10. 

8.  a  +  ^,  c-rt,  6-c,  3.  4.      -2,6,    -5,12. 

5.  -14,  a -3b,  2a- 3b,  5b -3a.  6.     7,  4,  a,  6. 

7.  ic,   5-=- a,  0,  1.  8.      -1,   {(a  +  b)^-a}-i-b,  a+b. 

9.  (/>-«),  a  +  i.         10.      1-^a-h,   l~(a-b},   l^(a-'  +  &£). 
11.  2b,  a.  12.  a +  6,  c-^(rt  +  6),  b-^{a-c). 

13.  (i-c)H-(«-6),  i  +  c.  14.     a+b,  a^-^ab  +  b^. 

15.  «2_a&+/,2^   1.  16.      -1,   (n-{-b)^{u-b). 

17.  (e  +  i)(e_/,),  2^15,   3---14.  18.    -1-12,  b~-ac,  a-^b. 

19.  («-+;>2)-^rt-7>2)-^a2/ja,  ^(63-|-c3J^5p, 

20.  10,   12,  4,  4.         21.   1000,  f,  f.  22.   9/^,  ab,  bc^a. 

23.  h^^ac,  c{n-\-b),  b{a  +  b)^a. 

24.  a-f-i,   {a-b)-^(a  +  b),    -{a+b)^^{a-h)^. 

25.  -1,    -1.  26.   (a2_c2)--(a+6)2,  2,  3^. 

27.  ab,  b^a,  nc^b,  12.  28.   12,    -ac-^6. 

29.  9,  2.        30.  12,  1.        31.  3,  1.        82.  (2fl -l)(2r,  +  2),  0. 

33.  l-!-m.         34.  1.  35.  {ab  ^-bc-\-ca)---{a^  +bc+c-^). 

36.  {a^Jrh'+c-)^{ahJ^hc-\-ca).         37.   a  +  b+c.  38.    1. 

39,  1.         40.    1.         41.  1.         42.  15.         43.   16^.         44.  6. 

45.  5.  46.  {npqa+j)qb  +  qc  +  d)-^vmpq.  47.    —  ^. 

48.  0.         49.    -25H-136.         50.  1. 


answers.  xxi 

Exercise  1. 
1.     2,  3.         2.   *,  h         3.   +2,  li.  4.  1,  H.         5.   ±f, 

d=(a4-*),  «•  6.  4,  S,  2,  2i.  7.    -8  or  2  ;  4,  -3  ; 

2i,   -li.        8.  1;  for-l;  i  01-3.       9.    -f  or  |,    ^  or  6; 

4  or  -|.         10.    -1,  2,    -i,  1.         11.  0,    -Z*,  3ft. 
12.      a,    +av'-l.  13.   1;  l(  — l+v/5).  14.    ±rt. 

15.      +6c,    -(/)+c).         16.  rt  +  2Z>.         17.  6or+rt. 
18.      -2ai,  iab{lztVl).  19.  «,  /;,     -  (a+i).  20.  a,  b. 

21.     aorl-rt.         22.    -a,   -b,  a-2b. 
.2a     a,  6, /;(1  -/;)^(l+«.-6).  2-1.  a;^ -6.c2  -  37.1-  +  210. 

25.  a;4-4fla;3-13rt2a;2+64rt3a;-48a4. 

26.  a;(cc-l)(a;H-2)(a;-4)=0.  27.  a;* -4A-3+a;2  +  6x-f-2  =  0. 

Exercise  li. 


« 


31 


1.  4.         2.    -7f         3.    -107.         4.  8.         5.  3./.      6 

7.  50|o,  17.         8.  22,  46^.         9.  7,  3.  10,  10,  10,  11. 

11.  0  or  11;  33.  12.3956-^3971.  13.     i(15+s/190). 

14.  3.         15.  3.         16.  4.  17.   If.         18.     l^-.        19.  3^. 

20  4.  21.    +3.  22.   11.  23.   2  aud  -1±:|/ -3. 

24.  2^.         25.  0.         26.  3a.         27.   S-  28.   \§.         29.  3, 

80.  10.         31.  0,  1,  or  (-5±a/-23)-^8.         82.  102|. 

33.  (-ll±i/4681)-^20.         34.  2,  i,  |.         35.    --4 

86.  0  or  ±:y  (fl2+62). 

Exercise  lii. 

1.  {l-a)^(l+r7),  rt(m  +  l)^(/»,-l),   h(m-^l)^a{m-l). 

2.  a -6,  0,  0.  3.      h,  ma^h,   h-^ca.  4.      1,    -1,  0. 
5.  -|or-l.         6.      (c-/;)(ft2_f_c2):^2a6c.*       8.     14,  4^. 
9.  2;  6^295.       10.     73 -=-210,   {a -^b+c  +  d)  ^(m+n). 


XXU  ANSWERS. 

11.  h-^a.  12.     h^a.  13.     a  or  0. , 

14.  ±:|/a2+l-j-2.         15.     |.  16.     ||.  17.     0  or  4. 

18.  c-^ab.  19.     83|/(2a;-l)  =  100A/(3x-3). 

20.  75 --52.  21.     8.  22.     34^^?- 

23.  l^«(n-l).  24.     ac-^{h-a).  25-.     4,  3i  or  13^. 

26.  a2j2^(rt_6)2^  3      27.     4a2^(l+a)2,    6(a  +  6)2-7-(a-6)2. 

28.  (1+62)  ^2a6.  £9.     ■,/(!-«)  =  2  ^  (a +  1)3. 

30.  -a  +  aV{(l  +  &  +  i2)-j-26}. 


U-1/     -     U-1/ 


5 


ExEEcisE  liii. 

1.  8.  2.     0.  3.     3.  4.     (v/m+|/n)». 

5.  ^;,^(1-2n/&).  6.     4-f-7.  7.     l-^(a-2). 

8.  18962^12393.  9.      V^-^  ( v/^v  +  2). 

10.  (c4-2/;c2)-T-(2c-2_26).  11.     i.  12.     18a. 

13.  .r2  =  80-^81.  14.     ±V°n/^t-         15.     +Av/-ll. 


16. 


hs/    !«=-  l^^-^^^^'U  17.     0.  18.     ^. 

19.     (c-a-6)-'''  =  27a?'c.  20.     a;2  =a2(«-l)2  s- (2?i-l). 

21.     16.r/y=(»-4x--2/)2.  22.     0,    -ff 

23.     /   ""^       -1)  ^  0.  24.     2s/(l -w2) -i-m  ^(4 -m2). 

\2rt  —  2         / 

25.  (rt3_i)|,,2+2+ V(a^  +  l)}H-a2.' 

26.  (m-an+c)2H-/;(n-l)2.  27.   ±5. 

28.     2V(3x2+*10)  =  (17i/17-3^3)---7.  29.   ±5. 

80.     ±i/(36-2a).  81  |/|(a2_fe2). 


ANSWERS.  XXm 

32.     {2i/  +  2z-2x)^-{-2Wx/jz^0.  33.     |«-^/6. 

34.  a(^^2-4/t+_8)  -  (2«-4).  35.     a^ +2a, 
36.     ±|/(3ft2+i2)^^3. 

Exercise  liv. 

3.  {(a-&)«2_2c(«2+a^  +  62)|^|„2_2t.(a3_/,3^j_ 

4.  —6.  5.  a  +  6-fc.  6.  ab-r-{b  —  a). 

7.  a;2  — 3«a;-<*2=0,  &c.      8.  a.       9.   K«  +  ^  +  <')      10.  I^a6c. 

11.  l3;(a  +  6+f).         12.   {a-b)[ac-2b)-^{a  +  byiG.        13.    -c. 
14.      (ifa4-^/>)3.  15.    ±2.  16.   c^{a-b). 

17.  {a-b)^{a  +  b).     18.  |a.  19.   ±2.         20.   ±2,  &c. 

21.  i(a  +  c)-^(«-c).    22.  a,  {Sab-Bb''  -a)^{l-\-'da~db). 

23.  «.      24.  a,  b,  2b.        25.  a,  {c- .+  Qab)^Gb.       26.  i(c  +  6rt). 

27."  ia.    28.  r(  +  6.  29.  (a6  +  k;+ca)H-(«  +  64-c). 

30.  ±b,   ±a.  31.  |/{l-=-(a     1)}. 

32.  {6{a-b)-4:c{c~b)}-^{4:C-db-a). 

33.  (c-2-a/.)^(a  +  6-2(;)  34.  -^(- 29+ v/37). 

35.  (a;  +  a)2=2i3-a3.  36.    v/(^^2._|«i),         37.  ^(&- a). 

38.  8h  I-     89.  x--6x  =  a.     40.  1±]/19.  41.  Z*,  b~a. 

42.  ,  («3  +  63)^(a  +  6).         43.  a;=-5-^2.         44.  -1(5+ a/3). 
45.     -2a,  -^a,  |a.  46.    -3a. 

Exercise  Iv. 
1.     bc-i-(a  +  c).  2.  (a3  +  6_2a5)-j.(a  +  ^>2). 

3.   (arf-66)-^(«-i).  6.     -^;X7-  C-  ^-         7.  ^(a+6) 

8.  a  +  6.         9.  0.  10.  0.  11.  abc. 

12.  («2+62+c3)--(a  +  fc  +  c).      13.   (a  +  &  +  c)-(a2  +  63+c2). 


AXi^'  ANSWEES. 


a^b  ,^        ah 

14.      {a^^h'-^-\-c^)M'tb  +  bc  +  ca)    15.^3^.  16.    ^^• 

17.     4*.      19.  4.        20.   -140.        21.  17.        22.  10.        23.  a. 
24.    ^!!^'7^l,.     25.  3i,0.     26.3^.     27.  («6-6-2)--(«  +  Z>). 

r^<  (lb 

28.  -i,  «,       29.  0,  0.       30.  U'l-^b-c).        31.    —-,. 

32.  d.       33.  fl6--(a3-62).       84.    -3|.        35.  f.        36.   -3§. 

37.  Infinity.  38.  10.         39.  abc-^{ah  +  hc  +  m). 

40.  {ab-\-hc  +  ca-ad  —  bd-cd)^(a+b-{-c-Bd). 

41.  a{b  +  c)^-^(b^  +  rJ-ab-]-bc-ca). 

42.  Z*c{f/-«)  +  (a-6)(&-c)(c-rf)-=-(«tH-ic4f'?-'^'d-&---c2)- 

43.  bc^—b^c-ac'^-\-b^d-abd+cicd^{ah-^bc-ac-h^). 

44.  _(fl_j_&.4.(.).  45.  rt  +  /)  +  c.  46.  {ab  +  bc  +  ca)-^abc. 
47.  _i(i4.c).  48.  («^  +  c)--2«.  49.9.  50.2.  51.7. 
52.  4.  53.  r,S(5±^/7S5).  54.  4,  (am.-nb)-^{n-'m  +  a-~b). 
56.     ^Ij,  b{a  +  o)-^{cr--]-ab-\-b^).     56.  0,    -|,  *.  57.  10 

n(i{ap-\-i)ih)  —  wp{cq-\-nd) 

58.  («prig - cmjpq)-^{apn^  +cqn,2);  ^^^^^^2^,nnH  ^^n^f^^^^m^d' 

59.  rt6-(6-c),  c{a2  +  (6-0«-ic}  +  a(62_f2)^      . 
((j2^i2_c24.rtJ_5c  — rtc).       60.  b-^{u  +  b), 

61.     mpcq-]-apnq-~{Cjtn^  —cqrn^). 

02.      {&m(a— c)4-CT«(6— rO  +  f(/)(c-5)}-^ 

{m((a-c)+n(i-a)+y>(6--i)}.         68.    (a2-|-,62)^,,6,  0,  ^. 

d{u-q)-q{h-d) 
64.     (a;.-cm)H-(«7i-im),  ^,, _ ^)  _ ,,,(^ _ ^y  ^  55.   i,    3. 

66.     100.  67.  13,  111.  68.  11,  7. 

a-2  +2ac+«rf+26c  +  2«fe 
69.     (rt  +  ft-m-n).  70.    ^3-^ ; • 


ANSWERS.  XXV 

71.     ^^^h)  +  i/{^(a-hy-^c^\,aoYb.     72.0.      73.  rt  +  6  +  c. 

74.     ^^1!+!!^+^^  75.     a  +  fc  +  c.  76.     a  +  b  +  c. 

abc 

77.     (a/*  +  /;c+r'^)H-(a4-?'  +  c).  78.     b^+a^-c^. 

79.     c-a-6.  80.     0.  81.     0  or  11. 

ExEECiSE  Ivi. 
1.     ^  =  0,  orB  =  0.  2.     A  =  0,  or  5  =  0,   or  C'=sO. 

3.     x-0,  or(i—b  =  0.  4.     x  =  0,  or;y  =  0. 

5.     In  the  first  case  either  .r  —  5?/  =  0,  or  x— 4y  +  3  =  0,  in  the 
second  case  both  conditions  hold.  6.     x  =  0,  or  x~a. 

7.  x  =  0,  orx=—b.  8.  x  =  a,  ora;-- c^-&. 

9.  a?  =  0,  ora:  =  3.  10,  x  =  0,  or  a- =  a  +  h. 

U.  3;  =  0,  or  «=+«.  /           12.  x  =  0,  or  x^b^-^  a. 

13.  :c=:0,  orx  =  a.  14.  x  =  0,  or  a. 

15.  x  =  0,  or  ic=rt  +  &.  16.  a;  =  0,  ora  +  i^. 

17.  -{2ab)-^{<i  +  h).  .  18.  a:  =  rt,  or  6. 

19.  «  =  a,  or6.  ore.  20.  5.           21.     1.          22.     21. 

23.  x  =  \,  «  =  3.  24.  a;  =  9,  .r  =  4. 

25.  x=l,  orB.  26.  (a/O  ^  («'  +  ^'). 

27.  x  =  a,orb.  28.     a;  =  (a^ +/,2)  h- («+i),  x  =  /v+a. 

'J.9.     (2r/.ft)-i-(rt  +  '0-  ^^-     «  =  «,  or^. 

1.     a;=i,   or  (l+a)-f-(l -a).  32.     x  =  a. 

34.     a;  =  a  -  6,  or  i(54-c).  35.     x  =  «+??,  or  i(rt  +  c). 

36.     X  =  — " — ,  orl.  37.     a-^b-c, 

a^rb-\-c, 

38.     x  =  rt,  or  ^(46-a).  39.     x=  -c,  or  a^h  +  c. 


XXVI 


ANSWERS. 


40.    x  =  lf  or 


m—n 


41.    X 


_    nc—pb 


n—p 

42.     P{'i—^)-<^{m-'>^) 
)n{c  ~  b)  —  a[n  —  p) 

•1:4.     x  =  2a  —  b,  or  36 — 2a. 

17(7-76 


46.     «  =  «  +  /',  or 


mc  —  ap 
43.     a;  =  i(a+6),  or  ^(6-4 

45.     x  =  a-\-c  —  b,  ora;  = 
4a+4c-26 


47.     x  =  4:a  +  h,  ov  a  +  b. 


48.     a; 


8 


• '      0  a 

- — ,  or_ 
b  —  c  0 


49.  {a -  h){b  -  c]x^  -  (^2 4-^3  ji^c-  -ab-bc-  ca)x+ 
{n-c){a-h)z=Q. 

50.  x=±d,  or  ±2.  51.  a;=±G,  or  +2. 


a-\-b 


52.     a;  =  3,  ori.     53    a;  =  ^^,  or  ^ 


a 


r.     54.  x  —  -r-,  or 


a 


55.     x  =  b  —  2a,  or  a  — 26. 


•  2rt  +  36         Sa  +  26 
56.    a;  =    — ^ ,  or 


5      '  "^         5 

57.  x  =  {tnb+na)-i-{m  +  n),  or  (wa-w6)-7-(m+n). 

58.  a;=v/{(»i  +  2/i)-  A/(w-2«)}^l/{(m-|-2;i)+ i/(»i-2n)}. 


59.    a 


[v/(c+i)+  A/rc-1)) 


60.  a{y(3c-2)  +  i/(2-c)}H-{i/(3c-2)-|/(2-c)}. 

61.  a{v/(2c-l)  +  l}-=-{l-'/(2c-l)}. 

62.  H«+26).  63.  i(a+6)-(a-6)v'(7n-2w)-=-y(w+2«) 


64. 


66.  « 


26 


,  or  — 2 •        ^^'    2a6-^(a+6). 


3a  4- 56         86— 5a 


8 


-,  or 


8 


67.    «  =  2a{v'(c+4)-i/(c-4)}--{v/(c  +  4)+  \/(c-4)}. 


ANSWERS,  XXVll 


68.  ir  =  4,  01-3. 

69.  ^{«±:|/a3-4w)  where  m  =  «2+^\/(''+«*);  3  or  1. 

70.  a,  b.  71.  x  =  ^[a  +  b±i/{{a  +  b)2—4.{ab-\-t)}], 

72.  a;  =  0,  or  a,  or  ■^rt(ldr-i/-3) ;  a;  =  4,  or  2. 

73.  x  =  0,  or  «+&,  or  ^{{a+b)±lV{a-b)^-4:ab}. 

74.  a;2-(a-6)x+a6  =  &c.  75.  x=  ^(3a-fc).  or  ^(3fe-a). 

76.  a;  =  3a— 2&,  or  3i-2a. 

77.  y^  -m^  =0,  where  y  —  m—x  and  2m  =  a+b.     See  Key. 

78.  2/^-»*^=0-         79.  ^2_^2=o.  80.  y^-m^  =  0. 
81.  i/^-m3=0.         82.  t/3-m3=o.  83.  ?/3-w»=0. 

84.  (?/2-^3)(52/2^.7;^.2)^0,  (where  also  k=i(a-b), 

85.  k^-y^  =  c.  86.  A.-  +  10^3y2^5^^4  =  c(^4_y4)&c. 
87.  s?/ihA:N/(A;  — 3c±r)  =  0,  where  s^^g^  +  c,  and  r2  =  (A;— 3c)* 

4-(^_c)(3/.-+6-).         88.   -3±:a/(9  + 12/24). 

89 — 102.  Work  with  a  variable  w  such  that  wx  =  x^+l. 

89.  w  =  {a±s)^b,  where  s  =  a^  +  262. 

90.  «j  =  (3a+26±i)-=-2(a-6)  where  s=  ±:6/(a3+2a6+462). 

91.  M;  =  (3±:s)^(l±i>)  where  s  =  (6  — 4a) -^6. 

92.  (M;  +  l)3=a-^(«-i).  93.  Mj3==2a-^-(Z»-«). 

94.  {x+l)~{x-l)  =  a^{a-%h). 

95.  (w  +  2)-^(w-2)=^(l  +  sJ  where  s=(16rt+6)H-6. 

96.  «;3(4a-6)-^-(a-6).  97.  ■«;3  =  (4a-36)H-(a-fc). 

98.  M)  =  (6±s)H-2a  where  s^  =  63  -(-16a2. 

99.  u;  =  (a  +  i»±s)-i-2(a  — 6)  where  s  =  (a  +  6)3 +8(^a -6).. 


•XXVlll  A>'SWERS. 

100.  Wz^(a-\-b±s)-^2{a-b)  where  s^-^{a-b)^  = 

101.  (it;-f2)it(t«-2)=+s-=-(4+3s)  wheres2  =  2a^(a  +  6). 

102.  {w-{-2)-i-iv=±]/{5a-i-{a+U)}. 

103.  i{2a  +  b),  i(a  +  26j.  104.  2a-6,  |(a+6),  &c. 
105.  1,  2,  4,  5.  •           106.  ±1,  2,  4.  107.  1,  2,  3,  4. 
108.  -^,  -1    1,  f.     109.    -1,  3,  4.              110.    -a,  o^t,  5a, 
111.  15,  20.                    112.  2ia.                        113.  4,   -1. 
114.  7,   —1.     115.  ^(6c-^-a  +  6•a-^ft  +  rt/;^c).     116.   ±a^m,  &c. 

117.  26(s-a)(s-6)(«-(-)^V^{s'2_a2j(s'3-6)(.s-'2_c2j}  where 

2s  =  a  +  />+c,  26i=a3+i2  4-c; 

118.  (2«6+2ac2  +  26c8-a3-6'--c4)H-4c2.    ll'j.  a,  b,  ^{a  +  b). 
120.     ±a  or  ±JaV^'  121.  a,  6.  i(ct+6). 

Exercise  ivii. 

2.  a;,  2;  ?/,  1.  3.  x,  8 ;  y,  1. 

5.  X,  — lOi;  «/,  51  6.  a;,  -2;  t/,  ^. 

8.  x,  -2;  2/,  -3.  9.  X,  -f;  2/,  i. 

11.  aj,  12;  y,S.  12.  x,  8 ;  ^,  -9. 

14.  X,  12;  t/,  15.  15.  x,  18;  2/,  13. 

17.  x,l\  y,  9.  18.  x,l;  y,  -3. 

20.  a;,  2 ;  y,  3.  21.  «,  3 ;  y,  4. 

23.  X.  -3;  y,\.  24.  «,  12 ;  //,  15 

26.  X,  8 ;  y,  9.  27.  a;,  3 ;  y,  1. 

29.  a;,  11;  ;/,  7.  30.  x,  17;  y,  13. 

32.  a;,  -4^;  ^,  -|«.  33.  a:,  13 ;  y,  10. 

84.     a;,  4|;y,  3tV     ^S-  x,ll',y,Q.  36.  x,  7 ;  y,  5. 


1. 

X,  7 ;  y,  9. 

4. 

x,  9 ;  y,  5. 

7. 

«,  -1;  2/,  1- 

10. 

X,  -i;  ^,  f. 

13. 

«,  10 ;  y,  12. 

16. 

X.  -3 ;  y,  -2. 

19. 

a;,  7 :  y,  3. 

22. 

«.  r ;  3'»  ^• 

25. 

»'  Tff ;  2/'  A- 

28. 

a;,  7  ;  ?/,  8. 

31. 

a;,  5;  y,  -4. 

ANSWERS.  XXIX 

37.  x,2;  y,  3.  38.     a;,  5 ;  y,  3.  89.     Equations 

40.  x,d\  y,  1.  41.     x,l;y,5.  not  independent. 

42.  x  =  0  =  y  =  0.        43.     0,0.         44.     a;  =  Oor  13 ;  y/  =  Oor  ^f. 

45.  X,  17;  y,  20;  z,  5.  46.     a.-,  ^„  y,  \%^,  z,  f|^. 

47.  11,7,9.  48.     21,22,23.  49.      -15,-0,-8. 

50.  3,  4,  5.  51.     12,  15,  10„  62.     5,  3,  1. 

53.  f,  1^,  |.  54,     3,  5,  7.  55.     11,  13,  17. 

56.  5,  3,  1.  67.     9,  7,  3.  58.     Ik,  8i,  9^. 

59.  3f,  2|,  1|.         60.     2-3,  3-4,  4-5.  61.     30,  20,  70. 

62.  88  H-  59,  1098  h-  59,  1004  ^  69.  63.     30,  12,  70. 

64.  6,  12,  20.  65.     5,  2,  0.  66.     1,  1,  1. 

67.  11,  9,  7.  68.     6,  3,  1.  69.     2,  3,  1. 

70.  3,  4,  5.  71.     h  h  i-  72.     5,  4,  3. 

73.  7,  3,  1.  74.     2,  3,  1.  75.     1,  3,  6. 

76.  0,1,2.  77.     1755-698,   360 -f  319,   -15705^698. 

78.  ^,i,  1.  79.     5,4,1,3.  80.    4f ,  3^*,  2^,  U- 

81.  31,  41,  51,  21.  82.     7,  4^  4,  8i. 

83.  20,10,0,30.  84.     11 --24,  J-,  1  ^24,  ^. 

85.  270 -- 117,   -52-- 117,  15-^-117,    -126--117. 

86.  Eaok  210. 

Exercise  Iviii. 

1.  [a'c—ac')-i-{a'b  —  ah').         2.  b(cn- dm)-^(ad—bc). 

3.  b{d-c){d-a)^d{b-c)(b-a),  c{d-a){d-b)-^d{c-a){c~b). 

4.  ?/  =  cz+f^'^  +  ^M^  +  ^aJ)  2  =  rfM-j-eif'-|-aa;+«t/, 
u  =  ew+ax-\-by  +  cz,  iv  =  ax  +  by-{-cz-{-du. 

5.  a;  =  ^m(a  — ^4-c),  &c.         6.  x=  {j:>(a3  —  6)  — ?n(aA  — l)-j- 


XXX  ANSWERS. 

n(b^ -a)'^{nS  +  b^  —  Sah  +  l},  &c. 

7.  x={l  — am  +  abn  —  abc27-\-abcdr)-^{l-\-abcdf),  &e. 

8.  l=a-^{l-i-a)  +  b-rr{l-{-b)-{-c-i-[i+c). 

9.  l=-ab  +  bc  +  ca-\-2abc, 

ExEEasE  lix. 

1.  (nc  —  bd)^(na  —  bm),  {'mc  —  ad)-~-{mb  —  na). 

2.  (??a+6d)-r(aw  +  6?n),  (mc  — acZ)^(6w+(m). 

3.  c{n  —  b)-i-{an  —  mb),  c{jin  —  a)^{hm  —  am). 

4.  (6  — c)a-^(/>— «),  Z/(a  — c)-i-(rt  — i).     5.  a5-T-(a+5),  t/,  same. 
6.  a62-^(a2  +  '^3)^  a,n^{a'^+b^).       7.  ac-^(a4-6j,  6c-r;a  +  /:') 

8.  {a-  —b')-T-{am  —  bnfy  {b^  —a^)-T-{bm—an). 

9.  «  +  6  — c,  c+a— &.  10.  a  +  c.  b+c. 

11.  a(c»— <f?n)-j-(ii  — ac),  b{cn  —  dm)-T-{ad  —  bc). 

12.  t/={93(a2_c2)_o(6+2a)}^{(a-Z>)'^-i-s  +  4ic}. 

13.  a  +  6-c.  a-b  +  c.  14.  a+6-6-,  c+a  — 6. 
15.  (TO-a)(?i  — a)-5-(6-a),  &c.       16.  l-*-(a-^)(a-c),  &c. 

17.  (Hi-ic)(?--«)-^(c— a)(a  — 6),  &c. 

18.  x=p-i-{pl+mq-r'iir)+a,  BO  y  and  g, 

19.  p{l  — (?rt4-w6  +  wc)}-^jp^+m3+nr)+«,  &0. 

20.  {m^  +  '2a^ -b- -c^)-^Sa,  &c.    21.  y  =  a  — 6-fc,  &c. 

22.  x=  {ab+bc-\-ca){b+c-2.a){2b-a~ c)-i-{{a- c){b  +c  ^  2a)-{- 
(^_c)(2&-a  — c)}.     Corrected  equation,  a;  =  i(i4-c),  &c. 

23.  nui-^{a-\-b+c),  &C,  24.   npr-^{anp'^bmp-^cmq). 
25.  l-=-(6-c),  &c.  26.  -^(i+c— a),  &c. 

27.  a«H-rf,  &c  28.  «=l-^(«+6-c). 

29.  a+fe,  &c  80.  l-^2a,  &c. 


ANSWERS.  IXXl 

31.  (mS+nS-ZSI-^-Smw,  &c.  S2.  i{a  +  c-b),  &G, 

83.  l{m'^ +n^)-i-'imn,  &G.  84.   l-=-(ft  +  c- a),  &c., 

85.  bc-^[b-\-c),  &c.  36.  />  +  c-a,  &c.^ 

37.  a,  6,  c.  38.  ^>^-c3,&c. 

89.  i(a4-26-c  +  3fZ),  &c.  40.   i(4a+^  +  3c-2(/  +  5«). 

Exercise  li. 
1.     ,,  +  ?,.  a-h.  2.  i>.2+;,),  ^(^2-6). 

3.  {8;^2+m2)-^-5m,  (2^2 -w3)^5?n. 

4,  a_^(a_&),  />-^(rt4.6).  5.   l~{a~h),  l-4-(a  +  6), 
8.     a-f-6-c,  a-h+c.                  7.  a  +  6-c,  o-fe  +  c 

8.  (a2+fi/j+&^)-^(«  +  &),   (a2-a/j+6-')-4-(a-&). 

9.  (a&-l)^(«-l)f^-l),  {a-Z;)--(a-l)(6-l}. 

10.  (l+a)--(aft-l),  {l+h)-^{ab-l). 

11.  («  +  l)(^'  +  l)-5-(«^-l).  {a-b)-^{ab-l). 

12.  a(a+fc),  b(a-b).  13.  a{6(a4-o)-c(a-c)}-4-(a3 -6c), 
a{6(a-6)  +  c(a+c)}-j-(a2_6c).         14.    -(a +  6).  at. 

15.  i(^+c),  &c.  16.  (a-26+3c)-f-38,  &c. 

17.  2-T-(/>+c),  &o.  18.  fl  +  6,  &c.     (by  symmetry). 

19.  b^-c^,  &c.  20.  b-2-c^,  &e. 

21.  ia&c,  (l-a)(l~^)(l-c),  (2~«)(2-6)(2-cy, 

22.  2a&c-4-(a&+fec  — ca).  28.  1,   1,  1. 

24.  ar-{ma  +  7ib+pc-\-qd),  &c. 

Id         \        Id^     dn 

25.  w  =  0,  or(— -l)-(^-^|- 

26.  (5-|.c-fl)-f-(rT  +  6+c),  2/  =  (6-c-a)-«-(a-*-«). 

27.  ^(a— 6+w-w),  &0, 


XXZll  ANSWEB9. 

28.  (4:a  +  2c—d—Bb),  y+z  by  symmetry- 

29.  —{a-b-\-c-{-d},  {nb-\-bc,  &.c.),  -{abd-\-&c.),  abed. 
80.     ^(a  — 6  +  e  — rf-}-e),  others  by  symmetry. 

31.     x  =  (a  —  l.b-\- Imc  —  Imnd  +  Imnpc)  —  (1  +  Imnpq),   the  others  bj* 
symmetry.  '        32.  a;  =  6  +  c— <?,  &c. 

34.  j^  =  (a+56  +  3c-7rf+9e)H-22,  &c. 

35.  z  =  ^(a+c),  then  symmetry.     36.  2  = -J+^  +  e,  &c. 
•  87.     a5  =  a— 2fe+3c  — 2(i+e,  then  by  symmetry. 

Exercise  Ixi. 

1.  a;=(2a^  +  a+fe+r)-=-2(a-/»)  where  r  =  4rt(&2-j.i  +  i)  _j. 
{da-h){U-a\ 

2.  x=(ar  +  l)^(fflr-l)  where  r«  =  (J2  -  1)  H-3(a3 -62) 

3.  a;={v/(l  +  a)(l+6)-i/(l-«)(l-6)}-5- 

{l/(l+a)(l  +  ?.)+v'(l-a)(l-fe)}. 

{^/(a  +  6  +  c)(«  +  ^>-c)-i/...[. 
6.     {a-\-b)^{l-ah).  7.     x=(a^-ai)s-(a^+6a) 

8.  /^+^\  *  _    («+»0(^>-f-n)  ^ 
\a;  — 1/     "    {a-~ni)[b  —  n)' ' 

9.  a;={aT/(l-6^)-6v/(l-a3)-^  ^(^S-js). 

10.  x={b  +  c-a)  ~  {(Vb-^c-a)(c-\-a-b){a  +  b-c).] 

11.  x  =  {b  +  c)^^(a  +  b){b-^c){c+a). 

12.  aj=  VC^+t  +  c) -j- rt,  &c. 

13.  a;  =  {62.^.^2  _^(t+c)}  -  i/2(a3  +  6»-f-c»  -  8«5c). 

14.  (fc+^-a),  &n.  16.     a  or  (a^— 6j -j- (i_ai). 


ANSWERS.  ZXXIU 


x-y=  V(a-i-b)(a  —  2b) -^  V{a  —  h),  &c. 

17.  ix  +  ,jr^  =  l[3b-  ^^,     {x-yy- =  il^^  -  b'j  . 

18.  (x  ■}-]/)  H-  (x  —  7/)  =  -/(«+ 36)  -j-  i/{a  —  b)  =  m  suppose. 

19.  y^  =  m  -j-  iam^  — 9??.+ 1)  where  m  = 

20.  a,  6.  21.     a;={^(a-c)  +  |/c[y-- {i/(a-c)-y/c}. 

22.  «=  (a-|-c)?/ -;- (a  — c),  &c.  • 

23.  x+y  =  {ab-l)^{a-b),  &,G. 

24.  a;2/={v/(o  +  2)-i/(Z»-2)}^-,/{(?<4-2)-i/(&-2)}  = 
p  suppose,  x-^y=  {]/«4-2)  +  y  («-2)}-j- 
{s/(a+2)-T,/(a-2)}. 

25.  x'1j^z'^  =  \{a  +  h-c)[h-\-c-n){c^a-b),  &c. 

26.  x=  (ai  — 6c  — ca)-^2s/a?y(;.  27.  a  —  x'^  =  ±m,  inhere  w  is 
the  value  of  t;  in  the  equation  4m  — 4(r +</)»-+ ^f*  =» 

(ca  -  ai  -  6c) 2  4- 46(ca  -ab-  bc)v  +  462^3 . 

/I        1   \ 
28.     a;  =  i|/(<5i6c)    \-t--\ I  ,  y  and  z  by  symmetry, 

29-       a(624.c^)^(52_,_c2)_^(J3_}_c2)^    ^c. 

30.     c{V{a-\-b)  +  i/{a-b)}^Via  +  b). 

81.  0  or  a  ib  +  c)^2bc,  &c. 

32.  »=  -1  or  ff/(a«-l)-^v''(ft'«  — 6'),  &e. 


xxxiv  answers. 

-Examination  Papers. 


I. 

1.     (a  +  h  +  r)(x  +  y+z). 

3.     x^-6x^  +  ldx^-12x+i,  x^+dx-^  +  d, 


X' 


nln- 


•  1)  J.;^"!"— 3)_l.a;«(»— 3) 


4.    2aj2'"-ix3m   A_  —  _ — .  5.   -jhl4|/ - 19-^-27,  0  or  ^. 

'*  0         c         a  ' 

6.  16,  7.  199,  8,  AB  37,  CA  52,  BG  45. 

II. 

1.  {a  +  h]^.  2.   (a+h  +  c)(a^  +  J,S-^c^ -^ahc).  8.   l-^3;3. 

4.  (a2'"4-2rt'"  +  2)^(a"'  +  2),  (<(+i-|-c)-^(rt-6-c)• 
7.      -I,  4  or  6.  8.  p'^q^{r-pq). 

9.     584.         10.  (am +  b)i)^  {a -\-b),  {by  common  rule). 

III. 

2.  ,/4-fca;+ca;2,  3  — 4a;-h7a;- -10a;3 

3.  2i  ;  6,  9,  12.  4.  160  eggs. 

5.  40,  35.  6.    /|(l+«)+i/Kl-«). 

7.  5  or  J;  4.  9.  (rt+6+c)(a— 6)(6-c)(a-c. 

IV. 
1.     (a  +  h  +  c)9.        2.  a-;-6.       3.   (4a;2 -92/2)(4a;«— 4?/5).  1  +  Va;. 

4.  L^3-|v^6,  17  5.4.        6.1.       7.  Cor  4;  8,1. 

8.  7i,  12.         9.  4  or  6.         10.  i(-3±  ^  -39). 

V. 
1.     8x3  +  1^125.  3.   IGa^b^.  4.  b-^8a.  5.  15,  12. 

6.  ia,  ia;  4  or  -9,  7.  6,  7,  8,  or  -6,  -7,  -*8. 


ANSWERS.  XXXV 

Vi. 

1.  (r,3-f-ft2).  2.  a^  -\-b^  +  e*  +  ah  +  ac -he. 

3.     .c'+r±%,xy,  {lx+6y-9){x-y-{-4:).        4.   -20,  0. 
5.     borl^i.  7.  x-  =  4  01-9,  i/  =  9  or4;  1,  2,  3; 

a;  =  i(_7  +  y^33).     8.    -1,  0,  1,  or  5,  6,  7,  or  -V.  -4.  ?• 
9.     xix'^d)[x^-tix-l)  =  0.       10.  x-^//-3^4. 

vn. 

2.  --01.  4.  a;2+x-l. 

5.  ^36x--^+l8a:+9)--fl6a;4-81),  (u;3 -a/yj--(x^ +«6). 

6.  9;  3i  .t;  =  -^(a-Z*). 

VIII. 

3.  (»3_y3)2(a;_y);  a=y;4.ry,  i=i?g. 

5.  <'.6-3  +  ^/x  +  ca;-^  ;  ^a;*  +  i^"  -  i« 

6.  x^+px+p'';  50{x+o){x-4:}{.c-6). 

7.  a=0  =  ior«.=  l,  ^1  =  2. 

8.  3  or  —  43-J-7  satisfies  the  equation  2  -  y  .  .    &c. 

IX. 

4.  axij+b.     7.   +^/a6;  A/(a4-6)-f-i/(«-6),  andt/  =  reciprocal 
of  this.     8.  P(22a-216)--20a(a-6).         9.  7,  15,  48. 

X. 
1.     aa;3  4-%3+2ca;?/.  B.  x-2.  4.  24.      . 

.5.     5x-^-Qux+ia^;y+-^-Vi-  7.  3;*.  8.4,6. 

XI. 
1.     2.  2.6,8.  8.  (4x*-9i/V,TV^^+-5%*. 

4_     ^3_^3  4.9a:-S  x'^ -{a-\-b)x-c.         6.  0.  ^ 

7.     Oor  -2i;  i/  =  di3or  ±|/-9,  &c. 


XXXVi  ANSWERS. 

XII. 


«         «• 


6.  m^  —vi=p,   q  =  0. 

7.  a;2=::(a_2)-^(«4-4),   y/iV=  {b{n~  I)  -  cn\ -i- y'a[n-y} 
a     a6-{(a-l))i-l)-l}.  9.  87.   ' 

XIII. 
1,     2.     3.  14-rja;+Coa;^4-&c.,  where  c^,  Cg.  <^c.,  repveseut  tbt 

combinations  of  rt|,  (''21  •  ■  ■  taken  one,  two,  &c..  at  a  time, 
4.     §4000.  5.   {m{b'c  —  bG')—n{ca'~c'a)}-i-{ah'  -  a'b), 

V=P""c^'        7  («  +  /3f+(ar-/i)'-2/  =  0.       9.66. 

XIV. 

1.  12rtie.  i.  Smiles.  5.  «tm-i-/i;  a;+^  =  ±5  or  ±1, 
X  -  ^  =  ±  1  or  ±  5.               6.  ■  20. 

XV. 

2.  A.  3.  c2(r2-6d!)  +  ci!2(62_^,c)+a/r(rt(i-ftc).  4.  2^(OT-n). 
If  m— w  is  negative  u;  is  neg.  -wbicli  shows  that  they  were 
together  before  noon.  If  m-n  =  Q,  x  is  infinite,  i.e.,  they 
are  never  together. 

5.  x^(x^  -r,2)(x-2a)  ;   (x^  -  a^)(;£^  ~  y^} . 

6.  (rt+6+c+3d)-=-(a-}-6  +  t-l-c/):   {x  +  y-^z)-^{x-y^z); 

3a-T-(a  +  i). 

XVI. 

4.     2x* -3a; 3+ 4a; +3.  5.   See  paper  XIX.,  prob,,  4. 

1         i  1 

&      i{l±l/(«"  +  4//)H-\/4ft".      8.    (i,Ci-^C2)--(ai63-fl2&j) 


AJ^SWEES. 


xxxvu 


1. 

8. 

1. 
7. 

1. 

4. 
6. 

4. 

5. 

7. 
8. 

1. 

4. 


XVII. 

8a^-2ab-10ac-Sb^+2c^  +  5bc.     6.   ?/-»i^  =  4. 
(w.  +  n){bq—pG)^niq  -pn),  {p-\-q)(j)ic  -  bn)-i-{mq  —pn). 

XVIII. 

l-aj»«.  2.1.  4:.  x-a-b.  6.  {x- y){x-z). 

1.  8.  9a;   ±:1-=-a/2  or  ±1^6.  9.  10,11. 

XIX. 

i(4a;-«  +  l)8.  3.  5;c2_i.  (3.3+^^^4.^2)3. 

(u  +  2)-'^4(a2+a>       7.  (76'-^ -a-')^12fl.       8.  18,  22,  50. 

XX. 
{l+w.)x+(l-n)y.  3.  a,-2_i. 

{a^b-2ab^  - a^ -^ab^  +  b^)-r-{a^  -b^). 

1.         6.  a>6'-6c')  +  /*^(ac'-a'6-)4-c"(^'6-«ft')  =  0. 


l/{^*^ 


3m 


;  2,  3,  4. 


10- 


a{b-c)-T-{b-a},  b{c—a)^{b-a).         9.  3.  10.  2000. 

XXI. 
8.         2.  24a&c;  2a^ +4:Sa^x^  +  Q2a^x'^+A4:a^x^ +  lSx». 

0;  rj7+16.     5.  16;  x+^Jx^ -2a.        7.  3377  oz.  of  gold, 
783  oz.  of  silver.     9.    -  (8±4-i/3)h-(3±2  \/3  ; 
a;==fc2  or  %/— 1,  y=  +1  or  q:2|/-l  ;  8,  4. 
y  =  cost  of  2iid  bale  =  60  ±  20  \/ 7. 


XXXVUl 


ANSWEBS. 


XXII. 

1.     -02997,  a^+aq+p^.  2.  ab^'~h^+c  =  0, 

{a).{a-b){8a-'db).   {b).x{x-  l){u—b){b - c){((  - c). 

8.  b^=A'ia.  (a),  (a  +  b)^.  5.  (a)  {ax-bi/)^{i:x  +  by) 

7.      (a)  U(^  +  b  +  c),   {b),   f,  -I,   2. 

(c).  b-c,c-a,a-b,     (d).    -1±V2. 

XXIII. 

1.     ^x'  +  4.r*-.T3  +  ia;3  +  ^^3^  +  (95a;3+21a;3  -  40a:+42)-- 

(3x4-21a;3-j-9a;-6);    -382.        -4.  x  =  a  +  'Ic,  y  =  b-[-Sc. 
6.     (2).     ^.         7.  (1).  a+i  +  c.  (2),  1,  2,  3,  4.  (3j,  0  or  ^. 


8      5'j(^2_i(3yi-|-2).       11. 


ayb^c^^d-^' 

a^b.^c.-^d.^ 

a^b„c^,l., 


=  0. 


12.     Coll.  to  Newmarket  63  miles. 

XXIV. 

1.     i^^x-y){y-z){z-x).     2.  -d-rti—n;  0.       3.  5,  3,  4^. 

4.  (1).  A  =  bc^2a,  £  =  ac~2b,  C  =  ab-ir-2c;  (2).  a^+b-^=c^. 
11^    y=+2,  x=±5,&c.;  x'-'-lOx=-lQov  -16, 

1  i  4pq 

:.  a;  =  8or  3,  &c.;  (6    '  ±a     )^ 

XXV. 

1     % 

1.     (a2-3x)-^(a2-x2);  1.       2.  —  -  — -i2(a;+a)H-a=a;3. 

5.  ^•2-2.<;  =  2,  a;  =  3,  y/  =  2;  z/  =  53--24,  &c.     6.  4  miles,  3  do. 


7.     (a;»-l)H-(x3-l).*;' 


.»-i 


8.  fl-IP-fl+f-      '■>■■'■ 


ANSWERS. 


xxxix 


1      1       1 

10      1+  -j-+Y-2~^r'2^'^^^-~^''^'^^'^^  approximately; 

{(5x)>3'"^^}{l-6-ll  ...  (5r-4)}^|jl.      11.  f  and  -^. 

XXVI. 

i.     a-^{nh-a^)',  4.         3.   2,  i,  or  ^(-8± '/S) ;  x-\-y-^z— 
v/(a3+2^*2),    .■.2  =  o+i/(rt3+2?>2),  &c.;|(_4±i/76).      ■ 

4.  3.         7.  )t>3H-4+r3-f-27  =  0.  10.  (l+a:)-^(l-a;)*- 
a;"-l{3.«-l+2«(l-a;)}-^-(l-a;)2  ;  n-^(15u+9j,  yV- 

xxvn. 

1.     {Tm-nltY-.  '  2.  (a  +  t+^)(rt-ft)(6-c)(a-c).      " 

3.     ia-b){h~c)-\-{b~c)(c-a)-^(c-a){a-b);  {a-b)^  + 
(6-c)2  +  (c-a)2.  4.  a;  =  a^(«2+62  4.c2),  &c. 

5.  N/a6;  18or-2;  i  +  Sj.  7.  i(«  +  />)&c.; 

a;={l+rt2_63  +  i/(l-a-6)(l-a  +  ?>)(l  +  a-i)(l+a+i)} 
-i-2a;    x-i-y  =  {l+n){l  —  b)-i-{l  —  ac),  &c. 

8.  —8;  (^4-4;j2^+87?r)-^(yj^-4g). 

9.  (m-}-n)'i-2>inn,  (?i  — mJ-5-2m» 


*"       ''^   Or  THE  • 

UNIVERSITY 


M.  J.  ©age  ^  (Eo's  Jleto  ©bucatioual  SEorks. 


NEW   BOOKS   BY  DR.  McLELLAN. 


The  Teacher's  Handbook  of  Algebra. 

Revised  and  ciilarg-ed.  By  J  A.  McLellan,  M.  A.,  LL.  D.,  Inspector  of 
High  Sciiools,  Ontario. 

Price,  $1.25 

Teacher's   Hand   Book  of  Algebra. ---Part  i. 

Abridged  Edition.      Containing'  portions  ot  the  a  ove  suitable  for  Inter-  ( 
mediate  Students. 

Price,  75  Cents. 

Key  to  Teacner's  Hand  Book.  Price,$i.50. 

It  contains  over  2,500  Exercises,  including  about  three  hundred  and  fifty 
solved  e.xaniples,  illustratingeverj'  type  of  question  set  in  elementary  Alge- 
bra. . 

ft  contains  complete  explanation  of  Horner's  Multiplication  and  Division, 
with  application  not  gi\  en  in  the  Text-Books. 

It  contains  a  full  explanation  of  the  principles  of  symmetry,  with  numer- 
ous illustrative  examples. 

It  contains  a  more  complete  i!lu=fiSz;ion  of  the  theory  of  divisors,  with  its 
neautifui  ai-iplications,  than  is  to  be  found  in  any  text-book. 

It  contains  what  aolc  niathematiaal  teachers  have  pronouriced  to  be  the 
"finest  chapter  on  factoring  that  has  ever  appeared." 

It  contains  the  latest  and  best  methods  of  treatment  as  given  by  the  great 
Masters  of  Anal.vsis. 

It  contains  the  finest  selections  of  properly  classified  equations,  witti 
methods  of  resolution  and  reduction,  that  has  yet  appeared. 

It  contains  a  set  of  practice  papers  made  up  by  selecting  the  best  of  the 
<|Ucstions  set  by  the  University  of  Toronto  during  twenty  years. 

It  is  a  key  ot  the  methods,  a  repertory  of  exercises,  which  cannot  fail  to 
make  the  teacher  a  better  teacher,  and  "the  student  a  more  thorough  alge- 
braist.   

Read  the  following  notices  from  the  leading  authorities  in  Great  Britain 
and  United  Slates: 

"  This  is  the  work  of  a  Canadian  Teacher  and  Inspector,  whose  name  is 
honorably  known  l)cyond  the  bounds  of  his  native  province,  for  his  exer- 
tions in  developing  rind  promoting  that  admirable  system  of  public  instnic- 
tion,  which  has  placed  he  Dominion  of  Canada  so  high,  as  regards  educa- 
tion, not  only  among  the  British  Colonies,  but  among  the  civilized  nations 
of  the  world.'  VVe-know  of  no  work  in  this  country  that  exactly  occupies 
the  place  of  Dr.  McLellan's,  which  is  not  merely  a  text  book  of  Algelna,  in 
the  ordinary  sense,  but  a  Manual  of  Methods  for  Teachers,  illustrating  the 
best  and  niost  recent  treatment  of  algebraical  prolileius  and  solutions  of 
every  kind." 

From  Barnes'  Editational  Montiilv,  N.  Y 

"The  best  American  Algebra  for  Teachers  that  we  have  ever  examined." 


M.  J.  ^age  ■&  Oto's.  JIclu  OSbumtional  oHork©.- 


EXAMINATION  SERIES. 


Canadian   History. 

By  James  L.  Hughks,  Inspectcr  of  Public  Schools,  Toronto. 
Price,  25  Cents. 

HISTORY  TAUGHT  BY  TOPICAL  METHOD. 

A   PRIMER   IN    CANADIAN    HISTORY,  FUR   SCHOOLS   AND   STUDENTS   PREPAKINO   FOR 

•EXAMINATIONS. 


1.  The  history  is  divided  into  periods  in  accordance  with  the  ^-leat  na- 
tional changes  that  have  taken  place. 

2.  The  history  of  each  period  is  given  topically  instead  of  in  chionolog- 
ical  order. 

3.  Examination  tiuestions  arc  given  at  the  ciul  of  each  chapter. 

4.  Examination  papers,  selected  from  the  official  examinations  of  the. 
different  provinces,  are  given  in  the  Appendix. 

5.  Student's  review  outlines,  to  enable  a  student  to  thoroughly  tost  his 
own  progress,  are  inserted  at  the  end  of  each  chapter. 

6.  Special  attention  is  paid  to  the  educational,  social  and  conunercial 
progress  of  the  country. 

7.  Constitutional  growth  is  treated  in  a  brief  but  comprehensive  ti\er- 

ciso. 

SS"  By  the  aid  of  this  work  students  can  prepare  and  revjew  for  exam- 
inations in  Canadian  History  more  quickly  than  by  the  use  of  any  other 
work. 


Epoch  Primer  of  English  History. 

By  Rev.  M.  Creioiiton,  M.  A.,  Late  Fellow  and  Tutor  of  Merton  College, 

Oxford. 

Authorized  by  the  Education  Department  for  use  in  Public  Schools, 
and  for  admission  to  the  High  Schools  of  Ontario. 

Its  adaptability  to  Public  School  use  over  all  other  School  Jlistories  will 
be  shown  by  the  fact  that — 

In  a  brief  compass  of  one  hundred  and  eighty  pages  it  covers  all  the 
work  required  for  pupils  preparing  for  entrance  to  High  Schools. 

The  price  is  less  than  one-half  that  of  the  other  authorized  Iiistories. 

In  usintr  the  other  Histories,  pupils  are  compelled  to  read  nearly  three 
times  as  much  in  order  to  secure  the  same  results. 

Creighton's  Epoch  Primer  has  been  adopted  by  the  Toronto  School 
Board,  »Hd  many  of  the  principal  Public  Schools  in  Ontario. 


Wi.  J.  ©age  ^*  €00.  <^ctp  ^iucational  ^orks 


MASON'S  GRADUATED  SERIES  OF  ENGLISH  GRAMMARS. 

V 

Mason's  Outlines  of  English  Grammar. 

For  the  use  of  junior  classes. 
Price, 45  Cents. 

Mason's  Shorter  English  Grammar. 

With  copious  and  carefully  graded  exercises,  243  pages. 
Price,  *  60  Cents. 

Mason's  Advanced    Grammar. 

Including  the  principles  of  Grammatical  Analysis.  By  C.  P.  M.\son, 
B.  A.,  F.  C.  F.,  ieilow  of  University  College,  London.  Enlarged  and 
thoroughly  revised,  with  Examinations  Papers  added  by  W.Houston.  M.A., 

27tli  Edition,  price,  75  Cents- 

;'  I  asked  a  grammar  school  inspector  in  the  old  country  to  send  me  the 
best  grammar  published  there.  He  immediately  sent  Mason's.  The  chap- 
ters on  the  analysis  of  dithcult  sentences  is  of  itself  sufficient  to  place  the 
work  far  be\  oiid  any  English  Grammar  hitherto  before  the  Canadian  nub- 
lie"— AJex.'Sims,  M.  A.,  H.  M.  H.  S.,  Oakville. 


English  Grammar  Practice. 

'  This  work  consists  of  the  Exercises  appended  to  the  "Shorter  English 
/Grammar,"  published  in  a  separate  form.  They  are  arranged  in  progressive 
lessons  in  such  a  maimer  as  to  be  available  with  almost  any  text  book  of 
English  Grammar,  and  take  the  learner  by  easy  stages  from  the  simplest 
English  work  to  the  most  difficult  constructions  in  the  language. 

Price,  30  Cents. 

Outlines  of  English  Grammar. 

These  elementary  ideas  are  reduced  to  regular  form  by  means  of  careful 
definitions  and  plain  rules,  illustrated  by  abundant  and  varied  examples  for 
practice.  The  learner  is  made  acquainted,  in  moderate  measure,  with  the 
most  important  of  the  older  forms  of  English,  with  the  w^ay  in  which  words 
are  constructed,  and  with  the  elements  of  which  modern  English  is  made 
up.  Analysis  is  treated  so  far  as  to  give  the  power  of  dealing  with  sen- 
tences of  plain  construction  and  moderate  diflculty.     In  the 

English  Grammar 

the  same  subjects  are  presented  with  much  greater  fulness,  and  carried  to  ! 
a  more  advanced  and  dilficult  stage.  Tlie  work  contains  ample  materials  ' 
for  the  reiiuircments  of  Comyjctitive  Examinations  reaching  at  least  the  | 
standard  of  the  Matriculation  Examination  of  the  University  of  London. 

The  Shorter  English  Grammar. 

is  intended  for  learners  who  have  but  a  limited  amount  of  time  at  their  dis-   ' 
posal  for  English  studies  ;    but  the  expsrience  of  schools  in  which  it  iias 
been  the  only  English  Grammar  used,  has  shown  that,  when  well  mastered, 
this  work  also  is  sufficient  for  the  London  Matriculation  Examination. 
. —      — ^,^ — . — . — ^— 


cB.  J.  Sage  ^  (E.o's.  |leiu"  educational  Mlorks. 

HAMBLIN  SMITH'S  MATHEMATICAL  WORKS. 

Authorized  for  use,  and  now  used  in  nearly  all  the  principal  Schools  of 
Ontario,  Quebec,  Nova  Scotia  and  Manitoba. 


Hamblin  Smith's   Arithmetic, 

An  Advanced  treatise,  on  the  Unitary  System,  by  J.  Hambltn  Smith, 
M.  A.,  of  Gonville  and  Caius  Colleges,  and  late  lecturer  of  St.  Peters  Col- 
lege, Cambridge.  Adapted'  to  Canadian  Schools,  by  Thomas  Kirkland, 
M.  A.,  Science  Master,  Normal  School,  Toronto,  and  William  Scott,  B.  A., 
Head  Master  Model  School  for  Ontario. 

12th  Edition,  Price,  75  Cents. 


KE  Y.^ — A  complete  Key  to  the  atjoy*  Arithmetic,  by  the  Authors. 
Price,  $2.00. 

Hamblin  Smith's   Algebra. 

An  Elementary  Algebra,  by   J.   H.\mblix  Smith,  M.  A.,  with  Appendix 
by  Alfred   Baker,  B.  A.,  Mathematical  Tutor,  University  College,  Toronto. 

8tli  Edison  Price,  90  Cents. 

KEY. — A  complete  Key  to  Hamblin  Smith's  Algebra. 

Price,  $2.75. 

Hamblin    Smith's    Elements    of    Geometry. 

Containing  Books  I.  to  VI.,  and  portions  of  Books  XI.  and  XII., of  Euclid, 
with  E.xercises  and  Notes,  ijy  J.  Hamblin  Smith,  M.  A..  &c.,  and  Examina- 
tion Papers,  from  the  Toronto  and  McGill  Universities,  and  Normal  School, 
Toronto. 
Price,  ■  90  Cents. 


Hamblin  Smith's  Geometry  Books,  i  and  2. 
Price, 30  Cents. 

Hamblin  Smith's  Statics. 

By  J.  Hameli.n"  Smith,  M   \  ,  with  .\ppendi,x  by  Thomas  Kirkland,  M.  A., 
Science  Master,  Normal  School,  Toronto. 

Price,  ~  90  Cijnts. 

Hamblin  Smith's  Hydrostatics.  75  Cents. 

KEY.  — Statics  and  Hydrostatics,  in  one  volume.  $2.00. 


Hamblin  Smith's  Trigonometry.  $1.25. 

KEY.— To  the  above.  $2.50. 


m.  J.  ©age  &  Clo'e.  ^etu  ^ixtratioital  gEorks. 

BOOKS  FOR  TEACHERS  AND  STUDENTS,  BY   DR.  McLELLAN. 


Examination    Papers   in   Arithmetic. 

By  J.  A.  McLellan,  M.  A.,  LL.  D.,  Inspector  of  Ili^h  Schools,  Out.,  and 
Thomas  Kirkland,  M.  A.,  Science  Master,  Nonnal  School,  Toronto. 

"  In  our  opinion  the  best  Collection  of  Problems  on  the  American  Con- 
tinent."—National  Teachers'  Monthly,  N.  Y. 

Seventh  Complete  Edition,  -  -  Price.  $1.00. 

Examination  Papers  in  Arithmetic. ---Part  I. 

By  J.  A.  McLellan,  M.  A.,  LL.  D.,  and  Thos.  Kirkland,  M.  A. 
Price,  -  .  .  .  .  5o   Cents. 

This  Edition  has  been  issued  at  the  request  of  a  large  number  of  Public 
School  teachers  who  wish  to  have  a  Cheap  Edition  for  the  use  of  their 
pupils  preparing  for  admission  to  High  School. 

Hints  and  Answers  to  Examination  Papers 
in  Arithmetic.  '' 

By  J.  A.  McLellan,  M.  A.,  LL.  D.,  and  Thos.  Kirkland,  M.  A. 
Fourth  Edition,  -  -  -  -  -  $1  o6. 


McLellan's     Mental     Arithmetic,---Part     I. 

Containing  the  Fundamental  Rules,  Fractions  and  Analysis. 

By  J.  A.  McLellan,  M.  A.,  LL.  D.,  Inspector  High  Schools,  Ontario. 

Third  Edition,  -  -  -  -  SO  Cents. 

Authorized  for  use  in  the  Schools  of  Nova  Scotia. 


McLellan's     Mental    Arithmetic. ---Part     II. 

Specially  adapted  for  Model  and  High  School  Students. 
Third  Edition,  -  -  -  Price,  45  Cents. 

The    Teacher's    Hand   Book   of  Algebra. 

By  J.  A.  McLellan,  M.  A.,  LL.  D. 
Second  Complete  Edition,  _  .  .         $1.25. 


Teacher's   Hand  Book  of  Algebra. ---Part  I. 

Prepared  for  the  use  of  Intermediate  Students. 
Price,  -----  75  Cents. 

Key   to    Teacher's   Hand   Book   of  Algebra. 
Second  Edition,  -  .  .  Price,  $1.5Q. 


WORKS  FOR  TEACHERS  AND  STUDENTS,  BY  JAS.  L.  HUGHES. 


Examination    Primer    in  Canadian  History. 

On  the  Topical  Method.     By  Ja.s.  L.  Hiohes,  Iiis[K3ctor  of   Schools,  To. 
rente.   A  Primer  for  Students  prejiaring  for  Examination.       Price,  25c 

Mistakes  in  Teaching. 

By  Jaj.  Laughmn"  Hughes.    Second  edition.  Price,  50c. 

M)0FTEI>  BY  8TATB    nNIVKllSITT   OF    IOWA,  A3   AS    8LRMBNTART   WORK    FOR    USE 

OF  TEACHERS. 


This  work  discusses  in  a  terse  manner  over  one  hundred  of  the  mistakes 
commonly  made  by  untrained  or  inexperienced  Teachers.  It  is  desifjned  to 
warn  young  Teachers  <>i  the  errors  they  are  liable  to  make,  and  to  help  the 
older  members  of  the  profession  to  discard  whatever  methods  or  habits  may 
be  preventing  their  hiyher  success. 

The  mistakes  are  arranged  under  the  following  heads  : 

1.  Mistakes  in  Management.  2.  Mistakes  in  Discipline.  S.  Mistakes  In 
Methods.    4.  Mistakes  in  Manner. 


How  to  Secure  and  Retain  Attention. 

By  Jas.  Lauqhlin  Hughes.  Price,  25  Cents. 

Comprising  Kinds  of  Attention.  Characteristics  of  Positive  AttentionI 
Characteristics  of  The  Teacher.  How  to  Control  a  Class.  Developing  Men 
tal  Activity.    Cultivation  of  the  Senses. 

(From  The  School  and  University  Maoazinr,  London,  E^-g.) 
"Replete  with  valuable  hints  and  practical  suggestions  which  are  evident- 
ly the  result  of  wide  experience  in  the  scholastic  profession." 


Manual  of  Drill  and  Calisthenics  for  use   in 
Schools. 

By  J.  L.  Hughes, Public  School  Inapector,  Toronto,  Graduate  of  Military 
School,  H.  M.  29th  Regiment.  Price,  40  Cents. 

The  work  contains :  The  Squad  Drill  prescribed  for  Public  Schools  in  On- 
tario, with  full  and  explicit  directions  for  teaching  it.  Free  Gymnastic  Ex- 
ercises, carefully  selected  from  the  best  German  and  American  systems, 
and  arranged  in  proper  classes.  German  Calisthenic  Exercises,  as  taught 
by  the  late  Colonel  Goodvon  in  Toronto  Normal  School,  and  in  England. 
Several  of  the  best  Kindergarten  Games,  and  a  few  choice  Exercise  Song^s. 
The  instructions  throughout  the  book  are  divested,  as  far  as  possible,  of 
unnecessary  technicalities. 

"A  most  valuable  book  for  every  teacher,  jnrticularly  In  country  places- 
It  embraces  all  that  a  school  teacher  should  teach  his  pupils  on  this  subject. 
Any  teacher  can  use  the  easy  drill  lessons,  and  by  doing  so  he  will  be  con- 
i  ferring  a  benefit  on  his  country." — C.  RadciiIffb  Dkarnaly,  Major  First 
i   Life  Guards,  Diill  Instructor  Xormal  and  Model  Schools,  Toronto. 


Authorized  for  use  in  the  Schools  of  Ontario. 
The  Epoch  Primer  of  English  History. 

By  Rkv.  M.  Creigiito.v,  .M.  A.,  Late  Fellow  and  Tutor  of  Meiton  College, 


Dxloid. 


Sixth  Edition, 
Most  thorough. 


Price,  SO  Cents. 

AEERnF.rS'   JOfRNAL. 


This  volume,  taken  with  the  eight  small  volumes  cciitaining;  the  ac- 
counts of  the  different  epochs,  presents  what  may  be  regarded  as  the  most 
thorough  co\u-se  of  elementary  English  History  ever  published. 

What  was  needed.  Toronto  Daily  Globe. 

It  is  just  such  a  manual  as  is  needed  by  pul))ic  school  pupils  who  are 
going  up  for  a  High  School  lourse. 

Used  in  separate  schools.  M.  Stafford,  Prik.st. 

We  are  using  this  History  in  our  Convent  and  Separate  Schools  in  Lind- 
say. 

Very  concise.  Hamilton-  Ti.mes. 

A  very  concise  little  hook  that  should  be  used  in  the  Schools.  In  its 
pages  will  be  found  incidents  of  English  History  from  A.  D.  43  to  1870,  in' 
tercsting  alike  to  young  and  old. 

A  favorite.  London  Advertisf.r. 

The  book  will  prove  a  favorite  with  teachers  preparing  pupils  for  the 
entrance  examinations  to  the  High  Schools. 

Very  attractive.  British  Whig,  Kingston. 

This  little  book,  of  one  hundred  and  forty  pages,  presents  history  in  a 
very  attractive  shape. 

Wisely  arrang'ed.  Canada  Presbtterian. 

The  epochs  chosen  for  the  division  of  English  History  are  well  marked 

— no  mere  artificial  milestones,  arbitrarilj'  erected  b}-  the  author,  but  roaj 

natural  landmarks,  consisting  of  great  and  important  events  or  remarkable 

changes. 

Interesting.  Yarmoi'th  TRiBrxF.,  Nova  Scotia. 

With  a  perfect  freedom  from  all  looseness  of  style  the  intei'est  is  so  well 
sustained  throuKhoirt  the  narrative  that  those  who  commence  to  it..  .</ 
will  liiid  it  difficult  to  leave  off  with  its  perusal  incomplete. 

Comprehensive.  Literary  World. 

The  special  value  of  this  historical  outline  is  that  it  gives  the  reader  a 
comprehensive  view  of  the  course  of  memorable  events  and  epochs. 


m.  J.  (gage  S:  Co's.  Jlcto  eiurational  moxks. 


THE  BEST  ELEMENTARY  TEXT-BOOK  OF  THE  YEAR. 


Gage's  Practical  Speller. 

A  MANUAL  OF  SPELLING  AND  DICTATION. 
Price, 


SO  Cents. 


Sixty  copies  ordered.  Mount  Forest  Advucate. 

After  careful  inspect  on  we  unhesitatingly  pronounce  it  tiie  best  spell- 
ing book  ever  in  use  in  our' public  schools.  The  Practical  Speller  secures 
an  easy  access  to  its  contents  by  the  very  systematic  arrangements  of  the 
words  *in  topical  classes ;  a  permanent  impression  on  the  memory  by  the 
frequent  review  of  difficult  words  ;  and  a  saving  of  time  and  effort  by  the 
selection  of  only  such  words  as  are  difficult  and  of  connuon  occurrence^ 
Mr.  Reid,  H.  S.  Master  heartily  recommends  the  work,  and  ordered  some 
sixty  copies.  It  is  a  book  that  should  be  on  every  business  man's  table  as 
well  as  in  the  school  room. 

Is  a  necessity.  Phesb.  Wit.ness,  Halifax. 

We  have  already  had  repeated  occasion  to  speak  highly  of  the  Educa- 
tional Series  of  which  this  book  is  one.  The  ■•  Speller"  is  a  necessity  ;  and 
we  have  seen  no  book  which  we  can  recommend  more  heartily  than  the  one 
before  us. 


Grood  print.  Bow.MAiNviLLE  Observer- 

The  "  Practical  Speller"  is  a  credit  to  the  publishers  in  its  general  get 
up,  classification  of  subjects,  and  clearness  of  treatment.  The  child  wh« 
uses  this  book  will  not  have  damaged  eyesight  through  bad  print. 

What  it  is.  Stk.miiroy  Aoe. 

It  is  a  scries  of  graded  lessonSj  .containing  the  words  in  general  use, 
with  abbreviations,  etc.  ;  words  of  similar  pronunciation  and  different  spell- 
ing- a  collection  of  the  most  difficult  words  in  the  language,  and  a  number 
of  literary  selections  which  may  be  used  for  dictation  lessons,  and  commit" 
ted  to  memory  by  the  pupils. 


Ex«ry  teacher  should  introduce  it. 
It  is  an  improvement  on  the  old  spelling 
introduce  it  into  his  classes 


C'ANADIA.V   STATESMA.N.    j 

book.    Every  teacher  should  | 


The  best  yet  seen.  Colchester  Sun,  Nova  Scotia. 

Itis  away  ahead  of  any"speller"that  we  have  heretofore  seen.  Our  public 
schools  want  a  good  spelling  hook.  The  publication  before  us  is  the  best 
we  ha\  e  yet  seen. 


.  J.  (iagc  S:  aro'0.  JlcU)  @^urationnl  ^orks. 


The  Canada  School  Journal 

HAS    RRCEIVED   AX    IIONOIiARLE     MKNTION    AT   PARIS    EXHIBITION,    1878. 

Adopted  by  nearly  every  County  in  Caiia«ia. 
Recommended  by  the  Ministei'of  Education,  Ontario. 
Reconimeiided  b.v  the  Council  of  Pul>lio  Instnictioii,  (Jftelioe. 
Recommended  by  Chief  Sujit   if  Education,  New  Brunswick. 
Recomniciidec  liy  Chief  Supt.  of  Education,  Nova  Scotia. 
Recommended  by  Chief  Supt.  of  Education,  Uriti.sh  t:o!umbia. 
Recommended  by  Chief  Supt.  of  Education,  Manitoba. 

IT  I.S   EDITED   BY 

A  Committee  of  sotnc  of  the  Leading  Educationists  in  Ontario,  assisted 
by  able  Provincial  Editors  in  the  Provinces  of  QucIjcc,  Nova  Scotia,  New 
Brunswick,  Prince  Edward  Island,  Manitoba,  and  T5ritish  Columbia,  thus 
having'  each  section  of  Vm  Dominion  fully  represented. 

CONTAINS  TWENTY-FOCR    PAGES  OF   READINO     MATTER. 

Live  Editorials  ;  Contributions  on  important  Educational  topics  ;  Selec- 
tions— Readings  for  the  School  Room  ;  and  Notes  and  News  from  each  Pro- 
vince. 

Practical  Department  will  always  contain  useful  hints  on  methods  of 
teaching  different  subjects. 

.Mathe.matical  Department  gives  salutions  to  diillcult  proljlems  also  on 
ENamination  Papers. 

Official  Department  contains  .such  regulations  a.s  may  be  issued  from 
time  to  time. 

Suhscription,  $1  00  j-er  annum,  strictly  in  advance. 

Read  THE  Followino  Lettkr  from  John  Greenleaf  Whittier,  the  Fa- 
Mois  American  Poet. 

I  have  also  received  a  No.  of  the  "  Canada  School  Journal,"  which  seems 
to  me  the  brightest  and  most  readable  of  Educationa  Magazines  I  am  very 
truly  tliy  friend,  John  Greenleaf  Whittier. 

A  Club  of  1,000  Sifoscribei's  from  Nova  Scotia. 
(Copj)  Em-CATiox  Office,  Halifax,  N.  S.,  Nov.  17,  1878. 

Messr.s.  Adam  Miller  &  Co.,  Toronto,  Ont. 
i  Dear  Sirs,— In  order  to  meet  the  wishes  of  oxir  teachers  in  \arious  part^ 
I  of  the  Province,  and  to  secure  for  them  the  advantage  of  your  excellent 
;  periodical,  I  hereby  subscribe  in  their  behalf  for  one  thousand  (1,000)  copies 
,  at  dub  i-ates  mentioned  in  your  recent  esteemed  favor.  Subscriptions  wiij 
!  begin  with  January  issue,  and  lists  will  be  forwarded  to  your  office  in  a  few 
,  days.  Yours  truly, 

1  David  Allison,  Chief  Supt.  of  Education. 

I  Address,  W.  J.  GAGE  &CO.,  Toronto.  Canada 


! 


J 


THIS  BOOK  IS  DUE  ON  THE  LAST  DATE 
STAMPED  BELOW 


AN     INITIAL    FINE    OF    25    CENTS 

WILL  BE  ASSESSED  FOR  FAILURE  TO  RETURN 
THIS  BOOK  ON  THE  DATE  DUE.  THE  PENALTY 
WILL  INCREASE  TO  50  CENTS  ON  THE  FOURTH 
DAY  AND  TO  $1.00  ON  THE  SEVENTH  DAY 
OVERDUE. 


i 


U 


FEB  27  r^<': 


19Feb'57BP 

FEB  5    19b/ 


If)/ 220